You are on page 1of 496

MIR

PUBL1SHF..RS

r. c.
C. M.

KOcUH,

c.

OeMU008U'l" B. A Et/JuMeHf(O, Jl JlyHU,. E cP nOpwHeea, E Cbllieea, B. et>PO/lOB, P. fl. lllocmaK, A. P. f/nnoAbcKuiJ.

napaHeHK08.. B.

n.

3AJlAlIH 11 ynPA)I(HEHI1S1

no
MATEMATHLJECKOMY
AHAJll13Y

nOG penaKt4lJl't1
B.

n. A

E M 1-1

A 0 B M q A

t/JU3UKO-ManteMamU'l.eCKOil

rocyiJapcmseHHoe U3aameAbCmpo AU mepDlIlYPbl

OCK.6Q

G. Baranenkov, B. Drm;dovich V. Epmenko, S. Kogan, G. Lunt~) E. Por~hnlva, E. ~!/(:h(ta, S. fl0lCJv, R. ~ho~takt A. Yanpolsky

PROBLEMS IN MATHEMATICAL ANALYSIS


Under the editorship

of

B. DEMI DOVICH
Translated /rorn the Russian. by
G. YANKOVSKY

MI~

PUBLISHERS Moscow

TO THE READER

M/R Publishers would be glad to have yo"r opinion of the translatton and the design of this book. P lease send your suggest tons to 2. Pervy Rlzh~kg Pereulok, Mo~(.ow, u. S. S. R.

Second PrintiRR

Printed In

the Union

of

Soviet Socialist

Republics

CONTENTS
Preface .
9

Cizapter 1. INTRODUCTION TO ANALYSIS


Sec. Sec. Sec. Sec. Sec.
1. Pu nctions

2 3 4 5.

. . . . . . Graphs of Elementary Functions Li mits . Infinitely Small and Large Quantities. Continuity of Functions . . . . . . . . . . .

11 16

22
33

36

Chapter II DIFFERENTIATION Of FUNCTIONS


Sec 1. Calculating Derivatives Directly . . . . . . . . . Sec 2 Tabular Differentiation Sec. 3 The Dprivat'ves of Functions Not Represented Explicitly Sec. 4. Geometrical and Mechanical Applications of the Derivative . Sec 5 DeflvatlveC) of Hig ler Orders ..... Sec 6 DifTerE:'ntials of First and Higher Orders Sec 7 Mean Value Theorems . Sec. 8 Taylor'\) Formula Sec 9 The L'Hospital-Bernoulli Rule for Evaluating Indeterminate Forms ,. . ..... 42 46 56 60
66

11 15 17
78

Chapter III THE EXTREMA OF A FUNCTION AND THE GEOMETRIC APPLICATIONS OF A DER IVATIVE
Sec. Sec. Sec Sec Sec.
1. The Extrema of a Function of One Argument

2 3 4. 5.

83 The Dirfct ion of Concavity Points of Inflection 91 AsymptoteC) . . 93 Graph Ing Functions by Characteristic Points 96 DilTerential of an Arc Curvature . . . 101
I~DEFINI"E

Chapter IV
Sec. 1 Sec 2 Sec 3 Sec. 4 Sec. 5.

INTEGRALS

Inte~ration

Direct lntellration . . by Substitution Integrat 1 0n by Part~ . . . . . Standard Integral'\ ContaininR a Quadratic Trinornlal Integration of Rational Functions

. . .

107 113
116

118 121

Contents
Sec. 6. Init'grating Certain IrratIonal Functions 125 ~c 7. lr;t~rating 1 rifonrrretric Functions . . 128 ~ec. 8 Integrat ion of H Yferbolic Functions . . . . . 133 Sec 9. USIng I ngonometric and Hyperbolic Substitutions for Finding

Integrals of the Form ~ R (x. Vaxl+bx+cl dx. Where R


tional Sec 10 Sec 11 Sec. 12. FunctIon . . . . . . . . . . . . . . . . . . . . I ntf,:rat ion of Varlouc; Transcendental Functions Using Reduction Formulas. ..... MIscellaneous Examples on Integration .

IS

a Ra133

135 135
136

Chapter V DEFINITE INTEGRALS


Sec. 1. The Denn ite I ntegral as the Limit of a Sunl . 138 ~c 2 Evaluatll";~ [(tirite Intfgrals by N.eans of lndeflniteIntegrals 140 Sec. 3 Improper Integrals . . . . . . . . . . . 143 Sec 4 Charge ~f Vari2ble in a Dffinite Integral 146 ~ec. 5. In1f~ration by Parts . . . . 149 Sec 6 Mean- Value Theort'm . . 150 Sec. 7. The Areas of Plant' Fi~ures 153 Sec 8. The Arc Length of a Curve 158 Sec 9 Volumes of Sol ids . . . . 161 5ec 10 The Area of a ~urface of RE'volution 166 Sec 11 Norrfnts Centres of Cravity CJuldln's Thforems 168 ~ec 12. Applyir.g [ef!r.ite Intgrals to the Solution of Physical Prcblems . . . . . . . 173

Chapter VI. FUNCTIONS OF SEVERAL VARIABLES


Sec. Sec. Sec Sec Sec Sec. Stc. Sec Sec Sec Sec. Sec Sec. Sec Sec Sec Sec.
1. Basic Notions 2. Contlnulty

180
184 185

3 4 5 6.

P.artlal Derivatives Total DlfJerential of a Function ... Dlffprpntiation of Comroslte F-unctions Dt-f1Vatlve In a GIven DirtAct10n and the Gradient of a Function 7 H I~lfel -Crder Cenvat lVfS and Differentials 8 I ntegration of 10tal D;f;erentials 9 Dif.erentiation of I mpllclt functions. 10 Chanf!e of Variables . . . . . . . . . 11. The Tangent Plant' and the Normal to a Surface 12 1 aylor'~ Formula for a F-un('tion of 5pveral Variables 13 The Ex tremum of a FunctIon of Several Variables 14 Fir dlf:~ 1ht- Createst and c. n~allt"st Values of Functions 15 Slnl'ular POInts ot Plane Curves 16 Envelope . 17. Arc Length of a Space Curve , .

181
190 193 197

202 205 211 217 220 222 227


230

2.32 234

Contents
S~c. 18. The Vector Function of a Scalar Argument Sec. 19 The Natural Trihedrnn of a Space Curve Sec. 20. Curvature and Torsion of a Space Curve

7
2JS 238

242

Chapter VII. MULTIPLE AND LINE INTEGRALS


Sec. 1 The Double Integral in Rectangular Coordinates . . . 246 Sec. 2 Chan~e of Variables in a Double Integral 252 Sec. 3. Com pu ti ng Areas . 256 Sec. 4. Computing Volumes . . . 258 Sec. 5. Computing the Areas of Surfac~s . . . . 259 Sec. 6 Applications of the Double Integral in Mechanics 230 Sec. 7. Triple Integrals . 262 Sec. 8. Improper Integrals Dependent on a Paralneter. Improper Multi lIe Integrals. . 269 Sec. 9 Line Integrals . . . . . 273 Sec. 10. Surface Integrals . . . . . . 284 Sec. 11. The Ostrogradsky-Gauss Formula 286 Sec. 12. Fundamentals of Field Theory 288

Chapter VIII. SERIES


Sec. Sec. Sec. Sec.
1. Number Series 2. Functional Series 3. Taylor's Series 4. Fourier's Series
293

304
. . . . . 311 318

Chapter IX DIFFERENTIAL EQUATIONS


Sec. 1. Verifying Solutions. Forming Differential Equations of FaOli lies of Curves. Initial Conet itions . . . . . . . . . . . . Sec. 2 First-Order Differential Equations . . . . . Sec. 3. First-Order Diflerential Equations with Variables Separable. Orthogonal Trajer.tories . . . . . . . . . . . . . . . . . Sec. 4 First-Order Homo~eneous Differential Equ:Itlons . . Sec. 5. First-Order Linear Differential Equations. Bernoulli's Equation . . . . . . . . . . . . . . . . . . Sec. 6 Exact Differential Equations. Integrating Factor . . Sec 7 First-Ordpr DIfferentIal Equations not Solved for the Derivative Sec. 8. The Lagrange and Clairaut Equations . Sec. 9. Miscellaneous Exercises on First-Order Differential Equations Sec. 10. I-ligher-Order Differential Equations . Sec. 11. Linear Differential Equations . . . Sec. 12. LInear Differential Equations of Second Order with Constant Coefficients 322 324 327 330 3::12
335

337 339 340


345

349 351

Contents

Sec. 13. Linear Differential Equations of Order Higher than Two with Constant CoefficIents ..... . . Sec 14. Euler's Equations . . . . . . . . Sec 15. Systems of Diff'l'rentlal Equa tions . . . . . . . . . . . . Sec. 16. Integration of Dit'ien'ntial Equations by Means of Power Series . . . . . . . . . . . Sec 17. Problenls on Fourier's Method

356 357 359


361

363

Chapter X. APPROXIMATE CALCULATIONS


Sec. 1 Operations on Approxarnatc NUlllbers Sec. 2. Interpolation of Functions . . . . . Sec. 3. ConlputtT1f.! thc,Rcnl Roots of EquCltions . ~ec. 4 NUlllprital, In tet!ration of FUllct ions . . . . . . Sec. 5. f\un encal Inte~'r(ltion of Crdlrary Dtf'1t:rtntial Sec. 6. ApprO}dITlating P<. tlfllr'S Ccefficlents ANSWERS ..... . . . . " Equations .
367 372

376
382 384 3Q3
396

APPENDIX . . . . . . I. Grrcl< A1ph a bpt . 11. SOlne COl1~tants . III. Inverse Quantities, Powers, Roots, Lo~arithlns . IV Trigollollletric Funct ton~ . . . V. EXrorel~ tIaI, 11 y ~erbolic and Trigon01l1etric Functions VI. Some Curves . . . . . . . . . . . .

475 475
475

476 478
479

480

PREFACE
This collection of problems and exercises in mathematical analysis covers the maximum requirements of general courses in higher mathematics for higher technical schools. It contains over 3,000 problems sequentially arranged in (~hapters I to X covering all branches of higher mathematics (with the exception of analytical geometry) given in college courses. Particular attention is given to the most important sections of the course that require established skills (the finding of limits, differentiation techniques, the graphing of functions, integration techniques, the applications of definite integrals, series, the sol ut ion of differential equations). Since some institutes have extended courses of mathematics, the authors have included problems on field theory, the Fourier method, and approxima1e calculations. Experience sho\vs that the nunlber of problclns given in this book not only fully satisfies the requirclT:n s of the student, as far as practical 11las~ering of the various sections of the course goes, but also enables the instructor to supply a varied choIce of problems in each section and to select problenls for tests and examinations. Each chap.er begins with a brief theoretical introduction that covers the basic definitions and formulas of that section of the course. Here the rnost inlportant typical problems are worked out in full. We beli('ve that this will greatly Silllplify the work of the student. Answers are given to all computational problems; one asterisk indicate~ that hints to the solution are given in the answers, two asterisks, that the solution is given. The problems are frequently illustrated by drawings. This collection of problems is the result of many years of teaching higher mathelnatics in the technical schools of the Soviet Union. It includes, in addition to original problems and exam pies, a large nunlber of commonly used problelTIs.

Chapter I

INTRODUCTION TO ANALYSIS

Sec. 1. Functions
1. Real nurrl:ers. Rational and irrational numbers are collectively known 1 he ab\olutf value of a real nurnber a IS undt'rstood to be the nonnegative numb~r I a, defined by the conditions I a I = a if a ~ 0, and la , = - a if a < o. lhe following in(qual1ty holds for all real numbers CJ

as

real numbers

and b:

'a+bl~lal+'bl.

2. Definition of a function. If to every val ue *) of a variable x, wh ich belongs to SOllie collect Ion (set) E, there corresponds one and onl y one finite value of the quantity y, 1hen y is said to be a funcllon (~lnRle-valued) of x or a dependent 1 artable def! ned on the ~et E, x is the argument or tndependent vanable The fact that II IS a functIon of x IS expressed In brief form by the notation y = f (x) or y == F (.\), and the I'ke If to every value of x belonging to somt set E there corresponds one or several values of the vanabl~ !i, then y is called a multIple-valued functton of x defined on E. Fronl now on we shall u~e the \\ford "functIon" only in the meaning of a ~ln[!./e-vQlupd functIon, 11 not other\vl~e stated 3 The domain of deltnition of a function. The collection of values of x for which the gIven functlon IS dt.\Hned IS called the dOTnaln 01 de{tTulton (or the domain) of thiS function. In the slnlplest cases, the' domaIn of a function IS either a clofied Interval [a, bl, which is the set of n~al nUlnbprs x that satisfy the inequal itJes a ~ ~ ~ b, or an open tnterval (a. b). wh Ich :s the set of real nuolhers that satisfy the In('quallLes a < x < b. AI~o pOSSible IS a ITIOre complex structure of the dOllJ3111 01 defInition of 3 function (see, for Instance, Probleln 21) Exanlple 1. Determine the donlaln of definItion of the funchon

Y=

V x2-1 .
> 0,

Solution. The function is defined if x 2 -1

that is, if , x I> 1. Thus, the domaIn of the function is a set of two intervals: - 00 < x < - I ~nd I < x < + 00 4. Inverse functions. If the equation y = f (x) may be solved unIquely for the variable x, that is, if there is a function x =g (y) such that y == f [g (y,
*) Hencelorth all values will be conSIdered
8S

real, if not otherwise

stated.

12

Introduction to Analysis

(Ch. 1

then the function x = g (y), or. in standard notation, y =g (x), is the tnverse of Y=f(x). Obviou~lv. g(f(x))=:x, that is, the function ,(x) is the Inverse of g (x) (a nd v ice versa). In 11 r J ereJ al ca~e, tre rquation y = f (x) refines a multiple-valued Inver~e fupct Ion x = f- J (y) ~uch that y == f If-) (y)J for all Y that are values of the function f (x) I:xhn Jjle 2. Cetennine the inverse of the funchon
y= 1-2- x .
(1)

Solution. Solving equation (1) for x, we have 2- x =1-y and x=Obviou~ly,

IO~~~;Y) *).

(2)

tl-'e dotr'ain of cefinition of the function (2) Is - co < Y < 1. 5. Corrros it e and ireplicit functicns. A function Y of x defined by a ~e ries of equalit1t~sy=f(u). whereu=cp(x), etc., is called a comoosite function, or a function of a fun~tiol,. A function dfined by an fqu~tion not solved for the derencent variable is calh.:o an ImplIcit function. For xam~le, the equation x' + yl= 1 defines y as an implicIt function of x. 6. The graph of a function. A set of poi nts (x, y) in an xy-plane, whose coordinates are connectEd by the equation y (x), is caIJed the graph of the given funct~on.

=,

1**. Prove that if a and b are real numbers then

Ilal-l b II ~ la-b I ~ lal+lb (.


2. Prove the following equalities: a) Iab I = I a 11 b I;
b)

c)
d)

I~ I= :::

(b =1= 0);

laI =a
2

2 ;

Vi2=-.:\al.

3. Solve the inequalities: a) Ix-II<3; c) 12x+ 11< I; b) Ix--t-1/>2; d) Ix-II<lx+ll. 4. Find /(-1), [(0),1(1), {(2), 1(3), {(4), if f(x)=x ' -6xl + Ilx-6. 5. Find 1(0), I( - ~), {(-x),/( ~), f~X)' if l(x)=V1+x 2

6. f (x) = arc cos (log x). Find f (~ ), f (1), 1(10). 7. The function {(x) is linear. Find this function, if /(-1)=2 and f (2) = - 3.
) Log x is the logarithm of the number x 10 the base 10.

Sec 1)

Functions

13

8. Find the rational integral function f (x) 01 degree two, if f(O)==1, /(1)=0 and f(3)=5. 9. Given that f (4) == - 2, f (5) = 6. Approximate the value of 1(4, 3) if we consider the function J (x) on the interval 4 ~ x ~ 5 linear (linear interpolation 01 a function). 10. Write the function

f (x) =

{ 0,

x, If x>O

~f x ~ 0,

as a single formula using the absolute-value sign. Determine the domains of definition of the following functions:
11. a) Y=

12.

b) Y= ~ x+ 1. Y-=4--2'
~

Vx+ 1;

16. Y=

17. Y= log ~+~


18. y==log
2

V X-Xl.
.
2x

13. a) Y = x 2 -2; 2 b) y~x x -2.

V -V
l

3 +2 x - xl . x-l-

19. y=arccos l+x'

14**. y=V2+x-x

15.

1 Y= V -x+ Y2+x'

20. Y= arc SIn log 10

X)

21. Determine the domain of definition of the function

Y= Vsin2x.

22. f(x)=2x -3x ' -5x 2 +6x-l0. Find


4

q> (x)

= 2" (I (x) -t- I (- x) ) and

'1'( x) = 2" [/ (x) - f ( - x) J.

23. A function

is called even if

Determine which of the following functions are even and which are odd: a) f (x) = ~ (a~ +a- X );
b) f (x) = VI

f (x) defined in a symmetric region - l < x < I f (- x) = I (x) and odd if f (-x) == - I (x).

+ x +x'- Vl-x +x
J ).

c) I(x)=
d)

V(x+ 1)1+ V(x-I)I;

l+x f (x) = log -1-x;

e) l(x)=log(x+Vl+x

24. Prove that any function f (x) defined in the interval - l < x < 1 may be represented in the fOfln of the SUln of an even function and an odd function.

14

Introduction to Analysis

(Ch. I

25. Prove that the product of two even functions or of two odd functions is an even fl'nctlon, and that the product of an even function by an odd function is an odd function. 26. A function f (x) IS called periodic if there exists a positive numter T (the period of the tunction) such that f (x + T) = f (x) for all valres of x within the de main of definition of f (x). Ce1ermine \\hirh of the rol1ewing functions are perIodic. and for tLe perlcdic ft:ncticns flnd tt.eir least period T:
a)

b)
c)

f (x) = 10 sin 3 x, f (x) = a sin AX + b cos AX; f (x) = Vtan x;

d)

f (x) =

sin l x;

e) / (x)

= sin (VX).

27. Express the length of the segment y.= M N and the area S of the figure AA1N as a fun~tl0n of x= AM (Fig 1). Construct the graphs of these functions. D C 28. The linear density (that is, mass per unit length) of a rod AB = I b (Fig. 2) on the segments AC = I" CD == 12 and DB == la (L \ + I" + La ==- l) A is equal to q J' q 2 and q a' respec/3

c
~--a--~

Fig. 1

Fig. 2

tively. Express the mass m of a variable segn;ent AM =X of this rod as a function of x. Construct the graph of this function. 29. Find q;'W(x)) and ;q;(x)), if <p(X)=K and 'I'(x)=2 x .
30. Find f tf If (xlI}, if f (x) = 1 ~x 31. Find f(x+ 1), if f(x-l)=x l 32. Let f (n) be the sum of n terms of an arithmetic progression.

Show that

/ (n + 3)-3/ (n

+ 2) + 3/ (n,- 1)-1 (n) = O.

33. Show that if

f (x) =kx+ b
and the numbers gression.
the nurn ters

J (Xl)'

Xl' XI'

x. form an arithmetic progression, then

/ (XI) and

f (XI) likewise form such a

pro-

Sec. J]

Functions

15

f (x) = aX (a > 0),

f (x) is an exponential function, that is, and the numters x" XI' XI form an arithmetic progression, then the numbers f (Xl)' f (x 2 ) and {(x,) form a geometric progression. 35. Let
34. Prove that if
l+x f(x)= log . i-x

Show that

f (x) + f (y) = f ( ;:~)


36. Let cp (x) = 2" (aX + a-X) and
1 1 'P (x) = 2 (aX _a-X).

Show that and

rp (x + y) = cp (x) <p (y) + 'P (x) '" (y)


'" (x

+ y) == cp (x) 'I' (y) +- cp (y) 'I' (x).


{

37. Find /(-1), {(O), f(1) if

f (x) =

arc sin x for-t ~ t" ~ 0, arctanxforO<x ':+00.

38. Determine the roots (zeros) of the region of positivity and of the region of negativity of the function y if: a) y == I -t- x; d) y = x - 3x; b) y = 2 -t- x - Xl ; 2x e) y- Iog-c) y = I-x -tXl; -

1 +x

39. F:nd the inverse of the function y if: a) y = 2x + 3; b) y=x 2 -1; .i/-l- 3 . c ) Y=v - X , d) y = log ~ ;
2

c) y=arctan3x.

In what regions will these inverse functions be defined? 40. Find the inverse of the function
I X, if x~O, y =) Xl, if x >

o.

41. Write the given functions as a series of equalities each member of which contains a simple elementary function (po\ver; exponential, trIgonometric, and the like):
a) y = (2x-5)10; b) y = 2cos x;

c) y = log tan 2 ; d) y = arc sin (3 -X2 ).

16

I_n_t ,_od_u_c_t,_on_t_o_A_n_a~ly:-.s_,s

.....:[::.-C_h_._I

42. Write as a single equation the composite functions represented as a series of equalities: 8) y=U', U= sin x; b) y=arctanu, v=logx; c) = { 2u, if u ~ 0, y 0, if u>O;

u=Vv,

tt=x'-l.
43. Write, explicitly, functions of y defined by the equations: a) arc cos y =---1(,; b) lOx + 1()Y = 10;

r-

c)

x+1YI=2y.

Find the domains of definition of the given implicit functions.


Sec. 2. Graphs of Elementary Functions
Graphs of functions Y= f (x) are l11ajnly constructed by marking a sufficiently dtln~e net of points Nli(xit I/i), where Y,=f(xj) (i==-O, 1,2, ... ) and by connecting the pOInts with a line that takes account of intermediate pOints. Calculations are best done by a ~ltde rule.

-... ... ......


. . ..........y,

...

Fig. 3
Graphs of the basic eleJnentary functions (see Ap pendix VI) are readily learned through their construction. Proceeding from the graph of

Y=f (x),

(f)

we get the graphs of the following functions by means of simple geometric construct ions: 1) YI = - f (x) is the mirror image of the graph r about the x-axis; 2) !I. = f (-I') is the mirror image of the graph r about the y-axis;

Sec. 2)

Graphs of Elementary P'J.nct,oni

17

4) Y4 = b I (x) is the r graph displaced along the y-axis by an amount" (Fig. 3). Example. Construct the graph of the function

3) Y. = f (x-a) Is the r graph displaced along th? x-axis by an amount a;

y = sin ( x - : ) .

Solution. The desired line Is a sine curve g = sin x displaced along the x-axis
to the: right by an amount

(Fig. 4)

x
Fig. 4

Construct the graphs of the


(straight I inps):

following

linear functions

44. y=kx, if k=O, I) 2,1/2, -1, -2. 45. Ij=x+ b, if b=O, 1,2, -1, -2. 46. y = 1.5'x -}- 2. Construct the graphs of rat ional integral fun.tions of degree two (parabolas). 47. y=ax 2 , if Q= 1,2, 1/2, -I, -2, O. 48. Y = x 2 + C, if c = 0, 1) 2, - 1. 49. y=(X-x o )2, ii xo=O, 1,2, -1. 50. Y = Yo + (x - 1)2, if Yo = 0, 1, 2, - I. 51*. y=axl-~-bx+c, if: I) a=l, b=-2, c=3; 2) a=-2, b=6, c=O. 52. y = 2 t- x-x2 Fi nd the points ot intersect ion of th is parabola with the x-axis. Construct the graphs of the following rational integral tunction~ of degree above two: 53*. Y = x 3 (cub ic parabola). 54. y=2+-(x-I)'. 55. y = x'- 3x -t- 2. 57. y=2x l -x4.. Construct the graphs of the following linear fractional functions (hyperbolas): 68*,
56. Y = x 4

y=+,

18

Intr,duct,on to Analysis

[eh. 1

59. y= I-x.

60. y= x+2.
2x-3 62 * Y=3x+2"

x-2

61*. y=y o -+.. x~, if xo=l, yo::::a-l, m=6. X


o

Construct the graphs of the fractional rational functions: 1 63. y=x+-. x


64. y= x

65*.

+ 1 !I = ;.
1

x2

66. y= x .

~.l~ 1 (W itch of Agnesi). 68. y = x.2~ 1 (Newton's serpentine). 1 69. y= x+ 2 x . 70. y = Xl +!. (trident of Newton). x
67*. Y =

Construct the graphs of the irrational functions: 71*. y= J/72. '1= v x. 73*. y= ViZ (Niele's parabola). 74. y= x (c;emicubical parabola).

Vi

75*. y.

f V25-x

VX-

(elltpse).

76. y=

V xZ-l
1

(hyperbola).

77. y= V 1- Xl
78*. y=

Vb
r

(cis.<;oid of Diodes).

79. y= xV 25-x 2 Conslruct the graphs of the trigonometric functions: 80*. Y = sin x. 83*. y =- cot x. 81*. y=cosx. 84*. y=sec x. 82*. y=-tanx. 85*. y=cosec x. 86. y= A sin x, if A = 1, 10, 1/2, -2. 87*. y= sin nx, if n= 1, 2, 3, 1/2. 88. y=sin(x-q, if q>=O, ~, n, - : . 89*. y = 5 sin (2x-3).

3;,

Sec. 21

Graphs of Elementary ftlnctions

19

90*. Y= a sin x+ b COS X, if Q= 6, b -= -8. 91. Y = sin x -1- cos x. 96. y = 1-2 cos x. 92*, y=cos1x. 97. g= sinx-{ sin3x. 93*. Y= x+ sin x. 94*. Y = x sin x.

98. Y= cos x+ 2 cos 2x.


n

99*. Y = cos -x . 95. y = tan! x. 100. y = sin x. Construct the graphs of the exponential and logarithmic runetions:

lOt. g=a", if a=2, ;, e(e=2, 718, .. )*).


102*. g= lo&z x, if a= 10. 2, ; , e. 103*. y=sinhx, where sinhx=lj2(eX-e-~. 104*. y=co~hx, where co~hx=1/2(ex+e-X).
105*. Y = tanh x, where tan h x = h- . cos x
1

sinh x

106. Y= lOx. J07*. Y = e- x2 (probab ility curve). 108. y=21

1 113. y=log-.
x

114. y=log(-x). 115. Y = log! (l + x). 116. y=log(cosx). 1 112. .l j = -og l- . 117. y=2- x sinx. x Construct the graphs of the inverse trigonometric lt8*. y=arc sinx. 122. y=arcsin...!...
119*. Y = arc cos x. 123. y=arccos-. x 120*. y=arctanx. 124. y=x+arccotx. 121 *. y = arc cot x. Construct the graphs of the functions: 125. Y=lxl. 1 126. y = 2 (x x I)
x I

109. y=logx 2 110. Y = logl x. lIt. y=log(logx).

functions~

+,

127. a) y=xlxl; b) y=logv7Ixl. 128. a) y=sinx+lsinxl; b) y=slnx-lsinxl. 3-X2 \vhen Ixf~ I. 129. y= { ,:, when Ixl> 1.
*) About the number e see p. 22 for more details.

20

Introduction to Anall/sis

(Ch. 1

130. a) y=[x], b) y=x-[x], where [xl is the In~egral part of the nurnLer x, that is, the grea~est in:eger less than or equal to x.

Constrt:ct the graphs of the following functions in the polar coordina.e system (r, <p) (r ~ 0):
131. r = 1.

132*. r (spiral of Archimedes). 133*. r":= e<P (Iog'lrithmic spiral). 134*. r =!:. (hyperbol ic sptral).
135. r = 2 cos 'P (cirlle). 136. r = _1- (straight line).
Sin

=!
q>

q>

137. r=sec! ~ (parabola).

CJns~ruct

13S*. r = 10 sin 3cp (three-leafed rose) 139*. r=a(l +-coscp) (a>O) (cardioid). 143*. ,1 = a l eos 2cp (a > 0) (lemniscate).

the graphs of the functions represented parametri(sernicubical parabola). y= sin t (ellipse). Y = 10 sin a t (astroid). t slnt), y=a(slnt-tcost) (involute of a

cally: 141*. x = t a, y = (' 142*. X= 10 cos t, 143*. x = 10 cos' t, 144*. x=a(cost+

circle ).
145*.
146

X=l~tal
X

Y=l~:a (folium of Descartes).


I

. . I) se"LtCtrc e . 1 t 147. x~~t +2-', y=2 _2- (branch of a hyperbola). 149. x=2co~Zt, y=2s1n 2 t (segntent of a straight line). 149. x==t-t 2 , y=t 2 __ t a 150. x==-~J2cost-co~2t), y=a(2sint-sin2t) (cardIoid). CJnstruct the graphs of the following functions defined implicitly: t 51 *.Xl + yl = 25 (c i r (l e) . 15l. xy= 12 (hyperbGla). 153*0 yJ=2x (parabola).

Ir , Y= r 1 + /2

at

I + tl

154. 100+64=1 (elllPse). 155. y! = Xl ( 100 - X


Z ).

rot

y2

156*. x ... + y"3 = a3" (astroid). 151*. x+y=-lOlogy. 158.. Xl = co~ y.

Sec. 2)

Graphs of Elementary Functions

2t

(logarlthmtc spiral). 160*. X 8 +y3_3x!J=0 (folium of Descartes). 161. Derive the conversion fornlula trom the Celsius scale (C) to the Fahrenheit scale (F) if it is known that OC corresponds to 32P and 1000e corresponds to 212F. Construct the graph of the function obtained. 162. Inscribed in a triangle (base b == 10, altitude h = 6) is a rectangle (Fig. 5). Express the area of the rectangle y as a func-tion of the base x.
159*.
A

VX -t- y" = earc tan .!.!.x


Z

'--'-----~ 8

Fig. 5

FIg 6

Construct the graph of this function and find its greatest value. 163. Given a triangle ACB with BC=a, AC=b and a variable angle ?J ACB = x (Fig. 6). Express y = area 6. ABC as a function of x. Plot the grapn of this function and find its greatest value. 164. Give a graphic solution of the equations:
b) XS + x-I = 0; c) logx=O.lx;
a) 2x"-5x-r2=O;

d) lO-x=x; e) x = I -+ 0 5 sin x; f) cot x=x (O<x<:t).

165. Solve the systems of equations graphically:


a) xy = 10, x -t y = 7; b) xy = 6, XZ y" = 13; c) x!-x+y=4, y!-2x=O; d) x" y= 10, x+ y"= 6;

e) Y= sinx. y=cosx,

(O<x<2n).

Introd'lction to Analysis

(Ch. 1

Sec. 3. Limits
XI'

to. The limit of a sequence. The number a is the limit of xJ ' . , x". . . . , or
lim xn=a, n + C1J
(8) such that

tJ

sequence

If for any e > 0 there is a number N = N Ixn-al<e when n>N.


Example 1. Show that
Ihn 2n
n-+ct)

n+l

+ 1=2.
__ 1_

(I)

Solution. Form the difference


2ft

n+1

+ 1_ 2 =

n+l'

Evaluating the absolute value of this difference, \ve have:


2n 1 1 j n + 1 - 2 = n + 1 < e,

(2)

if
n

>e

-1

=N

(e).

Thus, for every positive nUlnber e there will be a number N =.!-l such that for n > N we will have irequality (2) Consequently, the number 2 is the limit of the sequence x n = (2n-t- l)/(n + 1), henc~t formula (I) is true. 2. The limit of a function. We say that a function f (x) ...... A as x ~ a (A and a are numbers), or lim f (x):= A,
x -+a

if tor every e > 0 we have ~ = ~ (e) > 0 such that If (x)- A I < e for 0 < I x-a I < ~.
Similarly:
x

linl f (x) = A,
-+
00

if

I f (x) -

A I < e for
IS

I x I > tv (8).
also used:
CO,

The following conventional notation


x ..... a

lim

f (x) =

which means that I f (x) I > E for 0 < , x-a I < f) (E), where E i~ an arbitrary positive number 3. One-sided limits. If x < a and x -+ a, then we write conventionally x -+ a-O; s!ml1arly, if x > a and x a, then we Wrtte x a +0. The numbers f (a-O) = lim f (x) and J (a + 0) = lim f (x)
-P
-?

a -- 0

a+0

.are catted, respectively, the ltmlt on the left of the function f (x) at the point a and thf limIt on the f,ght of the function f lX) at the point a (if these nuolbers eXist).

Sec. 3)

Limits
~

23

For the existence of the linlit of a function f (X) as x and sufficient to have the follcw lng equal ity:

a, it Is necessary

f (a-O) =:-f (a+O).


I f the linlits lim f 1 (x) and lim f I (x) exist, then the following theorems.
x~a

x-.a

1old: 1) lim [f. (x)


x~a

+ f2 (x)] =

lim f. (x) + lim f2 (x);


x~a x~a

2) 3)

lim [f. (x)


x~a

'2 (xli

lim ,. (x) lim


x~a x~a

f" (x);
f 2 (x)
(lim
x~a

litn
x~a

If. (x)! f 2 (x) J = lim f.


x~a

(x)! Brn
x~a

f 2 (X) :I: 0).

The following two limits are frequently used:


lim sin x = 1
.

-+

and
1

Jim (1
X-+(;I)

+..!..)X = X

Jim (1
-+0

+ ala =e=2 71828


1

..

Example 2. Find the limits on the right and left of the functioD

f (x) =arc tanx


as x
-+

Solution. We have

o.

'(+0)= lim (arctan


and
x ~ +0 f(-O)= lim

-!..)=~ X 2
2

x-+ - 0

(aJctan..!..)=-~.
x
x

Obviously. the function f (x) in this case has no limit as

-..0.

166. Prove that as n - + 00 the linlit of the sequence


1, 4
1
t t

1 nz' ...

is equal to zero. For which values of n will we have the inequality


1
nz

<

(8 is an arbitrary positive number)?

Calculae numerically for a) 8=0.1; b) 8=0.01; c) 161. Prove that the limit of the sequence

8=O.OOl~

x"=n+l (n=1,2, ... )

24

Introduction to Analysis

(Cia. I

as n - t o 00 is unity. For which values of n> N will we have the inequality

Ixn-ll< e

(e is an arbitrary positive number)? Find N for a) 8=0.1; b) 8=0.01: c) 8=0.001. 168. Prove that

How should one choose, for a given positive number positive number 6 so that the inequality should follow from

8,

some

Ix -41< 8
l

Ix- 2 1<cS?

Compute 6 for a) 8=0.1; b) 8=0.01; c) 8=0.001. 169. Give the exact meaning of the following notations: a) liin log x = - 00; b) lim 2x = + 00; c) Iinl f (x) = 00.
x
-~

+0

...

+ 00

00

170. Find the limits of the sequences:


a) 1, -"2' 3'
2 4 6 b)

- 4-' ... ,
2n

(- I)n-l n , . .. ,

T' "3' '5' ... , 2n=1' ... ;

c)

V2. -( 2 Vf, 2 -.( 2 V'2, d) 0.2 0.23, 0.233, 0.2333, ...


1

'"

Find the limits:


( 171 Iun
n~oo

n~oo.n

1 2 3 2+2-12 n n

n-l) . + ... +-21l

172. lirn (n+l)(n~2)(n+3) .


n

173. liln
n-+oo

l!..:t3+5+7+"'+'(2n-I)_2n+IJ
n+l 2
2n + 1 +3 1Z + 1 2n +,n
~

174 I1m 175. lim 176.

n+(-I)n -(-1)". n-+oon


n~oo

}i~ ( ~ + ~ + ~ + ... + ~ ) . [ 1 I 1 ( _ 1) n - '] 177. 21~ 1-"3+ 9-27+ ... + 3"1

178. lim 1 +2 +3 : n .... < n

+n

Seco 3]

L_l_om_i_t.s

179. lim 180. htll


n

(V n + 1- Vn).
-r-+ 1 n
n "ill nl

n-fJ(J)

-+J

When seekipg the lilnit of a ratio of two integra t polynomials In x as x -+ 00, it is useful first to divide both ternlS of the ratio by x n , where n Is the highest de~ree of these l\oly~omia!so A si 1T1 ilar procedure is also possi ble in nlany cases for fractioos eontaiQfng irrational terms.
Example 1.

linl

(2x-3) (3t + 5) (4x-6)

3x'+x-1
-=-=- !ill)
t+7fJ

5)( 4 - 6) 3+(2 - 3)( x =2.3.4c= 8.


x'
X

3-~-~-~ 3
x2

Example 2o

x III I 1 x~rJ::3/~\3+10 V

=::

1 lIn x~oo

3V-TO == .
1 1

1--1->.J

181. 182.

.
.r

(x+ 1)2
OCt

hIll
~

2 _L
I-

-1

186.
t

2x~-3x-4 1 1In --;r -::---==4


~
00 ,.,

x+ooX-

lOOOx I 1:11 -.2-1


~

+1

187 1lIn
x

2.t+3

x2-5t+ 1 183. 1 1111 - 3 ..l~7


x
00

x + 00 x

-1- V x
J

188. HIn

x2

184. 11111
t -+0

2~2-x-1X

rJ::

3_8' \:

+5

-~ oc 10

+x V "t
X2

185. 11111
.t -+
00

(2x 3)3 (3x-2)! ---5---

V 189 1 lIn - - 1 - .
x~oo

- +1
r

x+

-t- 5

190.
t

Ii 111 ~ too

-V x + V x -f

Y:-==:x====~
V-;lll)

If P (x) and Q (x) arp integral polynomials and P (0) '# 0 or <J then the lilnlt of the rational fraction
lim P (x)
t

:f: 0,

~a

Q (x)
Ai&a-Q

is obtained directly. But if P (a) = Q (a) =0, then it is advisable to laJllel the binonllal

out of the fraction


Example 3.

~ ~:~
l

once or several times.

lIru
11

x -4
-

-+ ..tl

3~

+2

z::

lim (z-2) (x
. -+ 2

t- 2) =
1)

(x - 2) (x -

lim x +2 c::.ft ~ 2 x-I

Int,oduction to Analysis
x'-3x+ 2 1 195. X-+l~ 1m 4-4 x + 3 '

(Cho J

, x2_(a+ 196 IlIn 3


x ..... a

x -a

I) x+a a

197 I1m
h

-+

(X+h)'_X' h
X X

198. lim (I I - I 3 .).


x -.
I

tzed by introducing a new variableo Example 4. Find

The exrressions containing irrational terms are In many cases rational-

lim Vf+X-I .r~oVI+x_I

Solution. Putting "We have


Urn
%

I +x=y',

-+.

V" 1 + x-I
Vx-I
xYx -8
I'

Yf+X-l

Jim

y:- 1=

Y-+ly-I

Y~l

lim y!+Y+I=~. y+1 2

199o 1lIn
0

-+ J

2O1. 1 1m

X-+l

200. I1m
x
-"14

V x -4

202. 1 lIn

x-.

-I V Vx x-I Vx +1 V - (-2 -1)2


Xl
X

Another way of finding the litnit of an irrational expression is to transfer the Irrational term froln the nUlnerator to the denominator, or vice versa, froln the denominator to the numerator.
Example 5.

lim
~~Q

VX" - Va _
x-a

Um x-a -, --%-+Q(x-a)(Yx+Ya)
lim 1

%-+aVx+Va

Va

(a>

0),

203.

2- Vx-3 x2 - 4 9 ' 204. I.TI x-h x ... 8 x -2

-+,

1m

206.

x-+41-

r1m 3-Y5+x v--.


5-x

V-

VI +x- Yl-x 207. lim x


%-+0

205.

10

~-+IV..t-l

1m ~--.

Vx-I

208. lirn YX+hh,


11-+0

VX .

Sec.

~')

Limits

209. 210. 211.

h-.o

r1m
In)

v2

x+hh

Vx
2

212. lim
213. ILn
. -++

(V x (x + a)-x).
IS)

.-++ aJ

1.

y x -2t+6- y x + 2x-6
x-x+ .
2

JC~3

x-.-t aJ

iim

(V x

t a- Vx).

214.

(V x! -5x -+ 6-x). IL11 x <V x! + I-x).

~+oo

215. liln (x -t
x~eX)

V 1-

Xl).

The formula
Urn
x
-+ 0

Sitl

x= I

i~ frequently u~E'd when solvin~ the following examples. It is taken for granted that lin] Sin x = Sin a and lim cos x ==C<JS a_
-'~"

x-+a

Example 6.
JC-t> 0

lim sin 5x = liIn (c;{n 5x .5) = 1.5== 5. X x.....,. Q \ 5x


7 a) I - -I ; 22 J. lIn x sIn
x~o

216. a) lim
X -+2

si fl x ;
X

b) 1Lll
.

217. I en - - . X-+ 0 x 218 1 lin -2- .


. -.0

x -+ Cf) sin 3x
sin 5x
X

sin x -. t

b) Ij'n x sin
x.....,. eX)
X-+I

-!. . x
2

228. lim (1 -- x) tan nx 229. lLn cot


x .... o

Sill

2x cot ( ; - x) .

219. I I~n--.
.c .... l
Sill 311X

S(fl1(X

220.

liin
n-+cx>

(n sin ~). n
1- c('sx

230. lim
-.

I-sin~
2 3t-x 1- 2 C0S x 11- 3x
cos mx-cf'S nx

n
11

. 221. 1 Inl

...

231 I l~n

.t~o

x ..... -

222 1 1m - - - u

sin x-sin a

232. lim 233.

x-a

x2

223. ILIl
.1-+

cC"s x - CPs a

1 22 4 ,,_~C~2 x+ 2

x-a tan 3tX

art' sin.x 234 . 1 1:11--.


.t -+ Q

225. 1 un
22 6.

sin (x

+ h)-sin x

arc tcln 2t 23 5. I1m - - 3 - .


r ... O
4)Ul

I) -+-0

r
in]
~~-

11

sin X-CflS x i-tan x

l-x2 236 . IIrn - - .


.t --1> 1 Sll1llX

Introduction to Analysis

[Ch. I

237
,

1 1m

x-c;in 2x

:r .... 0 X

+ S1Il 3. X
1(X

239. lim
x -+0

1-

VCOS"X
x2

238.

lim

~. ~ +11x

cos-

240.

x...,. 0

liIn

YI +sinxx

Y I-sjnx

When taking limits of the form


linl (cp (x)]~
JC
(X)

= C

(3)

-+a

()ne should bear in mind that: 1) If there are final limits


X-+Q

lim cp (x) = A and lim '" (x) = B,


~-+a

then C= An; 2) if lim <p (x) = A #= 1 and lim 'l' (x)::..:


( "1>Q

00,

then the problem of finding put q> (x) = I + a (x).

X -+

(J

the linllt of (3) is 'iolved in straightforward fashion; 3) if linl q> (x) = 1 and lun 'i' (x) = 00, then we
x-+a
-+

x-+a
-+

where a (x)

0 as x

a and, hen\,.e,

c=
I

lim
l'

1 lim a (XI {[I+a(x)]a(x)~~(.~)~(X)=ex-+a

'1' (X)

lim [~(x) -IJ =~-+a

'1' (Xl

+a

where e = 2.718 ... is Napier's number.


Eample 7. Find
x-+o

sin 2x \ I+~ Urn ( - ) , X

Solution. Here,
~-+o

lim (Sin 2X) =2 and Urn (1 +x) = I;


x
x-.o

bence. 11m
%-+0

(Sin 2X)1 +x =2 =2.


1

Example 8. Find
lim
x-+ c

(X+ 1 )XJ
2x

+1

Solution. We have

1+ 1 x+l 1 11m - - = Urn --=~ -+ 2x + 1 x -+ 00 2 + l 2 x

fI:J

aod

Sec 3)

Limits

29
2

1"here{ore,

11m x-.
Example 9. Find

.J(X+I\X ---) 2x + 1
CX)

=0.

litn
% ..... 00

(X-I )'x . x+l


t--!..
x

Solution. We have

lim x-I = lim _ _ x =1 x-+CI>x+l ('~oo l+-!.

Transforming, as indicated above. \\'e have


I" 1m x-.

(X-I)~ - - = I"1m
x+1

(II

x -+ (YJ

l + (X-J -+ I

-1

)J" =
-2)1 - 2} I+x = eX l:n"" ;/,x _ e\x+1
0

X+

!X

= lim {[ I-t- ('


%-+ct'>

In this case it is ealijer to find the lilnlt without rec;orting to the general procedure:

lim
JC -+ '"

(X-l)Xo=: X+1

lim (1-+
X+ '" (

1+

' r J[I~mct 1( 1-+ f"r r + r


x

=e-l=e.' I ,

~I~

Generally, it is useful to remernber that


lim
%-+
CJ)

(l+~)x=efl
x

241. Itm 3 c.....o \ - x


%-+.

(2+X)"~

248. lim (
t ..... z.:

242. 1i~n(XI..!J)x+l.
x

249. (~i~ x+3

--;-)~ . x+

(X_')~+J

243. lim ( I X-+>D X 244 I

)-i+I .
sln%
%
2

250. lim ( 1 + n
n-+QD

_ X)n

. (Xl +2 ).t 245. :~~ 2xl + 1


246. lim ( 1 n.... QD

;~~ xZ -3x+2

(xl-2.t+3)
..!.)II n
%

251. lirn (1
-.o

+ sin x) .c
I

252**. a) lim (cos x) ~ :


%-+0
I

247 lin) (1
~QI

+ !)J& .

b) lim (cos x)iI.


~

...o

30

Introduction to Analysis

[Ch. 1

limit lim f (x) exists and is positive, then


.c-.a

When solving the problems that follow, it is useful to know that if the
Um [In f (x)] = In rUm f (x)].
x.... g

x-.a

Example 10. Prove that


x-+o

Urn In (1 +x) x

1.

(*)

Solution. We have

Formula (*) is frequently used in the solution of probleuls.

253. Iim[ln(2x+l)-ln(x+2)].
. lOR ( 1 + lOx) 254. I1m
K-+'

255.

~~ ( ~ In y: ~;) .
..... +GID
X-+O

260*. lim n (Via -1)


n-+oo

(a>O),

256. lim x (In (x+ 1) -In xl.

261. bIn
-+O
K....O

eaX_e fJX

x
1
-x
SlIl

, In (Cf'S x) 257 I1m 2


X

262. lim~.
x

258*. lim ,x_I.


" .... 0

263. a) lim ~irh x ;


%-+0

259*. lim
%-+0

aX

-=X

oX

(a

>0).

b) Iimcosh~-l
~ ... o

(see Problems 103 and 104). Find the' following limits that occur on one side:
264. a) liln
% -+ - .... OD

V ~! + 1
x

b) lim
%

OD

Vx + 1
2

.
267.

b) lim
,t-++o

I
I

. .

265. a) Ii.n tanh x;


-+-:

b) lim tanh x,
-++QD

eX _e- JI where tanh x = e -x--+e- Jt


1 +e"

1 +e In (1 +e-, . a) lim x x-+ -00 In (I + eX) b) lirn x JC-++oo


K

I ~tn x; J 268. a) rlin x-+ 0

266. a) lim - I, J ' ""'-1

b) lim 'sin x I
-.+o
X

Limits

81

269. a) 1 1m -, I I;
0

x-I
X

270. a) lim~2 ;
%-+2-0

X-+I-O

x-

x-I b) 1m x X-+I+O

I-

'--=-11
I+
x
n
X

b) 1im~2.
%-+2+0

x-

271 **. Y = lim (eOS Zn x)o


n-+~

Construct the graphs of the following functions:

272*. y=lim
n-+oo

(x~O)_

273. Y = lim
a-+o
n-+(J)

Vx-z-+-a,-z.

274. Y = lLn (arc tan nx). 275. y = lim


n-+~

V 1 +xn

(x ~ 0).

276. Transform the following mixed periodic fraction into a common fraction: a = 0.13555...

Regard it as the limit of the corresponding finite fraction. 277. What will happen to the roots of the quadratic equation
ax! + bx +c=O,

if the cotfficient a approaches zero while the coefficients band c are constant, and b#=O? 278. Find the ltmit of the interior angle of a regular n-gon as n --. 00. 279. Find the limit of the perimeters of regular n-gons inscribed in a circle of radius R and circumscribed about it as n - 00. 2E:O. Find the limit of the sum of the lengths of the ordinates of the curve y = e- x cos nx,
dra\vn at the poi nts x = 0, 1, 2. . n, as n -+ 00. ~f 1. Find the limit of the ~um of the areas of the squares constructed on the ordinates of the curve
0 .,

y=2 1 as on

inscrIbed in a logarithmic spiral


I

ba~es, where x=: 1, 2, 3, ... , n, provided that n ~ 00. 282. ~ind the limit of the perimeter of a broken line MoM I M"

-=- e-CP

Int ,oduct,on to Analysis

(Ch. 1

(as 11 - . (0), if the vertices of this broken line have, respectively, the polar angles
PI
283. A segment

=O ,

q:>1

= 2"'

... , q>tJ = 2
cHl

nn

is divided into n equal parts, isoscelf's triangle \\'lth base angles u .::~- 45. Sho\v th:lt the linlit of the perulletcr of tl1(~ hrok(,ll line thus fortned ciilTers frOl11 the len~~th of AB despite the fact that in the linllt the broken line "geornetrlcal1y nl{'r!~eS \Vlt~ the segrnent AB".
ench pnrt serving ns the bClse of

AB = a

(Fi~. 7)

Q~
Fig. 7

Fig 8

point C 2 divides a segnlent AC, in half; the point C~ divides a spglnent C 2 C, in half; the point C.. divides C:!.C 3 in half, and so on. Dctpfl11inc the linuting position of the pOInt e,. \vhen 11-'" cu. 285. The side a of a right triangle is divided into n equal

284. The point C1 divides a

sc~nlent

AB ---l in hCllf; the

pnrts, on each of \vhich is constructed an inscri bed rectangle


(rJig. 8). Deternlinc the linlit of the area of the step-like figure

thus formed if n ~ (X). 286. Find the constants k and b fronl the equation

:c~r~(kX+b-.::t: )=0.

(1)

\Vhat is the geolnetric I11eaning of (I)? 287*. A certain chemical process proceeds in such fashion that the increase in quantity of a substance during each interval of ti nle t' out of the in fi nite sequence of intervals (tT, (i -t I) i) (i ==- 0, 1, 2, ... ) is proportional to the quantity of the substance available at the comlnencement of each interval and to the length of the interval. Assuming that the quantity of substance at the initial time is QQ' deternline the quantity of substance Q~n) after the elapse of time t if the increase takes place each nth part of the time interval T = !.n Find Q. = li'n Q~n).

n-..

Sec.

41

Infinitely Small and Large Quantities

33

Sec. 4. Infinitely Small and Large Quantities


1. Infinitely small quantities (Infinlteslmals). If

lin1 a (x) = 0,
x-+a

Le., if

Ia
00.

(x)

as x

infinitesimal as x
~

1< e when 0 < I t-a I <


---+

a.

{j (e), then the function a (x) is an In 51ltlilar fashion we define the infinitesImal a (x)

The sum and product of a linlited number of infinitesimals as x ---. a are also infinitesimals as x -+a. If a (x) and p (x) are infinlteslnlals as x --+ a and
liJn a (x)_C
x-+a

P(xJ -

where C is sonle number diff~rent frolll zero, then the functions a (x) and P(x) are called infinitesimals of the same order; but if C =0, then we say that the function a (x) is an in fin ttesitnal of higher order than ~ (x). The function u (x) is called an in{initesit1lal of order t1 cotnpared with the function p (x) if Ihn where 0 < J C 1< If

+ 00.
and

,"~a

IP (x)]n
(x) -

a (x)

=c

lim a (x)_1
,"-+Q ~

,
equit'alent

are called a (x) - p (x). For exan1ple. for x --. 0 we have


(x)

then the function" a

P(x)

functions as x -+- a:

sin x-x; tan x-x; In (1 +x)-x and so forth. The sUln of t\\10 infinitesimals of different orders is equivalent to the tenn whose order is lo\\'er. The Unlit of a ratio of two infinitesimals remains unchanged if the tenns of the ratio are replaced by equivalent quantities. By virtue of this theoreln, when taking the limit of a fraction
lim a (x)
~-+a ~

(x) ,

where a (x) - + 0 and P(x) ~ 0 as x --+ a, we can subtract fronl (or add to) the numerator or denominator infinitesitnals of higher orders chosen so that the resultant quantities should be equivalent to the original quantities. Example 1. ~2(~ . 1 lIn1 11m - - = %-+0 In (1 + 2x) x-+o 2x 2

. Vii

VX"'

2. Inftnltely large quantities (InOnltes). If for an arbitrarily large number N there exists a ~ (N) such that when 0 < I x-a I < ~ (N) we have the inequality If (x) I> Nt

then the function


2-1900

f (x)

is called an infinite as x --+ Q.

34

Introduction to Analysis

[Ch. 1

The definition of an infinite f (x) as x ~ Q() is analogous. As in the case of infinitesimals, we introduce the concept of infinites of different orders.

288. Prove that the function

f (x) =
is an infinitesimal as x ~ quality
00.

sin x x

For what values of x is the ine-

If (x) 1<8

fulfilled if 8 is an arbitrary number? Calculate for: a) 8 = 0.1; b) 8=0.01; c) 8=-0.001. 289. Prove that the function

f (x) =
is an infinitesimal for quality
x~

i-Xl

1. For what values of x is the ine-

I f (x) 1<8

fuliilled if 8 is an arbitrary positive number? Calculate numerically for: a) 8=0.1; b) 8=0.01; c) 8=0.001. 290. Prove that the function

f (x) = x-2
is an infinite for x ~ 2. In what neighbourhoods of Ix - 21 < 6 is the inequality I f(x) I>N fulfilled if N is an arbitrary positive number? Find 6 if a) N = 10; b) N = 100; , o c) N = 1000. A ",.. ...----.,;~ 291. Determine the order of smallness B of: a) the surface of a sphere, b) the volume of a sphere if the radius of the sphere r is an infinitesimal of order one. What will the orders be of the radius of the sphere and the volume of the sphere with respect to its surface? 292. Let the central angle a of a cirFig. 9 cular sector ABO (Fig. 9) with radius R tend to zero. Determine the orders of the infinitesimals relative to the infinitesimal a: a) of the chord AB; b) of the line CD; c) of the area of d ABD.

Sec. 4]

Infinitely Small and Large Quantities

35

293. For x---+ 0 determine the orders of smallness relative to x of the functions: 2x d) 1- cos x; a) T+X; e) tan x- sin x.
b)
c)

Vx+ Vx;
Vii-Vi';

294. Prove that the length of an infinitesimal arc of a circle


of constant radius is equivalent to the length of its chord.

295. Can we say that an infinit~simally small segment and an infinitesimally small semicircle constructed on this segment as a diameter are equivalent? Using the theorem of the ratio of two infinitesimals, find 298 1 ~ 296 I sin 3xsin 5x
x-.o 1m (x-x1)2
S1n ..

X-+l

1m I -x

297. lim

arc

x
I-Xl

299. lim CO~ x-cos2x


%-.0

-cos X

x~0

1n (1 -- x)

300. Prove that when x~O the quantities and VI +x~i are equivalent. Using this result, demonstrate that when I x t is small we have the approximate equality
VI+x~I+;

-i

(I)

Applying formula (1), approximate the following:


a) Vl.06; b) VO.97; c) VI0; d) V120

and compare the val ues obtained with tabular data. 301. Prove that when x --+ 0 we have the following approximate equalities accurate to terms of order Xl:
a) _1_ ~
l+x
l

b)

I-x' x a -t- x~a + 2a

(a

> 0);

c) (1 +x)n~1 +nx (n is a positive integer); d) log (1 + x) = Mx, where M=10ge=O.43429... Using these formulas, approximate:
1) 1.02; 2) 0.97; 3) 105; 4)
1 1 1

V15;

5) 1.04'; 6) 0.93 4 ; 7) log 1.1-

Compare the values obtained with tabular data.


2*

36

Introduction to Analysis

ICh. 1

302. Show that for x ~ 00 the rational integral function p (x) = aox rl + a1xn - I -~ +an (ao::;bO) is an infinitely large quantity equivalent to the term of highest degree aoxn 303. Let X--'(X). Taking x to bean infinite of the first order, determine the order of growth of the functions: a) x!-lOOx-l,OOO;
Xl

c)Vx+VX;
d)

b)

x,-2 ;

V x-2x

Sec. 6. Continuity of Functions


1. Definition of continuity. A function f (x) is continuous when X= 6 (or ccat the point ~"), if: 1) this function is defined at the point ~, that is, there exists a number f (~); 2) there exists a finite limit Urn f (x); 3) this lim-

it is equal to the value of the function at the point

~,

Le.,

x-..;

Putting

X-+;

lim f (x) = f (~).


x=~+L\~,

(1)

where L\~ ---+- 0, condition (1) may be rewritten as lim L1f (~) = lim (f (~+ L\~)-f (~)) = 0,
d;-+O
~;-+o

(2)

or the function f (x) is continuous at the point ~ if (and only if) at this point to an infinitesimal increment in the argument there corresponds an infinitesi.. mal increment in the function. If a function is continuous at every point of some region (interval, etc.), then it is said to be continuous in this region. Example 1. Prove that the function

y=sinx ts continuous for every value of the argument x.


Solution. We have Ay = sin Since
Silly
dX-+O

(x + Ax) -

sin

2 sin

~x cos ( x+ ~x) = Sl:x2


"2

L1x cos (

x+ ~x) . Ax.

L\x

lim - - = 1 and Icos(x+


L\x

L\2X)I~I,

it fol lows that for any x we have lim /).y=O.


dX-+O

Hence, the function sin x is continlJOUS when - oo<x<

+ co.

Sec. 5J

Continuity of Functions

37

a dIscontinuity lat x=xo (or at the point xo) within the domain of definition
of the function or on the boundary of this domain if continuity of the function at this point.
Example 2. The lunct'ion
th~re

2. Points of discontinuity of a function. We say that a function

f (x) has

is a break in the
is discontinuous

f (x) = (1

1x)t

(Fig.

10 a)

when x= 1. This function is not defined at the point x= 1, and no matter

y
y=(x)
2
I __ ~
I
---~

I I

2 (b)

(a)

-I

(c)
FIg. 10

how we choose the number f (1). the redefined function f (x) will not be continuous for x = 1. If the function f (x) has finite limits: 11m f (x) = f (xo-D) and lim f (x) = f (xo D), X-+Xo - 0 X-+Xo +0

and not all three numbers f (xo)' f (xo-D), f (xo + D) are equal, then Xo is called a discontinuity of the first kind. In particular, if

f (xo-O)=f (xo+O).
then
X o is called a removable discont inuit y. For continuity of a function f (x) at a point xo, it is necessary and sufficient that

38

(CIl. 1

Example 3. The function

f (x) = ~I: ~
x-++o

has a discontinuity of the first kind

at x=O. Indeed, here,

f (+ 0) = lim
and

sin x = x

+1

'(-0)= lim sin x = - t .


.1-+-0

-x

Example 4. The ~unction y = E (x), where E (x) denotes the integral part of the number x [i.e., E (x) is an integer that satisfies the equal ity x = E(x) q.

where O<;q

Indeed, if n is an integer, then E (n-O):.= n-I and E (n 0) = n. At an other points this funct ion is, obviously, continuous. Discont inuities of a function that are not of the first kind are called
discont inuities of the second kind. Infinite discontinuities also belong to discontinuities of the second

I, 2, ... , and all the discontinuities are of the first kind.

<

I}, is discontinuous (Fig.

tOb) at every integral point: x=O,

kind. These ar.e points Xo such that at least one of the one-sided limits, f (xo-O) or f (xo+O), is equal to Q() (see Example 2).

Example 5. The function y=cos ~ (Fig. 10c) at the point x=O has a x discontinuity of the second kind, since both one-sided limits are nonexistent here:
%-+-0

lim cos ~
X

and

x-++o

lim cos ~
X

3. Properties of continuous functions. When testing functions for conti-

nu ity, bear in mind the following theorems: I) the sum and product of a limited number of functions continuous in some region is a function that is continuous in this region; 2) the quotient of two functions continuous in some region is a continuous function for all values of the argument of this region that do not make the divisor zero; 3) if a function f (x) is continuous in an interval (a, b), and a set of its values is cont~fned in the interval (A, B), and a function cp (x) is continuous in (A, B), then the cOlnposite function cp (f (x)] is continuous in (a, b). A function f (x) continuous in an interval [a, b) has the following properties: 1) f (x) is bOl'fnded on [a, b), Le., there is some number M such that If(x) f<M when a~x<;b; 2) f (x) has a minimum ann a maximum value on [a, b); 3) l (x) takes on all interrrlediate values between the two given values; ~hat is, if f (a) = A and f (~) = B (a ~ a < ~ ~ b), then no matter what the number C between A and B, there will be at least one valuex=v (a<v<~) 6uch that f (V)::::IC. In particular, if l(a) f (~)<O, then the equation
[(x) =0

has at least one real root in the interval (a,

~).

304. Show that the function y = of the argument x.

Xl

is continuous for any value

Sec. 5)

Continuity of Functions

39

305. Prove that the rational integral tunction p (x) = aox n + a1xn - 1 + ... -t- a n is continuous for any value of x. 306. Prove that the rational fractional function
aox n+ a.x n-l+ . . . + an boxm + b.x m- 1 + b rn

R( )X -

+ ...

is cont in uous for all val ues of x except those that make the denominator zero. 307*. Prove that the function y= Vi is continuous for x ~O. 308. Prove that if the function f (x) is continuous and nonnegative in the interval (a, b), then the function F (x) = V! (x) is likewise continuous in this interval. 309*. Prove that the function y == cos x is continuous for any x. 310. For what values of x are the functions a) tan x and b) cot x continuous? 311 *. Show that the function y = I x I is continuous. Plot the graph of this function. 312. Prove that the absolute value of a continuous function is a continuous function. 313. A function is defined by the formulas

f (x) ==

How should one choose the val ue of the function A = f (2) so that the thus redefined function f (x) is continuous for x = 2? Plot the graph of the function y = f (x). 314. The right side of the equation

" t

x2-4 --2 x-

for X=F 2, for x=2.

f (x) =

I-x sin-!.x

is meaningless for x=O. How should one choose the value {(O) so that f (x) is continuous for x == O? 315. The function

f (x) = arc tan x-2


is meaningless for x = 2. Is it possi ble to define the val ue of f(2) in such a way that the redefined function should be continuous for x = 2?

40

Introduction to Analysis

(Ch. 1

50

316. The function f (x) is not defined for x = O. Define f (0) that f (x) is continuous for x= 0, if:
a) f(x)

= (1 +x)n-l x

(n

is a positive integer);

b) f(x)= I-XC1osx ;

c)
d)

f (x) = In (I +X)-; III (I-x) _ ; f (x) =


eX _e- X
x
2

e)

f (x) = x sin 1.; x f) f (x) = x cot x.


Investigate the following functions for continuity:
1

317.

Y=XX 2 .
Y= 1 +x
1 +x'

324. y=lnltanil.

318.
319.

325. y=arc tan

1 x

Y=

V~;3
x

326. y=(1 +x)arctan I I x

320.

y= iXl

327. y = e X+l 328. Y= e


329. Y=
-i2
1

321. a) y= sin -; ;
b) y=

x sin ~. x

- - - J-

322.

x . Y= -:-.stn x
X2.

J +e 1 -

323. y = In (cos x). 330. y= {

2x+l

for x ~ 3, for x>3.

Plot the graph of this function.

331. Prove that the Dirichlet function X(x), which is zero for irrational x and unity for rational x, is discontinuous for every value of x. Investigate the following functions for continuity and construct their graphs:

332. y= lim 1+1


n-+<
n~CD

(x ~ 0).

333. y == lim (x arc tan nx).

S~c.

5]

Continuity of Functions

41

334. a) y = sgn x, b) y = x sgn x, c) y == sgn (sin x), where the function sgn x is defined by the formulas: + 1, if x >0, sgnx= O,~fx=O, { -I, If x<o. 335. a) y=x-E(x), b) y=xE(x), where E (x) is the integral part of the number x. 336. Give an example to show that the sum of two discontinuous functions may be a continuous function. 337*. Let a be a regular positive fraction tending to zero (O<a< I). Can we put the limit of a into the equality E (I + a) = E (l-Q) + 1, which is true for all values of a? 338. Show that the equation x'-3x.+ 1 =0 has a real root in the interval (1,2). Approximate this root. 339. Prove that any pol ynomial P (x) of odd power has at least one real root. 340. Prove that the equation tan x=x has an infinite number of real roots.

Chapter II

DI FFERENTIATION OF FUNCTIONS

Sec. 1. Calculating Derivatives Directly


1. Increment of the argument and increment of the function. If x and Xl are values of the argurnent x, and Y=f (x) and Yl =f (Xl) are corresponding values of the function y = f (x). then

Ax=xl-x
is called the increment of the argument
X

in the interval (x, Xl)' and

/1Y=YI-Y

or

AY=f (x 1 )-f (x) = f (x

+ Ax) -f (x)

\)

Fig. 11

Is called the increment of the function y in the same interval (x, xJ) (Fig. 1t, where l\x=l.1A and Ay=AN). The ratio

:~=tana
js the slope of the secant MN of the graph of the function Y= f (x) (Fig. 11) and is called the mean rate of change -of the function y over the interval (x, x+ Ax). Example 1. For the function

y=x l -5x+6

Sec. 1]

Calculating Derivatives Directly

43

calculate ~x and ~y, corresponding to a change in the argument: a} from x= 1 to x= 1.1; b) fromx=3 to x=2.
Solution. We have

Ax= 1.1-1=0.1, = (1.1 2 -5.1.1 + 6)-(1 2 -5.1 + 6) = - 0.29; b} Ax=2-3 = - 1, t,.y = (2:1-5 2 + 6) - (31 -5.3 + 6) :::::0.
a}

Ay

1 Example 2. In the case of the hyperbola y= - . find the slope of the

secant passing through the points M ( 3,

"3, 1)
and

t ) . and N (10, 10

Solution. Here, fiy 1 k= lix=- 30

fix

= 10-3=7

fiY=10-a=-M.

Hence,

2. Tbe derivative. The derivative y' = ~ of a function y = f (x) with reo

spect to the argument

JC

is the limit of the ratio

~~ when

Ax approaches zero;

that is.

The magnitude of the derivative yields the slope of the graph of the function y = f (x) at the point x (Fig. 11): y' =tan lp.

tan~ent

MT fo the

Finding the derivative y' is usually called diOerentiallon of the functlon. The deriv at ive y' = f' (x) is the rate of change of the functton at the point x. Example 3. Find the derivative of the function
y=x l

Solution. From formula (I) we have


fig = (x

+ fiX)t-xl =-2xAx4- (Ax)'


fiy ~x = 2x+ fix.

and

Hence,
y'= 11m liy= lim (2x+Ax)=2..,. ~x -+ 0 Ax 6.x -+ 0

ae
and

One-sided derivatives. The expressions

t'- (x) =
I'
+

lim

f (x+ Ax)-f (x)


fix

6.%-+-0

(x) = 11m AX ~+Q

f (x+ fix)-f (x)


~x

44

Differentiation of Function"s

[Ch. 2

f (x) at the point

are called, respectively, the left-hand or rtght-hand derivative of the function x. For f' (x) to exist, it is necessary and sufficient that

t'- (x) = f~ (x).


Example 4 Find f~ (0) and f~ (0) of the function

f (x)

=,

x I.

Solution. By the definition we have

f~(O)= lim lL\xl=_l,


L\x -+ -0

Ax

f~
4
0

(0) =

lim
L\x -+ +0

Il:!xl= 1.
I:!x

Infinite derivative. If at some point we have


lim
AX-+- 0

f (x+ I:!x)-f (x) =


~x

ClO,

then we say that the continuous function f (x) has an infinite derivative at x. In this case, the tangent to the graph of the function y = f (x) is perpendicular to the x-axis. Example 5. Pi nd I' (0) of the funct ion
y=

Solution. We have
L\x -+ 0

Vi:
L\x -+ 0

f' (0)= lim -A-= lim


uX

Vf:j.x

1 V-=oo. L\x
2

341. Find the increment of the function y == x 2 that corresponds to a change in argument: a) from x = I to Xl = 2; b) from x = I to Xl = 1.1; c) from x = I to Xl == I + h. 342. Find ~y of the function y ~ a) x=O, ~x~O.OOl; b) x=8, L\x==-9; c) x=a, Ilx=h. 343. Why can increment 6.y if Ax = 5, while for 344. Find the tions:

Vx if:

we, for the function y = 2x + 3, determine the all we know is the corresponding increlnent the function y = Xl this cannot be done? increment tJ.y and the ratio ~y for the funcx

a)

Y=(X t _2)2

b) y==

c) y = log x

Vi

forx=l forx=O for x= 100,000

and~x=0.4;

and ~x==O.OOOI; and t!.x = - 90,000.

Sec, 1]

Calculating Derivatives Directly

45

345. Find Ay and ~~ which correspond to a change in argument from x to x-~ ~x for the functions:
a)y=ax+b;
b) y === x' ;

d)y=Vx;

c) Y == ~ 2

e) y = 2x ; f) y = In x.

346. Find the slope of the secant to the parabola y=2x-x 2 ,


jf the abscissas of the points of intersection are equal:
a)x,~I,xl=2;

c)

b) xl==l, x 2 =0.9; Xl ~ 1, x t = I + h.

To what limit does the slope of the secant tend in the latter case if h ~O? 347. What is the mean rate of change of the function y = Xl in the interval 1 ~ x ~ 4? 348. I'he law of motion of a point is s = 2t 2 + 3t + 5, where the distance s is given in centimetres and the time t is in seconds. What is the average velocity of the point over the interval of time fron] t=1 to t=5? 349. Find the mean rise of the curve y = 2x in the interval
1~x~5.

lx, x-t-l\x].

350. Find the mean rise of the curve y = f (x) in the interval

351. What is to be understood by the rise of the curve y = f (x) at a given point x? 352. Define: a) the tnean rate of rotation; b) the instantaneous rate of rotation. 353. A hot body placed in a medium of lower temperature cools ott. What is to be understood by: a) the mean rate of cooling; b) the rate of cooling at a given instant? 354. What is to be understood by the rate of reaction of a su bstance in a chemical reaction? 355. Let In = f (x) be the mass of a non-homogeneous rod over the interval (0, Xl. What is to be understood by: a) the Inean linear density of the rod on the interval [x, x + l\x]; b) the linear density of the rod at a point x? 356. Find the ratio ~~ of the function y = at the point x=2, if: a) l\x=l; b) l\x=O.l; c) l\x=O.Ol.Whatisthederivative y' when x~: 2?

46

DitJerentiation of Functions

[Ch.2

357**. Find the derivative of the function y = tan x. 358. Find y' = lim ~ of the functions:
~X-+ 0

a) y = Xl;
b) Y= z; x
1

c)

y=Vx;

d) y = cot x.

359. Calculate f' (8), if f (x) = x. 360. Find f' (0), f' (1), f' (2), if t(x)= x(x-I)I(x-2)1. 361. At what points does the derivative of the function f (x) = x' coincide numerically with the value of the function itself, that is, f (x) = I' (x)? 362. The law of motion of a point is s = 5t l , where the dis.. tance s is in metres and the time t is in seconds. Find the speed at t = 3. 363. Find the slope of the tangent to the curve Y= O.Ix' drawn at a point with abscissa x = 2. 364. Find the slope of the tangent to the curve y = sin x at the point (n, 0). 365. Find the value of the derivative of the function f (x) = ~ x at the point x = X o (x o =/= 0). 366*. What are the slopes of the tangents to the curves y =.!. x and y = Xl at the point of their intersection? Find the angle between these tangents. 367**. Show that the following functions do not have finite derivatives at the indicated points: a) y= V Xl b)y= x-I c) Y = I cos xl

V-

V-

at x=O; at x = I; 2k+l at x=--rn, k=O, I, 2, ..

Sec. 2. Tabular Differentiation


v=

'i' (x) are functions that have derivatives, then


1) (c)' =0;
5) (uv)' = u'v+ v'u;

1. Basic rules for finding a derivative. If c is a constant and u = q> (x),

2) (x)' = 1;
3) (u

(!!..)'
V
I

VU' -v'U Vi

(v ,t: 0);
(v

v)'=u' v';

7) (

~ )' =

~~'

~ 0).

4) (cu)' =cu';

Sec. 2]

Tabular DltJerentiation

47

2. Table of derivatives of basic functions I. (x n )' = nxn - I

,I (x 2 fr x III. (sin x)' = cos x. IV. (cos x)' = - sin x.


_ V. (tan x)' = _1 1

II. (yX)' =

>

0).

cos x
-1

VI. (cot x)' = sin -.-1. x


VII. (arc sin x)' =
VIII. (arc cos x)'

y I-xl 1 (I x 1< 1).

y -1 (I x I < I-Xl
= 1+x
-I

I).

IX. (cue

lijl

x)

X. (arc cot x)' = x2+ 1 XI. (aX)' = aX In a. XII. (ex)' = eX.

XIII. (1" x)' =-!.


x

(x

> 0). > 0,


a

X IV. (loga x)' = -1-- = loga e (x x "a x XV. (sinh x)' = cosh x. XVI. (cosh x)' = sinh x.

> 0).

XVII. (tanh X)'=-!.-h cos 2 x


XVIII.
(cothx)'=~hl .

-1

sIn

XIX. (Mcsinhx)'= XX. (arc cosh x)'

~. 1 +x

x2 -1
1

( I x, > 1).

XX I. (arc tanh x)' == 1-(I x I < I). -x1 XXII. (arc coth x)' = xl-I (I x r > 1). 3. Rule for differentiating a composite function. If y=f(u) and u=q>(x). that is, y = f [q> (x)], where the functions y and u have derivatives, then Y~=Yu U x (1) or in other notations
dy_dy du

-1

dx- du dx This rule extends to a series of any finite number of differentiable functions.

48

D'Uerentiation of Functions
Example l. Find the derivative of the function

[Ch.2

Y= (xl

2x+ 3)5.

have

Solution. Putting y=u', where u=(xl -2x+3), by formula (1) we will

y' = (U5)~ (x 2 _ 2x + 3)~ = 5u 4 (2x - 2) = 10 (x-I) (x 2 -2x + 3)'. Example 2. Find the derivative of the function y=sin' 4x.
Solution. Putting

y=u';

u=sinv;

v=4x,

we find
y' = 3u 2 cos tJ4 = 12 sin:! 4x cos 4x.

Find the derivatives of the following functions (the rule for differentiating a composite function is not used in problems
368-408).

A. Algebraic Functions
368. y=x' -4x'

+ 2x-3.

375. y=3x3 -2x t 376*. Y = 377.


Xl
l .

-t-X-'.

369. y=i-370. y=ax 2

~ x+r-O.5x.

+ bx+c.
3

Vx a b y= Vx - V- x
2

-5x 371. y =a -.

378. Y=c+dx.
IR

a+bx

372 Y = at m

373. Y=

Va

+ bt . ax'+b
2

n +

379. y=x 2 -Sx+S . 380. y=2x-l-x.


_I + Vi 381 Y.r1- f Z
/I

2x+3 2

+1J2'

374. y=~~ln2. x

B. Inverse Circular and Trigonometric Functions 382. y = 5 sin x+ 3 cos x. 386. y= arc tan x + arc cot x. 383. y=tanx-cotx. 387. y=xcotx.
384. Y = sin x+ cos x
sin x--cos x

388 Y =

arc sin x.
2 )

385. y=-2t sin t-W-2) cost.

389. y= (I +x

ar; tan x-x.

Sec. 2]

Tabular DiOerentlation

49

c.

Exponential and Logarithmic Functions 396. Y = eX arc sin x.


397. Y=-l nx
x2
Xl

390. y==x 7 .eX


391. y=(x-l)e".
eX
Xl

392. Y=i. x
393. Y=-x. e

398. y=x Inx- 3 .


1nx 399. y=.!.+21nx_ . x x 400. y=lnxlogx-lna log a x.

394. f (x) == eX cos x. 39ii. Y= (x l --2x + 2)e".

D. Hyperbolic and Inverse Hyperbolic Functions


401. y=x51nhx. 402. y=-h-. cos x
Xl

405. Y = arc tan x- arc tanh x.

406. y = arc sin x arc 51 nh x.


407 408

403. Y= tanh x-x.


404

y Y

_ arc cosh x
x

=3cothx lnx

= arc coth x

I-x!'

E. Composite Functions
In problems 409 to 466, use the rule for differentiating a C0111posite function with one intennediate argument. Find the derivatives of the following [unctions:

409**. y=(1+3x--5x2 )'o.


Solution. Denote 1

+ 3x-5x! =

u; then y = ulO We have:


u~=3-10x;

y~=30ul8;

lI~ ~=30u28.(3-10x)=30(1 +3x-5x 2 )18.(3-IOx).

410. Y=

(UX/b)'.
24(2~-l)'
40 (2x -1)1

411. f(y)=(2a+3by)l.

412. y=(3+2xl )4.


413. Y=56(2:-l)7

414. y= Vf=XI. 415. y= Va+bx' . 416. y=(a2/1_x3/1)'/a.

50

DifJerenttation of Functions

[Ch. 2

417. y=(3-2sinx)'.
Solution. y' = 5 (3-2 sin x)' (3-2 sin x)' = 5 (3-2 sin x)' (- 2 cos x) == - 10 cos x (3-2 sin x). 418.

419. Y= Vcotx421 *.
X

y=tanx- ~ tan1x+ ~ tan'x.

Vcota.

1. 423. Y= - I -_ a

420. Y= 2x + 5 cos' x.

424. Y=

3 cos x cos X , 13 sin x-2 cos x

= cosec t + sec t.
1

425. Y= Vsin! x+_

cos x

1 _,

422. !(x)=-6(1-3Cosx)l.

426. Y= VI +arc sinx. 427. Y = Varc tan x- (arc sin x)'. 428. 429. 430. 431.

y arc tan x Y= V xe" +x. Y= V2e"-2" + I + In l x. y=sin3x+cos ~ +tanVx.

Solution. y'=cos3x(3x)'-sin
1 . x + 1 ...... -5 sIn -5 - .. ----r---'1-r- 2 ,. X cost ,. X

~(i)' +cos.lyx(YX)'=3COS3X-

432. Y= sin (x l -5x+ 1) + tan ~ x 433. {(Xl = cos (ax + P). 434. f (t) = sin t sin (t + q. 435. y = 436. 437.

~ ~~~: ~ .

f (x) = a cot!-. a 1 1 Y = - 20 cos (5xl ) - 4 cos Xl.


YI _(2X)1 y= arc sin ~ . x
1 (2x)' = 2 . Vl-4x 2

438. Y = arc sin 2x.


Solution. y'

439. 440.

441. Y = arc tan..!.. . x


442. Y = arc cot l
l

f (x) = arc cos Vi.

-x

+x

Sec. 2]

Tabular DitJerentiation

51

443. Y= 5e- xa 444. 445. 446.


452.
1

453.
454.

447. Y= arc cos eX. 448. y=In(2x+7). Y= . sX 449. Y = log sin x. y=x"IO"x. 450. y=In(l-xl ). f(t)=tsin2 t 451. y=Inlx-ln(Inx). y = In (eX + 5 sin x- 4 arc sin x). y = arc tan (In x) + In (arc tan x). y= Vln x+ I + In I).

evx+

F. Miscellaneous Functions
115** Y = sin 5x cos " '3. x 4u
11 4 456. y=-2{x-2)I-x-2.
15

457. y=- 4 (X-3)4


458. y = 8 (1 -Xl).

10 3 (X-3)1

1 2{x-3)1-

- 2x l -2x+ 1 459 yx

460. Y= 461. Y= 462. Y=


463.

al
3

x y-_. a -1- x
2 2

Y (1 -}-

Xl

A 2)1

~ VXi+~xVX+~ x V?+l~X- Vi
4

Y=~ V (l

464. Y=3 x+2 4 465. Y = x (a - 2X S )1


466. y= ( a-bx n 9 3 467. Y = 5 {x+2)S- (x + 2) 468. y
a +bXn)",

VX-l

+ XS)I- ~ V (l +X)I.

+ (x+ 2)' -2" {X+2)1

469. 470.

= (a + x) Vli=X. y = V(x+ a) (x-t- b) (x+ c).

z=Vy+Vy.

471. f(t)=(2t+l)(3t+2)V3t+2.

62
1

D"Oerentiation of Functions

[Chi 2

472. x= ..yr~' 2ay- y l 473. y = In (VI +ex-I)-ln (VI +ex 474. y = 1~ cos' x (3 cos' x-5). _ (tan! x-l)(tan 4 x + 10 tan 2 x +]) 475 Y 3 tan' x 476. Y = tanS 5x.

+ I).

485. y=arc sln~.

x2 -1

477. Y = ~ sin (x').


478. y = sin l (t 3 ),

479. Y = 3 sin x cos x + sin' x.


480. y="3 tan 3 x-tanx+x. 481. y = - .3 a +-3 cotx. SlIl X
cos x
4

482. y= Va sin I x+ Pcos l 483. Y = arc sin Xl + arc cos

x.

x.

491. y=arcsin(1-x)+V2x-r.

484. y=}(arcsinx)'arccosx.
492. y=

1) arcsin 1/-1V ,x- 2 v x+ 2 x-x'. (


x sin a

493. y = 1n (arc sin 5x). 494. y = arc sin (In x).


495. y =r.:: arc tan 1 -xcosa 496. y = ~ arc tan
2

x 5tan 2 +4
3

497. y=3b'arctan

498. y=- V2 arc cot


499. y= 500. y = es1n2 x,

Yb

x-(3b +2x) Vbx-x'.

tanx

V2- x .

Ve

ax

501. F(x)=(2ma mx +b)p. 502. F(t)=eCltcos~t. 503 = (a sin ~x - p cos ~x) e'J.x y a2 -1- ~2

Sec. 2]

Tabular DiOerentiation
X

53

504. Y = 110 r

(3 sin 3x-cos 3x). 507. Y = 3cot

+.

505. y=xna- 506. y = V cos xaYC"Oi'X.


510. y=x-2Vx+2In(1-l-VX) 511. y=In(a-t-x-1-V2ax+x l ). 1 512. Y=~l nI x
x-I 5 13. Y = I n cos -x- .

xi

508. y=In(ax!+bx-J-c). 509. y === In (x -1- Val + Xl).

514* y=In(x-2)S

515 y -I n
516

(x 1)' (X-I)3(X-2) -~----..;.

x-3

Y- -

2 sin! x

+ n anx.

I t

517.

y= ~ Vxl-a l _

a ;

In(x+ Vxl-a l ),

518. Y= In In (3-2x'). 519. Y = 51n s (ax + b).

520. y=ln
2

y~+x. y x -i-a! -x
2

52 I. Y = ~ In (x 2 - at) -l- ~ In ~ .
2a x+a

522.

y=x.sin(lnx-~).
1

523. Y = -2 In tan -2 --2 sin --2- x

1 cos x

524. f (x)

=
I

V7tl -In 1+ YX2+1 , x


x 2 -2x -+-1

525. Y=3 1n .\I-+-x+l


526. Y = 2arc sin IX -1- (1- arc cos 3X)I.
527. Y =
sin ax

3cOS b~ +

~ ~~:.~ .
2-1-

tan ~+ 2- Y3 528. y===~ln __ 2_---

Va

tan~+
2

Y3
2

529. Y = arc tan In x.

530. y= In arc

sinx+~ In x-t arc sin Inx.


1

531. y = arc tan In -. x 532. y

-3- arc tan

Y2

y'2' + (fIn X'D

x-I

54

DiOerentiation of Functions

[Ch.2

533. Y= In
3

1+ V' VSinx V-.+2 are tan sm x.


1Xl

534. Y=4 1n xl _1 +41nx+l+"2arctanx.


I 1 I I tan 2x -I 535. f (x)=2"ln(l +x)-6 n(xI -x+ I )+ (:rare Y3

+I

sin x

x-I

536.

f (x) =

VIC sin x + In V I-xl.


i-Xl

037. Y = sinh' 2x. 538. y = erJx cosh px. 539. y = tanh' 2x. 540. y = In sinh 2x. 541. y = arc sinh 547. Y =
x
2

542. y = arc cosh In x. 543. y = arc tanh (tan x). 544. y = arc coth (sec x). 545. y = arc tanh I ~XI
1

al'

546. Y= 2 (xl-I) arc tanh

x+ 2 x.

(~

Xl
J

+ ~ ) arc sinh x- ~ x V I + Xl
if:

048. Find y'


a) Y= Ix I; b) y=xlxl.

Construct the graphs of the functions y and y' . .549. Find y' if y=lnlxl (x*O). 550. Fin d f' (x) if f (x) = { 1- x for x ~ 0, e- x for x>O. 551. Calculate I' (0) if , (x) = e- x cos 3x. Solution. l' (x) =e- X (-3 sin 3x)-e- X cos 3x; l' (0) = eO (-3 sin O)-eO cos 0 =-1 .

..

552. f (x) = In (l +x) + arc sin ~.


553. Y = tan' ';;. Find
a)
b)

(:~

t=,

Find f' (I).

554. Find t+(O) and ''-(0) of the functions:

f (x) = V sin (Xl) ; -x a a +x f (x) = arc sin


2 l

d)
l 2 ;

f (x) = Xl sin..!. , x

x~O;
%

e) f (x) = x sin

x:

=F 0;

f (0) = 0; f (0) = 0

c) f(x) =_x, 1
1 +e ~
\-

x=t=O; 1(0)=0;

Sec. 2]

Tabular DifJerentiation

55

. l' (0) find cp' (0)


. cp' (I)

555. Find f (0) xl' (0) of the function f (x) = e- x 556. Find f (3) (x-3) I' (3) of the function f (x) = VI -+ x. 557. Given the functions f (x) = tanx and cp (x) = In (I-x).

+ +

558. Given the functions

f (x) = I-x and cp (x) =

1- sin ~x ,

find f'

(1)

559. Prove that the derivative of an even function is an odd

function, and the derivative of an odd function is an even function. 560. Prove that the derivative of a periodic function is also a periodic function. 561. Show that the function y = xe- x satisfies the equation
xy' = (I-x) y.

562. Show that the funct ion y = xe-;: sat isfies the equat ion xy' == (I-xl) y. 563. Show that the function y = 1+x+ln x satisfies the equation xy' = y (y In x-I).
1

x2

G. Logarithmic Derivative
A logarithmic derifJatit1e of a function Y=f (x) is the derivative of the logarithm of this function; that is, (In y)' ==!L = f' (x) y f(x) Finding the derivative is sometimes simplified by first taking logs of the function. Example. Find the deri vative of the exponential function
y=u'D,

where u = cp (x) and v = W (x). Solution. Taking logarithms we get In y == v In u. DifTerentiate both sides of this equation with respect to x:

(Iny)'=v' In u+v (In u)',


or

-y'=v'lnu+v -u', y u
whence y'=y

(V'lnu+: u').

56
or

DiOerentiation of Functions

[Ch.2

y'=Uf1(V'lnu+~ u')
564. Find y', if
y=

- I-x V x2 1 + x2 sin' x cost x.


l+x 2 + sinx
3_1 _ cosx_2s1nx .

2 Solution. In y =31n x + In (l-x)-ln (1 +x2 ) + 3 In sin x +2 In cos x;

y y

'=~-.!-

3 x

+ I-x

(-1) __~

cosx

whence

Y'=Y(3:-1

1 X-l~XI+3cotx-2tanx).

565. Find y', if y=(sinx)".


Solution. In y = x In sin x;

-.!y' = In sin x + x cot x; y y' = (sin x)" (In sin x + x cot x).

.In the following problems find y' after first taking logs of the funet ion y = f (x):
566. Y= (x+ 1) (2x+ 1) (3x+ 1).

574. Y=

Vx.
v%.

567. Y=

(X+2)2
(x+l)8(x+3)4.

575. y=x

568. Y = ' / x (x - I)

x-2
x2

569. Y = x

+ VX2
3

577. y=xs1nx 578. Y = (cos x)Sin x.

570. Y =

(X-2)9

Y(x-I)S (X-3)1I

571. Y= x +2)1 V(x+3)' 572. y=x". 573. y=xx3

vr

Yx-l

579. Y=

(1 +~

r.

580. y= (are tan x)..

Sec. 3. The Derivatives of Functions Not Represented Explicitly


1. The derivative of an inverse function. II a function Y =f (x) has a derivative Y~:I= 0, then the derivative of the inverse function X=/-1 (y) is
XII

=-,

Yx

Sec. 3]
or

The Derivatives of Functions Not Represented ExplIcitly


dx 1 dy= dye dx

51

Example 1. Find the derivative x~, if


y=x+ In x.
1 x+ 1 x Solution. We have Yx = 1+-=--; hence, x y =-+ . x x x 1 2. The derivatives of functions represented parametrically. If a function y Is reI ated to an argu ment x by means of a parameter t,
I

then

x==- q> (t), { y= 'I' (t),

or, in other notation,

Example 2. Find

:~.

dy dy di dx= dx . dt

if
x == a cos y=a sin

t, } t

Solution. We find

:~ = -a sin t

and
a sin t

~ = a cos t.

Whence

t!J!.= _
dx

a cos t =-cot t.

3. The derivative of an implicit function. If the relationship between x and y is gJven in implicit form,
F (x, y) = 0,

(1)

then to find the derivative y~ == y' in the simplest cases it is sufficient: 1) tocalculate the derivative, with respect to x, of the left side of equation (I), taking y as a function of x; 2) to equate this derivative to zero, that is, to put d
dx F (x, y) =0,

(2)

and 3) to solve the resulting equation for y'. Example 3. Find the derivative y~ if

x+y-3axy=0.

(3)

Solution. Forming the derivative of the left side of (3) and equating it. to zero, we get 3x 3yy' - 3a (y XI/') = 0,

58

DiOerentiation of Functions

[Ch. 2

whence

581. Find the derivative x~ if


a) y=3x+x l
;

b)

y=x-~ sinx;
x

c) y=O.lx+ea .

In the following problems, find the derivative functions y represented parametrically:

y' = :~ of the

58 {2 . ,
X

x=2t-l, y=t.

89 f x=a cos i, 5 . '\ .


t

y=b

Sin

t.
I

= t~I
=

583.

y
X

(_t_)'
t+ 1

'

J x = a cos' t

590. \ y= b sin' t.

= I ~Itl ,
a (1-12)

584.

r
591. if
\

cos't
X

y
X

1+ t 3at

Y=

Yeos 2t ' sinS t V cos 2t

585.

= 1+ tS ,
1

Y=

3at 2 + t'

592.

1
\

( X = arc cos .r 1 , ,. 1 + t Z
t

Y = arc sin .. r - . 2
, 1
t

586. \ Y = 587.

~=

Vi , VI.
594.

{ x = e- ,
693.

+t

y=elt

{X =" ~T'
y

{x = a( .In tan {+ cos t- sin t ) ,


y=a(slnt+cost).

Yt:l+l

588. { Y= a (sin t-t cos t).

x=a(cost+tsint),

595. Calculate

:~

when t =

~ if

x= a (t- sin t), { y=a(l-cost).


dy Solution. dx a sin t sin t a (l-cos t) = I-cos t

Sec. 3]

The Derivatives of Functions Not Represented Explicitly

59

and
( dx d Y)

sin "2

1t

t=-;:-

n: =

l-cos2

1.

596. Find ddY when t x


d dy 7 F In 59. d-x

= 1 if { In t Y=-t' t w hen t __ .::.. I' f { X = et co. 5 t, 4 Y =e sIn t.

x=tlnt,

598. Prove that a function y represented pararrletricaJly by the equations

satisfies the equation


Y=

(dY)' (dy)1 dx +2 dx

599. When x = 2 the following equation is true:

x' = 2x. Does it follow from this that


(Xl)'

when x == 2? 600. Let y= Val-xl. Is it possible to perform term-by-term different iat ion of Xl + yl == al? In the examples that follow it is required to find the derivative y' =:~ of the implicit functions y.
601. 2x-5y+lO=O.

= (2x)'

609. acosl(x+y)=b.

602. ::+~:= 1.

610. tany=xy.
x

603. x+y=a'. 611. xy=arctan-. I l O Y 64 o. x -}-xy+y = 612. arctan (x+y)=x. 605. 606.

Vx-t-VY=Vli.

613. e>'=x+y.
11
l

Vx -f-V y 2=Va
l

614. Inx+e-X- =c.

607. y'=-. x+y

x-y

615. lny+~=c.
Y

608. y-O.3 siny=x.

616. arc tan

~ = ~ In (x' + y').

60

DiUerentiation of Functions

lCJ1. 2

617.

. Vx 1+y2=carctan lL x

618. x'=yx.

619. Find y' at the point M (1,1), if 2y:=; 1 + xy.


Solution. Differentiating, we get 2y' = y' 3xy 2 y'. Putting x = 1 and y=l, we obtain 2y'=1+3y', whence y'=-l.

620. Find the derivatives y' of specified functions y at the indicated points:

a) (x + y)' = 27 (x-y) b) ye' = eX+ J


c) yl ==
X

+ In JL x

for x=2 and Y= I; for x=o and Y= I; for x=1 and y= 1.

Sec. 4. Geometrical and Mechanical Applications of the Derivative


1. Equations of the tangent and the normal. From the geometric significance of a denvative it follows that the equation of the tangent to a curve :1=/ (x) or F (x, y) =0 at a point M (x o, Yo) will be .

\vhere y~ is the value of the derivative y' at the point M (x o, Yo)' The straight line passing through the point of tangency perpendicularly to the tang~nt is called the normal to the curve. For the normal we have the equation 8

x-xo+Y: (y-Yo)=O.
2. The angle between curves. The angle between the curves
Y=/l (x)

and

at their common point M o (x o' Yo) (Fig. 12) is the angle ro between the tan~ents MoA and MoB to these curves at the point Mo. Using a familiar formula of analytic geometry, we get F' 12 Ig.

Y=f2 (x)

3. Segments associated with the tangent and the normal in a rectangu lar coordinate system. The tangent and the normal determine the following four

Sec 4]

Geometrical and Mechanical Applications of the Deritatit'e

61

segnlents (Fig. 13): t = TM St=TK n == N M Sn = KN

is the so-called segment of the tangent, is the subtangent, is the segment of t he normal, is the subnormal.

o
Since KM = , Yo
r

St
Fig. 13

If Sn N

and tan q> = Yo, it follows that

t = TM=

I~o~ V + (Y~)! I;
1

n = N M = Yo

I VI + (Y~)! I;

St=TK=/

~:I; Sn=IYoy~l

4. Segments associated with the tangent and the normal in a polar system of coordinates. If a curve is given In polar coordinates by the equation r=f(cp), then the angle J1 fornled by the tangent MT and the radius vector , = OM (Fig. 14), is defined by the following fonnula:

tanll.=r -=-. r dr r' The tangent MT and the normal MN at the point M together with the radius vector of the point of ta ngency a nd with the perpendicular to the radius vector drawn through the pole o determine the following four segments (see Fig. 14):

dq>

x
T
Fig. 14

t = MT is the segment of t he polar tangent, n = MN is the segment of the polar nornzal, St=OT is the polar subtangent, Sn = ON is the polar subnormal.

62

Differentiation of FunctIons These segments are expressed by the following fornlutas:


2 t=MT=-, ,r , Yr +(r')I;

[Ch. 2

St=OT=-1 r-'I; Sn=ON=lr'l

rl

n=MN=Yrl+(r')I;

to the curve y =

621. What angles cp are formed with the x-axis by the tangents
X-Xl

at points with abscissas: a) x=O; b) x=1/2; c) x=l? Solution. We have y' = 1-2x. Whence
a) tan q> = 1, q> =45; b) tan q> =0, q> =0; c) tan q> =-1, cp = 135 (Fig. 15).

622. At what angles do the sine curves y = sin x and y = sin 2x intersect the axis of abscissas at the origin? Fig. 15 623. At what angle does the tangent curve y = tan x intersect the axis of abscissas at the origin? 624. At what angle does the curve y = eO. IX intersect the straight line x=2? 625. Find the points at which the tangents to the curve y==3x 4 +4x'-12xl +20 are parallel to the x-axis. 626. At what point is the tangent to the parabola

y=x l -7x+3 parallel to the straight line 5x+ y-3 = O? 627. Find the equation of the parabola y = Xl + bx+c that is tangent to the straight line x=y at the point (1,1). 628. Deter4mine the slope of the tangent to the curve Xl + yl - xy-7=O at the point (1,2). 629. A t what point of the curve yl = 2x' is the tangent perpendicular to the straight line 4x-3y+2=O? 630. Write the equation of the tangent and the nornlal to the parabola Y= x at the point with abscissa x = 4.

v-

Solution. We have y' =


k = [y'l." = 4 =

Since the ;ointof tangency has coordi nates x = 4, Y = 2, It follows that the equation of the tangent is y-2=lj4(x-4) or x-4y+4=O. Since the slope of the normal must be perpendicular. k 1 =-4; whence the equation of the normal: y-2=-4(x-4) or 4x+y-18==O.

~_;

whence

the

slope

of

the tangent

is

Sec. 4J

Geometrical and Mechanical Applications of the Derivative

63

631. Write the equations of the tangent and the normal to the curve y=x'-l-2x l -4x-3 at the point (-2,5). 632. Find the equations of the tangent and the normal to the curve Y= x-I at the point (1,0). 633. Form the equations of the tangent and the normal to the curves at the indicated points: a) y = tan 2x at the origin; b) y=arc sin x 2 I at the point of intersection with the x-axis; c) y=arccos3x at the point of intersection with the y-axis; d) y = In x at the point of intersection with the x-axis; e) y=e1 - x2 at the points of intersection with the straight line y = 1. 634. Write the equations of the tangent and the normal at the point (2,2) to the curve

V-

x=-ta ,
Y=2t 2 + 2t.
3 1

1 +t

635. Write the equations of the tangent to the curve

x = t cos t,

Y = t sin t

at the origin and at the point t = ~ . 636. Write the equations of the tangent and the normal to the curve X l .t- y 2+2x-6=O at the point with ordinate y=3. 637. Write the equation of the tangent to the curve X S + yS_ --2xy==0 at the point (1,1). 638. Write the equations of the tangents and the normals to the curve y = (x-I) (x-2) (x - 3) at the points of its intersection with the x-axis. 639. Write the equations of the tangent and the normal to the curve y4=4x4+6xy at the point (1,2). 640. Show that the segment of the tangent to the hyperbola xy==a 2 (the segment lies between the coordinate axes) is divided in two at the point of tangency. 641. Show that in the case of the astroid x 2/1 + ylll = a2/1 the segment of the tangent between the coordinate axes has a constant value equal to a.

DIOerentiatlon of Functions [Ch. ~ 64 --. - - - - ,-_---=:.._----=-----------=-------

642. Show that the normals to the involute of the circle

x=a(cost+t sint), y=a(sint-tcost) are tangents to the circle x l +yl=a 2. 643. Find the angle at which the parabolas y = (X-2)2 and Y= --4 +6x-x 2 intersect. 644. At what angle do the parabolas y = Xl and y = Xl intersect? 645. Show that the curves y=4x l +2x-8 and y=xs-x+lO are tangent to each other at the point (3,34). Will we have the saIne thing at (-2,4)? 646. Show that the hyperbolas
XY= a 2 ;

x 2 _y= bl

intersect at a right angle. 647. Given a parabola y2=4x. At the point (1,2) evaluate the lengths of the segments of the subtangent, subnormal, tangent, and normal. 648. Find the length of the segment of the subtangent of the curve y =-= 2x at any point of it. 649. Show that in the equilateral hyperbola X 2_ y 2=a 2 the length of the normal at any point is equal to the radius vector of this point. 650. Show that the length of the segment of the subnormal in the hyperbola Xl_yl = a2 at any point is equal to the abscissa of this point. 651. Show that the segments of the subtangents of the ellipse x2 y2 (i2+ fj2 = I and the circle x 2.+ yl = at at points with the same abscissas are equal. What procedure of construction of the tangent to the ellipse follows from this? 652. Find the length of the segment ot the tangent, the nor.. mal, the subta..ngent, and the subnormal of the cycloid
X {

= a (t - sin t), y = a (1 - cos t)

at an arbitrary point t = to. 653. Find the angle between the tangent and the radius vector

of the point of tangency in the case of the logarithmic spiral

r=ae kCP
654. Find the angle between the tangent and the radius vector of the point of tangency in the case of the lemniscate r l = a l cos 2q>.

Sec. 4]

Geometrical and Mechanical Applications of the Derivative

65

655. Find the lengths of the segments of the polar subtangent, subnormal, tangent and normal, and also the angle between the tangent and the radius vector of the point of tangency in the case of the spiral of Archimedes

r=acp
at a point with polar angle <p=2n. 656. Find the lengths of the segments of the polar subtangent, subnormal, tangent, and normal, and also the angle between the tan gent and the radius vector in the hyperbolic spiral r = ..!:.. at an arbitrary point cp = CPo; r = r o. 657. The law of ITIotion of a point on the x-axis is
cp

x=3t-t'.
Find the velocity of the point at to=O, t 1 =1, and /2=2 (x IS in centimetres and t is in seconds). 658. Moving along the x-axis are two points that have the following laws of motion: x= 100+5t and x= 1/2/ 1 , where t~O. With what speed are these points receding from each other at the time of encounter (x is in centinletres and t is in seconds)? 659. The end-points of a segment AB =-~ 5 nl are sl iding along the coordinate axes OX and OY (Fig. 16). A is moving at 2ol/sec.
y

Fig. 16

Fig. 17

What is the rate of motion of B when A is at a distance OA = 3 m from the origin? 660*. The law of motion of a material point thrown up at an angle a to the horizon with initial velocity V o (in the vertical plane OXY in Fig. 17) is given by the formulas (air resistance is
3-1900

66

Difjerentlation of Functions

[Ch.2

disregarded):
X= V o t

coso,

where t is the time and g is the acceleration of gravity. Find the trajectory of motion and the distance covered. Also determine the speed of motion and its direction. 661. A point is in motion along a hyperbola y = 10 so that its x abscissa x increases uniformly at a rate of 1 unit per second. What is the rate of change of its ordinate when the point passes through (5,2)? 662. At what point of the parabola y2 = IBx does the ordinate increase at twice the rate of the abscissa? 663. One side of a rectangle, a = 10 em, is of constant length, while the other side, b, increases at a constant rate of 4 cnl,'sec. At what rate are the diagonal of the rectangle and its area increasing when b = 30 cm? 664. The radius of a sphere is increasing at a unifortTI rate of 5 cm/sec. At what rate are the area of the surface of the sphere and the volunle of the sphere increasing when the radius becomes 50 cm? 665. A point is in motion along the spiral of Archimedes r=acp (a = 10 cm) so that the angular velocity of rotation of its radius vector is constant and equal to 60 per second. Determine the rate of elongation of the radius vector r when r = 25 cm. 666. A nonhomogeneous rod AB is 12 cm long. The mass of a part of it, AM, increases with the square of the distance of the moving point, M from the end A and is 10 gm when AM =- 2 em. Find the mass of the entire rod AB and the linear density at any point M. What is the linear density of the rod at A and B?
Sec. 5. Derivatives of Higher Orders to. Definition of higher derivatives. A derivatzve of the second order, or lhe second derivative, of the function y= f (x) is the derivative of its derivalive; that is,
y" = (y')'.

The second derivative may be denoted as 1 d2 y

II,

or

d,x2'

or

f" (x).

If

X=

f (I)

Is the law of rectilinear motion of a point, then

:;~

is the accel-

eration of thIs motion.

Sec. 5]

Derivatives of Higher Orders

67

Generally, the nth derivative of a function y=/ (x) is the derivative of derivative of order (n-l). For the nth derivative we use the notation

y (n),

or

dny
dx n
'

or

f (n) (x).

Example l. Find the second derivative of the function y=ln(l-x). ' 1 , , ( - I )' 1 I Ii on. Y=l-x; Y= I-x Sou =(I-x)2'
2. Leibniz rule. If the functions u=<p (x) and v ='1' (x) have derivatives up to the nth order inclusive, then to evaluate the nth derivative of a product of these functions we can use the Leibl1iz rule (or formula):
(uv) (n) = u(n)v

+ n.u (n-I, v' + n (~~ I) UCx=<p(t), { Y='i'(t),

2)

v"

+ ... + uv(n).

3. Higher-order derivatives

of functions represented parametrical ~j'. if

then the derivatives

y;= :~ ,
II'

can successively be calculated

by the formulas

y xx -- y x

" - ( ')'x ( y'~); ,


Xt

y xxx = - - , - and so iorth.

(Y:x)~

x,

For a second derivative we have the formula


"

Yxx=
Example 2. F\nd y", if
{

, " x t Ytt - XttY, ( x~)a


X=Q

cos t,

Y =- b sin

t.

Solution. We have y'= and


(b sln

t) ~

(a cost),

,=

b . cos t b = - - cot t. -a SUI t Q

'l'

-~ colt ):
(a cos t);

b -1 --a. Sfi1i7

-a sin t

=- -

at sin' t

68

Differentiation of Functions

[Ch.

A. Higher-Order Derivatives of Explicit Functions


In the examples that follow, find the second derivative of thl given function. 667. y=x l +7x 8 -5x+4. 671. y=ln(x+Va 2 +x Z ). 668. Y = eX!. 672. f (x) = (1 + Xl). arc tan x. 673. y=(arcsinx)l. 669. y = sin l x.
670. y= In

VI +x

674. y=acosh~ .
a

675. Show that the function y = Xl + 2x 2 + 2 sa t IS fles th e dlfTer ential equation 1 +y,z=-=2yy". 676. Show that the function y = -} x'e x satisfies the differen tial equation y"-2y' +y=e x . 677. Show that the function y == C 1e- x -1- C 2 e- 1X satisfies th equation y" +3y' -1-2y=0 for all constants C 1 and Cz. 678. Show that the function y = eZx sin 5x satisfies the equa tion y" -4y' + 29y = O. 679. Find y"', if y=x 3 -5x 2 -f-7x-2. 680. Find f"' (3), if f (x) = (2x-3)5. 681. Find yV of the function y::= In (1 + x). 682. Find yVI of the function y = sin 2x. 68:3. Show that the function y = e- x cos x satisfies the differ ential equation yIV +4y==0. y 684. Find {(O), f' (0), f"(O) and f'" (0: if f (x) = eX sin x. 685. The equation of motion of a paill along the x-axis is

x = 100 + 5t-0.00lt 3
1.40 X Find the velocity and the acceleration ( the point for tinles to = 0, t 1 = 1, an

686. A point M is in motion around circle x2 y2 = at with constant angul2 Fig. 18 velocity 00. Find the law of motion of i1 projection M, on the x-axis if at time t = the point is at M o (a, 0) (Fig. 18). Find the velocity and the a< celeration of motion of MI' What is the velocity and the acceleration of M 1 at the in tial time and when it passes through the origin? What are the maximum values of the absolute velocity and tt absolute acceleration of M J ?

= 10.

'ec. 8J

l),rlvatlves of 811her Orders

69

687. Find the nth derivative of the function U= (ax + b)lI, where n is e Il.atural number. 688. Find the nth derivatives of the functions:
8) 1/=-1 -; -x
1

and

b) y= v x.
1

1/-

689. Find the nth derivative of the functions:


a)lJ=sinx;

e)Y=I+x;
f) y =

b) y = cos 2x; c) y = e-' ; d) g=ln(l+x);


Jt

+: ;
l+x

g) y = sin l x;

h) y=ln(ax+b).

690. Using the Lei bniz rule, find ycn), if:


a) y=x.e";
b) y=x l e- 2X ;
c)

d) y=

Yx;

c) y == x 3 In x.
Ix

y=(l-xl)cosx;

691. Find fln) (0), if

f (x) = In I

B. Iltgher-Order Derivatives of Functions Represented Parametrically and of Implicit Functions


In the following problems find

:~ .
c)

692. a) {x=ln t, . b) {x=arctant, y=t l ; y=ln(1-t-t 2 );

{x=arcsint
y=l-t l

693. a) { y = a sin t;
X=

x=acos t,

c)

a cos' t,

b) { y=asin't;

694. a) { y = sinl t;
b) {
.

x=cos2t,
x=e-at, at y=e
d1x

{x=a(t-sint), . y = a (1 - cos t), { x = a (sin t - t cos t), d) y=a(cost+t sint). { x=arctant, 695. a) y = ~ tl ;

{ x=lnt,
b)
1

Y=l-t'

696. Find d---a,


Y

If

{X =

t . t Y ==e sin

e' cos t,

70

Differentiation of Functions

[Ch.2
2 ),

697. Pmd dx 2 for / =0, If Y= /2. 698. Show that y (as a function of x) defined by the equations x = sin t, Y = aet Vz + be- t Vi-for any constants a and b satisfies the differential equation
(l-x)dx l
2

d2y

. {x=ln(l+t

d2 y
-

x dx=2y.

dy

In the following examples find y'" = X = sec i, { x = e- t , 699. { t 701. a y= an t. y=t . x=e-t cost, . dny { X= lo t, 700. { y=e- t 'sin. . t 702. Find -d n' if m x y=t. 703. Knowing the function y = / (x), find the derivatives x", x'" of the inverse function x = /-1 (y). 704. Find y", if x2 .-t- y2 = 1.
Solution. By the rule for differentiating a composite function we have x an d !:J ,1' = - -x )' =----2-. y -xy' 2x-f- 2yy ' = 0 ; wh ence y ' = - Y y x y Substituting the value of y', we finally get: , y2+ X 2 1 y' = III '

:1;1 .

l'

---r=-

In the following examples it IS required to determine the derivative y" of the function y = / (x) represented implicitly. 705. y2 --; 2px.
706. Q2
x2

+ b2 =

y2

1.

707. y=x-tarctan y.

708. Having the equation y = x + In y, find ~2;t and ::~. 709. Find y" at the point (1,1) if x 2 +5xy+y2_2x+y-6=0.
710. Find y" at (0,1) if

x-xy+y4= 1.
711. a) The function y is defined implicitly by the equation Xl 2xy + yl-4x-t- 2y-2 = O.

Find :~ at the point (1,1).


b) Find dx"
. dly

if x 2t y2 = al.

Sec. 6]

DiOerentials of First and Higher Orders

11

Sec. 6. Differentials of First and Higher Orders


1. First-order differential. The diOerential (first-order) of a function, Y = f (x) is the principal part of it~ increment, which part is linear relative to the increment I1x=dx of the independent variable x. The differential of a

o
Fig. 19 function is equal to the product of its derivatIve by the differential of the independent variable
dy== y'dx,

whence
1/
1

:::::.dy .

dx

If M N is an arc of the graph oi the function y = f (x) (Fig. 19), MT is the tangent at M (x, y) and
PQ
=~x==dx,

then the increment in the ordinate of the tangent


AT=dy

and the segment AN = ~y. Example t. Find the increment and the differential of the function y=3x 2 -x. Solu tion. first method:
~y =

3 (x+
~y =

~X)I_(X+~x) -3x 2

+x

or
(6x - I) ~x

+ 3 (~X)I.

Hence,
dy= (6x-l) Ax= (6x-l) dx.

Second method:

y' =6x-l; dy=y' dx= (6x-l) dx.


Example 2. Calculate ~y and dy of the function y=3x 2 -x for x=l

and
and

~x=O.Ol.

Solution.

~y= (6x-I). ~x

+ 3 (~X)2= 50.01 + 3 (0.01)1=0.0503

dy=(6x-l) ~x==50.01 =0.0500.

72

DiOerentiation of Functions

[Ch.2

2. Principal properties of differentials. 1) de = 0, where c = canst. 2) dx == /ix, where x is an independent variable.


3) d (cu) = c du. 4) d(u v)=du dv. 5) d (uv)=u dv -f- v duo

6) d ( ~ )
V

= v du -

v2

u do

(0

#= 0).

3. Applying the differential to approximate calculations. If the increment Ax of the argument x is small in absolute value, then the differential dy of the function y=f(x) and the increment fiy of the function are approxilnately equal;

7) df (u) =- f' (u) du.

that is,
whence

~y=:::;dy,

f (x + fix) - f (x) ~ f'


f (x + fix)
~

(x) fix,

f (x) + l'

(x) dx.

Example 3. By how much (approximately) does the side of a square change if its a rea increases from 9 m2 to 9.1 m2 ? Solu lion. If x is the area of the sq uare and y is its si de, then

y=yx.
It is gi ven that x = 9 and ~x = o. 1. The increment fiy in the side of the square may be calculated approximatply as follows:
Ay===dy=y' Ax= 2 f 9

~r- O.1=O.016m.

4. Higher-order differentials. A second-order dtOerential is the differential of a first-order differential:

We similarly define the dtOerentials of the third and higher orders. If Y == f (x) and x is an independent variable, then
d 2 y = y" (dX)2, d 3 y = y'" (dx)',

dny

= y(n) (dx)n.

But if y = f (u), where u = cp (x), then


d 2y = y" (dU)1 y' diu, day == y'" (du)a 3y" du. d 2 u y' diU

and so forth. (Here the primes denote derivatives with respect to u).

712. Find the increment ~y and the differential dy of the function U=5x-t Xl for x=2 and L\x=O.OOl.

Sec. 6]

DiOerentlals of First and Higher Orders

73

713. Without calculating the derivative, find S d (I-x )

for x = 1 and ~x = - -;;. v 714. The area of a square S with side x is given by S=x l Find the increment and the differential of this function and explain the geonlctric significance of the latter. 715. Give a geometric interpretation of the increment and differential of the follo\ving functions: a) the area of a circle, S = nx 2 ; b) the volume of a cube, v=x s 716. Show that \vhen ~x -+0, the increnlent in the function y = 2x , corresponding to an increment ~x in x, is, for any x, equivalent to the expression 2-'- In 2 ~x. 717. For \vhat value of x is the diITerenti al of the function y == x 2 not equivalent to the incretnent in this function as L\x ---.. O? 718. Has the function Y==lxl a differential for x=O? 719. Using the derivative, find the differential of the fUllction

y = cos x for x = ~ and /),X --= 3~ 720. Find the diITerential of the function
Y==

Yx

for x==9 and L\x==-O.OI. 721. Calculate the diITerential of the function Y == tan x for x::=. 3 and ~x -== 180 In the following problenls find the differentials of the given functions for arbitrary values of the argunlent and its increluent. 1 722. Y=--=x tn 727. y=xlnx-x.
723
x 'Y==T-x
Jl

728.

y=nl-f-x

I I-x

724. !J= arc sin : .


x 725. Y = arc tan . a

729. r = cot <p

+ cosec <p.
t

730. s = arc tan e

726. Y = e- x2 731 Find dy if x 2 -}-2xy- y l = a l


Solution. Taking advantage of the invariancy of the fOrln of a differential, we obtain 2xdx-{-2(ydx+xdy)-2ydy=O Whence X+Yd x. dy=--x-y

74

DiOerentiat ion of Funct ions

In the following examples find the differentials of the functiollS defined implicitly. 732. (X+y)2. (2x+y)' = 1.
733. Y = e-Ii .
734. In V Xl
x

+ y. = arc tan JL x

735. Find dy at the point (1,2), if y'-y=6xl 736. Find the approximate value of sin 31.
Solution. Putting x=arc300=i and Ax=arc 1= 1~0' from formula (1) (see 3) we have sin 31:::::::: sin 30

+ 1;0 cos 30=0.500+0.017. ~3 =0.515,

737. Replacing the increment of the function by the differential, calculate approximately: a) cos 61; d) In 0.9; b) tan 44; e) arc tan 1.05.
c) eO. 2 ;
738. What will be the approximate Increase in the volume of a sphere if its radius R = 15 cm increases by 2 mm? 739. Derive the approximate formula (for I ~x \ that are small compared to x)

vx + ~x ~ VX +

~x

lJsing it, approximate Y-5, V17, V70, V640. 740. Derive the approximate formula

- Vax Vx+~x~ x + 3 V= Xl
and find approximate values for VI0, V70, V200. 741. Approximate the functions: 8) y=x'-4x l +5x+3 for x=I.03; b) f(x)=Vl +x for x=O.2;
c)

for X= 0 1 ; d)y =e f or x= 105 .. 742. Approximate tan 453'20". 743. Find the approximate value of arc sin 0.54. 744. Approximate VI7.
1 x l+x
1 xl

f (x) =

Sec. 7)

Mean-Value Theorems

15

show that a small change in the current, due to a small change in the resistance, may be found approximately by the formula

745. Using Ohm's law, I

=~,

8/=-

I R~R.

746. Show that, in determining the length of the radius, a relative error of 1% results in a relative error of approximately 2 loin calculating the area of a circle and the surface of a sphere. 747. Compute diy, if Y = cos 5x.
Solut ion. d 2 y = y" (dx 2) = - 25 cos 5x (dX)2.

748. u = i-x2 , find d 2 u. 749. y = arc cos x, find d 2 y. 750. y = sin x In x, find d 2 y. In x f d d 2 751. z=-X' In z. 752. z = x2 e- X , find d'z. 753.
Z = (-)- ,
xo&

11

... -x

find d

01

z.

754. II = 3 sin (2x -t- 5), find dnu. 755. y = eX cos a sin (x sin u), find dray. Sec. 7. Mean-Value Theorems
a ~ x ~ b, has a derivative

1. Rolle's theorem. If a function f (x) is continuous on the interval f' (x) at every interior point of tillS interval, and

f (a) -= f (b),
then the argunlent x has at least one value
~,

where a <

I'

,<

b, such that

(~) =0.

a ~ x ~ b and has a derivative at every interior point of this interval, then


where a < ~ < b. 3. Cauchy's theorem. If the functions f (x) and F (x) are continuous on the interval a ~ x ~ b and for a < x < b have derivatives that do not vanish simultaneously, and F (b) :/:- F (a), then
F (b)-F (a)
f (b)-f (a) = (b-a) I'

2. Lagrange's theorem. If a function f (x) is continuous on the interval

(6),

f (b) -f (a)

= F' (~)'

f'

(~)

where a <

<

b.

756. Show that the function f (x) = x-x' on the intervals -1 ~ x ~ 0 and 0 ~ x ~ 1 satisfies the Rolle theorem. Find the appropriate values of 6e

76

DiUerenttation of Functions

[Ch.2

of x, and f (-1) =f (0) = f (1) =0. Hence, the Rolle theorem is applicable on the intervals -1 ~ x <: 0 and 0 ~ x < 1. To find we fornl the equation

Solution. The function f (x) is continuous and differentiable for all values

f' (x) = 1-3.r=O. Whence and 0 < ~I < l.

~1 = -

-V ~ ;~2= -V ~.

where

-1

<

~1 < 0

takes on equal values 1(0)=/(4)= V4 at the end-points of the interval [0.4). Does the Rolle theorem hold for this function on [O.4]? 758. Does the Rolle theorem hold for the function f (x) = tan x on the interval [0, n]? 759. Let

757. The function /(x)=V(x-2)'

f (x) =
Show that the equation has three rea i roots. 760. l~he equation

x (x .,- 1) (x

t- 2) (x + 3).

f' (x) =0
eX = 1 t- x

obviously has a root x=O. Show that this equation cannot have any other real root. 761. Test whether the Lagrange theorem holds for the function
{(x)

= x-x

on the interval (-2,1] and find the appropriate internlediate value of Solution. The function I (x) =x-x' is continuous and differentiable for nil values of x, and I' (x)= 1-3x2 Whence, by the Lagrange fornlula, we h3ve f(1)-f(-2j=O-6==(I-(-2)]f'(~), that is, f' (s)~~2 Hence,

s.

1-3~2=-2 and ~=l; the only suitable value is inequality -2 < ~ < I holds

~=-l,

for which the

762. Test the validity of the Lagrange theorem and find the clppropriate intermediate point ~ for the function f (x) = X"/. on the interval [-1,1]. 763. Given a segment of the parabola y = x! lying between two points A (1,1) and B (3,9), find a point the tangent to which is parallel to the chord AB. 764. Using the Lagrange theorem, prove the formula sin (x

+h) -

sin x = h cos S,

where

x<;<x+h.

Sec. 8]

Taylor's Formula

77

765. a) For the functions f (x) = r -t- 2 and F (x) = x' -1 test whether the Cauchy theorem holds on the interval [1,2] and find ~; b) do the same with respect to f(x)=sinx and F(x)=cosx on the interval [0, :].

(n-I)th order inclusive on the interval a ~ x ~ b (or b ~ oX ~ a), and there IS a finite derivative fen) (x) at each interior point of the interval, then Taylor's fornzula
(x - a)2 t" (a l + (x - - a) - I f'" (a)+ ... a + (x-a ) t ' (a) + -2-1f (x) == f () 3t

Sec. 8. Taylor's Formula If a function f (x) is continuous and has continuous derivatives up to the

... + (~;~~~l-I rn-I'(aH (x:~)~ fIn) (;),


where ~==a+8(x-a) and 0<8<1, holds true on the interval. In particular, when a==O we have (Maclaurin's formula)
f (x) =f (0) +xf' (0)

+fu f"

(0)

+ ... + (~n-:)I

fIn_I) (0)

+ :~ fin) m.

where

~=Ox,

0<8<1.

the polynomial f(x)=x ' -2x l +3x+5 in positive integral powers of the binonlial x-2. Sol ution. f' (x) == 3x 2 - 4x + 3; til (x) == 6x - 4~ f'" (x) = 6; fen) (x) = 0
766.
for n ~ 4. Whence
rherefor~,

[~xpand

f (2) = 11; f' (2) = 7; {" (2) == 8; f"' (2)


x - 2x 2 + 3x+ 5 = II
or
x l -2x 2 +3x+5= II

= 6.

(x - 2)2 8 (x - 2)1 6 + (.r - 2). 7 +---. +--. 2! 31

+ 7 (x -

2)

+ 4 (X--2)2 -l- (x-2)'.


x -r-l to the

767. Expand the function term containing (x+ 1)1. Solution. 1(") (x) =e X for all n,
eX

f (x) = eX in powers of
ten) (-1) =..!... Hence,

=..!- +(x+ I)..!.. + (x+ 1)2 ..!.. + (x + I).! + (x+ 1)


e e

21

31

41

e~

'

where ~=-1+8(x+l); 0<8<1.

768. Expand the function the term with (x-I)I.

f (x) = In x in powers of x-I up to

78

DiOerent iat ion of Functions

[Ch. 2

taining

ing x n - 1 771. Show that sin (a +h) differs from

f (x) = sin x in powers of x up to the term conand to the term containing Xl. 770. Expand f (x) = eX in powers of x up to the term contain769. Expand
Xl

sin a -+h cos a by not more than 1/2 h 772. Determine the origin of the approximate formulas:
a)
1 1 1 +x~ 1 +2"x-S-x V-1 1 b) V 1 + x~ 1 + "3 x- 9 x
2 ,
2 , 2

Ixl

<

1,

I x I< 1

and evaluate their errors. 773. Evaluate the error in the formula

e ~ 2+

1 2!

+ 3! T

1, 1

4i

774. Due to its own weight, a heavy suspended thread lies in a catenary line y = a cosh -=-. Show that for small Ix I the a

shape of the thread is approximately expressed by the parabola

y=a+ 2a
approximate equality
x ea ~

x2

775*. Show that for Ix I~ a, to within

(~

r,

we

have

the

y-.
a+x a-x

Sec. 9. The L'Hospital-Bernoulli Rule for Evaluating Indeterminate Forms


1. Evaluating the indeterminate forms

%and :

. Let the single-valued

functions f (x) and cp (x) be differentiable for 0</ x-a I <h; the derivative of one of them does not vanish. If f (x) and cp (x) are both infinitesimals or both inflnites as x ~ a; that
is, if the quotient

~(~:)'

at x = a, is one of the indeterminate forms

or

~, then co

lim

x~a <p

f (x) _lim l' (x)


(x) - %-+-a <p' (x)

provided that the limit of the ratio of derivatives exists.

Sec. 9]

L' Hospital-Bernoulli Rule for Indeterminate Forms

79

The rule is also applicable when a= 00.


If the quotient

~,(~1)

again yields an indeterminate form, at the point

x=a, of one of the two above-mentioned types and I' (x) and cp' (x) satisfy all the requirements that have been stated for' (x) and p (x). we can then pass to the ratio of second derivatives, etc. However, it should be borne in mind that the limit of the ratio f (x)

may exist, whereas the ratios of the derivatives do not tend to any

(see Example 809). 2. Other indeterminate forms. To evaluate an indeterminate form like 0-00, transform the appropriate product (x).,. (x). where lim (x)=O and

Ir~~~

'1

~a

lim!2 (x) =~, into the quehent /1 (IX) (the

form -0 (or, I( -) (the 1 x


00,

'I

-wJ

'1

(x)

form 00).
00

In the case of the indeterminate form 00 -

12 (x)

one should transform the

appropriate difference fdx)-f 2 (x) into the product f'<x)

first evaluate the indeterminate form duce the expression to the form
1

',IX; x
I

if

lim

%~a

[I-~: ~:n ',1 (x = 1, then


1 X

and

we re-

1_'t.

'I

(x) (x)

(the form 0-. 0)

(x)
(X) ,

The indeterminate forms I 0<, 00 0 are evaluated by first taking loga e rithms and then finding the limi t of the logarithm of the power ['I (x 12 (' (which requires evalua ting a form like O 00). I n certain cases it is useful to combine the L'Hospital rule with the finding of limits by elementary techniques. Example 1. Compute In x 00 l1m - (form -). X'~o cot x 00 Solution. Applying the L'Hospital rule we have
lim ~ = Urn (In x)'
%-+0

cot x

x~o

(cot x)'

== _ Urn
x-+o

sin

We get the indeterminafe form

~;

however, we do nof need fo use the

L'Hospital rule, since


lim
~-+o

sin x I sin x --= 1m -sln x= 1-0=0. x %-+0 X

We thus finally get


x~o

lim In x =0. cot x

80
Example 2. COlTIf'ute
x ~o

DiOerent iat ion

of Funct ions

[Ch. 2

lim

(_l__ ..!-) (form 00-00). sin x x


2
2

Reducing to a common denomi nator, we get


x-+o

ltm

(1 I) = ---sin 2 x

x2

x~o

2 x (form -) 0 ltm x -sin! x 2 sin! x 0


0

Before applying the L'Hospital rule, we replace the denominator of the latter fraction by an equivalent infinitesinlal (eh. I, Seco 4) x 2 sin! x_x4 We obtain x 2 - sin 2 x 11m - . - - 4 (form -0). 2 = Iun

. (1

x-+o

S111 2 X

1)
0

X-+O

The L 'Hospital rule gives


x-.+o

1- - 1) - 1 2x - 11 m 2-2cos 2x 1 Jm ( 1m 2x-sin sin 2 x x 2 - x-+o 4x 8 x-+o 12x2


0 0

Then. in elementary fashion, we find

lim
x-+o

(_I__ .-'!')=liIn l-cos2x=lim 2sin2x=~. sin x x 6x 6x 3


2
2

x-+o

x-+o

Example 3. Compute
a

lim (cos 2x) Xi (fonn 100 )


x-+o

Taking logarithms and applying the L'Hospital rule, we get


a

lim ln (tos 2x) X2 = HIn 3ln cos 2x = _ 6 lim tan 2x = _ 6


x-+o

X-i-O

x2

X -+0

2x

lienee, lim (cos 2x)Xi:y:e- s


x-+o

F"ind the indicated limits of functions in the following exatTIpleso


XS_2XI_X+2 776 1lin 3-7 +6 X-+l x X

Iimx'-2xl-x+2_Ilm3x2-4x-I_1 _ __ So IU t Ion. X-'l x'-7x+ 6 X-+J 3x 2 -7 2

777.

rlin x cos x-sin x a


x -+Q
ot

779. lim cosh x-I 1 -cos x


x~o

778. lim
.r--+J

I-x
1- sin ~ 2

780.

lim x-sinx
.t~o

tan x--sin x

Sec. 9]

L' Hospital-Bernoulli

Rule for Indeterminate Forms

81

781. hm n
4

x?-

sec 2 x-2tanx I -1- cos 4x

785. lim _x_ .


2 . In (sin nlx) 1 786. 1m 1 x---+o n sin x
x-+cotj(x

tan x 782 1 Im - an Sx nt
X-i--

783. hm --;.

eX

. 784. 1 1m

t'---+oo X

787. lim (1- cos x) cot x.


x -+0

In x
Vi -

X-~'7J

Solution.lil11 (l-cosx)cotx=lim (I-cosx)cosx== litn (I-cosx).t=z


x-?o
c:: lilll
X

x-?o

Sill X

x -+0

SlIl .\.

sin

= 0

-?>O

cos \"
nx

788. liln (I-x) tan "2 .


.\"-...,.1

792. linl x n sin!:., n>O.


X-x'

789. lilll arc sin x cot x.


x~o

793. hnllnxln(x-I). \- - ( 794. I lIn X+l

790. Hln (x ll e- X ), n>O.


t"~0

.\-1

Inx

1)

791. Hln x sin!!:.. .


x

. Itnl . Solutaon.
X~1

(X ----.\-}

In x

1) =Itm . xlnx-x+1 == In \
X-i-l

(x-I)

= Htll

I ..\.--+Inx-I x I =linl

1
Ill'
~

x-~11I1.\-I-x(\-1)

x-+1}nx-

x -+-l

= ltnl1- I-=2
~71~+X2

( -:--3- . 2 _ -6 . 795. I lIn x -+3 X X.\ 796. I lIn


X---+l

5)

[ I JI
2 (1 -

'x)

3 (1 -

1] V
x)

d.

79~

. (X hIll ---n cotx 2cosx


t~-

1t)

798. liln xx.


x~o

Solution.

We

have

xX=y;

Iny=-xlnx:

linllny=limxlnx=x-+o .r-+o

Inx ==lim-=lim--==O, whence limy=l, that


%-+0

-.!X

x-+o _

x !

IS, ltnl.\..\.=
%-+0

l.

x2

%-+0

82
1

DifJerent iat ion

of

Funcf ions
1

[Ch.2

799. linl xx.


x-+>+ tI)

804. lim Xl-X.


C-+J
%.

800.

a limx4 + 1n
%-+0

805. lim(tan
%-+J

7) ".
1
%

tan~

801. lim x,'n x.


%-+0

806. lim (cot x)ii1i.


%-+0
rEx

802.

lim (l_x)cos a ,
~-+1

807.

Iim( .!-.)tan
%-+0 X

803. lim (I
%-+0

+ x fi .
2

808. lim (cot x)~in


%-+0

%.

809. Prove that the limits of


x a) 1 Im_.-=o;
x~o

Xl

stnJ.

sin x

b) lim x-s~n x = 1
%-+ClDx+slnx

cannot be found by the L'Hospital-Bernoulli rule. Find these limits directly.

Fig. 20

810*. Show that the area of a circular segment with minor central angle a, which has a chord AB=b and CD=h (Fig. 20), is approximately
2 S~-bh

wi th an arbitrari Iy small reI ati ve error when a --+ O.

Chapter 11/

THE EXTREMA OF A FUNCTION AND THE GEOMETRIC APPLICATIONS OF A DERIVATIVE

Sec. 1. The Extrema of a Function of One Argument


1. Increa~e dna decrease of 'unctions. Thl 1u11ctlon y= f (x) is called on some Interval if, fo. any points Xl and X 2 which belong to this Interval, frolll the inequality X I <X2 we ~et the Inequality I (x I )< <I (x 2 ) (Fig 21a) [{ (XI! (.x 2 ) (Fig. 21b). i i f (x) is conttnuous on the interval la, b) and f' (xO [t' (x)<OJ for a<. <b, then f (.x) increases (decreases) un the Interval [a. b).
increasing (decreaslItg)

o
(a)
Fig. 21

(b)

o
Fig. 22

111 the sinlplest cases, the don13ln of definition of f (x) may be subdivided Into t.l finite nUlllber of intervals of incrense and decrease of the function (intervals of 11l0notontcity). These Intervals are bounded by Cl itic poi nts x [where I' (x) =~ 0 or f' (x) does not exist]. Example 1. Test .the following function for increase and decrease:
r '

y=x 2 -2x+5.
Solution. We find the derivative

y'=2x-2=2(x-l).
Whence y'=O for x= 1. On a nUlnber scale we get two intervals of monotonicity: (-00, I) and (I, co). Frolll (1) we have: I) if -oo<x<l, then y'<O, and, hence, the functioll t (x) decreases in the interval (-00. I); 2) if 1<x< -1- 00. then y'>O. and. hence. t he function f (x) increases in the interval (1, 00) (Fig. 22).

84
tion

Extrema and the GeometrIc Applications of a Derivative

[Ch.3

Example 2. Determine the intervals of increase and decrease of the rune


1 Y=x+2
Solution. Here. x= - 2 is a discontinuity

of the function and y' =

(x+ ) - oo<x<-2 and -2<x< + 00.

= -~22<0

for x:j:.-2. Hence, the function y decreases in the intervals

Example 3. Test the following function for increase or decrease:

1 y == 5
Solution

1 xS-a Xl.
(2)

Here,

Sol ving the equation X4 _X 2 =:.0, we find the points Xl = -1, x 2 =0, xs == 1 at which the derivative y' vanishes. Since y' can change sign onl y when passing through points at which it vanishes or becomes discontinuous (in the given case, y' has no discontinuities), the derivative in each of the intervals (-00, -1), (-1,0), (0,1) and (1, +00) retains its sign; for this reason, the function under investigation is monotonic in each of these intervals. To determine in wh ich of the indicated intervals the function increases and In which it decreases, one has to determine the sign of the derivative In each of the intervals, To determine what the sign of y' is in the interval (- 00, -1), it is sufficient to determine the sign of y' at sorne point of the interval; for example, taking X= -2, we get from (2) y' = 12>0, hence, y'>O 111 the interval (-00, -1) and the function in this interval increases Sirnilarly, we find that y'<O in the interval (-1, 0) (as a check, we can take
X= -

~).

y'<O

in the

interval

(0,1)

(here, we can use x = 1/2) and y'>O in the interval (1, -t- 00). Thus, the function being tested increases in the interval (- 00, -I), decreases in the Interval (-1, 1) and again increases in the interval (I, + 00). 2. Extremum of a function. If there exists a two-sided neighbourhood of a point Xo such that for every pC'int x:j:.x o of this neighbourhood we have the inequality X, f (x! (x o), then the point Xo is called the minimum potnt of the fu nct ion y =-= f (x), Fig 23 whtle the nUlnber I (x o) is called the mInimum of the function y= f (x). Similarly, if for any point x=x l of some neighbourhood of the point Xl' the inequality f (x)<t (Xl) is fulfilled, then X J is called the maximum point of the function I (x), and f (Xl) is the maximum of the function (F ig. 23). The rTiI nimum point or maximum point of a function is its ext remal point (bending point), and the minimum or nlaximum of a function is called the extremUtn of the' funct ion. If Xo is an extremal point of the function I (x), then I' (x o) = 0, or f' (xo) does not exist (necessary condition for the existence of an extremum). The converse is not true: points at which f' (x) =0, or I' (x), does not exist (critical polnts) are not necessarily extremal points of the function I (x).

Sec. 1]

Tlte Extrelna of a Function

of

One Argument

85

The sufficient conditions for the existence and absence of an extremum of a continuous function f (x) are given by the following rules: I. If there exists a neighbourhood (x o-6, xo+6) of a critical point Xo such that f' (xo for X o- 6<x<x o and I' (x)<O for xo<x<xo +~, then Xo is the maxinlum point of the function I (x); and if I' (x)<O for xo-6<x<xo and f' (xO for xo<x<x() + 6, then Xo is the minimum point of the function Finally, if there is SOlne positive number () such that f' (x) retains its sign unchanged for 0<1 x-xo /<6, then Xo is not an extremal point of the functIon f (x). 2. Iff' (x o) =. and t" (xo)<O, then Xo is the Inaximum point; iff' (x 0) == 0 and (x 0) > 0, th en Xo is the 1T1 i n imum po in t; but if f' (xo) = 0, 1" (xo) -= 0, and f'" (xo):#:O, then the point Xo is not an extremal point. ~I\ore generally: let the first of the derivatives (not equal to zero at the point x o) of the function f (x) be of the order k. Then, if k is even, the point Xo IS an extrenlal point, nalnely, the l11axinlUl11 point, if t<k) (xo)<O; Clnd it is the 111ininlum point, if t<k) (xoO But if k IS odd, then X o is not (n ext re111 a I poi nt . Example 4. Find the extrenla of the function

f (x).

y:;= 2x

i +3 V x

Solution. Find the derivative


y'=2-t-

Vx

2 V-x+I). -=V-(
t

(3)

Equating the

derivativ~ y'

V x+ 1 ==0.

to zero, we get:

\VhenCL', we find the critical point Xl =-= -1. From fonnula (3) \ve have: if x==- - : - h, where h IS a sufficiently sl11all positive nUl1lUer, then y'>O; but if x=---= -1 h, then y' <0*). lienee, Xl::::: - I IS the maXlmU111 point of the fu nctton !I, and ymax ::-:-- 1.

Eqllatln~

thr dcnolninator of the expression of y' in (3) to zero, \ve get

V-

x~

o,

whence \\e find the second critical point of the functIon xt=O, \\There there IS no derIvatIve !I' For x== -h, \ve obviously have y'<O; for x== Iz \ve have U'>O. Con"cquently, xt=-=O is the Il1inl111UI11 point of the functIon y, and !/rnin==O (FIg. 24). It is also possible to test the behaviour of the function at the point x= -1 by means of the second derivative

!I=-, ;/ ' 3x V x
Here, y" <0 for Xl = - I and, hence, Xl = - I is the 111aXlInUm point of the function. 3. Greatest and least val ues. The least (greatest) val ue of a cont inuous function f (x) on a given interval [a, b] is attained either at the critical points of the function or at the end-points of the interval {a, b].
*) If It is difficult to deternline the sign of the derivative y', one can calculate arithmetically by taking for Iz a sufl1cillntly snlall positive number.

86

Extrema and the Geometric Applications of a Derivative


Example 5. Find the greatest and least values of the function
y=x 8 -3x+3

[CIl.8

on the interval -P/2~X~21/1. Solution. Since

y'=3xl -3,

it follows tha t the critical points of the function yare

Xl

= -I

and

XI =

I.

x
Fig. 24 Fig.
~\J

Comparing the values of the function at these points and the values of the function at the end-points of the given interval
Y(-I)=5;f(I)=I;Y( - I

~)=4 ~;

Y(2 ~)=ll ~.

we conclude (Fig. 25) that the function attains its least value, m = I, at
the point x= I (at the minimum point), and the greatest value M = II at the point x=2 1 /. (at the right-hand end-point of the interval).

i-

Determine the intervals of decrease and increase of the fune . iions:


811. 812. 813. 814.

Y= 1-4x-xl Y=(X-2)1.
Y = (x -t- 4)1, y = Xl (x- 3).

816.

Y=(x_I)2.
:

817. Y = xZ 818.

x- Hi )

815. Y=--!-2. x-

y=(x-3)Vx.

Sec. 1]

The Extrema

ot

a Function of One Argument

87

x 819. Y==3-

Vx.
I

820. y=x

+sinx.
+ x).

821. Y= X In x. 822. y = arc sin (1

824. Y = 2X::-a. eX 825. y= x'

Test the following functions for extrema: 826. y = x" + 4x -t- 6.


Solution. We find the derivative of the given function, y'=2x+4. Equating y' to zero, we get the critical value of the argument X= -2. Since y'<O when x<-2, and y'>O when x>-2, it follows that X= -2 is the minimum point of the function, and Ymln = 2. We get the same result by utilizing the sign of the second derivative at the crit ical point y" = 2>0.

827. y=2+x-x l 828. y=x'-3x' +3x+2. 829. y=2x'+3xz -12x+5.

Solution" We find the derivative y'=6x l +6x-12=6 (x l +x-2). Equating the derivative y' to zero, we get the critical points Xl =-2 and XI = 1. To determine the nature of the extremum, we calculate the second derivative y"==6(2x+l). Sinee y"(-2)<O, it follows thatxl =-2 is the maximum point of the function y, and ymax=25. Similarly, we havey" (10; therefore, Xz = 1 is the minimum point of the function y and Ymln= -2.

830. y=x (x-12)2. 831. y = X (x-I)1 (X-2)'.


x 832. Y=x l +3'
x -2x+2
x-I
l

840. y=2 cos

-i +3 cosi-.

841. y=x-ln(l+x).

833. y=

842. y=x Inx.

834. Y = (X-2~~8-X) 835. y-= X (4-x 2 ) 836. Y = 837.


4
l

843. Y =

In x,

16

844. y=coshx. 845 . y=xe x .

yXi"+8 x + '

x Y=V xl - 4 '

846. Y=XIe-~.

838. Y= V(.r-I),

847. y=~ ,

839. Y = 2 sin 2x + sin 4x. 848. y = x - arc tan x. Determine the least and greatest values of the functions on the indicated intervals (if the interval is not given, determine the

88

Extrema and the Qeometric Applications of a DerivativI

[Ch. 8

~f

greatest and least values of the function throughout the domain definition).
849. Y= I 850. y = V-x(-10---x-). 851. Y= sin" x + cos" x.

;xi'

853. y = Xl on the interval [-1,3]. 854. y = 2x' -1- 3x l


12x + 1 a) on the interval r-1,5]; b) on the interval [-10,12].

852. Y = arc cos x. 855. Show that for positive values of x we have the inequality
x+-~2. x

856. Determine the coefficients p and q of the quadratic trl'nomial y=xl+px+q so that this trinomial should have a minimum y=3 when X= 1. Explain the result in ~eometrlcal terms. 857. Prove the inequal ity

,It> 1+x
In the usual Hence,
wa~

when x + o.

Solution. Consider the function f (x) :a::,X -(1 +x).

we find that this function has and so eX

single minimum

f (0) =0.

f (x) > f (0) when x = O. > 1 + x when x#: 0,

8S

we set out to prove.

Prove the inequa1ities: Xl 858. x- 6 < sinx<x


859. cos x > 1- '2 860. x-"2<ln(l+x)<x
x
2

when x>O. when x+O.

Xl

when x>O. 861. Separate a given positive number a into two summands such that their product is the greatest possible. 862. Bend a piece of wire of length 1 into a rectangle 80 that the area of the latter is greatest. 863. What right triangle of given perimeter 2p has the greatest area? 864. It is required to build a rectangular playground so that it should have a wire net on three sides and a long stone wall on the fourth. What is the optimum (in the sense of area) shape .of the playground if I metres of wire netting are available?

Sec. 1]

The Extrema of a Function of One ArgulfLCnt

89

865. It is required to make an open rectangular box of greatest capacity out of a square sheet of cardboard with side a by cutting squares at each of the angles and bending up the ends of the resulting cross-like figure. 866. An open tank with a square base must have a capacity of u litres. What size will it be if the least amount of tin is used? 867. Which cylinder of a given volume has the least overall surface? 868. In a given sphere inscribe a cylinder with the greatest volume. 869. In a given sphere inscribe a cylinder having the greatest lateral surface. 870. In a given sphere inscribe a cone \vith the greatest voluIne. 871. Inscribe in a given sphere a right circular cone \vith the greatest lateral surface. 872. About a given cylinder CirCUITIScribe a right cone of least volunle (the planes and centres of their circular bases coincide). 873. Which of the cones circumscribed about a given sphere has the least volume? 874. A sheet of tin of width a has to be bent into an open cylindrical channel (Fig. 26). What should the central angle cp be so that the channel will have maxinlunl capacity?
IJ

C
N

o
/
~,

'f
a

,
A
B

A~8
M
Fig. 26

875. Out of a circular sheet cut a sector such that when madeinto a funnel it will have the greatest possible capacity. 876. An open vessel consists of a cylinder with a hemisphere at the bottom; the walls are of constant thickness. What will the dimensions of the vessel be if a mininlUlTI of rnaterial is used for a given capacity? 877. Determine the least height h=OB of the door of aver tical tower ABeD so that this door can pass a rigid rod MN of length i, the end of which, M, slides along a horizontal straight line AB. The width of the tower is d < 1 (Fig. 27).

Extrema and the Geometric Applications of a Derivative

[Ch.3

878. A point Mo (x o' Yo) lies in the first quadrant of a coordinate plane. Draw a straight line through this point so tha t the triangle which it forms with the positive semi-axes is of least area. 879. Inscribe in a given ellipse a rectangle of largest area with sides parallel to the axes of the ell ipse. 880. Inscribe a rectangle of maximum area in a segment of the parabola y" = 2px cut off by the straight line x = 2a. 881. On the curve y = 1,~XI find a point at which the tangent forms with the x-axis the greatest (in absolute value) angle. 882. A messenger leaving A on one side of a river has to get to B on the other side. Knowing that the velocity along the bank is k times that on the water, determine the angle at which the messenger has to cross the river so as to reach B in the shortest possible time. The width of the river is h and the distance between A and B along the bank is d. 883. On a straight line AB=a connecting two sources of light A (of intensity p) and B (of intensity q), find the point M that receives least light (the intensity of illumination is inversely proportional to the square of the distance from the light source). 884. A lamp is suspended above the centre of a round table of radius r. At what distance should the lamp be above the table ~o that an object on the edge of the table will get the greatest !1lunlination? (The intensity of illumination is directly proportional to the cosine of the angle of incidence of the light rays and is inversely proportional to the square of the distance from the \ig ht source.) 885. It is required to cut a beam of rectangular cross-section 01lt of a round log of diameter d. What should the width x and the height y be of th is cross-sect io n so that the beam will offer maximum resistance a) to compression and b) to p bending?
,~~--:t :A---~----.vB

Note. The resistance of a beam to compression is proportional to the area of its crosssection, to bending-to the product of the width of the cross-section by the square of its height.

886. A homogeneous rod AB, which can rotate about a point A (Fig. 28), is carrying a load Q kilograms at a distance of a em from A and is held in equilibrium by a vertical force P applied to the free end B of the rod. A Iinear cent imetre of the rod weighs q kilograms. Determine the length of the rod x so that the force P should be least, and find P min.

Fig. ~

Sec. 2]

The Direction of Concavity. Points of Inflection

91

887*. The centres of three elastic spheres A, B, C are situated on a single straight line. Sphere A of mass M moving with velocity v strikes B, which, having acquired a certain velocity, strikes C of mass m. What mass should B have so that C will have the greatest possible velocity? 888. N identical electric cells can be formed into a battery in different ways by combining n cells in series and then combining the resulting groups (the number of groups is ~) in parallel. The current supplied by this battery is given by the formula 1= Nne
N R+n 2 ,
,

where <fj is the electromotive force of one cell, , is its internal resistance, and R is its external resistance. For what value of n will the battery produce the greatest current? 889. Determine the diameter y of a circular opening in the body of a dam for which the discharge of water per second Q will be greatest, if Q = cy Vh-y, where h is the depth of the lowest point of the opening (Iz and the empirical coefficient care constant). 890. If Xl' XI' .. , x n are the results of Illeasurernents of equal precision of a quantity x, then its Inost probable value will be that for wh ich the sunl of the squares of the errors
(}=

~ (x-xi)Z
i=l

is of least value (the principle of least squares). Prove that the nl0st probable value of x is the arithmetic mean of the measurements.
Sec. 2. The Direction of Concavity. Points of Inflection
to. Thr concavity of the graph of a function. We say that the graph of a differentiable function y = f (x) is concave down in the interval (a,b) [concave up in the interval (al,b l )] if for a < x < b the arc of the curve is below (or for a < x < bl above) the tangent drawn at any point of the interval (a,b) or ot the interval (a1,b/)] (Fig. 29). A sufficient condition for the concavity downwards (upwards) 0 a graph Y=f (x) is that the following inequality befulfilled in the appropriate interval: f" (x) < 0 If" (x) > 0].
2. Points of Inftectlon. A point (x o, f (x o)) at which the direction of concavity of the graph of some function changes is called a point 0/ inflectIon (Fig. 29).

'92

Extrema and the Geonletrtc Appllcatlons of a Derivative


For the abscissa of the point of inflection
Xo

[Ch. 3

y = {(x) there is no second derivat ive {" (x o) -= 0 or t" (x o). Points at wh ich I" (x) == 0 or f" (x) does not exist are called critical points of the second kind. The critical point of the second kind Xo is the abscissa of the point of inflection if {" (x) retains constant signs in the intervals x o-6 < x < X o and X o < x < xo +6, where 6 is some posi-

of the graph of a function

tive number; provided these signs are opposite. And it is not a point of inflection if the signs of fIt (x) are the same in the above-indicated intervals. Example t. Determine the intervals of concavity and convexity and also the points of inflection of the Gaussian curve

y=e- x2
Solution. We have

x
and

y' = -2xe- x

y" = (4x 2 - 2) e- xz .

Equating the second derivative y" to zero, we find the critical points of tHe second kind 1 X 1 = - Y2 and x t = Y2 These points divide the number scale - 00 < x < + 00 into three intervals: 1 (-00, Xl)' II (Xl' XI)' and III (x z' +(0). The signs of y" will be, re~pec-

I f I}'

.
Fig. 30 Fig. 31

1ively, +, -, + (this is obvious if, for example, we take one ~oint in each of the intervals and substitute the corresponding values of x into y) Therefore:
II the curve is concave up when curve
IS

00< x < -

concave down when- Y2

1 < x < Y2' 1


+

;2 ;2
and

< x < + 00;

2) the

The poInts

( Y2' \ Ye 1) are

points of inflection (Fig. 30). It will be noted that due to the symmetry of the Gaussian curve about the y-axis, it would be sufficient to investigate the sign of the concavity of this curve on the semiaxis 0 < x < 00 alone.

Sec. 3]

Asymptotes
x+2. Vs

93

Example 2. Find the points of inflection of the graph of the function y= Solution. We have:

if =

2 (x --t- 2) - . "9

-2

(1 )

It is obvious that y" does not vanish anywhere. Equating to zero the denominator of the fraction on the right of (I), we find that y" does not exist for x==-2. Since y">O for x<-2 and y"<Ofor x> -2, it follows that (-2,0) is the point of inflection (Fig. 31). The tangent at this point is parallel to the axis of ordinates, since the first derivative y' is infinite at X= -2.

Find the intervals of concavity and the points of Inflection


of the graphs of the following functions: 891. y=x' -6x2 +12x+4. 896. y=cosx. 892. y=(x+l)~. 897. y=-x-sinx.

893.

Y=X~3'

898. y=x1lnx.

894. Y =

X2+12.

Xl

899. y = arc tan x-x.


900. y = (1

895. y =

V 4x'-12x.

+x

2 ) eX.

Sec. 3. Asymptotes
{I::-=

f (x) in such a \\'uy that at least one of its coordinates approaches infinity

to. Definition. If a point (x,Y) is in continuolIs Inotion along a curve

(aud at the salllC tinlc the distance of the point fronl sonle straight line tends

to zero), then this straight 1ine is called an asynzptote of the curve. 2. Vertical asymptotes. If there is a nunlber a stich that
Ihn
x~a

f ( t) =-= 00 ,

then the straight line x=a is an aSYlllptote (vertical asymptote). 3 Inclined asymptotes. If there are limits
X~+CX)

lim f(x) =k x I

and
lim
X-++
00

rf (x)-ktx] = bt ,

then the straight line y= ktx+ b1 will be an asymptote (a right inclined asymptote or, when k 1 = 0, a right horizontal asymptote). If there are limits

9_4

E_x_t_'e_rn_a_Q_n_d_t_h_e_G_e_o_m_et_'_ic_A..;..p..;...p_li_ca_t_to_n_s_o-=-'_a __D_e_'_lv_a_t_tv _e__ [ell. ,1

and
x-+-oo

Hrn [f (x) -krl = b2 ,

then the straight line y = k1x + b2 is an asymptote (a left inclined asymptote or, when k 2 = 0, a left horizontal asymptote). The graph of the function y = f (x) (we assume the function is single-valued) cannot have more than one ri~ht (inclined or horizontal) and more than one left (inclined or horizontal) asytnptote. Example t. Find the asymptotes of the curve x2 Y= xl-l
Solution. Equating the denominator to zero, we get two vertical aSYlnp-

1ote"
x=-l
y

and x=1.
~

We seek the InclIned asymptotes. For x


k 1 = lim ~ -~ +
- = lim x x-+-+ 00
2

-+ 00
2

we obtain

IX)

Y x2 -

x _ _ --1, 1
2

b,=--=lJm(y-x)==ltm
X~+-:r..

. x -x Yx -1 ==0, V-I) t'~+oo x"-l

-1

a
Fig. 32

h~nce,

the straight line y=x is the right asymptote. Similarly, whenx---..-oo, we have
k a= lim
x-+- CJ) x
x-+-CJ)

1..= -1;

ba=

lim (y+x)=O.

Thus, the left asymptote is y= -x (Fig. 32). Testing a curve for asymptotes is simplified if we take into consideration the symmetry of the curve. Example 2. Find the asymptotes of the curve

y=x+lnx.

Sec. 3]

Asymptute~

95

Solution. Since
-++o

lim y=

-00,

the straight line x=Q is a vertical asymptote (lower). Let us now test the curve onl y for the inclined right asymptote (since x> 0). We have:
k=

Urn
X-++(J)

1L=1,
x

b= lim (y-x)=
X--"'+
00

Urn In X=
X -++
00

00.

Hence, there is no inclined aSYlllptote. If a curve is represented by the paratnetric equations x=cp (t), Y=1J' (t), then we first test to find out whether there are any val ues of the parameter t for which one of the functions cp (t) or 1J' (t) becomes infinite, while the other remai ns finite. When cp (to) = 00 and 1J' (to) = c, the curve has a horizontal aSYlllptote y=c. When 1J'(t o)=oo and cp(to)=c, the curve has a vertical aSYlnptote x == c. If cp (t 0) = 1J' (t 0) = 00 and
I-+t o

lim ' ~) === k; lim ['I' (t) - k({1 (t) 1== b,


cP
t -+ t n

then the curve has an incltned aSYlllptote y==kx+b. If the curve is represented by a polar eq uation , === f (cp), then we can find its asynlptotes by the preceding rule after transfortlling the equation of the curve to the parailletric form by the fortnulas x =, cos cp = f (cp) cos q>; y sin cp ==- f (q sin (p.

==,

Find the asynlptotes of the following curves:


901. Y =
1
(X-2)2

908. y=x-2+V~' -\1+ 909. y=e- x2 +2.


910. Y=-l -ex 911. y=e x _sinx 912
1

Xl

902. Y=x 2 -4x-1-3.


903. Y = x 2 -4 904. Y=Xi+9 905. y =
x'

x2

V xl--l.
Y xl-t x

y- x

906. Y=

907. y=

3 x2 + 1 xl-l

913. y = 1n (1 -t- x).

914. x=t; y=t+2arctant.


a

915. Find the asymptote of the hyperbol ic spiral r=-. q>

96

Extrema and the Qeo/netrle Applications of a Derivative

[Ch. 3)

Sec. 4. Graphing Functions by Characteristic Points


In constructing the graph of a function, first find its domain of definition and then determine the behaviour of the function on the boundary of this domain. It is also useful to note any peculiarities of the function (if there are any), such as symnletry, periodicity, constancy of sign, Inonotonicity, etc. Then find any points of discontinuity, bending points, points of inflection, asymptotes, etc. These elelnents hel p to determine the general nature of the graph of the function and to obtain a mathematically correct outline of it. Example t. Construct the gra ph of the function x Y= x2 _ 1 .

Solution. The function 0(0, 0). This b) The

a) The function exists everywhere except at the points x== 1. is odd, and therefore the graph is sytTlmetric about the point simplifies construction of the graph discontinuities are x = - 1 and x = 1; and lim y = 00 and hence, the straight lines x=

lim y=
X~-l

graph. c) We seck inclined asymptotes, and find


kl =
bl

to

00;

~-H+O

are vertical asymptotes of the

lint
X---++
00

!I =0,
X
00,

x-.+oo

lim y=

thus, there is no right asymptote. From the symmetry of the curve it follows that there is no left-hand asymptote either. d) We find the critical pOints of the first and second kinds, tha tis, pOints at wh1ch the first (or, respectively, the second) derivative of the giveu function vanishes or does not exist. We have: . x 2 -3 y' (I) 3 (x 2 -1)'" 2x (9-x 2 ) y" (2) 9V(x 2 -1)7

The derivatives y' and y" are nonexistent only at x= 1, that is, only at pOints where the function y itself does not exist; and so the critical points are only those at which y' and y" vanish. From (1) and (2) it follows that
y'=o y"=O

when x= V3; when x=O and x= 3.

Thus, y' retains a constant sign in each of the intervals ( - 00, - V3)t -1), (-1,1), (1, 3) and (V~ +00), and y"-in each of the intervals (-00, -3), (-3, -1), (-1,0), (0, 1), (1,3) and (3, +00). To determine the signs of y' (or, respectively, y") tn each of the indicated intervals, it is sufficient to determine the sign of y' (or y") at some one point of each of these intervals.

(-3:

Sec 4]

Graphinl! Functions by Characteristic Points

91

It is convenient to tabulate the results of such an investigation (Table I), calculating also the ordinates of the characteristic points of the graph of the function. It will be noted that due to the oddness of the function y, it is enough to calculate only for x ~ 0; the left-hand half of the graph is constructed by the principle of odd symmetry.
Table I

(0, 1)

1(1,

V3) I V3~1. 73 1( V3. 3) I


-1-

1(3. +00)

IJ

-I

oo

V'2

----=~1.37

V3

1.5

-t-

IeXIst non\lIon l~X I~t


o

II"

----ConeluSlOlh

I
del

Function
I t::a-;c~, grdph h

I I
I

+
+

+
0

+
Min. point

I I

Point
of

mllcctlon

lOl1l.1ve do\\n

Fund Ion Ol,-"on- dcer ea~c'5, ttnui t I graph IS conCAve up

FunctlOn Inrrca-,e'l; graph IS lOlll3ve up

Point Function il1r:-rcn"cc;; of graph illf1ec15 con.::ave han down

e) Using the results of the investIgation, \ve construct the graph of the function (FIg 33).

,
----.-r---r----1~~r--~-~--~I---__

Fig. 33
4-1900

98

Extrema and the Geometric Applications


Example 2. Graph the function In x y=-.
x

of a Derivative

Solution. a) The domain of definition of the function is 0 < x < + 00. b) There are no discontinuities in the domain of definition, but ae; we approach the boundary point (x = 0) of the domain of definition we have In = x - 00. I Imy= I'1m x-+-o x-+-o
X

the straight line x=O (ordinate axis) is a vertical asymptote. c) We seek the right asymptote (there is no left asymptote, since x cannot tend to - 00):
k=
X--++(SJ

~Ience,

lim lL-o
X -

b=

lim
X~+(SJ

y=O.

The right a~ymptote is the axis of abscissas: y =0. d) We find the critical points; and have
Y =-x-2- '
y

I-Inx

,/' = 2 In x-3 .
x3
'

'I' and yIP exist at all points of the domain of definition of the f unction and y'=O when Inx=l, that is, when x=e;

if =0

when In x =

that is, when x =e*,

We form a tabl~, including the characteristic points (Table II). In addition to the characteristic points it is useful to Hnd the points of intersection of

Fig. 34 the curve with the coordinate axes. Putting y=O, we find x= 1 (the point of intersection of the curve with the axis of abscissas); the curve does not ;ntersect the axis of ordinates e) Util izing the results of investigation, we construct the graph of the .unction (Fig. 34).

Table II

I
I

I
I
I

I
,
(1, e)

I I
e:::::: 2.72

i
I I
f

(0. 1)
I

I I

I
y

I ;
I
I

I
-cc
!

;
I

I
-::::::0.37 e
1

(e. e +) I eI +
2
I

4.49

(3-.+00)
+
-

I 0

I
I
j

I
!
I
r

I
y'
I
I

JI--~O 33 3

I I
noneXlSt.

I I
i : I

I
-;-

I
I
I
I I
I

-+I

-r

I I
I

I
0
I

y"

I
I

I
-

nonexist

I
I

i
I

I
I
Funct
1"1(r

I
I

I
I

I
POlnt or inflec t lon

I
I

,
Funct
IIICrCtl"cl::.

I
I

Conclu Slons

Boundary pOint of domain or dt..f. 0' :unctlOO

;
I

I
~

F ,In' t
11" 1

I
~

gTdph

I"

Vert1C.al asymptote

concave.:
do\'"n

RI .lr It
conca\ c

~
IS.

do\\. n

J:!f,lph lone-ave do\\n

I I
I
Of

J\\ax
pOInt

funl1.

Funct decr . graph l~ c.oncave dov.n

Function decreases; graph l~ concave up

100

Extrema and the Geo/netrlc Applications of a Derivative

[Ch.

Graph the following functions and determine for each function Its domain of definition, discontinuities, extremal points, intervals of increase and decrease, points of inflection of its graph, the direction of concavity, and also the asymptotes. -- d - a l 916. y=x -3x 939. Y= V x+ I-V x-I. 6x 2 _X 4 940. Y= (X+4)2- (X-4)1. 917. Y=-g-' 941. y=V(X--2)2+V(x-4)~ 4 918. Y = (x-I)I (x+ 2). 942. Y= Y .

V- -

919.

y=(X-2):(X+4).

920. Y= 2 921 = x -2x+2 Y x-I' x4-3 922. Y=-x-' 923. Y=


4

(x 2 _ 5)a 125 .

943. Y= x Y~2_4' 944. Y=

4-x 2 8

V_x _.
x 2 -1 x

945. Y=V~'
946. Y = xe- x .

+3 x

924. Y = x 2

+~ . x
I

925. Y= x 2 -t- 3'


926. Y = x Z--4 - .
927. Y= 4+x 2
4~

947. 948. y=e8X-x2-14. 949. y = (2+ x 2 ) e-x~. 950. Y =

y=(a+x;)eci-.
21 x 1- x
In x
2

928. Y = (x 2)2
x 929. Y=~4' x-

4x-12

951. Y= V-x' x2 x 952. Y=2"ln(j.


953. Y = fiiX . 954. Y= (x -t 1) In:! (X-i- 1).
955. Y = In (x
2 -

930. Y == x 2 (x-':4j
931 Y

16

3x

+1

1)
x

+- :t2~ I .
.
).

x:J'

932. Y =

933.

V-x 4- V4- x. Y~ VS -i-x- V~8-x.


Vx
3

956 Y= In

VX2 +)-1
+eX

957. y=ln(1

934. y=-xVx+ 3.

958. y = In ( e + ~ ) . 959. y = sin x + cos x.

935. Y=

-~.

936. Y= Vl-i2. 937. Y= VI-x', 938. y == 2x -1- 2-3

+sin 2x . 960 Y= sin x -2-. 961. y=cosx-cos l x. 962. Y = sin a x + cos' x.

'1/' (x +1)1.

963. Y= 8111 X +COSX .

S_e_c_._fi~l

D--::ifJ:.-e_'e_n_tt_Oa_1_0:-'_a_n_A_'_c_,_C_u_,v_a_tu_'_e sin x
sin (

l01

964. Y= 965. 966. 967. 968.

x+ :)

976. Y = arc cosh ( x 977. Y == e,lnx. 978. y = earc sin Vx. 979. y=e arc tanx.
98~.

+ ~ ).

y = sin x sin 2x. y = cos x co:; 2x.

y=x+ sinx.
y = arc sin (1dIe

V x::).

969. Y=

y._-' I-x!

~lTl

y = In sin x.

981. y = In tan (

T-i )

982. Y= Inx-arc tanx. 983. y = cos x-In cos x. I 972. Y = x arc tan -x \vhen x =1=0 981. y = arc tan (In x). and y=O when x=O. 985. !J = arc sin In (Xl + 1). 973. y = x-2 arc cot x. 986. Y = xx. x I 974. y = 2+ arc tan x. 987. y=x x 975. y =--= In sin x. A good exercise is to graph the functions indicated in Examples ~~6-84Ho Construct the graphs of the following functions represented paralnetrica 11y. 988. x=(1.-2t, y==II_t-2t. 989. x=-::al'o~3t, y-==asint (a>O). 990. x=te', y=te- t t 991. x=l +e- , y=2t+-e- 21 992. x=a (sinh/-I), y===a(cosh/-l) (a>O).
Sec. 5. Differential of an Arc. Curv&ture
1. Differential of an arc. The dIfferential of an arc s of a plane curve repre"ented by an equation in Cartesian coordinates x and y is expressed by the fornl ula
ds === V(dX)2

970. y = 2x-tan x. 971. y=xarctanxo

+ (dy)2;
dx ;

here, if the equation of the curve IS of the form


a) y={(x), then ds=

vl+(tf

b) x={.(y).

then ds=

vl+(~friy;
V-(-~-)-2-+-(-~-)-2dt;
II F'l + F'l
dx=
It

c) x=q> (I). y= (t), then ds=

V F'2 + F'l
d) F(x, y)=O, then ds=

I~~I

I~~I

"dy.

102

Extrema and the Geometric Applications of a Derzvatlve

[Ch. 8

Denoting by a the angle fornled by the tangent (I n the direction of increasing arc of the curve s) with the positive x-direction, \ve get dx cos a::=; ds '
.
Sill

u== ds.

dy

In polar coordinates,
ds=

y' (dr)t+(r drp)t =

rt-l-

(~)). dIp

Denot1n~ by ~ the angle between the radius vector of the point of the curve and the tangent to the curve at this point, \ve have dr cos A.==_
t-'

ds'

sin

~ =-= r~;

.
ds '
dt

2. Curvature of a curve. Thr curvature K of a curve at one of It,;; points AI is the 11n1lt of the ratIo of the angle between the POSltIVl~ directinns of the tangents at the points M and N of the curve (angle of COl1ftll6ence) to tlH. length of the arc 1\1 N -.=: As when ~V -~ M (FIt::. 35), that I~,
~

K = Ii rn ~\~ = da
~I\ ~
() /\')

\\hC're

U IS the angle oetwe(:'!J the posItIve directIons of the tangent pOInt 1\-1 and the x-aXIS.

the

o
FIg.

35

The radius of curvature R is the reciprocal of the absolute value of the curvature, i. e., 1

R=iKl

The circle

(K =~.

where a is the radius of the circle) and the straIght

line (K=~ are lines of constant curvature.

Sec 5]

Differential of an Arc. Curvature

103

We have the fo))owing formulas for computing the curvature in rectangular coordinates (accurate to within the sign): 1) if the curve is given by an equation explicitly, Y=f (x), then

K-

y"

y)

- (1 .,- y'2)'/J '

2) If the curve IS gIven by an equation implIcitly, Ftx,

=-=0, then

F;x F: u F~

K= (F't + F'2)3/ x . y
y = (t), then

F~x F~y F~ F~ F11 0


J

3) if the curve is represented by equation.; in paralnetric form, x=<p (t),

K=
\vhere
, dx rtt '

I
(x't

x' x"

+ y'2)J/

Il' .

y' \

,
2

x
\vr have

In polar coordInates, \vhen the curve is given by the equation r=f

(~),

\vhcre
r =-

dr

dcp

an d

3. Circle of curvature. The clrcle of curvature (or osculati1t~ ctrcle) of a curve at the pOInt AI IS the lill1itln~ position of a circle dra\vn throu~h M ~nci t\VO other pOlllts of the curve, P and Q, as P ---+ M and Q- .. 1\-1. The radius of the circle of curvature is equal to the radius of curvature, and the centre of the circle of cur\'ature (the centre of curvature) ties on the nOrInClI to the curvr dra\\'n at the point M in the dIrectIon of concavity of the curve. The coordInates X and Y of the centre of curvature of th~ curve are conl puted fr0J11 the fornllJl 3'i

X=x-

y' ( 1

+ [1'2) 1 + y'l " ,Y=y+--,,-.


y

The eLlo[ute of a curve is the locus of the centres of curvature of the curve. If in the fonnulas for deternlining the coordinates of the centre of curvature we regard X and Y as the current coordinates of a point of the evolute, then these fOrtl1Ulas yield pnralnetric equations of the evolute \\ ith parameter x or Y (or f, if the curvE' itself is represented bV equations in paralTIctric f orin) Example 1. FInd the equation of th~ evolute of the parabola y =x~.

104

Extrema and the Geometr,c Applications of a Derivative

(Ch. 3'

6x Solution. X = - 4x', Y = I +2 '. Eliminating the parameter x, we find

the equation of the evolute in explicit form,

1 3 (X Y=2+ T

)2/.

The lnvolute of a curve is a curve for which the given curve i~ an evolute. The normal Me of the involute r 2 is a tangent to the evolute r,; the """--" length of the arc CC l of the evolute is equal to the corresponding increnlent in the radius of curvature CC.:::-::M.C1-MC; that is why the involute r 2 is also called the evolvent of the curve r 1 obtained by unwinding a taut thread wound onto r, (Fig. 36). To each evolute there corresponds an infinitude of involutes. which are related to dIfferent initial lengths of thread. 40 Vertices of a curve. The vertex of a curve is a point of the curve at which the curvature has a maximum or a minimurn. To dpternline the vertices of a curve, we fornl the expression of the curvature K and find its extrelnal points. I n place of the curvature K we can take the radius of curvature R= I ~I and seek its extremal points if the computations are simpler in this case. Example 2. Find the vertex of the catenary x y=a cosh - (a > 0).
a

Fig. 36

Solution.

Since

Y'=sinh!-andy,,=J..cosh~, a a a
x dR

it follows that K=
x

u cosh 2 !..
a

and, hence, R=acosh 2 - . We have -d =slnh2a x a

Equating

the derivative dd

R
x

to zero, we get sinh

2~=O, a

whence we

find the <;ole

critical point x=Q Computing the second derivative


it the value x=O,

~2X~

and putting IOto Therefore,

wegetdd2~1 o=~cosh2!..-1 o=!>o. x x= a a x== a

x=O is the minimuln point of the radius of curvature (or of thfl nlaximum

of curvature) of the catenary. The vprtex of the catenary y = a cosh.!.. is, (l thus, the point A (0, a).

of the angle forIned, with the positive x-direction~ by the tangent

Find the differential of the arc, and also the cosine and sine

to each of the following curves: 993. Xl + y2 = a 2 (circle). x2 y2 . 994. a 2 +b 2 =1 (ellipse). 995 y" = 2px (parabola).

Sec. 5]
996.
X 2fs

DiUerenttal of an Arc. Curvature

105

997. y =

a 2 / S (astroid). a cosh.!(catenary). a
y2{a

998. x=a(t-sint); y===a(l-cost) (cycloid).

999. x=acosSt, y=asin1t (astroid).

find the differential of the arc, and also the cosine or sine of the angle formed by the radius vector and the tangent to each of the following curves: 1000. r = acp (spiral of Archimedes).
1001. r=!!.. (hyperbolic spiral).
q>

1002. r=a sec


1003.

2 ;

(parabola).

r = acos 2 ; (cardioid). 1004. r=aCP (Iogarithlllic spiral). 1005. r'l.=a 2 cos2q> (lemniscate). COIllpute the curvature of the given curves at the indicated points: 1006. y=x4.-4x ' -18x z at the coordinate origin. 1007. Xl + x!I + y2 = 3 at the poi 11 t (I, 1). x2 ,l1008. a l +;;2 = 1 at the vertices A (a, 0) and 8(0, b). 1009. x=f\ 1I=t 3 at the point (1, I). 1010. r 2 = 2a 2 cos 2ep at the vertices {p = 0 and <p == 1t. lOll. At \vhat point of the parabola 1/2=8x is the curvature equal to O.12H? 1012. Find the vertex of the curve y=-e"'. Find the radii of curvature (at any point) of the given hnes: 1013. y=x s (cubic parabola). x2 ~~ 1014. a 2 + b 2 = I (ellIpse).
1015. x=-=

~~_In:/ .

1016. x=acos1t; y=asinSt (astroid). 1017. x===a(cost-+ tsint); y=a(sint-t~ost) involute of a

circle).
1018. , = aekq> (Iogarithlllic spi ral). 1019. r=-a(l +COSfp) (cardioid). 1020. Find the least value of the radius of curvature of the

para bola y2 = 2px. 1021. Prove that the radi us of curvature of the catenary y = a cosh ~ is eq II aI to a segnlent of the nornla 1. a <:ofilpute the coordinates of the centre of curvature of the given curves at the indicated points:

]06

Extrema and the Geometric Applications of a DerIvative

(Ch.3

1022. xy = 1 at the point (1, 1). 1023. ayl = x' at the point (a, a).

Write the equations of the circles of curvature of the given curves at the indicated points: 1024. y=x z -6x+ 10 at the point (3, 1). 1025. y = eX at the point (0, I). Find the evolutes of the curves: 1026. y2 = 2px (parabola). x2 y2 1027. a2 b2 = 1 (ellipse). 1028. Prove that the evolute of the cycloid

x=a(t-sint),

y=a(l-cost)

is a displaced cycloid. 1029. Prove that the evolute of the logarithmic spiral

r = aekcp
is also a logarithmic spiral with the same pole. 1030. Show that the curve (the involute of a circle) x = a (~os t + t sin t), Y =- a (sin t - tens t) is the Involute of the circle x = a co~
t~ y =

a sin t.

Chapter IV

INDEFiNITE INTEGRALS

Sec. 1. Direct Integration


to. Basic rules of integration.
1) IfF' (x)
=- -;

f (.\), t h P 11

~f
where C is
all

(x) d x -= F

~x) t- C

nrbttrary

~onstant.

2) ~ Af (x) dx ~- A ~ f (x) dx, when' A is a constant quantity.


3)

~ [f1{X)::l:f,{"')ld\-'~fl(X)dx~ f 2 (x)dx.

4) 1f

Sf (x) d x --- F ( \") -t- C


~ f

nd

II --:

(f' (t), the n

(ll) du --:: F (u)

-f- (;.
{(/. ~ 0) .

In particular,

Sf

(Ll.\

b) d \:

-!r F (a.\.
I.

j b)

1- C

2. Table of standard integrals.


l

1.
t

\ "d \" - - -- IZ-:

.\"+1

! - (;
I ,

11

=1= -

II.
I I l. IV

Cd~ ~-= In I x I -; J x

C.
t

5---=:-x 2 -t-a 2
Z

dx

tlrc tan -

f-C==- -

(l

arc cot -

\:

+C

(a

=F 0).

r X~:-_:.!\nlx-al-t_C J 2a x -1- a
_(12

(a#O).

V. (' y~_==lnJx+yx2-I:aJ+C (a#O). J x +1 . x +C =-arccos-+ x C VI. ..r dx arc~ln2


to.

5
\

a"-- x

"t

dx .2=-==2 In a+ ",]

Ia-x
(a

+c

(a

0).

y a 2 -x 2

VII.

5aXdx~:

In a

aX

+C

>

0);

(a> 0).

eXdx =e X

+ c.

108

Indefinite Integrals

(Ch.4

VIII. ~ slnxdx=-cosx+C.
lX. ~ cosxdx=sinx+C.
X X 1.

5 d~
Sll1 Z

5.d~ x cot + C. Xli 5 s~:x=ln Itan ~ I+C=ln Iczosl'C x-cotxl+C. X II I. 5 c~:x = In Itan ( i + : ) 1+ C = In I tan .. + sec x 1+ C.
= x

cos x

=tan x-l-C.

XIV. XV.
XVI. XVII.
EX81npie 1.

S sinhxdx=coshx+C.
Scosh x dx= sinh x+ C.

5 .dh
sin

r cos dX =tanhx-{-C. h2 x J
X2

=-cothx+C.

(ax 2 +bx+c) dx=

Sax dx+ Sb.l.dx+ Scdx= =a 5 x dx + b 5 x dx -f- e


2

5 =dx

x -f- b -i x~ -1- ex -t- C. a "3


3

Applying the basic rules 1, 2, 3 and the formulas of integration, find the following integrals:
1031. ~ 5d 2 x 6dx.
1032. 1033. 1034. 1035. 1036. 1037.

1040. 1041

.J

r (X2+.:j~-2L dx.
,\'~

S (6x +Rx+3)dx. S x (x -+ a)(x -t b) ix.


S(a + nx3)2 dx.

1042. 1043. 1044. 1045.

SV2pxdx. S;~.

5(nx)-n- dx.
l-n

5X2~ 5X2~ JV +-

S(x:-=-\Il~ I JI"l lX. jl (V-a- V-;)4 - V a~-- dx.


7'
10'

dx 4 x2

1038.5

1039.

('J(Vx-l-l)(x-VX-J--l)dx. -

(aT -xTrdx.

JY2-t-x V2-x 1047. --Y4_;4-- dx .


2 -

1046..

~ V:~X2'

__

Sec 1]

Direct Integration

109

1048*. a) ) tan 2 xdx;


b) ) tanh l x dx.

1049. a) )cot'lxdx;

b) ) cothl X dx.
1050. ) 3 x ex dx,

3. Integration under the sign of the differential. Rule 4 consid(lrably expands the table of standard integrals: by virtue of this rule the table of integrals holds true irrespective of whether the variable of Integration is an independent variable or a differentiable function. Exanlple 2.

Y5x-2 S~~

~S (5x-2) -+ d (5x-2) = 5
1

= -5

S -- du==- 1 u 2 (5x-2) -+C=1 2

1 2

1 2

2-+C=- Y5x-2 f-C


5

'

2"

where we put u == 5x-2. \Ve took advantage of Rule 4 and tabular integral l. ~ x dx 1 ,. d (x 2 ) 1 -Example 3. V---=-2 I 2 =-2 In (xl + Yl+x~)+C. 1 +.r .J 1 + (x )2

y-

We iJllplied u =x2 , and use \vas made of Rule 4 and tabular Integral V. Example 4. x1e X" dx,~ ~ eX I d (x") = ~ ex1 + C by virtue of Rille 4 and tabular intt.\grat VII. III exatllples 2, 3, and 4 \\'(1 r(\duced the given integral to the following fornl before ITlaklng u,;e of a tabular integral:

) f (q> (x)

q" (x)

dx =

) f (u) dll,

where u = 'P (x),

This type of transfornlatioll is called IntegratIon under the differential sign. SOlne COlnnlon transforrnattoll~ of d Iff~\rcntials, which were lIsed in EX31npi es 2 and 3, are:
a) dx ==
LJ~ing

1 a d (ax -I

b)

(a

# 0); b) x dx = 2" d (x 2 ) and so on.

the basic rules and forIuulas of integration, find the following in-

tegrals:

1051**.5 ~dx . a-x 1052** S2X-I-~d


2.\ -1-1

1055.

x.

1056.

+ 5x +
a.x
2

ax+b
1

(1 dx.

1053.

5~~t ix.

Sx-I

dx.

r.:7sx2-t-fiX+7d IO Q x +:~ )C.


1058.

1054.

5aX::x'

SX4~':~ I dx.

110

Indefinite Integrals

lCh.4

S(a-l x b aYdX. 1060*. S (x ~ l)2 dx.


1059.

1078.

S2:'~3'
S
J
ax -1- b
a2x2

1079.

-t- b2 dx.

1061.

S
J

bdy ..r-. y I-y

1080.
1081.

r ;~. a -x4
4

1062. ~ Va-bxdx. 1063*.


1064. 1065.

SI ;2xe dx.
X2
t.

r .r
y

1082.

x2 + 1

dx.
1083.

..r-==== Y x 6 -1
2

dx

SYxt In x dx.

S3X~~-5' 1066. S 7X~~-8' 1067. S (a-t b)-~:-b) x'


(O<b<a).
1068.

1084. 1085

J JI
llfC

r . . / arc sin x dx. i-x


tan !-. 4 +X22 dx.

rx-

1086.

fV(l+x~)ln(x+ V-I
dx

Yafltan2x d
1 +4x 2

x.

+x 2 )

1087. ~ ae- mx dx. 1088. ~ 42 -ax dx. 1089. ~ (et_e- t ) dt.

SX'~:2dx.
Sa ~2dx. -x
x -f-4

1069.

1070 S~X-t-6d 2 1071. 1072.


1073.

x.

1090.

r Y~' J 7 +8~2
",'

r dx \ Y __ .
7 -5x 2

S(e-~ +- e-~ Ydx. (aX -- bX)2 1091. S dx. a -l 1092. J Ya" dx.
u>"b X
n
2X
X2

S3x2 __ 2dx.

2x-5

1093. ~ e-ex'+J) X dx. 1094. ~ x 7 dx.


" eX dx. 1095. J

4 107.
1075. 1076.

S 3x-t-1 __ dx. -t-l J Y 5x


f'

3-2x 5x2 -t- 7 dx.


2

- dx. x -4 Sy
2

x+3

1096.
1097.

J5

x2

yx

Yx'

dJl

1077.

~. Sx
-0

xdx

eX

eX - l dx.

Sec. 1]

Direct I ntegrat ion

III

1098. ~ eX

1099.

Va- be x dx. S(e-~ + l)~ e~dx.


S~ 2x +3
I
aX
2X

1119.

Stan xdx.

H20. Scot xdx.


1121. 1122. 1123.

1100 1101

5cota~bd"x.

5 +a dx

J tans

r~.

1102. 1103.

e-bx

l __ e-:.b."dx.
..I -==-==:t

Stan Vx

;-x.
+ 1) dx.
x
a

1104. ~ sin (a + bx) dx.

Sr

e'dt

l-e'

1124. ~ x cot (Xl


1125. 1126.

j' sin xdxcos x .


x
a

Ccos V x 1105. J 2 dx.


1106. ~ (cos ax 11 07. 1108.

5cos sin

sin - dx.

+ sin ax)! dx. dx .1r l'OS V-x Y-.t

1127. ~sin6xcos6xdx.
1128.

5sin (lg x) df

5sin' ax:3x .x.d II 29 x. 53 -tcos 3x


2

cos ax d

1109*. ~Sitl!xdx.

1130.

j" ..rsin.\ x - -C0S -- d ,X. y cos X-S1I1 x


2

1110*. ~ cos! xdx. III I. ~ sec (ax I b) dx.


1112. ~coeaxdx.

1131. ~
t 132 1133. 1134.

V 1 -1- 3 cos x sin 2x dx.


3

5t an
l

X 2 X d "3 sec "3 x.

J cos

ytanx I
2

x lX.

1113.)'~. . x
Sttl-

1114.
1115

J 3 cos ( 51:- :

dx

sIn (ax+b)

dx

51cos 4tx. 3x 1136. 5 t- sin


1135.
si t1 3x
2

-lX 2
Stn

cot

x f x .
l

(cos ax

ax)2

dx.

sin ax
2

1116.

1117. ~ x sin (1 - r) dx.

5cos

xdx
X

-2-2.

1137.

Sb-a cosec 3x d cot 3x x.


S (2 sinh 5x-3cosh 5x)dx.

1118.

SCinx\'2- 1

1138.

dX

1139. ~ sinh! xdx.

112

Jndefinite Jntegrals

(GIL 4

1140.

1141.

5 dx
Sill

-=--h. X

dx

1143. ) tanh x dx. 1144. ) cothxdx.


.

1142..

5sInh

x 5-h-. dx
cos

x cosh x

Find the indefinite integrals:


1145. ~ x tl5-xl dx. 1146. 1147.
1163.

5X.~';:-~
5X8~
xl

cosa

dX ' x

1 dx. 1164.

5 dx.

SV Ix+ In x dx.
dX StanVx-l .J'r x-I

1148. ~ xe1149.

dx.

1165. t 166.

t 150.

5:' /dx.
I-sin x

3- Y2t3x2 2-1- 3x 2 dx.

5!
J

'" dx 1151.JYex

1167.

Slll X Z

dx

eare t::ln x -1-X

1n( 1+x 2)-+_1

I -1"'\ 2
SUI

dx.

1168. 5SinX-C('SXdx.
x

1152. 1153.

5 t 154. 5 x
...\

5x + cos x dx.
tan 3x. -3cot 3x dx.
sin x
d 1

+ COS X
.
sin

(I-SilJ
1169.
t..

y'-2

V2Y dx.

1155. 1156.

V tan

X na x c;;ec2 x
2

1170.

5X,x'
5
J

2 dx.

x-2

dx.

1171.

(1 -1- ~f X(I+x 2 )dx.

S(2 +2XI~-I) 2X~~ I'

1172. ) e,ln' x sin 2x dx. 1173. \ 1174.


1175.

1157. ) a1nxcosxdx.
1158.

SV
,
v

X'
.\ 3

-1- I

dx.

V4-3x

5-3x

dx.
2

11 59.

xdx ~r--=. y I-x"


l

SeX~I'
J (l -i- b) + (a x S.r:.... dx. e _2
y
2X

dx

b) Xl

1160. ) tan ax dx. 1161.


1162.

(0< b <a).
1176.

5sin' ; dx.
SY4-tan s~_x x
2

I t 77.

r. dx J Sill ax cos ax

Sec. 2]

Integration by Substitution

113

1178. 1179. 1180.

Ssin C;t + fj>o) dt. Sx (4 -ln dx x) ) arc cos i


2

1185. 1186. 1187.

SY sec x tan x dx. secl x+ I


S4+C05 cos 2x d 2x x.
2

4-x 2

dx.

S1 +cos dx

x
2

1181. ~ e- tan x secl X dx. t 182. , sin x cns x dx.


" Y2-su1" x

Vx + I) 1188. ) Vln (x+ 1+,\2 dx.

1183. 1184.

'-

S f

1189. ~ Xl cos (x 3 -+ 3) dx. 1190.


\

SI11 2

dx x CCJs 2

afC SIll

X+x dx. l-x 2

'" cr.sh 2 x

--Jx

atanllA

Sec. 2. Integration by Substitution


to. Change of variable in an indefinite integral. Putting
x~qJ(t).

wIH'r~

t is a new variable and (p is a continuously differentiable function, \ve will have:

~ f (x)dx= ~ f [ljl (t ljl' (I) dt.

(1)

ThE' attelnpt is made to choose the function ({1 ill such a way that the right side of (I) beCOlnl'~ 1110rc convcl1i~nt for int~gratlon. EX31npie 1. Find

) x Y-t-I d.t.
Solution. It is natural to put t == Hence,

V x-I, \vhcnce x = t + 1 dnd


2

dx = 2t dt.

SOlnetilnes substitutions of the fornl are llsed. Suppose we succeeded in transforming the integrand
u = q> (x)

f (x) dx

to the foraD

f (x) dx = g (u) du, where u = q> (x).

llt

Indefinite Integrals

(Ch. 4

If ~ g (u) du is known, that is,

~ g (u) du = F (u) +0,


then

~ f (x) dx= F [q> (x)] +C.


Actually, we have already made use of this method in Sec. i ,3. Exam pIes 2, 3, 4 (Sec. 1) nlay be sol ved as follows:
Example 2. u = 5x- 2;
-=-d__...x......--..
du = Sdx;
1

dx = 5' duo

SV 5x-2
=-21SVdU 1+ x 1+ S.;tdt
4

J.. _du - = ~ ~ + C = ~
5

V "it

5 ~
2

Y-Sx---2 + C.

du Example 3. u =x 2 ; du = 2x dx; x dX="2 .


u
2

=-21tn(u+V1+U2J+c=-21tn(x2+Vl+x4)+C.
du

Exalnple 4. u = x 3 ;

du = 3x2 dx; x2 dx = 3" .

Sx eXJ
2

dx

=~

SeU

du

= ~ en +c = -} eX' + C.

2. Trigonometric su bstitutions. 1) If an integral contains the radical x= a sin t; whence

Va -x
2

the usual thin~

IS

to put

2) If

an integral contains the radical

whence

Vx 2 -a 2 , we put X~ll

"-CC

t.

3)

If an integral contains the radical V.\2+ a2, we put x=a tan t; whence

V x2 + a2 =

a sec t.

It should be noted that trigononletric substitutions do not al ways turn out to be advantageous. It is sometimes Illore convenient to make use of hyperbolic substit lltlOI1S, which are ~iInilar to trigonometric substitutions (see Exarnple 1209). For more details about trigonometric and hyperbolic substitutions, see Sec. 9.
Example 5. Find

JY?+1d
X
2

x.

Sec. 2)

Integration by Substitution
dt t
2

115

Solution. Put x= tan t. Therefore, dx=

5yX2+T S f
dx===
x2

cos 2

Ytan t -1--1 ~=S sec t cos t -.-!!:!.-= ta n2 t cos 2 t SIn 2 t cos 2 t dt = sin! 2t i- cos! t dt = ~ cos t dt = Sin! t cos t sin t . cos t cos t + sin 2 t
2

S
2

=--=\11 I tan t-{-sec t 1--.I_-t-C=ln Itan t-I- 1-1-tan 2 t 1Sin

- YTtTafiit +C=lnl x+ Y x + 11_ yx +1 +C. tan t x


1191. J\pplying the indicated substitutions, find the follo\vlng

integrals:
a

) I

. x V~ 2-~-2

d.\

'

x == 7 ;

b) c)
(1)

SeX~I'

x=-In(,

"' .I V
(.
fJ

~x(5x' --3)'dx,
~.t__

5x 2 ---:3=I;
r __

l\) ,

x -t- f ' cos x dx

1==1/x ~ 1;

"Vl~- ~1I12~~ ,
Stl i ta hIe

t :- - sin x.
su bst itut ions, find

App) ying

the f 0\ 1o\\'i ng integrals:

1192. ~ x (2x -\- 5)10 dx.

1W3.
t

\ I l-ry_.dx.

1194.
1195.
1196.

Jx
\

t -l-

/~_.
2~+ 1

y._.

SIn2xdx. In 4x x

dx eX-l

1199.
1200*.

e"

"' sin' x ..r-dx. \ Y cos \'

J x Vl-t-.\2

dt

Applying trigonolnetric substitutions, find the follo\ving integrals:


1201.

. j .r-
X2

dx
dx

1203.
1204*.

y I-x
Xl

j"'

1202.

lr
y

2-x2

.. ,\

I y_.
x
dx x 2 -1

~F.~

a! dx.

116

'"defi/ute /ntet!ral.s

rCll. 4

1205.

1206*. 1207.

1208. Evaluate the integral

j YX(l-x)
by means of the substitution x = sin 2 t.
1209. Find

,..

dx

~ Vl1 +x 2 dx,
by applying the hyperbolic substitution x=asinh t. Solution. We have: Va 2 + x 2 = Va 2 + a 2 Sl11h 2 t=a cosh t and dx=a cosh t dt.

Whence

~ Va2 +x2 dx= )


= a2

Scosh t dt = a 5 cosh ;t + 1 dt = a; ( ~ sinh 2t + t ) + C =


2
2

a cosh ta cosh t dt =

a2

(sinh t cosh t

+ t) +C.

Since

sinh t = and

a '

cosh t =

Ya+x a
2 2

et = cosh t -1- sinh t = - - - a

x+ Va _+x
2

we finally get
x a 5V q2+ X2 dx =2 a 2+.\2+ 2 I n(x+

Ya 2 +x 2 )+Cl'

--

where C. ~ C- a In a is a new arbitrary constant.


2

1210. Find

putting x==acosht.

Sec. 3.

Integr~tion

by Parts

A formula fer integration by parts. If u = <p (x) and v = '" (x) are differentiable functions, then

~ II dv = uv - ~ v duo

Sec. 31
Example 1. Find

IntegratIon by Parts

111

~ x In x dx.
Putting u=lnx, dv=xdx, we have dU=-X'
dx x2

v=2 Whence
x~

x2

5
We have

xlnxdx=2 Inx -

X2

52x=2 Int -4+ C.


t
2

dx

Sometimes, to reduce a ~iven integral to tabular foro), one has to apply the furmula of Integration by parts several tirnes. In certain ca~es, integration by parts yields an equation froIll which the desired integral is determined. Example 2. Find

~ eX cos xdx.

~ eX cos x dx= ~ eXd (sin x) = eX sin x- ~ e:' sin x dx= eX sin x-l-

+ ~ eX d (cos x) ~= eX 5111 x + e:c cos x- ~ eX cos x dx.


lienee,

~ eXcosxdx=eXslllx+excosx-~eXcosxdx,
whence

5
1211. ~ In xdx.

x
t

eot; cus

dx =--= e (sin x 2

+ cos x) + c.

Applying the forrTIula of integration by parts, filld the follo\ving integrals:


1221. \ x sin x co~ x d.\
..I

121 2. ~ arctan x dx. 1213. ~ arc


sill

1222* ~ (x 3 +5x+6) I.:OS 2xdx. 1223. ~


Xl

xlix.

In x dx.

1214. ~ x sill xrlx. 1215. ~ xcos3xdx.


1216.
x

1224. ~ In 3 xdx. 1225. 1226.

5eX dx.
Xl

5':.=- dx. S ~.;dX.


5111

1217. ~ x2- X dx. 1218**. ~


e dx.
_ 3X

1227. ~ x arc tan x dx.

1228. ~ x arc

x dx.

1219*. ~ (x l 1220*.

2x+5) e- x dx.

1229. ~ In (x + V 1 + x 2 )dx.
1230. j

5xae-~ dx.

xdx
SUI1X.

118

Jndefinite Jntegrals

[Ch.4

1231.

1232. ~ e" sin xdx. I 233. ~ 3" cos x dx.

Ssin! x

x cos x d

x.

1234. ~ eax sin bxdx. 1235. ~ sin (In x) dx.

Applying various methods, find the following integrals:


1236. ~ x 3 r
x2

dx.

1246.

1237. ~ eV~ dx. 1238. ~ (x 2-2x+3) In xdx.


I 239. 1240.

J
J

r"rt:~x dx.
l--x

1247. ~ xtan 2 2xdx.


1248.

Sx In : +; dx.
7dx. S

r sin; x dx. e

I 249. ~ cos 2 (In x) dx. 1250**.

1241.

In2x

5(X2~
r(X -1~xa J
2

1)2 dx .
2)2'

(~n x) dx.

1251 *.

I 242. ~

Xl

arc tan 3x dx.

1252*. ~Va2-x~dx.

I 243. ~ x (arc tan xY dx.

1253*. ~ V A -1- x 2 dx.


1254*. C x:!~
~

1244. ~ (arc sin X)I dx.


1245.

Sarc :~11 x lx.

, ..r _;(2 r 9 .

Sec. 4. Standard Integrals Containing a Quadratic Trinomial


to. Integrals of the form

5 mx+n+
ax 2 -l- bx

c x.

The principal calculation procedure is to reduce the quadratic trinomial to the for III ax! + bx+c=a (x+ k)2 + 1. (1)
where k and 1 are constant~. To perforrn the transformations in (1), it is best to take the perfect square out of the quadrattc trinomial. The follo-

wing substitution may also be used:

2ax+b=t. If m=O, then, reducing the quadratic trinomial to the form (1), we get the tabular integrals II I or IV {see Table).

Sec 4]

Standard Integrals Containlng a Quadrattc Trtnomtal

119

Example t.

5 I 1 x- 4 ----arc tan --+C= 2 V31 -4--4-

y3T

lr- arc tan 1r--+ C.


y 31 y 31

4x-5

If In : 0, then from the nUlnerator we can take the derivative 2ax + b out of the quadrattc trtnolnial

mx -1- n dx == ax 2 -t- bx -1- c

m (2ax -l- b) -12a 2a ax 2 bx + c

(n- nlb) dx =-=

n1 2. (mb ) -2a ln / ax +b.\:+c/+ n- 2a -

5axl+b,+c' dx

and thus we arrive at the Integral discussed above.


Example 2.
' x--l ) - - - - dx-= \2- x - l

ii(2X--l)-;
,\2-

x _1

- dl.:.=- In I x 2 _ x-l/2

}V + n dx. The luethods of calculation J 1l.\2+bx+c are sinlilar to thosE' analyzed above. The integral is finally reduced to tabu2. Integrals of the form
III X

lar integral V, if a > 0, and VI, if


Example 3.

<~

o.

J Y2\-3x-2x2=
Example 4.

r
t

dx

Y 2

J V~_ ('t- ~
dx

r
2

r y1
=

1 . 4x-3 arc Sill -5- + c.

dX=..!..S 2x+2 dx+2f dx j Yx x+3 +2x+2 2 Y x +2x+2 J Y (x+ 1)2+ 1


"

= Y x + 2x +2 +21n (x + 1 + Y x----+-2x-+-2) +O.


2

J20

I ndefi,nite Jntegrals
3. Integrals of the form

(Ch.4

verse substitution

S(mx+n) Vax +bx+c


dx
2

. By means of the in-

mx+n

1 --=t

these integrals are reduced to integrals of the form 2. Example 5. Find


Solution. We put

s+
(x

1)

~ x +1
2

x+l=T'
whence
dx:~

dt

f2.

We have:

(x+l) Vx 2+1
= - ; 2-

dx

--~ f 2 +V(+-lr+ 1 =-, Y-I-2t+2t = r y ( d\ r ;:1 nit - ~ + -V t2- t + ~ I + J


1'1
dt
t

1 =-

t- 2

+4

+c=- ;Cjlnll-X+x?t2+1)I+c.
4. Integral~ of the form ~ Y ax2 + bx + c dr. By taking the perfect square out of the quadratic trinOlninl, the given integral is reduced to one of the following two basic integrals (sec examples 1252 and 1253): 1)

a -x dx= - Va -x +-- arc j-'1r-'2 2


y
2 2

2 -2

a2

Sin

x -=-1- C; a

2)

(a > 0);

V x2+A dx= ; VA 2+A + ~

In I x+

Vx 2+A l+c.

5YI-2x-x 2dx= ~ Y2-(l+X)2d(l+x)=


=-2-

Example 6.

l-1-x .. r

. l+x y 1-2x-x2 +arcslO Y2 +C.

Pind the following inte6rals:


1255.

SX2+~:+5

1256.

S x ~2X
2

Sec. 5)

Integral ion of Rational Functions

121

1259. 1260. 1261. 1262. 1263.


1264.

1269.
1270.

x S '"
(l

dx yxz--t-x-I . dA-

"\ (x-I)

V.\2 -2
d-<
2

t 271.

J(x+l) Yx
2

1-2,'

1272. ~ x +2x+5dx.

1273. ~ Yx=?dx
1274. ~ 2-x-x 2 dy,
xdx
2

1275. 1265. 1266.


1267. 1276. 1277.

+3 Sx -4x cos x
4

J
f}

Sl11 2

X-6s1I1X+12

dx

..

eXdx
1_
SlIl \

" \+fx

1278.
t'

~2X d.\
2

Vc~ X + 4-~~~s~~- 1
In x d\
t'

1268.

1279.

Jx YI-lInx-li1

Sec. 5. Integration of Rational Functions


to. The nlethod of undeiernlined coefficients. IntegratIon of a rational function, after takIng out thf whole part, reduces to lIltehratlon of the proper ratlollal traetloll p (x)
l

Q (x)'

(1)

where P (x) and Q (x) are Integral polynollllals, and the degree of the numerator P (x) is lower than that of the denolninator Q (x). If Q (x) == (x-a)'1 . . . (x-l),\

where a, l are real distinct roots of the polyllotniiJl Q (x), and a, A are natural nUlllhcrs (root lllultiplicities), then decomposition of (1) i~t~ partial fractIons is justified: P (x) At A2 (-----)2 x-a +. Q (x) =:=:i x-a

a + (--)'X x-a +

- L1 Lz L). +x-l+(x-l)2++(x l)1\.

(2)

To calculate the undetertllined coefficients At, A 2 , , both sides of the identity (2) are reduced to an integral fornl, and then the coefficients of like powers of the variable x are equated (tlrst nlethod). These coefficients may like\vise be detennined by putting {in equation (2) or In an equi. valent equation] x equal to suitably chosen nUlnbers (second Inethod).

122
Example 1. Find

Indefinite Integrals

[Ch. 4

Solution. We have:

xdx (x-I) (x+ 1)2=1.

Whence
x==A (x-t-I)~-+-81 (x-I) (x+ l)+8 2 (x-I).
(3)

a) First "Iethod of deternunlnR the coefficients. We rewrite identity (3) in the (onn x = (A 8 1 ) x 2 (2A + B 2) x + (A - 8. - 8 2 ) Eq uati n~ the coefficients of identical powers of x, we get: O-==A+B.; 1==2A,-B 2 ; O==A-8.-B 2 Whence I A=- 4 '

b) Second method of determtning the coefficlents. PuttIng x= 1 in identity (3), we will have:

J=A.A, Putting x= -1, we get: -1=-8 2 -2,


\ve

Le.,

8 2 ==1/ 2

Further, puttIng x=O,

will have:
O==A--B.-8 2 ,

Example 2. Find

Solution. We have:

x3 -2x 2+x

1 A 8 x (x-I)2=X-+ x-I

+ (x-I)2
(4)

-and
I=A(x-I)2+Bx(x-l)-f-Cx.

When solving this example it is advisable to conlbine th e two methods of determining coefficients. Applying the second method, we put x=O in identity (4). We get 1= A. Then, putting x= I, we get I =C. Further, applying the first nlethod, we equate the coefficients of x 2 in identity (4), and
~et

Hence,

O=A+8,

Le.,

8=-1.

11=-1,

8.=.-1,

and

C=l.

Sec. 5]

Integration of Rational Functions

123'

Consequently,
1=

dx 5dx 5 dx I x-l+ (X_l)2=lnlxl-ln/x-11-x_lTC. 5-XI

If the polynonllal Q (x) has cOlnplex roots a partial fractions of the fornl
A1x+B, .\2+ px -f-q

of mu It1phcity k, then
(5)

+.

(x2+PX-~_(/)k

Akx -t-R k

will enter Into the expansion (2). lIere, x2+px+q=[x-(a-r-lb)] [x-(a--lb)J

and AI' B I , , A k , Bk are undetennined coefJiclents \vhlch are detennlned by the nlcthods given above For k =-= 1, the fraction (5) IS integrated din:ctIy; for k > 1) USl' is llHlde of the redLLctzoll Inethod; here, It is first advisable to represent the qua drahc trinonllal x'

+ px + q in the forIII ( x + f

+( q_~2)

-r

and make the substitution

x+%=z.

Example 3. Find

Solution. Sillce theil,


PUltlfl~ .\'-1-2---:2, Wl' gl\t

1 === J {2 2 t-

2-

02 az:=:;

(1:;- t 1) -

~ dz

J (Z2 +112

II (1 -t- z:!) -

z'1.

dz-==

1 C dl I ~- --~ lz~t") .--- J i~-n l- J Id . -2 (z:!+ I) --- -2 (2 2 + 1)z 1 z -1- 1 --drr tall z- - -- --- -1- - -life tan z= - - - - - ~ (2' 2

ell
2

-1-

1)

2 (Z2 . ~- 1)
2

2. The

~ aI ( t a 11 z --t- C :::.:: 2

2 (x 2 -t- 4 \'

\ -t- :3

+ 5) -

...!.- alet a 1J (x -.J - C)) _L C


I 1

Ostrograd~ky

fllcthod. If Q (.\) has Illultiple roots, then


(6)

tJ

\P(X)d\,:-=-:.~~t-1_rY(X) dx, Q (.\) (J 1 (.\) J Q2 (x)


COlll1110n

where Q I (x) is the greatest dcri vativ e Q' (x);

diVisor of the polynoillial Q (x)

and Its

Q! (x)

== Q (x): Ql (x);

x (x)

and Y (x) are polynonriClls \vtth undetcrrnincd coefficients, whose degrees arc, respectively, less by Ullity than those of Ql (x) and Q2 (x). The undetermined coefficients of the polynonllals X (x) and Y (X) are cOlnputed by differentiating the identity (6). Example 4. Find dx

J (x 3 -1)2

124
Solution.

Indefinite Integrals
~=AX2+Bx+C +5Dx2+Ex+F dx 3 Xl - I , x3 - 1 5 (x - I )2
(2Ax+B) (x 3 -1)-3x 2 (Ax 2 +Bx+C)
(x 3 _1)2

Different i at i ng th is identi ty, we get


I
{x -1)2 '
'Or

+ Dx 2+ Ex+F
x3 -1

1= (2Ax+ B) (x 3 -1)-3x l (Ax 2 + Bx+C) + (Dx 2 +Ex+F) (x3 _1).

Equating the coefficients of the respective degrees of x, we will have: D=O; E-A=O; F-2B:.=O; D+3C=0; E+2A=0; B+F=-l; whence
A=O;
B=

-"3; C=O; D=O; =0;


dx

F=-3"
(7)

and, consequently,

S(x --1)2=-'3 x - I - a
3 3

Sx dx -1
3

To compute the integral on the right of (7), we decompose the fraction


-.!-I into partial fractions:

x-

lhat is,
1=L{x 2 +x+I)+Mx{x-I)+N{x-I).
(8)

Putting x= I, we get

L=}.
L+M=O; L-N=I, N=--a.
2

Equating the coefficients of identical degrees of x on the right and left lof (8), we find

-or
M--_o 3'
I

Therefore,

dx

Xl -

I=

Sx -dx I -

3
I

Sx + x+2 d + I x=
2

=3 1n I x-ll-filn (x
and

+x+ I) - Y3 arc

tan

2x 4- I

V3

+C

dx

(x'-1)2

x J x2 X 1 2 2x 3(x3 _1)+gln (x-I)! +3 Y3"arctan

+ +

+I Y-3 +C.

FInd the fol1owlng integrals: dx 1280. (x+a) (x+ b)

1281

SX

1282.

-5t+9 .\,2-5x+6 dx

5ex + 1283. 5

dx
I) ex

+ 2) (x + ~)

2X2

-f- 41 x - 91

(x-I) {x+3)lx-4)dx

See. 6]

/ntegratinf;! Certain I,rational Functions

125

5 1285. 5 (Xd~_
1284.
x

5X'+2 x3_5A2+4xdx.
1)2'

1293. 1294. 1295. 1296. 1297. 1298. 1299. 1300.

1286.
1287. 1288.

S
5

4-3

-x dx. x4-6x3+ 12~2+6


X

r-I

(X_t-1)2 dx 5 x2-8x+7 1289. 5 3x_IO)2 dt . 2x-3 1290. 5 (x -3x+2)3 dx. -t-x+ I 1291. 5 (x2 + I) dx. 1292. 5 JI./ 1 dx.
(X-3)2
(-\2_
2

x3 -6-\2-t-12x-8 dx. 5r2 -t- 6x+ 9

X3

Applying Ostrogradsky's method, find the following integrals:


1301

5
5
C

{x

-1- 1)2 (x 2 + 1)2

dx

1303. 1304.

~)

r (.t d'( + 1)4 .


2

1302.

dx (t 4 _1)2'

J(.l2_2x+2 2dx .
n

xl - 2 \ 2 ~- 2
j

Applying different procedures, find the integrals:


1305.

5(.1. + 1)'"~X3 13

8)

dx.

1306. J x I2 -2\,4 1- 1 dx.

1310*5 Xlx~Xt I)' 1311.

\'3

5 1308. 5 1309. 5
1307.

X2_ \ -t- 14 (x-tl):' (x-2) dx.


d'(

5xlS~

1)2'

1312.
1313. 1314.

5(x2-1-2~+-2){X2+2x+5).

dx

.t4 {x3

-t- 1)2

dx x3 - 4x 2 -t- 5 ( - 2

5lt~I\'. 5~
Xl AG

Sec. 6.
1
0

Int~grating

Certain Irrational Functions


PI

Integra Is of the f .. rm

5
where

aX+b)Q; R [ .x. ( cx+d

ax+b\q~ ex +d) , ... dx,

P2

(1)

R is

a rattonal functIon and

PI' ql' Pa' qa are whole l1ul1lbers.

12J

Jnde{iniie Jniegra[s

(Ch.4

Integrals of form (1) are found by the substitution

ax -l- b n --=2 cx+d

where n is the least cornmon lTIulti pie of the lllllnbcrs q., q2' Example 1. Fi nd

\----~--J V 2x - 1- V 2x - 1

J V2x-I- V2t-1 J
Find the integrals:
1315.
1316.
t.\

Solution. The
d.\

~ubstitution

2.\ -1
= 2

C2z dz
3

Z2_ Z

J 2-1

= zo1 leads to
z dz =
2

an Integral of the fOrIn

=2S(Z+I+z

II)dZ=(Z+ll2+21nlz--ll+C=
==

(1-1- V2x=f)~ -I-ln (V2x-1-1)2 +c.


1321. 1322.

r ./3 _dx. r x-I


"

t.

-V
j

dx

lb. 1.\ -1)3'

'VX" I
\

dx.

x+2
_d_t_ ~r-1----\

1117.

,.. 1318. \)

JV~+l ~XV(~
V-=-l '-1- V- X
V\,+l
dx
t.

I ("2-- .\)
'V\r
'1

132:3.

.\

" J/.t--=-r - - - ([
\+ 1

.\..

i'

1324. C

'Ir "Y t- --1 laJ9. , ~-- -..==- dx.

13~5.

1 x -1- 3 -.r-'==-- dx. x 2 r 2x ~- J

~__1=1 dx.

\-

t 320. ('

J(x-l-l)2- YX-1-1

_tt+! +-=-_ dx.


SV~a).,-2~+-b-x-+-c dx
Pn (x)
I

20 I ntegraJ~ of the form

(2)

where Pn (x) is a polynol1llal of degree Put


~\ f

tt

lrax2

Pn(x)

+ bx + c dx~ Qn-l (x)

lr r axl+bx+c+",

S.r----.:.=' dx
r
a.\2

-t- bx -t- c

(3)

where Qn_l(X) is a polynolnial of degree (n-l) with undeternlined coefflcients and A is a num b~r. The coefficients of the polynomial Qn-l (x) and the nUlllber A are found by differentiating identity (3).

Sec. 6}

Integrating Certaln Irrational Functions

127

Whence

.\4~_4x2 =(3Ax2 +2Bx+C) Y x2 + 4


Multiplying by
x, \ve obtain

Yx

+4+(Ax +Bx +Cx+D)x+ .r.\ 2 -f- 4

Y +4
.l2

Yx +4
2

and equating the coefficients of identical degrees of


8==0,
1 C=-;
2

1 A =4 '

D=-=O;

A=-2

Hence,

3. Integrals of the form

5
Find the integrals:
1326.

(x-a)n

;:X

+bX+C'

They are reduced to lntrgra\s of the form (2) by the substitution:


..\-a
_I_==t .

1:J27.
1328.

r2dt
"If - - '- - - .

1329. 1330. 1331.

J'

,\2_ ~-1-1
r~

1r ~-==-"l dx.
J' )-"\"
X6

,\ r 1-1- x 2

1/--=====

dx.

4. Integrals of the binonlial differentials

x ln (a

+ bt")P dx,

(5)

where tn, nand p are rational Ilunzbers. Cheby~hev's conditions. The integral (5) can be expressed in tenns of a finite COlnbll1ation of e1rl11entary functions only in the following three cases: I) If P i" a whole nUlnber;
2) if 111-t-l is a whole llulllber. I-Iere, we Inake the substitution a+ bxn = n

= zS, where s is the denonlinator of the fraction p;

n ax-n+b=zs.

3) if

111-1-

I + P is a whole nUll1ber. Here, use is made of the substitution

-------------~---=------------=----

128

Indefinite Integrals

(Ch. 4

Example 3. Find

1 1 Solution. Here, nt= -2 ; n=T;

P=3" ;

--!"+l 2 m+ 1 1 n
4

=2. Hence,

we have here Case 2 integrability.

The substitution
1 +x 4
2

=2 3

yields x= (z3-1)4; dx= 12z (zl-I)3 dz Therefore,


I

x-

~(I +x+)+ dx= 12 5 z~;:~~:r dz=

where

Z=

I-I-

x.
3

= 12

S(z'-ZS) dz = z1-3z +C,


4

Find the integrals:


1332. ~
1333.
Xl

(l

+ 2x") -2 dx.

1335.

, x

"

, i / - 5.

dx

it'

1 +- x

SV~, 1 +~ .

1336. " 133 7. \

~ x"
"

dx
(2

~.
dx

+ x") -;-

1334.

'y 1+ x . ~4
2

dx

J Y.t

-3

V + Vx--
1
J

Sec. 7. Integrating Trigonometric Functions


1. Integrals of the form

sinmxcosnxdx=I m n'

(1 )

where nz and n are integers. 1) If m=2k+ 1 is an odd positive nUlllber, then we pUf
1m n = -

~ sin"k x cos n xd (cos x) = - ~ (I-cos" x)k cos n xd (cos x).


n is an odd positive number.
51.1
11

We do the sanle if
Exall1ple 1.

~ s1n1o x cos'x dx = ~

10

x (I-sin" x) d (sin x) =

sin x sin 13 x ;:::::-11--13+ C.

Sec. 7]

Integrating Trigonometric Functions

129
tran~

2) If m and n are even positive numbers, then the integrand (1) is formed by tneans of the formulas 1 sln2 x= 2 (I-cos 2x), 1 cos! x="2 (I
1

+ cos 2x).

sin x cos x="2 sin 2x.


Example 2. ~ cos 2 3x sin' 3x dx = ~ (cos 3x sin 3X)2 sln 2 3x dx =
=

5
8

sin2 6x 1- cos 6x 1 -42 dX=8

r (sin2 6x-sin 2 6x cos 6x) dx= J

15(I-COS12x. ="8 2 stn 2 6x cos 6x ) dx=

= ~ (~_ sin 12x _ .-!- sins 6x ) + C


2 24 18

3) If m = parity, then
1 m," =

~t

and n = - v are integral negative nunlbers

of

identical

5 5(
\

S1I11'- ::os' x
~

5
2

cosec'" x sec' -2 xd (tan x) =


V-2

1 -t- t - 2 -

1)

an x

(1

+ tan

x)

d (tan x)

=5 +
(1

tan 2 x) t lJ. an' x

--1

~+v

d (tan x).

In particular, the fotlo\ving integrals reduce to this case:

Example 3.

5 X=21'-~Jj~ -i) 54 = 5
SI::

dx ( x and d sintJ. - cosP- sin" x + 2 2 2

5co~'~ x=s

((+ ~\
seeS

cos x

sec 2 xd (tan x) =

5+
(I

tan 2 x) d (tan x) =

1 =lan x +"3 tan 3 x+ C.


Example 4.

5-; =ds ~
d:
Sill

sina

!.- cos' .!.


2 2

dx

=i 5

tan -3

1- dx-=

=8

1~(1+tan2 ;)2
tan
sX

sec 2 .!-dx=2 8

2)[

tan-a

2x !-.+-+ 2
tan

+tan

~] d( tan i )= ~

[-

2tan 22"

_I-x +21n Itan ~ 1+ ta:

2 ;]

-I- C.

5-1900

130

Indefinite Integrals

[Ch.4

4) Integrals of the form ~ tan m x dx (or ~ cot m x dx). where m is an integral positive number, are evaluated by the formula tan 2 x=secx-l (or, respectively, cot l x = cosec 2 x-i). Example 5.

tanS x tanS x -=-3-- (sec2x-l)dx=-3--tanx+x+C. 5) In the general case, integrals Jm n of the form (1) are evaluated by means of reduction formulas that are usually derived by integration by parts. 2 2 E 16 dx SSln x+cos x d xamp e . cos' x = cos' x x=

5
S

t an' x dx =

tan' x (sec' x-I) dx = ta~' x -

tan' x dx =

==

sin x S dx. sinx--dx+ --=slnx cos' x cos x 5 sin x


1

e ----

1 2 cos 2 X

1 2

5cos cos x 5dx --dx+ -= cos x


2

C:: 2-

2 cos x

+-2Inltanx+secxl+C.

Find the integrals:


1338. 1339. 1340. 1341. 1342. 1343. 1344. 1345. 1346. 1347. 1348. 1349.

~ cos' xdx.
~ sins xdx.

1352. )

sin - cos a -

dx

~ sin' x cos' x dx. S "x sIn 2" cos sX 2 d x.

1353. 1354. 1355. 1356. 1357. 1358. 1359. 1360. 1361. 1362. 1363. 1364.

) sin (x+ ~ ) . sin x cos x

dx.

Ssin' cos x d x x.
S

sinS x

dx

~ sec s 4x dx.

~ ~in' xdx.
~ sin~xcos'xdx.
~ sin! x cos' x dx.

~ tan' 5x dx.
~ cot' xdx.
~ cote xdx.
a

~ cos' 3xdx.

Ssin dx x Scos' dx x Ssin' cos' x d x x.


4

5'l tan "3 + tan 4' X) dx. ~ x sin' x'dx. 5 x.


x
4

cos' x d sin. x

J350.

1351.

Ssin' dxcos' x
x

sin l x cos 4 x

dx

S dx x SYtan dx x
YSin

~ sinS x Vcosxdx.
cos' x

Sec. 7]

1ntegrating T rigonometrlo Functions

131

2. Integrals of the form ~ sin mx cos nx dx, ~ sin mx sin nx dx and

~ cos mx cos nx dx. (n these cases the following formulas are usedl
I)

slnmxcosnx=~
I

[sin (m+n)x+sln(m-n) x];

1 2) sin mx sin nX="2 [cos (m-n) x-cos (m + n) xl;


3) cos mxcos nX="2 [cos (m-n) x +cos (m + n)

xl.

Example 7.

Ssin9xsin xdx= S~ [cos ax-cos lOx) dx=>


1369. ~ ros(ax t- b)ros(ax-b)dx. 1370. ~ sin rot sin (rot +cp) dt.
1371. ~ cos x cos 3x dx. 1372. ~ sin x sin 2x strl 3x dx.

=~ sin 8x- 2~ sin lOx + C.


Find the integrals: 1365.
1366.

~ sin 3x cos 5x dx. ~ sin lOx sin 15 xdx.


2" cos 3 x. Scos . x . 2x d
x x d

1367
1368

Ssin 3" sln"3

x.

30 Integrals of the form

~ R (sin x, cos x) dx,


where

(2)

1) By means of substitution

R is a rational {unct ion.

tan'j=t,
whence
. 2t
'

slnx=l+t l
new

COSt=l+tl'

I - t2

2dt

X=I+t l

'

integrals of form (2) are reduced to Integrals of rational functions by the variable t.
Example 8. Find

5+
1
2dt

dx sin x + cos x

Solution. Putting tan ~ =t, we will have

1=

1+t 2 2t I-t!

1+ l+ta+l+t l

132

Indefinite Integrals
2) If we have the identity R (- sin x, -cos x) e i R (sin x, cos x),

[Chi 4

and

Iben we can use the substitution tan x = t to reduce the integral (2) to a rational form. Here, t 1 sinx= '1r ' cosX= '1r2 f 1 t f I t2

x = arc tan

t,

dx = I
=/.

dt + t

Example 9. Find
Solut ion. Putting tan x= t,

51+ sin x
dx
2 l

(3)

sln x = 1 + tl
2

t2

'

dx=

1+ t2

dt

we will have
J= ~

(l+t

2 )

51+2t dt 1 S d (t y'2) t) =Y2 1+(tY-2)2 ( 1+ l+t


dt
2

= Y12

arc tan (t ,. 2) + C =

lr-

lrY1 2 arc tan (,. 2 tan x) + C.

We note that the integral (3) is evaluated faster if the nurnerator and denominator of the fraction are first divided by cos x. In individual cases, it is useful to apply artificial procedures (see, for example, 1379).

Find the integrals:

1374.

5 1375. 5
1376. 1378.

1373.

53+::0SX
dx sin x+ cos x ' cos x d 1+ cos x XI

1382* 1383*

5sin!..( + 3 sin xcos x-cos x 1384*. 5. :X. . sin x- sin x cos x

dx
2

3 sin 2 x + 5 cos 2 X

dx

1377.

58-4sinx+7cosx'
dx

SI-sin sin x d x XI

1385.

S(I-cos sin x
5

X)I

dx
I

1386.

sin 2x 1+ sin2 x dXI cos 2x

Scos x + dx 2 sin x + 3

1387.
1388. 1389*.

1379**. 53s~nx+2cosx d 1380.

Scos4 x + sin4 x

d
XI

51+

2 sin x

+ 3 cos x x.
I

1381 *

S1+:x . cos x
I

tanx dx I-tan x

5 .

cos x d sin 2 x-6sillx+5 X.


dx (2-s1n x) (3-sin x)

1390*,

Sl+s1nx-cosx I-s~nx+cosx d X

Sec. 8]

Integration of Hyperbolic Functions

133

Sec. 8. Integration of Hyperbolic Functions


Integration of hyperbolic functions is completely analogous to the integration of trigonometric functions. The following basic formulas should be remembered: 1) cosh! x-sinh! x = 1;
2) sinhl x =
3) cosh 2 x =

~
1

(cosh 2x-I);
2x + 1);

"2 (cosh

4) sinh x cosh x =

sinh 2x.

Example t. Find Solution. We have

Scosh

x dx=

5; (cosh 2x+ I) dx= ~ sinh 2x+ ; x+C.


~ cosh' x dx.

Example 2. Find Solution. We have

S cosh a x dx= Scosh l xd (sinh x) = S (I +sinh! x) d (sinh x) =


. ==slnh sinha x x+- +c. 3

Find the integrals:


1391. 1392. 1393.

S sinh' x dx.

1397. 1398.

S tanh' xdx.

S cosh

&

x dx.

Ssinh' x cosh xdx.

1399. 1400.

Scoth& x dx.
sinh! x + cosh!
dx

dx

1394. Ssinhl xcosh l xdx. 1395.

52 sinh x + 3 cosh x
Ssinh x dx

5smh xd;OSh x 1396. 5 sinh" :~OShl x


2

1401*.
1402.

5tan::-I

Vcosh 2x

Sec. 9. Using Trigonometric and Hyperbolio Substitutions for Finding Integrals of the Form
SR(x, Yaxl+bx+c)dx.
(1)

where R is a rational function.

134

Indefinite Integrals

[Ch.4

Transforming the quadratic trinomial ax! bx c into a sum or difference of squares, the integral (I) becomes reducible to one of the following types of integrals:

+ +

I) ~ R (z,
2)

V ml -Zl) dz;
V ml+zl ) dz;

~ R (z.
~ R (z,

3)

Y Zl

ml) dz.

The latter integrals are, respectively, taken by means of substitutions:


1) Z= m sin t or z= m tanh t, 2) z=mtan tor z=msinh t, 3) z m sec t or z = m cosh t.

Example 1. Find

Solution. We have

x!+2x+2=(x+ I)!+ 1.

J-

_r dx S J (x+ I)' Y(x+ 1)'+ 1


Example 2. Find Solution. We have

Putting x+ l=tanz, we then have dx=sec 2 zdz and sec! z dz = tan! z sec z

Scos z dz = sin' z
SUI Z

= __ .1_ + C =

V x2 + 2x + 2 + X+I

C.

x! + X + I =
Putting
x+'~=-2- slnht

1)2 + 4" 3 (x + 2"


and

Y3.

dX=-2- cosh tdt,

Y3

we get

J=

Y3. S( -

sInh t- - - cosh t -2- cosh t dt= 222

I) Y3

Y3

=-8-

}ras .

sInh t cosh! t dt-

cosh! t dt = cosh t-l--

Since

3 Y3 cosh' t 3 (I . = - -3- - -sinh t 8 8 2

1 t)

+C

sinh t = .,} 3
and

(x +

~).

cosh t =

V Xl +

+ 1

t=ln (x+

;+YXI+x+I)+ln

;3'

Sec. 111
we finally have

V sing Reduction Formulas

135

1 (X 2 + X + 1) -I - - 1 ( X + -1 ) J = -3 ,4 2

Y Xl + X + 1-~ In (x+ ~ + Yx l +x + 1) +
C.

Find the integrals:


1403. ~ V3-2x-x l dx. 1404. ~ V2 1405.
"

1409. 1410.

SV x

-6x-7 dx.
I

-+- Xl dx.
2

S(Xl +x+ 1)1 dx.


2

J V 9+x dx.
Xl

1406. ~ V x l -2x

+2 dx.

S(x-I)Vx dx -3x+2 1412. r


1411.
dx "

J (xI-2x+5)1

1407. ~ V Xl -4 dx.
1408.

1413. 1414.

S (I +

dx
Xl)

SVx +x dx.
l

VI-x'
l

(l-x 2 )

dx

Yl+x

Sec. 10. Integration of Various Transcendental Function s

Find the integrals:


1415. ~ 1416. ~
(Xl Xl

+ 1)1 elx dx.


cosl 3x dx.

1421.
1422.

5 :;x
elX
Xl

-2

1417. ~ x sin x cos 2xdx. 1418. ~ e2X sinl x dx.

1423.
1424. 1425. 1426.

5 In ~ +; dx.
SIn' (x + VI + x') dx.
Sx arc cos (5x 2) dx.

Ye2~ +e~ + 1

dx

1419. ~ eX sin x sin 3x dx.


1420. ~ xe x cos x dx.

Ssin x sinh x dx.

Sec. 11. Using Reduction Formulas

Derive the reduction formulas for the following integrals:


1427.

In=5(xl~xal)n; find II and I .


~ sinn xdx; find I. and II'

1428. In =

136

Indefinite Integrals

[Ch.4

1429.

In=S cosd~ x ;

find I. and I".

1430. In = ) xne- x dx;

find 110"

Sec. 12. Miscellaneous Examples on Integration


1448.

S + x xdx V 1449. S Yl- 2x


(1
2 )

I-

XC

xdx

1450. 1451 *.

J (x + 1)

r x+

2 - X4

1 ..!. dx.
2

5
X

dx (x 2 +4x) Y4-x 2

1452. ) 1453. ~
1454.

lR=9 dx.

V x- 4x
2

dx.

S Yx dx +x + 1 S x
4

1455. ) xVx 2 +2x+2dx.


1456. /x
xZ-1

1457. \ v x
1458.
1459.

Sy

'"' dx j VI+X"

Y dx . 1 -x'

5x dx. 1+xt

1460. ~ cos x dx.


1461.

cos x sInS x
1
Sln 2

dx.

5 + yCOtX dx x 1463. S x dx. Vcos


1462.
sinS
ax

1464. ) cosec s 5x dx.


14655sin2xd
COSO 1&

X.

Sec. 12]

Miscellaneous Examples on Integration

137

1466.

Ssin ( : -x )Sin(: + x) dx.


+ ~ ) dx.

1484. 1485. 1486.

1467. Stan' ( ; 1468. 1469. 1470.

Scos x+2 sin xcosx+2 dx sin x 1471. S dx sin x sin 2x dx 1472. S (2 + cos x) (3 + cos x) 1473. S seclx dx. Ytan x+4 tan x+ 1
2 2
2

S2sinx+3cosx-5 dx S2 + 3dx cos!


X

Yl-x dx. Yl-x S sinh x cosh x d sinh x+ cosh x x.


sinh
2 2

~ sinh x cosh x dx.

1487. \ y d x . ~ sIn x 1488. 1489. 1490.


1491.

S S

e2X -2e x
eX

dx

e!X _ 6e x
lX e

+ 13 dx.

..!.. dx.
4

(eX

+ 1)
X

1474. 1475.

SYu +sln ax dx. ax


cos
2 2

S1_4xdx. 2
l/
eX

Scos! xdx 3x
l
2X

1492. ~ (x' -1) 1O-2X dx. 1493. ~


1494.

t 1 dx.

1476. ~ x sin x dx. 1477. ~ x'-e~' dx. 1478.

~ xe dx.

1495.

Sarc::" x dx. Sx arc sin ~ dx.

1479. ~ Xl In Vl-xdx. 1480. 1481. 1482. 1483.

1496. ~ cos (In x) dx. 1497. ~ (Xl - 3x) sin 5x dx. 1498. ~ x arctan (2x+3)dx.
1499. 1500.

S'IX 3x sIn 2 cos 2dx.

x arc tan x dx. YI +x 2

5(sin x + dx cos x)!


S(tan x+ dx 1) sin
a

Sarc sin y"X dx. SI x I dx.

Chapter V

DEFINITE INTEGRALS

Sec. 1. The Definite Integral as the Limit of a Sum


1. Integral sum. Let a function f (x) be defined on an interval a~x~b, and a=xo < XI < ... < xn=b is an arbitrary partition of this interval into n subintervals (Fig. 37). A sum of the form
n-l

Sn = ~
[=0

f (~i) AXi'

(I)

where
Xi~6ie=;;;;;Xi+l; &xi=xi+l-xi;

i=O, 1, 2, ... (n-l),

is called the integral sum of the function !(x) on [a, b]. Geometrical1y, S" is the algebraic area of a step-like figure (see Fig. 37).

o1
Fig. 37

to
Fig. 38

2. The definite integral. The limit of the sum Sn' provided that the number of subdivisions n tends to infinity, and the largest of them, dX', to zero, is called the definite integral of the function f (x) within the limits from x=a to x=b; that is,
n-l

max Ax, -. 0 I =0

11m

~ f (M tui = ~ f (x) dx.


a

(2)

Sec. 1]

The Definite Integral as the Limit

of a Sum

139

If the function f (x) Is continuous on la, b), it is integrable on [a, b); Le., the limit of (2) exists and is independent of the mode of partition of the interval of integration [a, b) into subintervals and is independent of the choice of points ~i in these subintervals. Geometrically, the definite integral (2) is the algebraic sum of the areas of the figures that make up the curvilinear trapezoid aABb, in which the areas of the parts located above the x-axis are plus, those below the x-axis, minus (Fig. 37). The definitions of integral sum and definite integral are naturally generalized to the case of an interval [a, b), where a> b. Example 1. Form the integral sum Sn for the function f (x) = 1 +x on the interval [1,10] by dividing the interval into n equal parts and choo!ing points that coincide with the left end-points of the subintervals [xi, xi+tl. What is the lim Sn equal to?

ai

-+

CJ)

So I ' H ere, I:! xr=-n-=--; 10-1 9 an d tfoi=xI=xo+t'A xi= 1 utlon.


(~i)=l+l

9i +n.

WIlence

+-=2+-. Hence (Fig. 38), n n


'\.~ ( 9i) 9 18 81 =~ 2+--; n=n n + n 2 (0+ 1 + ... +n-l)=
1=0
n-l

9i

9i

n-l

S'n = ~f (;i) dXi


1=0

=18+8In(n-l)=18+81
nZ

2
n-+CJ)

(1_-!.)=58-.!.._81
n
2 2'

2n '

lim Sn =-= 58 -.!..

Example 2. Find the area bounded by an arc of the parabola y =x2 the x-axis, and the ordinates x = 0, and x = a (a > 0). Solution. Partition the base a into n equal y parts dX=~. Choosing the value of the funcn tion at the beginning of each subinterval, we will have
Yl = 0; Y. =

(:

Yn=[<n-l):r
The areas of the rectangles are obtained by multiplying each Yk by the base Ax= ~ (Fig. 39). n Summing, we get the area of the step-like figure
1 1

y, = [ 2 ( :

YJ .,. ,
0

X
Fig. 39

Sn=: (:Y l l+2 +3 +,..+<n-l)').


n

Using the formula for the sunl of the squares of integers,


n (n

+ 1) (2n + 1)
6

140

Definite Integrals aSn (n-l) (2n-l) 6n l

[Ch.5

we find

and. passing to the limit. we obtain


S:'C lim Sn
n -+
(J)

n -+

lim a l (n-l) n (2n-l) _ al 6n I 3


a;)

Evaluate the following definite integrals, regarding them as the limits of appropriate integral sums:
b
1

1501. ~ dx.
a
T

1503.

~ x'dx.
-!

10

1502.

S(v. +gt) dt,


o

1504. ~ 2 x dx.
o
5

V.

and g are constant.

1505*. ~ x' dx.


1

1506*. Find the area of a curvilinear trapezoid bounded by

the hyperbola by two ordinates: x = a and x = b (0 < a < b), and the x-axis. 1507*. Find
x

f (x) = ~ sin t dt.


o

Sec. 2. Evaluating Definite Integrals by Means of Indefinite Integrals


1. A definite integral with variable upper limit. If a function continuous on an interval [a. bl. then the function
x

t (t)

Is

F(x) =

~ f (t) dt
a

is the an\iderlvative of the function f (x); that is,


F' (x) =

f (x) for

a ~ x <; b.

2. The Newton-Leibniz formula. If F' (xl = f (x). then


tJ

~ f (x) dx=F (b)-F (a).


a

Sec. 2]

Evaluating Definite Integrals by Indefinite Integrals

141

The antiderivative F (x) is computed by finding the indefinite integral

Sf (x) dx = F (x) + C.
Example 1. Find the integral
s

S y;4 dx.
-I

1508. Let

I=

Find

5I::
a

(b

> a>
2) db
dJ

1).

1) da;
X

dJ

Find the derivatives of the following functions:


1509. F (x) = ~ 1n t dt
0

x'

(x> 0).

1511. F (x) 1512. I =

= Sr /2 dt.
x

1510. F(x)= ~ Vltrdt.


X

vX'
1

S cos (r) dt
x

(x

> 0).

1513. Find the points of the extremum of the function


x

Y=

S-to

sin

t dtIII th e region . x> 0 .

Applying the Newton-Leibniz formula, find the integraisJ


1514. 1515.

Sl+x
1

dx

1516.

S et dl.
-x
x

0
-

S dx

X'

1517. S:cos t dl.


0

-I

Using definite integrals, find the limits of the sums:


1518**. lim 1519**. lim
'l-+a;)

n-'a;)

1520.

P lim IP+2 +;;.+n (p>O).


p
n-+CIO

21 + ... +n 11). (';+n n n ( +1 1+ +1 2+ ... +_+1 ). n n n


fl

nil

142

Definite Integrals

[Ch.6

Evaluate the integrals:


2

1521.

S(x l -2x+3) dx.


1

1534.
1535.

J Y 5+4x-xl
I

r
1

dx

1522. ~
o

(V 2x + Vi") dx.

5Y +4
o

y2dy y

1523.

51+yrii dye
1

n
4.

1536.

Scos! ada.
o
:t
I

1524. 1525. 1526.

SYx-2dx.
I

-8

1537. ~ sin 8 q> dq>.


o

5
o
-2
1

y 25+ 3x.
1538.
1dX X I

dx

e2

S
5
o
1 -1 1

x In x
sin

dx

1539.
xdx

I
1
4.

e e

~n x) dx.

1527. 1528. 1529. 1530.


1531.

x!+3x+2'

:t

~ ~':~.

1540.

S tanxdx. .
:t

n
8

5
o
8

X!+d:x +5'

lMl.

Scot' q> dq>.


n
e

5 X!-~:+2
5Z8~
o

1542. 1543.

5 :elX dx.
I
o
1

I dz.

Scosh xdx.
o

n
4.

1632. ~ sec l ada.


n

1544.

trco::!x'
In
11

VI'
P-

1533.

I VI-x

dx

154.1).

S sinh! x dx.
o

Sec. 3]

Improper Integrals

14~

Sec. 3. Improper Integrals


1. Integrals of unbounded functions. If a function I (x) is not bounded in any neighbourhood of a point c of an interval [a, b] and is continuous for a ~ x < c and c < x <; b, then by definition we put

r f (x) dx = elim r f (x) dx + e-+o lim r f (x) dx. J ~o J J


a

c-e

(1)

c+e

If the limits" on the right side of (1) exist and are finite, the improper integral is catted convergent, otherwise it is divergent. When c = a or c == b, th..: definition is correspondingly simplified. If there is a continuous function F (x) on [a, b] such that F' (x) = f (x) when x rI: c (generalized antiderivative), then
b

~ f (x) dx=F(b)-F (a).


a
b

(2)

If /I (x) I <;; F (x) when a <;; x <;b and


tegral (1) also converges (comparison test).

SF (x) dx
a
00,

converges, then the in-

If f (x)

~0

and 11m
x~c

r(x)

I c-x 1m ~-" A t=

A '1= 0, i. e.,

f (x)- I

when x -+ c, then 1) for m < 1 the integral (1) conver~es, 2) for In ~ 1 the integral (1) diverges. 2. Integrals with infinite limits. If the function f (x) is continuous when a ~ x < 00, then we assunle
t:rJ

c-x

liIJ

r f (x) dx = J

lim b-+~

r f (x) dx J
ri~ht

(3)

and depending on whether there is a finite limit or not on the the respective integral is called convergent or divergent. Similarly,

of (3),

-QO

(x) dx=

lim
a-+-rD

r f (x) dx

b
OIl

and

~ f (x) dx= a~~ Sf (x) dx.


-QO

b-+-+rD a

rD

If

II (x) 1<; F (x)


f (x) ~ 0 and

and the integral

SF (x) dx
a
00,

converges, then the Infe

gral (3) converges as well.

If

x -+ co. then 1) for m > 1 the integral (3) converges, 2) for m ~ 1 the integral (3) diverges.

%-+rD

11m f (x) x m = A #r.

A #:

o.

i. e.,

f (x)-Ii when
x

144
Example t.

Definite Integrals

(Ch.5

-= 5ax
1

x2

2 -+ 0

lim

-52 2+ dx
x
-1

2 -+ 0

lim

5' dx . (1 )=00 2= 11m --1 ) + l'1m (1 --1


e
x
2 -+ 0

e~0

and the integral diverges. Example 2.


GO

51+x
dx
o

= lim

b-+GO

5I +x = lim
dx
2

b-+(I)

(arc tan b-arc tan O)=~

Example 3. Test the convergence of the probabllity integral


(4)

Solution. We put
~

~ e- xl dx= ~ e- x2 dl:+ Se- x2 dx.


0 0 1

The first of the two integrals on the right is not an improper integral, while the second one converges, since e- ~ ~ e- x when x ~ 1 and
(I) b

e-Xdx= Urn \ e-xdx= lim (-e- b +e- J )=e- 1 ;

b-+~

b-+GO

hence, the integral (4) converges. Example 4. Test the following integral for convergence:

(5)
Solution. When' x-+-+ 00, we have 1 1

VxI+1 yxa(l+;a)
Since the integral

converges, our integral (5) likewise converges. Example 5. Test for convergence the eltiptic integral
1

S VI-x o

dx

(6)

Sec. 3)

I mproper Integrals

145

Solution. The point of discontinuity of the integrand is x= 1. Applying the Lagrange formula we get I I VI-x. = .. /(I-x).4x a =

(I-x)"

!.. . - . ' 2x:

where x <

Xl

< 1. Hence, for x -. 1 we have

Since the integral

converges, the given integral (6) converges as well.

Evaluate the improper integrals (or establish their divergence):


1546. 5 dx tx
1

1554. 1555.

-e
(X)

5 1548. 5dx
1547.
-1

d :

5 5
(X)

I +x l

dx

x!+4x+9

dx

-00
(X)

xP

1556. ~ sin xdx.


0

1549. 5
0

(x -1)2

dx

1557. 5 dx In
2

0
1

1550. 1551.

5 5
GO

dx

VI-xl

1558. 1559.

Sx In dx x
0

d:.

1
GO

S x dx Inx
n

QI)

(a> I).

1552.

Sdx Xl

dx 1560. Sx Inlx
a
I

GO

(a> 1).

1553.

S7. dx
1

GO

1561. ~ cot X dx.


0

J46
CIO

Definite Integrals
CIO

[Cia. ,
dx

1562. ~ eo
CIO

kx

dx

(k

> 0).

1565.

o
1

5x'+ 1
dx
X -

1563.

Sa~t~nt dx.
o

1566.

xa S-'-----5 o

1564.

5(XI~
I

CIO

1)1
integrals~
I

Test the convergence of the following


100

1567. )

Vx+2 V x+x'

dx

1571.

dx V=::e. S l-x4 o
I

1572.

5l:XX 1573. 5~ d x.
1 ex>

Xl

n
I

1570.

S
o

co

Yx + 1
5
1

xdx

1574*. Proye that the Euler integral of the fil st kind (betafunction)

B (p, q) = ~ xp - t (l-x)q-t dx
o

converges when p > 0 and q > O. 1575*. Prove that the Euler integral of the second kind (gamma-function)
QI)

r (P) = ~ xP-1eo

dx

converges for p >

o.

Sec. 4. Change of Variable in a Definite Integral If a function f(x) is continuous over a<;x<;b and x-q>(t) II. function continuous together with its derivative <p'(t) over a..;t";Pl where a==-cp(a) and b= cp (~). and f [q> (t)) is defined and continuous on the Interval a<t<p.

Sec. 4)
then

Change
b

0/ Variable in a Definite Integral


fJ

147

~ f (x) dx = ~ f [q> (t)) q>' (t) dt.


a

Example 1. Find

(a> 0).
Solution. We put

x=asint; dx=a cos t dt.


Then t=arcsin~ a and, consequently, we can take a=arcsinO=O,

P=
a

arc sin 1= ; . Therefore, we shall have


n
2 2

Sx
o

Va 2

x2 dx=
n
2

~
0

a2 sin 2 t Ya 2 -a 2 sin 2 tacostdt= n


2

n
I

=a&

Ssln
o

t cos t dt =

~&

Ssln 2t dt = ~. S(I-cos 4!) dt =


2

n
(

=8 t -4 sln4t

a4

=16

nat

1576. Can the substitution x = cos t be made in the integral


2

~ Vl-x1dx?
o

Transform the following definite integrals by means of the indicated substitutions:


a

n
I

1577. 1578.

~ Vx+ Idx, x=2t-1.


I I

t 580.

S f (x) dx, x = arc tan t.


o
b

S YI-x '
1

dx

X=

sin t.

1581. For the integral

1579.

SYxdx+l
.
l

's "

Sf (x) dx (b > a)
a

x= sinh t.
t

148

Defil1ite Integrals

[Ch. 5

indicate an integral linear substitution

x=at + p,
as a result of which the limits of integration would be 0 and 1t respectively. Applying the indicated substitutions, evaluate the following integrals:
4

1582.

S1+ V-'
o

dx

18

1583.
1584.

S
1nl

SVex-l dx,
o

2/ (x-2) , 2/ 3 (x-2) I

dx,

x-2=z'.

t 585

S3 + 2 cos t
n
I

dt

1586.

1 +a sin1x S o
l

dx

tan x = t.

Evaluate the following integrals by means of appropriate substitutions:


1

1587.

VI

S ~I dx.
x

Ins

1589.

S o S
o
I

eX reX-l eX dx.

lr--

+3

15S8.

5y~
I

dx.

1590.

2x+

Y3x+ 1

dx

Evaluate the integrals:


1591.

Sx Y x dx +5x+l
l

1593. ~ Vax-x dx.


o

1592.

S(l ~XX.)I
-I

In

1594.

S,,5 ~X cos
o

1595. Prove that if

f (x) is an even function, then


~ f(x) dx = 2 ~ f(x) dx.
a a
0

-a

Sec. 5]

Integration by Parts

149

But if

f (x)

is an odd function, then


a

~ f (x)dx =
-0

o.
~

1596. Show that

-~

5e- X'dx=2 5e-X'dx= se-;dX.


0 0

1597. Show that


n
1

S
o

x -dx - - Ssin x arc cos x x d


0

1598. Show that


n
I

2!.
I

~ f(sinx)dx= ~ f(cosx)dx.
o
0

Sec. 6. Integration by Parts


If the functions u (x) and v (x) are continuously differentiable on the Interval [a, bl, then
b b

~ U (x) v' (x) dx=u (x) v (x)


a

1- ~
a

v (x) u' (x) dx.

(I)

Applying the formula for integration by parts, evaluate the following integrals:
n
I

co

1599.

S x cos xdx.
0

1603.

~ xe-xdx.
co

1600.

~lnxdx.
I

1604.

~ e- ax cos bxdx
0

(a> 0). (a> 0).

co

1601.

Sx'elXdx.
0

1605. ~ e -axSIn bxdx


0

11.

1602.

~ eX sin xdx.
0

150

Definite Integrals

[Ch.5

1606*. Show that for the gamma-function (see Example 1575) the following reduction formula holds true:

r (p + 1) = pr (p)

(p

> 0).

From this derive that r (n + 1) = nl, if n is a natural number. 1607. Show that for the integral
! I

I n= ~sinnxdx= ~ cosnxdx
o
0

the reduction formula


1
n=-n-

n-l

l n-I

holds true. Find In' if n is a natural number. Using the formula obtained, evaluate I, and 110 1608. Applying repeated integration by parts, evaluate the integral (see Example 1574)
1

B(p, q)=~xP-J(l-x)q-Jdx,
o

where p and q are positive integers. 1609*. Express the following integral in terms of B (betafunction):
2

I n.m = ~ sin m x cos n x dx,


o

if m and n are nonnegative integers.


Sec. 6. Mean-Value Theorem
1. Evaluation of integrals. If ,(x)
b

~F(x) for a~x~b,


b

then
(I)

~ f (x) dx ,e;;;; ~
a a

F (x) dx.
~

If ,(x) and q> (x) are continuous for a ~ x


b b Ip (x)

b and, besides, q> (x) ~ 0, then


b Ip

~
a

dx,e;;;;

~ f (x) Ip (x) dx 0;;;; M ~


a a

(x) dx.

(2)

where m is the smallest and M is the largest value of the function f (x) on the in terval [a, lll.

Sec. 6)

Mean-Value Theorem

151

In particu Jar, if cp (x) == 1, then


b

m (b-a) EO;; ~ f (x) dx..;;; M (b-a).


a

(3)

The inequalities (2) and (3) may be replaced, respectively, by their equiva lent equalities:
b b

~ f (x) <p (x) dx = f (c) ~


a
b

<p (x) dx

~ f (x) dx = f (~) (b-a),


a

where c and ~ are certain numbers lying between a and b. Example l. Evaluate the integral !!.

/= ~
o

1+

sin' x dx.

Solution. Since O.c;;; sln l x E:; I, we have

that is,
1.57 < I

<

1.91.

2. The mean value of a function. The number


b

.... = b 1 a

Sf (x) dx
a

Is ca lied the mean value of the function

f (x) on the interval a <; x <; b.

1610*. Determine the signs of the integrals without evaluating them:


I

a) ~ x' dx;
-1

C) SSi~X dx.
o

In

b) ~ x cos x dx;
o

152

Definite Integrals

[Ch. b

1611. Determine (without eval uating) which of the following integrals is greater:
1

a)
b)
c)

SVI +x
0

dx

or or or

~
I

dx;

S
0
I

Xl

sin I x dx

~ x sin l xdx;
0

Se%'dx
J

Se"dx.
I

Find the mean values of the functions on the indicated intervals:


1612. f(x)=x
l ,

o~x~

1.

1613. f(x)=a+b cos x, 1614. f(x)= sin! x, 1615. f (x) = sin 4 x,


1

-n~x~3t.

O~x~rc. O~x~rc.
dx bt 2 ~ 0.67 and .. r1 ~ lies e ween -3

1616. Prove that


~

5V
o

2+x-~

y2

0.70. Find the exact value of this integral. Evaluate .the integrals:
11.

1617.

SV 4 t
o

Xl

dx.

1620*.
n
I

SxVtanx.
o

1618.

-I

58~XI.
In
o

+1

1621.

5sin x dx.
)(,

&

1619.

5lO+~cosx
200n

1622. Integrating by parts, prove that

0<

5~d <_I_
x
x

lOOn'

lOon

Sec. 7]

T he Areas of Plane Figures

153

Sec. 7. The Areas of Plane Figures


1. Area in rectangu lar coordinates. If a conti nuous curve is defined in rectangular coordinates by the equation Y=f (x) [I (x) ~O], the area of the curvilinear trapezoid bounded by this curve, by two vertical lines at the

1(

a
Fig. 40

J
Fig. 41
ae=;;;;;x~b

points x=a and x=b and by a segment of the x-axis is given by the formula b
S=

(Fig. 40).
(1)
2

Sf (x) dx.
a

x Example 1. Compute the area bounded by the parabola y = -2 ' the straight I ines x = 1 and x -= 3, and the x-axis (Fig. 41).

x)
0

I I I

-2
Fig. 42

g::f, (z)
Fig. 43

Solution. The sought-for area is expressed by the integral

154

Definite Integrals

[Ch. 5

Example 2. Evaluate the area bounded by the curve x=2-y_ y l and 1he y-a xis (Fig. 42). Solution. Here, the roles of the coordinate axes are changed and so the sought-for area is expressed by the integral

s=

S(2-y_ y2)dy=4 ~,
-I

where the limits of Integration Yl = -2 and Y. = 1 are found as the ordinates of the points of intersection of the curve with the y-axis.

r
y

x
Fig. 44

a
Fig. 45

In the more general case, if the area S is bounded by two continuo us curves y ='1 (x) and y = f 2(x) and by two vertical lines x =a and x = b, where 11 (x) <af. (x) when a<x~b (Fig. 43), we wilt then have:

S=

S
a

[f2 (x)-f.(x) dx.

(2)

Example 3. Evaluate the area S contained between the curves


y=2-x2 and yl =x.
(3)

(Fig. 44).
Solution. Solving the set of equations (3) simultaneously, we find the Jinlits of integration: x.=-1 and x 2 = 1. By virtue of formula (2), we obtain
1

s=

-1

x' - 3 xa) 1 =22 . (2-x -x2/ 8) dx= ( 2x-3 5 -I 15

If the curve Is defined by equations in parametric form x = q> (t), Y = "I' (t) then the area of the curvilinear trapezoid bounded by this curve, by tw ~

Sec. 7]

The Areas

of Plane Figures

vertical lines (x =a and x= b), and by a segment of the x-axis is expressed by the integral

S = ~ (t) cp' (t) dt.


t.

t3

where t 1 and t. are determined from the equations a=cp(tt) and b=cp(t 2 ) ['P(t)~O on the interval [tt, t z ]]. Example 4. Find the area of the ellipse (Fig. 45) by using its parametric equations x=- a cos t, { y == b sin t. Solution. Due to the synlnletry, it is sufficient to compute the area of a quadrant and then multiply the result by four. If in the equation x=a cos t we first put x:=: 0 and then x =a, we get the lilnits of integration i 1 = ~ and
t 2 = O. Therefore,
n

= ~ b sin a (n
2

sin t) dt = ab

Ssin! t dt = ~b
0

and, hence, S=Mb. 2. The area in polar coordinates. If a curve is defined in polar coordinates b~' the equation r =-= f (cp), then the area of the sector AOB (Fig. 46), bounded by an arc of the curve, and by t\\'o radiUS vectors OA and OB.

o~-...

....._--X
Fig. 47

Fig. 46

which correspond to the val ues <PI = a and <PI =~, is expressed by the integral
1 S = 2"

r[f (cp)]2 dq>. J


a

f}

,2

= a Z cos 2q>

Example 5.

Find (Fig. 47).

the

area

contained inside

Bernoulli's

lemniscate

156

Definite Integrals

[Ch. 5

Solution. By virtue of the symmetry of the curve we deter111ine first one ,quadrant of the sought-for area:

Whence S::: al.

1623. Compute the area bounded by the parabola y=4x-x l and the x-axis. 1624. Compute the area bounded by the curve y = In x, the x-axis and the straight I ine x = e. 1625*. Find the area bounded by the curve y=x (x-i) (x-2) and the x-axis. 1626. Find the area bounded by the curve y3 = x, the straight line Y= 1 and the vertical line x=8. 1627. Compute the area bounded by a single half-wave of the sinusoidal curve y= sinx and the x-axis. 1628. Conlpute the area contained between the curve y=tanx, the x-axis and the straight line x=1- . 1629. Find the area contained between the hyperbola xy=m'l., the vertical lines x = a and x = 3a (a> 0) and the x-axis. t 630. Find the area contained between the witch of Agnesi

Y=

Xl

+a and the x-aXIS.


2

03

1631. Compute the area of the figure bounded by the curve y=-=x 3 , the straight line y=8 and the y-axis. 1632. Find the area bounded by the parabolas y"c:2px and x" = 2py. ,. 1633. Evaluate the area bounded by the parabola Y= 2x-x and the straight line y=-x. 1634. Compute the area of a segment cut off by the straight line y=3-2x from the parabola y=x". 1635. Compute the area contained between the parabolas yrc:r.x, Y=i and the straight line y=2x. 1636. Compute the area contained between thQ parabolas

Y="3 and y=4-3"x 1637. Compute the area contained between the witch of

x2

Agnesi y= 1+x2 and the parabola y= 2. 1638. Compute the area bounded by the curves ya:ae". and the straight line x == 1.

Xl

II-a;s-~

Sec. 7)

T he Areas of Plane Figures

157

al

x2
-

1639. Fi nd the area of the figure bounded by t"he hyperbola y2 b2 = 1 and the straight line x = 2a. 1640*. Find the entire area bounded by the astroid

x' +y' =a'.


1641. Find the area between the catenary

Y= a cosh!., a

the y-axis and the straight line Y= (e t + 1). 1642. Find the area bounded by the curve a2y2=x2(a2_xl). 1643. Compute the area contained within the curve

:e

1644. Find the area between the equilateral hyperbola X 2 _ y 2 = = 9, the x-ax is and the diameter passi ng through the point (5,4). the x-axis, 1645. Find the area between the curve y=~, x and the ordinate x=l (x>l). X 1646*. Find the area bounded by the cissoid y2=2a ' -x and its asymptote x = 2a (a> 0). x (x-a)2 1647*. Find the area between the strophoid y2 = 2a-x and its asymptote (a> 0). 1648. COlllpute the area of the two parts into which the circle X 2 _t- y 2=:8 is divided by the parabola y2=2x. 1649. Compute the area contained between the circle Xl + y2 = 16 and the parabola x 2 = 12 (y-l). 1650. Find the area contained within the astroid X= a cos' t; y= b sin' t. 1651. Find the area bounded by the x-axis and one arc of the cycloid X = a (I - sin t), { y = a (1 - cos t). 1652. Find the area bounded by one branch of the trochoid
{ x=at-bsint, y=a-b cos t (O<b:s;;;;;a)

and a tangent to it at its lower points.

158

Definite Integrals

{Ch. 6

1653. Find the area bounded by the cardioid


{ X = a (2 cos t-cos 2/), y = a (2 sin t - sin 2/).

1654*. Find the area of the loop of the foli urn of Descartes
3ut 3at 2

x=l+t S ;

Y=l+t"

1655*. Find the entire area of the cardioid f=a (l+cosq. 1656*. Find the area contained between the first and second turns of Archimedes' spiral, r = aq>

(Fig. 48). 1657. Find the area of one of the leaves of the curve r = a cos 2fp. 1658. Find the entire area bounded by the curve = a l sin 4(p. 1659*. Find the area bounded by the curve , = a sin 3(p. 1660. Find the area bounded by Pascal's limac;on

,2

Fig. 48

= 2 + cos cp.

1661. Find the area bounded by the parabola r = a secl ~

and the two half-lines cp

=:

and cp

=;.

1662. Find the area of the ellipse f = 1+ e P (8< 1). cos q> 1663. Find the area bounded by the curve , = 2a cos 3(p and

lying outside the circle r = a. 1664*. Find the area bounded by the curve x4 + y4 = Xl + yl.
Sec. 8. The Arc Length of a Curve
1. The arc length in rectangular coordinates. The arc length s of a curve Y=f (x) contained between two points with abscissas x=a and x=b is
b

s= ~
a

VI +U'2dx.

Example 1. Find the length of the astroid x 2/1 y2 /1 :=a 2/a (Fig. 49). Solution. Differentiating the equation of the astroid, we get

y,=_L,. x/

II,

Sec. 8]

The Arc Length of a Curve

159

For this reason, we have for th~ arc length of a quarter of the astroi d:

{s=

SVI+ ~:;:
o

dx=

5:::: dX=~ ~
dt,

Whence s = 6a. 2. The arc length of a curve represented parametrically. If a curve Is represented by equations in parametric form, x = cp (t) and y = "I> (t), then the arc length s of the curve is
tJ
S

~ YX'I+y'l
tI

where t l and t 2 are values of the parameter that correspond to the extremities of the arc.

x
---1~--------_----I"-'-."-'-'-

27Ca

Fig 49

Fig. 50

Example 2. Find the length of one arc of the cycloid (Fig. 50) X = a (t - sin t), { y=a (I-cos t). Solution. We have dt =a (1- cos t) and dt = a sin t. Therefore,
S

dx

dy

= S Val (I-cos
o

In

t)l+ a l

sln l t dt =

2a

Ssin ~ dt =8a.
0

2n

The limits of integration t 1 =0 and t l =2Jt correspond to the extreme poinfs of the arc of the cycloid. If a curve is defined by the equation r=f (cp) in polar coordinates, then the arc length s is P s= ~ V,I +,,1 d<p.
a

where a and ~ are the values of the polar angle at the extreme points of the arc.

160

Definite Integrals

(Ch. 6

Example 3. Find the length of the entire curve , =a sln l ~ (Fig. 51).

The entire curve is described by a point as cp ranges from 0 to 3n.

Fig. 51
Solution. We have

,'= a sin t

cos : ' therefore the entire arc length of


a~

Ihe curve is
an
S

= S 'V i
o

a2 sin' ..!. 3

+a

sin' ..!. cos 2 ..!. dlp = a 3 3

J. sln ..!. dlp = 3na 3 2


2
0

1665. Compute the arc length of the semicubical parabola yl = x from the coordinate origin to the point x = 4. 1666*. .Find the length of the catenary y = a cosh ~ from the a vertex A (O,a) to the point B (b,h). 1667. Compute the arc length of the parabola y = 2 from x =0 to X= I. 1668. Find the arc length of the curve y = eX lying between the points (0, I) and (I ,e). 1669. Find the arc length of the curve y = In x from .t = Va to x= VB. 1670. Find the arc length of the curve y = arc sin (e-') from x=O to X= 1. 1671. Conlpute the arc length of the curve x=lnsecy, lying

Vx

between y=O and y= ~ . 1672. Find the arc length of the curve x= ~ y2_ ~ Iny from Y= 1 to y=e.

Sec,

9)

Volumes of Solids

161

1673. Find the length of the right branch of the tractrix


X= Va z

yZ+aln

la+ Y~I

from y=a to y=b (0< b<a).

= x (x-3a)l.

1674. Find the length of the closed part of the curve 9ay l =

1675. Find the length of the curve y= In (coth ~) from x=a

to x==b (O<a<b).

1676*. Find the arc length of the involute of the circle

x=a(c?st+tsint),} from t=O to t=T y=a(slnt-tc05t)


1677. Find the length of the evolute of the ellipse

1678. Find the length of the curve


x ==- a (2 cos t- cos 21), } y=a(2 sint-sin2t).

1679. Find the length of the first turn of Archimedes' spiral r = atp. 1680. Find the entire length of the cardioid r=a(l+cos<p). 1681. Find the arc length of that part of the parabola , = a sec!; which is cut off by a vertical line passing through the pole. 1682. Find the length of the hyperbolic spiral r<p = I from the point (2"/2) to the point C/2,2). 1683. Find the arc length of the logarithmic spiral r = aemcp , lying inside the circle r = a. t 684. Find the arc length of the curve cp = ~ + ~) from r = 1 to , = 3.

(r

Sec. 9. Volumes of Solids


to. The volume of a solid of revolution. The volumes of soB4s formed by the revolution of a curvilinear trapezoid (bounded by the curve !I ~f (x), tbe x-axis and two vertical lines x = a and x = b J about the x- and 'y-axes are
6 - 1900

162

Definite Integrals

[Ch.6

expressed. respectively, by the formulas:


b
b

I) Vx=n

~ yldx;
a

2) V y =2n

~ xydx*).
a

Example I. Compute the volumes of solids formed by the revolution of a figure bounded by a single lobe of the sinusoidal curve y = sin x and by the segment 0 < x ~ n of the x-axis about: a) the x-axis and b) the y-axis.
Solution.
n

a) VX=-n

Ssin
o
:t

xdx=T;

n2

b) Vy=2n

~ xsinxdx=2n(-xcosx+sinx)~=2nl.
o

The volume of a solid formed by revolution about the y-axis of a figure bounded by the curve x=g(y), the y-axis and by two parallel1ines y=c and y=d, may be determined from the formula
d

Vy=n ~ xldy,
c

obtained fronl formula (I), given above, by interchanging the coordinates x and y. If the curve is defined in a different form (parametrically, in polar coordinates, etc.), then in the foregoing formulas we must change the variable of Integration in appropriate fashion. In the Inore general case, the volumes of solids formed by the revolution about the x- and y-axes of a figure bounded by the curves Yl ='1 (x) and Y2 = f 2(x) [where fl(X)~f2(X)], and the straight lines X=Q and x=b are, respectively, equal to
b

Vx=n

~ (y:-y:) dx
a

and
b

Vy=2n

~ x (YI-YI) dx.
a

Example 2. Find the volume of a torus formed by the rotation of the circle x 2 + (y-b)1 =a2 (b ~ a) about the x-axis (Fig. 52). *) The solid is formed by the revolution, about the y-axis, of a curvilinear trapezoid bounded by the curve y=f (x) and the straight lines x=a, x=b, and y=O. For a volume element we take the volume of that part of the solid formed by revolving about the y-axis a rectangle with sides y and dx at a distance x from the y-axis. Then the volume element dV y =2nxydx, whence
b

Vy=2n

~ xydx.
CI

Sec. 9]

Volumes of Solids

Solution. We have Therefore,


Yl ==b- Val-xl and Y2=b+
u .

Va -x2.
2

Vx=n ~ [(b+ Val-xl)I-(b- Va 2-xl )2J dx =


-a
a

= 4nb

~ Val-xl dx =
-a

211,1

alb

(the latter integral is taken by the substitution x =a sin t).

-Q

x
Fig. 53

Fig 52

The volunle of a solid obtained by the rotation, about the polar axis, of a sector formed by an arc of the curve r == F (rp) and by two radius vectors ~ =-~ a, cp == ~ may be computed froln the formula
Vp = ; n

5,1
o

sin Ipd Ip.

Thls same formula is conveniently used when see{<ing the volume obtained by the rotation, about the polar axis, of some closed curve defined in polar coordinates. Example 3. Determine the volume formed by the rotation of the curve , == a sin 2cp about the polar axis. Solution.
n n
,1

Vp= 2.~ n

5
I

sin Ip dip =

na'5
0

siu l

21p

sin Ip dip

= ~ na S
3

5
I

sin 4 ep cos' (p dq>

== ~ .na'.
105

164

Definite lntegrals

[Ch. 5

2. Computing the volumes of solids from known cross-sections. If S = S (x) Is the cross~ctional area cut ofT by a plane pprpendicular to some strai~ht line (which we take to be the x-axis) at a point with abscissa x, then the volume of the solid is
XI

V=

~ S (x)dx,
Xl

where

K 1 and x! are the abscissas of the extreme cross-sections of th~ solid. Example 4. D~termine the volume of a wedge cut oft a circular cylinder by a plane passing through the diampter of the base and inclinpd to the base at an angle u. The radius of the base is R (Fig. 53). Solution. For the x-axis w? take th~ diam~ter of the base along which the cutting plane intersects the base, and for the y-axis \ve take the dialneter of the base perpendicular to it. The equation of the circumference of the base is Xl y2 = R2. The area of the spction ABC at a distance x from the origin 0 is I 1 2 S (x) = area ~ ABC = 2" AB. Be = 2"' yy tan a = Y2 tan Q. Therefore, the soughtfor vol urne of the wedge is

V=2

SyZtanadx=tana S(RZ-xZ)dx= ~ tanaR z


o
0

1685. Find the volume of a solid formed by rotation, about the x-ax is, of an area bounded by the x-axis and the parabola y= ax-r (a >0). 1686. Find the volume of an ellipsoid formed by the rotation Xl y2 of the ellipse aa-+/;1-=1 about the x-axis. 1687. Find the volume of a solid formed by the rotation, about the x-axis, of an area bounded by the catenary y = a cosh!..., the a x-axis, and the straight lines x == a. 1688. Find the volume of a solid formed by the rotation, about the x-axis, of the curve y = sin 2 x in the interval between x = 0 and X=1t. 1689. Find the volume of a solid formed by the rotation, about the x-axis, of an area bounded by the semicubical para bola y2 = x=s, the x-axis, and the straight line x == ]. 1690. Find the vol ume of a sol id formed by the rotation of the same area (as in Problem 1689) about the y-axis. 1691. Find,t,he volumes of the solids formed by the rotation of an area bounded by the lines y=e x , x=O, y=O about: a) the x-axis and b) the y-axis. 1692. Find the volume of a solid formed by the rotation, about the y-axis, of that part of the parabola yl = 4ax which is cut otT by the straight line x = a.

Sec. 9]

Volumes of Solids

165

1693. Find the volume of a solid formed by the rotation, about the straight line x = a, of that part of the parabola yJ. = 4ax which is cut 011 by this line. 1694. Find the volume of a solid formed by the rotation, about the straight line y = - p, of a figure bounded by the parabola
yl = 2px and the straight line x = ~

1695. Find the volume of a solid formed by the rotation, about the x-axis, of the area contained between the parabolas y = x2 and y= V"t. 1696. Find the volume of a solid formed by the rotation. about the x-axis, of a loop of the curve (x-4a) y2 = ax (x- 3.2) (a >0). 1697. Find the volume of a solid generated by the rotation

of the c)'ssoid y2=-2 AI about its asymptote x=2a. a -x


1698. Find the volume of a paraboloid of revolution whose base has radius R and whose altitude is H. 1699. A right parabolic ~egment \vhose base is 2a and altitude h is in rotation about the base. De~ermine the volume of the resulting solid of revolution (C:lvalieri's "lemon"). 1700. Show that the volume of a part cut by the plane x = 2a off a solid formed by the rotation of the equilateral hyperbola x z _ y2 = a" about the x-axis is equal to the volulne of a sphere of radius a. 1701. Find the volume of a solid formed by the rotation of a figure bounded by one arc of the cycloid x=-a (t- sin t). y=a(l--cost) and the x-axis, about~ a) the x-axis, b) the y-axis, and c) the axis of symmetry of the figure. 1702. Find the volume of a solid formed by the rotation of the astroid x=acos't, y=b sin' t about the y-axis. 1703. Find the volume of a solid obtained by rotating the cardioid r=a(l +coscp) about the polar axis. 1704. Find the volume of a solid formed by rotation of the curve r=acos 2 cp about the polar axis. 1705. Find the volume of an obeli<;k whose p~rallel bases are rectangles with sides A, B and a, h, and the altitude is h. 1706. Find the volume of a right elliptic cone whose base is an ellipse with semi-axes a and b, and altitude h. 1707. On the chords of the astroid x2/a+y"J/3.=a2/J, which ar,e parallel to the x-axis, are constructed squares whose sides are equal to the lengths of the chords and whose planes are perpendicular to the xy-plane. Find tHe volume of the solid formed by these squares.

_16_6

D_e...:..fi_n_ite_In_t-:;eg~ra_l_s

[Ch. 5

1708. A circle undergoing deformation is moving so that one of the points of its circumference lies on the y-axis, the centre describes an ellipse ;: +~= I, and the plane of the circle is perpendicular to the xy-plane. Find the volume of the solid generated by the circle. 1709. The plane of a moving triangle remains perpendicular to the stationary diameter of a circle of radius a. The base of the triangle is a chord of the circle, whi Ie its vertex slides along a straight line parallel to the stationary diameter at a distance h from the plane of the circle. Find the volume of the solid (called a conoid) formed by the motion of this triangle from one end of the diameter to the other. 1710. Find the volume of the solid bounded by the cylinders x" -t- Zl = at and y2 + Zl == al. 1711. Find the volunle of the segment cut off froIll the elli py2 22 tic paraboloid 2p-l-2q=x by the plane X=Q. 1712. Find the volume of the solid bounded by the hyperbox2 y" Z2 loid of one sheet a 2+b and the planes z=O and z=h. 2 -2"=1 C
1713. Find the volume of the ellipsoid a Sec. 10. The Area of a Surface of Revolution
The area of a surface formed by the rotation, about the x-axis, of an arc of the curve y == f (x) between thE' poi nts x = a and x = b, is expresse-d by the form uta b b
Sx=2n
x 2+ b! +2"= 1. c
y2 2

22

Syd;x dx =2:t S Y VI+y'ldx


a u

(I)

(ds is the differen tial 01 the arc of the curve).

,y

y
2a

o
Fig. 54

-....&.---__
no
Fig. 55

--L

___

21Ca

I f the eq ua lion of the curve is represented differen t1 y, the area of the surface Sx is cbtained from formula (1) by an appropriate change of variables.

Sec. 10]

The Area of a Surface of Revolution

167

Example 1. Find the area of a surface formed by rotation, about the x-axis, of a loop of the curve 9y 2 =x (3-X)1 (Fig. 54). Solution. For the upper part of the curve, when 0 e=;;;;; x <; 3, we have y= ; (3- x)

yx.

Whence the differential of the arc ds= ;~ dx. From for-

mula (I) the area of the surface


S = 2n

r31 J
a
o

.. r- x+ 1 (3 -x) r x 2 Yx dx = 33t.

Example 2. Find the area of a surface formed by the rotation of one arc of the cycloid x==a (t-sln t); y==a (I-cos t) about its axis of symmetry (Fig. 55). Solution. The desired surface is formed by rotation of the arc OA about the straight line AB, the equation of which is x=na. Taking y as the independent variable and notIng that the axis of rotation AB is displaced relative to the y-axIs a distance na, we y will have
2a

= 2n

5
o
J

ds (Tea-x) dy dYe

Passing to the variable t we obtain


n

S =2n

j" (no-ot+a sin t)


o

y (~~r + (~~r
+ 0 sin t) 2a sin ~
t sin
dt

dt

=-

= 2n = 4no
l

5 5(n
o

(no-at

=
B
dt =
FIg.

sin

~-

~ + sin t sin ~)

56

t t t =4na 2 [ -2ncos2+2tcos2-4sin2+3s1n'2

4.

tJ'" =8Jt (n- 4) at.


0

1714. The dimensions of a parabolic mirror AOB are indicated in Fig. 56. It is required to find the area of its surface. 1715. Find the area of the surface of a spindle obtained by rotation of a lobe of the sinusoidal curve y= sinx about the x-axis. 1716. Find the area of the surface formed by the rotation of a part of the tangential curve y = tan x from x = 0 to x about the x-axis. 1717. Find the area of the surface formed by rotation, about the x-axis, of an arc of the curve y::::z e-~, from x = 0 to x ==

=: '
+00.

168

Defin ite Integrals

lCh.6

1718. Find the area of the surface (called a catenoid) formed by the rotation of a catenary y = a cosh!... about the x-axis from a X = 0 to x === a. 1719. Fi nd the area of the surface of rotation of the astroid
%2/8 _~ y2/3 ==

a2/3 about the y-axis. 1720. Find the area of the surface of rotation of the curve x={ y2_ ~ loy about the x-axis from y= 1 to y=e. 1721 *. Find the surface of a torus formed by rotation of the circle x 2 +(y_ b)2-==a 2 about the x-axis (b>a). 1722. Find the area of the surface forlned by rotation of the ellipse ~:+~2=1 about: I) the x-axis, 2) the y-axis (a>b). 1723. Find the area of the surface formed by rotation of one arc of the cycloid x = a (t - sin t) and !J = a (1- cos t) about: a) the x-axis, b) the y-axis, c) the tangent to the cycloid at its highest point. 1724. Fincl the area of the surface forlned by rotation, about the x-axis, of the cardioid
x=a(2cost-cos2t), } y = a (2 sin t- sin 2/).
1725. Determine the area of the surface formed by the rotation of the lemniscate r 2 == a l cos 2cp about the polar axis. 1726. Determine the area of the surface formed by the rotation of the cardioid, = 2a (1 + co~ cp) about the polar axis.

Sec. 11. Moments. Centres of Grav:ty. Guldin's Theorems


to. Static moment. The static moment relative to the i-axis of a material point A having mass m and at a distance d fronl thz I-axis is the quantity
The static moment relative to the I-axis of a system of n material ('oints with Inasses ml , m2 , , mn 1ying in the plane of the axis and at distances d1 , d z , , dn is the sum
n

M,=md.

M l = ~midi'
I=J

(1)

where the distanct's of points lying on one side of the l-axis have the plus sign, thost' on th~ other side have the minus sign. In a similar manner we define the statIc moment of a system of potnts rt'lative to a rlane. If the massps continuously fill the line or figure of the xy-plane, then the Rtatic moments Mx and My about 1he x- and y-axes are exoressed ,respectively) a~ integrals and not as the sums (1). For the cases of geometric figures, the density is considered equal to unity.

Sec. 11]

Moments. CentrtS of Gravity. Guldin's Theorems


wh~re

169

Is the arc length, we have

In particul ar: 1) for the curve x = x (s); y = y (5),


L L

the parameter s

Mx=~y(s)dS; My=~X(S)dS
o
0

(2)

(ds =

Y (dx) 2 + (dy)2

is the differential of the arc);

v
b

!---~ T---~
a
Fig. 57

t--- b

I
--I

Fig. 58
th~

2) for a plane fi~ure bounded by th~ curve y = y (x), vertIcal ltnes x= a and y.==.b, we obtain

x-axis and two

MX =
Example 1.

5Y I y I dx;
a

My =

Sx r y I dx.
a

(3)

Find the statIc

monl~nts

bounded by the straight lines: ~

+f = 1, x::=O, y=O (FIg. 57)

about the

x- and

y-axe3 of a triangle

Solution. Here. y=b ( 1- : ) . Applyir.g formula (3), we obtain

and
My=b

J xli-ao

{.,

X )

dx=

a2 b

ti-

2. Moment of inertia. The nl0mellt of Inertia, about an l-a'Ci5, or a rn1fcrial point of rna~s m at a dIstance d fronl th~ l-aXl'i, I') the llulnber II-=- fnd 2 Th~ moment of cnertia, about an I-axis, o[ a systel1l of n nlalenal pOints with masses mJ , m:. __ , mil IS the surn
Il

J,=-=-~ nllj~,
i ... l

170

Definite Integrals

[Ch.5

where d1 , d l , dn are the distances of the points from the I-axis. In the case of a continuous mass, we get an appropriate integral in place of a sum. Example 2. Find the moment of inertia of a triangle with base band altitude h about its base. Solution. For the base of the triangle we take the x-axis, for its altitude, the y-axis (Fig 58). Divide the triangle into intlnitely narrow horizontal strips of width dy, which play the role of elementary masses dm. Utilizing the similarity of triangles, we obtain h-y dm=b --dy
h

and
dl x =y"dm=7i y2 (II-g) dy.

Whence

r 1 I X--"= Ii J y2 (h-y) dy = 12 biz'.


b
o

3. Centre of gravity. The coordinates of the centre of gravity of a plane figure (arc or area) of mass M are computed from the formulas
-

x=M' y=,

My

Mx

where M x and My are the ric figures, the mass M is For the coordinates of curve y=f (x) (a ~x ~b), we have
B
b
X

static moments of the mass. In the case of geolnetnumerically equal to the corresponding arc or area. the centre of g;ravity (~ Y) of an arc of the plane connecting the points A la, f (a)l and B [b, f (b)],
B

~
A

d!>

~
a

Y 1 + (y')Zdx
-

~
A

yds

~
II

y
b

y 1 + (y')2 dx
V I + (y')' dx

x= -.-5- =

~b------

~ Vl+(y')2dx
a

y=--s

~
a

The coordinates of the centre of gravity (~ Y) of the curvilinear trapezoid a ~ x ~ b, 0 <;; y ~f (x) Inay be computed from the formulas

Sxy dx
x=-S- ,
b
a

Sy2 dx
a

y = --S-

where S =

~ Y dx is the area of the figure.


a

There are similar formulas for the coordinates of the centre of gravity of a volume. Example 3. Find the centre of gravity of an arc of the semicircle x 2+ y2 = a 2; (y ~ 0) (Fig. 59).

Sec J11

Moments. Centres of Gravity Guldin's Theorems

171

Solution. We have
Y -==

Y-a-_-x2

2;

y' == _-_x_

Ya z

x!

and
ds
--=.

~r
y

+ (y')! dx.:.=
u

adx . _ Yal-x!
t ---=

Whence
M y::-:=
a

-a

' f ax J ds =-= Ya -x d
X
Z 2
OJ

0,

-a

(l

Mx== I'"' yds--- \'

'-a

'" -a
C
a

Ya -x
2

Va -x
Z

_udx -

_: 2a 2 ,

M-

J -V-a--x-'
2

adx

na.

l-1ence,

-a
x~-O;

Y=-=

Fa.

4. Guldin's theorems. Theorem t. The area of a surface obtaIned by the rotation of an arc of a plane curve about SOIne axis lying in the same plane as the curve and not intersecting it is equal to the product of the length of the curve by the circunlference of the circle described by the centre of gravity of the arc of the curve. Theorem 2. The volulne of a solid obtained by rotation of a plane figure about sonle axis lying in the plane of the figure and not intersecting it is equal to the product of the area of this fi~ure by the circumference of the circle described by the centre of gravity of the figure.

-a
Fig. 59

1727. Find the static moments about the coordinate axes of


a segment of the straight line
~+1[=1 a b

lying between the axes.

172

Definite Ifltegrals

(Ch. I

1728. Find the static moments of a rectangle, with sides a and b, about its sides. 1729. Fi nd the static momen~s, about the x- and y-axes, and the coordinates of the centre of gravity of a triangle bounded by the straight lines x+y=a, x=O, and y=O. 1730. Find the static moments, about the x- and y-axes, and the coordinates of the centre of gravity of an arc of the astroid
I I I

x'-ty'=a', lying in the first quadrant. 1731. Find the static moment of the circle r = 2asin q> about the polar axis. 1732. Find the coordinates of the centre of gravity of an arc of the catenary x y=acosh -a
from x= -a to x=a. 1733. Find the centre of gravity of an arc of a circle of radius tJ subtending an angle 2a. 1734. Find the coord inates of the centre of gravity of the arc of on~ arch of the cyeloid X= a (t- sin t); y = a (I-cos t).
1735. Find the coordinates of the centre of gravity of an area x2 LI I bounded by the ell ipse Q2 -1- b 2 = 1 and the coord i na te axes (x ~ 0,
y~O).
I

1736. Find the coordinates of the centre of gravity of an area

bounded by the curves

y=x, Y=~ x. 1737. Find the coordinates of the centre of gravity of an area bounded by the first arch of the cycloid
x=a(t-sinl), y=a(l-cost)

r-

and the x-axis. 1738**. Find the centre of gravity of a hemisphere of radius a lying ahove the xy-plane with centre at the origin. 1739**. Find the centre of gravity of a hOlno~eneous right circular cone with base radius r and altitude h. 1740**. Find the centre of gravity of a honl0~eneous hemisphere of radius a lying above the xy-plane with centre at the origin.

Sec. 121

ApplginR Definite lnieRrals to Solution of Phl/sical Probltms

173

1741. Find the moment of inertia of a circle of radius a about its d iarne~er. 1742. Find the mOITlents of inertia of a rectangle with sides a and b about its sides. 1743. Find the mOlllcnt of inertia of a right parabolic segment with base 2b nnd altitude h about its axis of sYlnrlletry. 1744. Find the moments of inertia of the area of the ellipse ~: + ~: = 1 about its principal axes. 1745**. Flnd the polar monlent of inertia of a circular rinq with radii R 1 and R, (R,<R 2 ) that is, the nlonlent of inertia about the axis passing through the centre of the ring and perpendicular to its plane. 1746**. Find the moment of inertia of a homogeneous right circular cone \\lith base radlus R and altitude H about its axis. 1747**. Flnd the Inonlent of inertia of a hotnogeneous sphere of radius a and of rnass M about its diaillcter. 1748. Find the surlace and volullie of a torus obtained by rotating a cIrcle of radIus a about 8n aXIs lying in its plane and at a distance b (l> >a) frOll1 its centre. 1749. a) Dcterrnine the po~ition of the centre of gravity of
2 2
l

an arc of the astroid x 3 -~ {/3 =a7 lying in the first quadrant. b) Find the Cl'n~rc of gravity of an area bounded by the curves 2 y2 = '2px and x == 2py. 17fiO**. a) Find the centre of gravity of a sen~icircle usin~~ Guldin's theorerTI. b) Prove by Guldin's theorem that the centre of gravity of a triangle is distant from its base by one thlrd of its altitude
Sec. 12. Applying Definite Integrals to the Solution of Physical Problems
1. The path lraverfied by a point. If a point IS In motion ;lloll~ some curve and the Clhsolute value of th~ velocity u =-1 (t) is :J known fUI1(uon of the time t, then the path traversed by the pOint in an Interval of time (t l , tIl is

Example 1. The velocity of a point is


(J=O.l, m/~ec. Find the path s covpred by the point In the interval of time T = 10 ~ec followIng the conlOlc.celnent 01 motion. What is the Iuean velucity ~f mollon durlng lhis interval?

174 Solution. We have:

Definite Integrals

[Ch.S

s=

5
o

10

t4110 O.lt'dt=O.lT 0 =250 metres

and
V mean

S = y= 25

mjsec.

2. The work of a force. If a variable force X = f (x) acts in the direction of the x-axis, then the work of this force over an interval [XI' x 2 ] is
X2

A= Sf (x) dx.
Xl

Example 2. What work has to be performed to strC'tch a spring 6 Cln, if a force of 1 kgf stretches it by 1 cm? Solution. According to Hook's la\v the force X kgf stretching the spring by X m is equal to X == kx, where k is a proportionali ty constant. Putting x=O.OI m and X=l kgf, we get k=lOO and, hence, X==IOOt. Whence the sought-for work is
0.06

A=

0.06

100xdx=50x21
0

=0.18 kgm

3. Kinetic energy. The kinetic energy of a material point of mass nz and velocity v is defined as

The kInetIC energy of a system of n material points with masses ml , ma, , mn having respective velocities VI' VI' , v n ' is equal to

K=

L m~Vi.
I. =1

(I)

To compute the kinetic energy of a solid, the latter is appropriately partitioned into elementary parttcles (which play the part of material points); then by summing the kinetic energies of these particles we get, in the limit, an integral in place of the sum (I). Example 3. Find the kinetic energy of a homogeneous circular cylinder of density tJ with base radius R and altitude h rotating about its axis with angular velocity roo Solution. For the elementary mass dm we take the mass of a hollow cylinder of altitude h with inner radi us r and wall thickness dr (Fig. 60). We have: dm = 211:' .h6 dr. Since the linear velocity of the mass dm Is equal to ki net Ic energy is
V

= fro,

the elemenfary

dK =- -2- = n,'ro 2 h6 dr,

v 2dm

Sec. 12]
Whence

Applying Definite Integrals to Solution of Physical Problems


R

175

K = 3tCl)2hfJ

r d, ~ S o
3

noo2fJR4h 4 .

4. Pressure of a liquid. To compute the force of liquid pressure we use Pascal's law, which states that the force of pressure of a liquid on an area S at a depth of immersion h is p=yhS, where V is the specific weight of the liquid.

Fig. 60

FIt! 61

Example 4. Fi nd the force of pressure experienced by a selniclrcle of radiu5 r submerged vertically in water so that it5 diameter is flush with the \vater surface (Fig 61). Solution. We partition the area of the semicircle into elements-strips parallel to the surface of the water. The area of one such element (ignoring higher-order infinitesimals) located at a distance h frotn the surface is
ds =-= 2xdh = 2

V,2 -

h2 dlz.

The pressure experienced by this elernent is


dP =yh ds = 2yh

V ,2-h" dh,

where y is the speciflc weight of the water equal to unity. Whence the entire pressure is

1751. The velocity of a body thrown vertically upwards with initial velocity Vo (air resistance neglected), is given by the

17i

Definite Integrals

Ie.":. 6

formula

v= vo-gt,
where t is the time that elapses and g is the acceleration of gravity. At what distance from the initial position will the body be in t seconds from the time it is thrown? 1752. l"he velocity of a body thrown vertically upwards with initial velocity Vo (air resistance allowed for) is given by the fornlula

v=c.tan( - ~

t+arctan

~O),

\vhere t is the time, g is the acceleration of gravity, and c is a constant. Find the altitude reached by the body. 1753. A point on the x-axis performs harmonic oscillations about the coordinate origin; its velocity is given by the fornlula v = V o cos (J}t,

where t is the time and [Jo, CJ) are constants. Find the law of oscillation of a point if when t = 0 it had an abscissa x=O. What is the nlean value of the absolute magnitude of the velocity of the point during one cycle? 1754. The velocity of motion of a point is v = ie- o. o't m/sec. Find the path covered by the point {roIn the commencenlcnt of motion to full stop. 1755.. A rocket rises vertically upwards. Considering that when the rocket thrust is constant, the acceleration due to decreasing weight of the rocket increases by the law J = a A bt (a- bt > 0), find the velocity at any instant of time t, if the initial velocity is zero. Find the altitude reached at tim~ t = t 1 1756*. Calculate the work that has to be done to pump the \vater out of a vertical cylindrical barrel with base radi us Rand altitude H. 1757. Calculate the work that has to be done in order to pump the water out of a conical vessel with ver1ex do\vnwards, the radius of the base of which is R and the altitude H. 1758. Calculate the work to be done in order to pump wateJ out of a semispherical boiler of radius R =]0 m. 1759. Calculate the w.Jrk ne~dcd to pum~ oil out of a tank through an UP4Jcr opening (lhe tank has the shape of a cylinder with horizontal axis) if the specific weight of the oil is y, the length of the tank H and the radius of the base R. 1760**. What work has to be done to raise a body of massm from the earth's surface (radius R) to an altitude h? What is the work if the body is removed to infinity? ..

Sec. 121

Appl"int! Definite lntellrals to Solution of Ph,,-;lcal Problems

171

1761**. T\\'o electric charges eo = 100 CaSEand e1 =200 CGSB lie on the x-axis at points X o = and x, -= J em, respectively. What work will be done if the second charge is moved to point XI = 10 em? 1762**. A cylinder with a movable piston of diameter D=20 em and length l = 80 em is filled with steam at a pressure l p = 10 kgf em What work must be done to halve the volume of the s1eam with temperature kept constant (isotherlnic process)? 1763**. De~ermine the work perforrned in the adiabatic expansion of air (having ini1 ial volume Vo = 1 rna and pressure P. -= I I<gfjcm l ) to volume VI = 10 In'? 1764**. A vertical shaft of weight P and i radius a rests on a bearing AB (Fig. 62). The frictional force between a small part a of the base of the shaft and the surface of the su pport in contact with it is F = Jlpa, \\'here P = canst is the pressure of the shaft on the surface of the su pport referred to unit area of the support, \vhile Jl is the coefficient of friction. Find the work done by the frictional force during one revolution of the shaft. 1765**. Calculate the kinetic energy of a Fig. 62 disk of Inass M and radius R rotating with angular velocity (1) about an axis that passes through its centre perpendicu lar to its plane. 1766. Calculate tile kinetic energy of a ri~ht circular cone of mass M rotating \\lith angular velocity (1) about its axis, if the rad j us of the base of the cone is R and the aItitude is H. 1767*. What work has to be don~ to stop an iron sphere of radius R = 2 mc~r~s rotating \vith angular velocity 0) = 1,000 rplD about its dianleter? (Specific \veight of iron, y = 7.8 g,tcnl J . ) 1768. A vertical trian~lc wIth base b and altitude h is submerged vertex dJwn\\'ards in wa1er sa that its base is on the surface of the water. Find the preSSUie of the \vater. 1769. A vertical dam has the sh lP~ of a trapezoid. Calculate the water pressure on the dam if w~ kn)w that the upper ba~e a=70 In. the lower bU5e b=:5Q m, and the height h=~O ffi. 1770. Find the pressure of a liquid, whose sp~cific weight is V, on a vertical ellipse (wl1h axes 2a and 2b) \vhose centre is subnlerged in the liquid to a distance h, \vhile the lTIaJur axis 2a of the ellipse is parallel to the level of the liquid (h;::: b,. 1771. Fl nd the water pressure on a vrrllcal circular cone with radius of base R and altitude H subln~rged in \va~el verlex downwards so that its base is on the surface of the walcr.

,P

178

Definite 1ntegrals

[Ch. 5

Miscellaneous Problems 1772. Find the mass of a rod of length 1 = 100 cm if the linear density of the rod at a distance x cm from one of its ends is 6=2+0.001
Xl

gjcm.

1773. According to empirical data the specific thermal capacity of water at a temperature to C (O:s;; t :s;; 100) is

c= 0.9983-5. 184 x 10- 5 t + 6.912 X 10- 7 t 2


What quantity of heat has to be expended to heat I g of water from 0 C to 100 C? 1774. The wind exerts a uniform pressure p gjcm l on a door of width b cm and height h cm. Find the moment of the pressure of the wind striving to turn the door on its hinges. 1775. What is the force of attraction of a material rod of length 1 and mass M on a material point of lTIaSS m lying on a straight line with the rod at a distance a from one of its ends? 1776**. In the case of steady-state laminar now of a liquid through a pipe of circular cross-section of radius a, the velocity of flow v at a point distant r from the axis of the pipe is given by the formula P (2 2) t1 = 4fll a - r , where p is the pressure difference at the ends of the pipe, f.1 is the coefficient of viscosity, and 1 is the length of the pipe. Determine the discharge of Liquid Q (that is, the quantity of liquid flow.ing through a cross-section of the pipe in unit tinle). 1777*. The conditions are the same as in Problem 1776, but the pipe has a rectangular cross-section, and the base a is great compared with the altitude 2b. Here the rate of flow t1 at a point M (x,Y) is defined by the formula

v = 2~l [b 2-(b- y)2J,


Determine the discharge of liquid Q. 1778**. In studies of the dynamic qualities of an automobile, use is frequently made of special types of diagrams: the velocities v are laid off on the x-axis, and the reciprocals of corresponding accelerations a, on the y-axis. Show that the area S bounded by an arc of this graph, by two ordinates V=OI and V=OI' and by the x-axis is numerically equal to the time needed to increase the velocity of motion of the automobile from vJ to VI (acceleration time).

Sec. 12) Applying Definite Integrals to Solution

of Physical Problems

179

1779. A horizontal beam of length 1 is in equilibrium due to a downward vertical load uniformly distributed over the length of the beam, and of support reactions A and B (A =B = ~ ) t directed vertically upwards. Find the bending moment Mx in a cross-section x, that is, the moment about the point P with abscissa x of all forces acting on the portion of the beam AP. t 780. A horizontal beam of length I is in equilibrium due to support reactions A and B and a load distributed along the length of the beam with intensity q=kx, where x is the distance from the left support and k is a constant factor. Find the bending moment M x in cross-section x.
Note. The intensi ty of load distribution is the load (force) referred to unit length.

1781*. Find the quantity of heat released by an alternating si nusoidal current

1=loSin(~t-q
during a cycle T in a conductor with resistance R.

Ch'lpter VI

FUNCTIONS OF SEVERAL VARIABLES

Sec. 1. Fa1ic Notions


1 is called a single-valued function of two variables x, y, if to ec)ch set of their values (x, y) in a givpn ron~t' there corre-;ponds a unique vatul' of z The variables x and yare called arguments or independent variubles. The functional relation is denoted by

A variable quantity

1. The concept of a function of stveral variables. Functional notation.

2=f(x, y).

Similarly, we define functions of three or more arguments. fxample 1. Express the volume of a cone V as a function of its gcneratrix x ann of its base radIus y . Solution. Frorn geometry we l<now that the volume of a cone is

where h is the altitude of the cone. But h=

Vx 2-

y2. Hence,

z
This is the desired functional relation. The value oi the function z=t(x.y) at a pOI nf P (a.b). that is, when x = a and y =: b, is drnoted by I (alb) or f (P) Generall y speakin~. the ~~eornetnc rerresntation of a runct ion like z -=-! (x,y) in a r'ctangular coordi nate system X, Y. Z is a surface (Fig. 63).

Example 2. Find I (2, -3) and t ( I,


Fig. 63

~) if

f (X,y) = -,-,~xy

x2 + ,)2

Solut;on. Substitutinq r= 2 and g= -3, we find


:l2

Il2,

-J)= :l.2.(-3)

+ (_3)2

-12

13

Sec.

I}

Basic Notions

181

Putting x= 1 and replacing y by

!L., x

we will have
y

Y) f ( I, x =
thaI is, f ( 1.

1+ ( X x 2 y2 ( 1I)' = '2xy , 21 x

)1

1L \ = f (x, x)

y).

2. Domain of definition of a function. By the domain of definition of a function 'I == f (x. y) WP undcr~tand a set of point~ (x, y) in an xy-plane in which the given function is d~fin~d (that is to say, in w:,ich It takes on defInite real value~) I n the slnlplest cases, the rlotnain of definition of a function i~ a finite or infinite part of the xy-plane bounded by one or several Curves (the boundan, of the domain). Slnlll(lrly. for a funrtion of three variables u=f (x, y, z) the domain of definit ion of the functlon IS a vol ulne in .\yz-space. Example 3. Find the domain of definition o[ the function

z= Y4_.\2 __ 2 . y
Solution. The run~tion haC\ real values if 4_X 2 _ The I at ter I nrq ua I it Y is sat i ~fi('rl hy thr coord inates of circle of radil1\ 2 with centre ~t the coordinate orll!ln. nition 01 the [unction is the InterIor of the circle (FIg

y2 > 0 or x 2 + y2 < 4. poi nts I yi ng inside a The domain of defi64).

FIg. 64

FIg 65

Example 4. Find the domain of definition of the function z = arc sin 2'

+ V--).y
IS

Solution. The first tenn of the funct ion


-2,s=;x~2.

defined for - I E:;


xy~O,

i ~ 1 or

when {

~~

g. or

The second term has rt'al values if


when {

~:z

g .. The dom a

Le., In two C35es:

In

of

definition of the entire

function is shown in fig. 65 and includes the boundaries of the domain.

1_8_2

F_u_n_c_t i_on_s_o~f_S_e_ve_r_al_Va_r_ia_b_le_s

,[Ch. 6

30 Level lines and level surfaces of a function. The level line of a function z = f (x, y) is a line f (x, y) -== C (in an xy-plane) at the points of wh ich the function takes on one and the same value z=C (usually labelled in drawings). The level surface of a function of three arguments u = f (x, y, z) is a surlace f (x, y, z) = C, at the points of which the function takes on a constant value u=C. Example 5. Construct the level Ii nes of the function z=x2 y. Solution. The equation of the level lines c has the form x 2 y == C or y =.: I .
x

Putting C=O, I, 2, ... , we get a fanlily of level lines (Fig. 66).

Fig. 66

1782. Express the volume V of a regular tetragonal pyramid as a function of its altitude x and lateral edge y. 1783. Express the lateral surface S of a regular hexagonal truncated pyramid as a function of the sides x and y of the bases and the altitude z.
f(x, y)===xy+-=-. y

1784. Find 1(1/2,3), 1(1, -1), if

1785

Find
X2 _

f (Y,x), f (-x,

-y),

f(

f, ~ )

1
I

f (x,

y) ,

if

/(x, Y)=-2-. xy 1786. Find the values assumed by the function

f (x,

y) = 1
2

+ x-y

at points of the parabola y = x , and construct the graph of the function F (x) = f (x, x 2 ).
1787. Find the value of the function
Z==--~2

xC + 2X 2y2+ y4
l-x
_yl

at points of the circle x 2 y2 1788*. Determine !(x), if

f(~)= ~(y>O).
1789*. Find

f (x,

y) if

f(x+y, x--y)=xy+yl.

Sec. 1]

Basic Notions

183

1790*. Let z = VY + f (Vi-i). Determine ihe functions z if z==x when y= i.


1791". Let
Z==

f and

z = xf ( ~ ). Determine the functions f and z if

Yi +yl when x= i. 1792. Find and sketch the domains of definition of the following functions:
a) z=Yi-X 2 _ y2; b) z = i-t_(X_y)2;

i) z=Yysinx; j) z = In (x 2 + y);
k)

c) z=ln(x+y);

d) z=x-f-arccosy; e) z=Yi-x 2 -!- Yi_ y 2; f) z=arcsin lL ; x


g)

z = arc tang +1 2 2 , xy
1 Z=-_ x +!J
2 2 ,

x-y

I) m) n)

z == y7=4 + Y4=iI;
(a> 0);

I Z=V y-y .; x

h) z==Y(xl+y2-a2)(2a2_x2_yl)

Z=x=r+y ;

0) zY sin (x 2 -+_ y2).

1793. Find the domains of the following functions of three arguments: a) u = ~rx + yy + V"z; c) u = arc sin x + arc sin y + arc sin z; b) u = In (xyz): d) II = Y l-x 2 _yl_ZI. 1794. Construct the level lines of the given functions and determine the character of the surfaces depicted by these functions:
a) z==x+y;
d) z=Yxy;

g) z= ~ ; 11)

z=

h;
2X
x
2

c)

Z=X

2 _

y 2;

f) z=

I-lxI-luI;

i) z=

+ y I.

1795. Find the level lines of the following functions: a) z=ln(x2 +y); d) z=f(y-ax); b) z=arcsinxu; e) c) z = f <YXl + y2); x

z=f(JL).

1796. Find the level surfaces of the functions of three independent variables:

c) u=xl-t

a) u=x+y-t-z; b) u = Xl yl + Zl;

gl_ZI.

184

Funct ions of Several Variables

(Ch.6

Sec. 2. Continuity
1. The limit of a function. A number A is called the limit of a function 2=1 (x, y) as the point P' (x. y) approaches the point P (a, h). if for any B > 0 there is a tJ > 0 such that when 0 < Q < tJ, wherE:' Q -== Ylx-a)1 (y-b)'

is the distance between P and p', we have the inequality

Jn th is case we write

I!(x, y)-AI<e.
lim
x-+a

f (x,

y) = A.

y-+b

2. Continuity and points of discontinuity. A function continuous at a pOint P (a, b) if linl f (x, y) = f (a, b). x-.Q
y-+b

= f (x,

y) is called

A function that is continllous at all points of a given


continllO:.lS over this range

ran~e

is called

A function f (x, y) may cease to be continuous either at separate points (isolated pOlnt of discontinuity) or at points that form one or several lines (lines of dlscontinuity) or (at times) more complex geometric objects. Example 1. find the discontinuities of the function

z=A 2 _y
Solution. Tile function will be m~aningless if the denomin3tor becomes zero. But )(2_ y -==O or y=x 2 i~ the equation of a parabola. Hence, the given function has [or its discontinuity the parabola y==.x 2

xy+ 1

1797*. Find the following limits of functions: . sit] xy e) Ii n a) Ii itl (x~ T yl) sin! C) I (Ln _ _ ;
x-+o

xy ,

X-+O
"-~2

x-~ox+y

_X_ ;

IJ -+ J

1/-+0

, x+y. b) ILn --z-+ 2 t


X ...

oo.x

d) Ii.n (1
X-+OO

+ 1!.. ) x;
XJ

f) lIn ~+ 2X ~o..\. IJ ..... 0

X 2 _!J2

l/

00

y-.k

1798. Test the fotlowi ng III nction for conti nuity:


f(x, y)= { Vl-x"l.-y2 when x

+ y2 ~ It when x"l. +y"l. > 1.


2

1799. Find points of discontinuity of the functions:


a) z = tn Vx~
b)
Z

+ yl,.
;

c) Z -

l - x2 _y2

= l,x,-y --) 2

d)

z =cos -.!-
>..u

Sec.

31

Partial DeritJatives

185

1800. Show that the function


2xy h I I 0 z= { Z--+ x y 2 W en x + y =F , o when x=y=O

is continuous with respect to each of the variables x and y separately, but is not continuous at the point (0, 0) with respect to these variables together.

Sec. 3. Partial Derivatives


1. DeOnltion of a partial derivative. If z =f (x, y), then assunling, for example, y constant, we get the derivative
OZ _ 10
uX
~

~t'~O

1m

f (x + A",

y) A

uX

f (x, y) - f'% (X, Y),


-

which is called the part ial derivat ive of the function z with respect to thevarIable x. I n similar fac;;hion we define and denote the partial derivative of the function z w!th respect to the variable y It is obvious that to find partial derivativ~~, one can use the ordinary formulas of differentiation. Example 1. Find the partial derivatives of the function

z=lntan~.
y

Solution. Regarding y as constant, we get


iJz

ax = t---X an - cos
g
1 tan y

-2X

Ii = ---:2X.
YSlng

Similarly, holding x constant, we will have


(11 = - - x - _ - x

iJz Y

1 ( x) 2x -2 =. 2x. cos! Y yl sin y u

Example 2. Find the partial derivatives of the follOWing fUJlction of three arguments: 1 U =X y 2 z + 2x-3y + z + 5. Solution.
iJu =.3 . X I y 2 z +2,
iJu iJy=2x 'y z-3,

ax

au -=x y2+1.
iJz
'

2. Euler's theorem. A function f (x, y) Is called a homoleneous function of degree n if for every real factor k we have the equality

f lkx,

ky) ==: kill (x. y)

186

Functions of Several Variables

(Ch. 6

A rational integral function will be homogeneous if all its terms are of one and the same degree. The following relationship holds for a homogeneous differentiable function .of degree n (Euler's theorem):
xf~ (x, y) + Yf~ (x, y) = nf (x, y).

Find the partial derivatives of the following functions: 1801. z=x'+y'-3axy. 1808. z=xy
1802. Z=x+y
1803 1805.
Z

x-y

1809. z=e
Z=

sin

.!L
%

= 1L X

1 8 1 0 . .. /

arc SI n JI

x2

X _

1804. z =: VX 2 _ y l.
Z=

2 y2

1811.

z = In sin x+a yy

Yx 2x+y2

1806. z==ln{x+Vr+y').
1807. 2=arctan.[. x
"

1812. u=(xy)z. 1813. u=zxy.


x Vr---xY+ y .

1814. Find f1&(2, 1) and f y (2, 1) if f(x, y)=

181 5. Fin d f~ ( 1, 2, 0),

2, 0), (1, 2, 0) if f(x, y, z)=ln(xy+z).

';J (1,

f:

Verify Euler's theorem on homogeneous functions in Examples 1816 to 1819: x+y 1816. f (x, y) = Ax 2 + 2Bxy-Cy 2. 1818. f (x, y) = V"j""'f'"';j
x!+y

1817.

Z=

x2+ y 2.

1820. Find

~(~~,

1819. f(x,y)=lnJ!.. x
where r=VxZ+yZ+zl.

1821. Calculate ::

az 1822. S how t h at x

'x g; ax /ar +
acp

,if x=rcos<p and y=rsin<p.


dz Y ay=2, If z= In (xl+xy+yl).
OZ
If

ax

1823. Sh OVl t h at x(jX+U ay=xy+z,

OZ

z=xy+xe%

JL

h 1824. Sh ow tat

au au au 0, If u=(x-y)(y-z)(z-x). ox+ay+az:Z:
ox

1825. Show that ~+aau+~u=lt if u=x+x- Y

J826. Find z = z (x, y), it ~ = !+x I . uy X y

uZ

y-z

Sec. 4]

Total DiUerential of a Function

187

1827. Find z==z(x, y) knowing that


02
uX

x 2 y2 ~=-x

and z(x,y)=siny when x=l.

1828. Through the point M(1,2,6) of a surface z=2x l +y. are drawn planes parallel to the coordinate surfaces XOZ and YOlo Determine the angles formed with the coordinate axes by the tangent Ii nes (to the resulting cross-sections) drawn at their

common point M. 1829. The area of a trapezoid with bases a and b and altitude h is equal to S=I/ 2 (a-l-b)h. Find

:~, :~, :~

and, using

the drawing, determine their geonletrical meaning. 1830. Show that the function I 2xy . f 2 , 2 =1= 0 J ~+ 2' 1 X -1- Y , f (x, y) --=--' x y , 0 , if X=IJ==O
~,

has partial derivatives f~ (x, y) and f~ (x, y) at the point (0, 0). although it is discontinuous at this point. Construct the geometric image of this function near the point (0,0).
Sec. 4. Total Differential of a Function
to. Total increment of a function. The total incretnent of a function z = f (x, y) is the difference ~z-= ~f (x, Y)-=f (x-t- Ax, y+ ~y)-f (x, y). 2. The total dift'erential of a function. The total (or exact) differential of a function z==f (x, y) is the principal part of the total increlnent ~z, \vhich is linear with respect to the increments in the argulnents f1x and Ay. The difference between the total increnlent and the total differential of the function is an infinitesimal of higher order compared with Q= VAx 2 + li y 2. A function definitely has a total differential if its partial derivatives are continuous. If a function has a total differential, then it is called differentiable. The differentials of independent variables coincide with their incrernents, that is, dx= ~x and dy= Ay. The total differential of the function z = f (x, y) is computed by the fonnula
dz = 0)( - dx -t- oy" - d'l. Similarly, the total differential of a function of three arguluents u =/ t x, y, z) is computed fronl the formula

oz

dz

du=

ou dx au ou ax + ay dy+ iJz dz.


y)=x l t-xy_ y 2

Example 1. For the function

f (x,

find the total increlnent and the total differential.

J88

Functions of Several Variables Sol ution.


~f {x, y)
=.: {( ~

(Ch. ,

f (J( + f!x,

= [(2x+ y) Ax + (x-2y) Ay) + (Ax! + Ax Ay- ~~l').

==- 2x.Ax + A.\2 +x Ay + y. Ax + ~x.Ay-2y. Ay-fl y 3 =

y + ~y) = (x + AX)I + (x + Ax) (y + ~y) - (y + Ay)2; + l\X)2 + (x + Ax) (y + Ay) - (y + Ay)21- (x 2 + xy- y2) =

Here, the expression df=(2x+y)Ax+(x-2y)Ay is the total differential of the function. while (Ax 2 + tuAy-Ay 2) is an Infinitesimal of higher order compared with V ~.t2 + Ay2. Example 2. Find the total differential of the function

z=
Solution.

Yx
Y

+y2.

az
dz

ax .r l x +
2

x
X

;
y2

a-= . 1--'::' Y r.\.2 + yl


11"-'-dy 2
r .\

az

.. r - - dx+ r AI + y!

+ y2

xdr+ydy .. r - - r l! -t- yl

3. Applying the total differential of a function to approximate calculations. suffirifn~ly small I Ax I and I Ay I and, hence. for ~ufficiently small Q= Y A\I+ Ayl, we have for a differentiable function z=flx. y) the approximate equality Az:::::: dz or For
Az ~

az Ax+ay az Ay. ax

Example 3. The altitude of a' cone is H = 30 em, the radius of thE' base R = 10 cm. How will the volume of the cone change. if we increase H by 3 mm and ditninish R by 1 mm? Solutlol\. The volume of the cone is V = nR 2H. The change in volume we replace approximately by the differential

AV::.;:dV=3' n(2RH dR+R 2 dH)=


1 = 3" n (-210300.1 + 1000.3) = -IOn =:::-31.4 em'.

Example 4. Compute 1.02'01 approximately. Solution. We consider the function z =-= xY . The desired number may be considered the increased value of this function when x= 1, y=3, Ax=0.02, dy=O.01. The initial value of the function z = 1'= 1,
f!z::::::: dz= yxY - 1 Ax+xY In x Ay= 3.1.0.02+ 11n 10.01 =0.06. Hence, 1.02'01::::::: 1 + 0.06= 1.06.

1831. For the function f (x, y) = x 2 y find the total increment and the total diflerential at the point (I, 2); compare them if
a)
~x=l, ~y=-=2;

b)

~x=O.I,

~y=O.2.

1832. Show that for the functions u and v of several (for example, two) variables the ordinary rules of differentiation holdJ
a) d (u

c) d ( ~ )

+ u) = du + dv;

b) d (uu) = u dv

= v dU;u dv

+ v du;

Sec. 4]

Total Differential of a FunctIon

189

Find the total differentials of the following functions: 1833. z==x'+y'-3xy. 1841. z=lntan1!.... 1834. z=x 2y'. x
x 2 _l/!

1842. Find

1835. Z= 1+"1
1836. z=sin x+cos y. 1837 . z=yx. y 1838. z == In (x 2 -~- yl).
1839. 1840.
.\
2

df (1, 1), if
x
Y

f(x.y)=z.

1843. u = xyz . _ 1844. U = Vx 2 -t- y2 + zj.

f(x,y)=ln(l+i).
1=

1845.
1846.

u=(xy+i-r
u =

arc tan y x
Y

-t..

arc tan x~ z

+aretan~.

1847. Find df(3, 4,5) if 2 j(x,y,z)= .. r_ _2.


r
AI

1848. One side of a rectangle is a = 10 em, How will a diagonal 1 of the rectangle change if the side a is increased by 4 rum and b is shortened by 1 mIn? Approximate

+y the other b = 24 cm.

the change and compare it with the exact value. 1849. A closed box with outer dimensions 10 em. 8 em, and 6 em is made of 2-mm-thiek plywood. Approximate the volume of nlaterial used in making the box. 1850*. The central angle of a circular se~tor is 80 it is desired to reduce it by 1. By how much should the radius of the sector be increased so that the area will remain unchanged, if theoriginal leng:h of t he radius is 20 cm? 1851. Approximate:
0 ;

a) (1.02)1. (O.97)2~ b) V(4.05)2 + (2.93)1; c) sin 32cos 59 (when converting degrees into radius and calculating sin 60 take three significant figures; round otT the last digit).
0

1852. Show that the relative error of a product is approximately equal to the sum of the relative errors of the factors. 1853. Measurements of a triangle ABC yielded the following da1a: side a = lOOm2m. side b = 200m3 m, angle C :== 60 10. To what degree of accuracy can we conlpute the
side c?

1854. The oscillation period T of a pendulum is computed from the fOflnula

T= 2nyr _

I g ,

190

Funct ions of Several Variables

[Ch. 6

where l is the length of the pendulum and g is the acceleration of gravity. Find the error, when determining T, obtained as a result of small errors ~l = a and ~g = p in measuring land g. 1855. The distance between the points Po (x ot Yo' and P (x, y) is equal to Q, while the angle formed by the vector PoP with
the x-axis is a. By how much will the angle a change if the point P(P o is fixed) moves to PI (x+-dx, y+dy)?
Sec. 5. Differentiation of Composite Functions
to. The case of one independent variable. If z = f (x, y) is a differentiab Ie function of the arguments x and y, which in turn are differentiable function~ of an independent variable t, x = cp (t), Y = 'I> (i), then the derivative of the composite function z=f (cp (i), lJ' (t)] may be puted from the form ula
dz dt
C0I11-

= ~ dx + az dy

ax dt ay d t

(I)

In particular, if t coincides with one of the arguments, for instance x, then the "total" derivative of the function z with respect to x will be:
dx- ax

dz _~ + QZ eJJ!

ay dx

(2)

Example I. Find ::. if


z=eIX + 2Y, where x=cos t, y=t 2

Solution. From formula (I) we have: dz ~ IcOSt+2t 3 dt =e,x+2Y3 (-sin t) eIX + 2Y 22t =e'X+2Y (4t-3 sin t) =e (4t -:3')int).

Solution. ~=yeXY. From formula (2) we obtain

Example 2. Find the partial derivative iJiJZ and the total derivative dz Ii x dx' z=e"Y, where g=cp(x).

dx = yexy

dz

+ xexy cp' (x).

2. The case of several Independent variables. If z is a composite function of several independent vari~bles, for insta~ce, z = f (x,y), where x= cp (u,v), Y=1I' (u, tI) (u and v are Independent variables). then the partial derivatives z with respect to u and v are expressed as
iJz = ~ ~ + ~ iJy iJu ax au ay iJu
(3)

Sec. 5]
and

DiOerentiation of Composite Functions


az az ax az ay

191

au = ax au + au au .

(4)

In all the cases consi dered the following formul a holds: az az dz= axdx+aydY

(tile invariance property of a total dzgerential).


Example 3. Find : : and

:~. if

z = t (x,

y),

where

u x=uv, y=--V.

Solution. Applying fonnulas (3) and (4), we get: az, , 1 == f x (x, y). u fy (x, y)

au

+
F'

and

oz _ ' (x, av -fx

y) u-, y (x, y) fj2 .

Example 4. Show that the function z == cp (Xl

+yZ)

satisfies the equation

yax -x ay=O.
Solution. The function cp depends on x and y via the intennediate argutHent x2 + y2 = t, therefore,

az

az

az dz at ax=dl ax=CP , (x I + y 2) 2x

~= ~ aat =q>' (xl y2) 2y. uy dt y Substituting the partial derivatives into the left-hand sid{) of th{) equation, we get OZ az Y -x ay = ycp'(xl + y2) 2x-xcp' (Xl y!) 2y = 2xyq>' (Xl + y2)-2xy q>'(x2 y2)ii5iO,

and

ax

that is, the function z satisfies the given equation.

1856. Find :: if
z=~, where y

x=ef , y=lnt.

1857. Find

:~ if

u = In sin
1858. Find ~: if

..(y, where x = 3t

l ,

Y=

vt'+l.

u=xyz, where x=tl+l. y=lnt, z==tant.

192

Functions

of Several Variables

(Ch.6

1859. Find

:~ if
Z

U=

X 2+yl

,where x=R cost, y=R sin t, %=H.


if

1860. Find :

Z = ltv,

where u == sin x, v = cos x.

1861. Find :; and :; if

z=arc tan!. and y=x". x


1862. Find :; and

~ if

z =xY , where y = cp (x).


1863. Find :; and :; if
z=f(u, v), where u=xl_y', v=e xy

1864. Find ~ and ~ if

z = arc tan...!., y where


1865. Find :: and
Z

X=

u sin 0, Y= u cos v.

~ if
u = xy

= f (u), where

+y x

1866. Show that if

then

u = cD (Xl + y' + 2 1 ), where x = R cos q> cos '1'. y = R cos cp sin '1', z = R sin cp,
~-o oq>-

au and 0lJ'=0.
y = cp (x), z = 1J' (x, U).

1867. Find ~ if
U = f (x, y, z), 1868. Show that if

where

z=f (x+ay), where f is a differentiable function, then

oz

iJy =a

ax

az

Sec. 61

Derivative in a Given Direction

193

1869. Show that the function


w=f(u, v),

where u=x+at, v=y+bt satisfy the equation


iJt

aw _ aW+ b ow -a ox oy
z = y cp (X 2_ y 2)

1870. Show tl-)at the function


fles th e equa tIon x 1 oz I az z sa t IS a;+-y iJy=!j2 1871. Show that the function

z = xy + xcp (
f th e equa tIon x-a az -I-Y~=xy az sa t ISles x uy 1872. Show that the function

~)

+ z.

z = eY cr
satisfies the equation 1873. The side of of 5 mjsec, the other is the rate of change
(X 1_ y 2)
a

(ye::' )

~+xy~=xyz.

rectangle x -:~ 20 m increases at the rate side y == 30 m decreases at 4 m/sec. What of the perilneter and the area of the rectx = t, y

angle?
1874. The equations of Illotion of a material point are
~= t
2 ,

z:= t l

What is the rate of recession of this point from the coordinate origin? 1875. T\vo boats start out from A at one time; one moves northwards, the other in a northeasterly direction. Their velocities are respectively 20 kmfhr and 40 knljhr. At what rate does the distance between them increase?
Sec. 6. Derivative in a Given Direction and the Gradient of a Functioll
10 The derivative of a function in a given direction. The derivative of a function z=f(x, y) ift a given direction I=PP, is

az =
iJl
7_ 1900

lhn
P1P ~ 0

f (PI )-f (P)


PIP

194

Funct ions of Several Variables

[eh. 6

where f (P) and f (PI) are values of the function at the points P and PI If the function z is differentiable, then the following formula holds:
iJz iJz iJz . m=axcosa +aySlna,
(1)

where a is the angle formed by the vector I with the x-axis (Fig. 67).

o
Fig. 67

In similar fashion we define the derivative in a given direction I for a function of three argunlents u = f (x, y, z). In this case
du

all at = ax cos u

+au au cos ~ + au az cos y,

(2)

where a, ~, yare the angles between the direction I and the corresponding coordinate axes. The directional derivative characterises the rate of change of the function in the given direction. Example 1. Find the derivative of the function z=2x 2 -3 y 2 at the point P (1, 0) in a direction that makes a 1200 angle with the x-axis. Solution. Find the partial derivatives of the given function and their values at th~ point P:

az = 4x; (az) ax Ox p = 4;

oz = -6y.' a (~) iJy y p


Here,

=0.
1

cosu=cos 1200 = - 2 sin a = sin 1200 = -2-3 Applying formula (1), we get
OZ ( - - +0. lIr3" -=4 f 2 a1 2 -2-= -

1)

The minus sign indicates that the function diminishes at the given point and in the given direction. 2. The gradient of a function. The gradient of a function z == f (x, y) IS .a vector whose projections on the coordinate axes are the corresponding par-

Sec. 6]

Derit.'ative in a Given Direction

195

tial derivatives of the given function: grad z == ox I + oyJ


02 02 (3)

The derivative of the given function in the direction I is connected with the gradient of the function by the following formula:

02 = pro)., gra d z. at
That is, the derivative in a given direction is equal to the projection of the gradient of the function on the direction of differentiation. The gradient of a function at each point is directed along the normal to the corresponding level line of the function. The direction of the gradient of the function at a given point is the direction of the maximum rate of increase
of the function at this point, tlUtt is, when l=grad z the derivative on its greatest value, equal to

~~

takes

u -= f (x, y, z):

In sitnilar fashion we define the gradient of a function of three variables, grad u=iJ~ J + a- k. x 1-+- uy Z
au iJu iJu
(4)

The gradient of a function of three variables at each point is directed along the nonnal to the level ~urface passing through this point. Example 2. Find and construct the gradient of the function z.=;x 2 y at the point P (I, I).

y 2
1

-~ - - --~
PI
I
2
Fig. 68

3 X

Solution. Compute the partial derivatives and their values at the point P.

~=2xY; (~)p=2;

(~)p=l.
Hence, grad
7*

z=21+J

(Fig. 68).

196

Funct ions of Several Variables

[Ch. 6

1876. Find the derivative of the function z = x!-xy-2y J 2) in the direction that produces an angle '()f 60 with the x-ax is. 1877. Find the derivative of the function z=x 8 -2x 1 y-+ xy2+ 1 .at the point M (1, 2) in the direction from this point to the point N (4, 6). 1878. Find the derivative of the function z == InVx! -~ y! at the point P (1, 1) in the direction of the bisector of the first

at the point P (1,

quadrantal angle. 1879. Find the derivative of the function u=x!-3yz+5 at the point i'A (1, 2, -1) in the direction that iorms identical angles with all the coordinate axes. 880. Find the derivative of the function u = xy + yz ,- zx at the point M (2, 1, 3) in the direction from this point to the point N (5, 5, 15). t881. Find the derivative of the function u = In (eX + eY -t-~ e1) at the origin in the direction which forms with the coordinate axes x, y, z the angles a, ~, V, respectively. 1882. The point at which the derivative of a function in any direction is zero is called the stationary point of this function. Find the stationary points of the following functions:
a) z==x!+xy+y2-4x-2y; b) z === Xl yl -3xy; c) u == 2y 2 -t- z!-xy-yz 2x.

1883. Show that the derivative of the function z=L taken x at any point of the ellipse 2x2 -1- y2 = C! along the normal to the

ellipse is equal to zero. 1884. Find grad z at the point (2, 1) if

z = Xl + yl- 3xy.
1885. Find grad z at the point (5, 3) if
Z==

Vx

_y!.

1886. Find grad u at the point (1, 2, 3), if u =xyz. 1887. Find the tnagnitude and direction of grad u at the point (2, -2, 1) if

1888. Find the angle between the gradients of the function Z= In ~ at the points A (1J2, 1/4) and B (1, 1).

Sec. 7]

Higher-Order Derivatives and DiUerentialb

197

1889. Find the steepest slope of the surface z=x! + 4y at the poi llt (2, 1, 8). 1890. Construct a vector field of the gradient of the following functions: a) z=x+y: b) z=xy;
Sec. 7. Higher-Order Derivatives and Differentials
1. Higher-order partial derivatives. The second partIal derivatives of a function Z=== f (x, y) are the partial derivatives of its first partial derivatives. For second derivatives we use the notations

:x (a:)

::~ =
2

t:

x (x, y);

o (DZ) a z ::::: f" ay ax = axay Xli (x,

y) and ~o forth.

Derivatives of order higher than second are similarly defined and denoted. If the partial derivatives to be evaluated are continuous, then the result of repeated dtUerentlailon is independent of the order in which the differentzatlon IS perfornled. Example 1. FInd the second partIal derIvatives of the functIon
Z ==-

arc tan !... y .

Solution.

Flr~t

find the first partial derIvatives: az 1 I y ax=~ Y=.\2+ y 2 '

l-t-y'l
1

ay=~

az

+ y2

2xy

!/t = -

x )

x2+y".

Now differentiate a second time:

ax
2

iJ'lz _ ~
2 -

ax

(_y_ )__ + y! x!

(x 2

+ y2)1
xl_gl
(x 2

az

ay2 =

ay -

0 (

x2

+ y2

X)

= (xt

cJ2 Z 0 ( Y ) ox ay = oy x! + y2 =

2xy + y2)1 l 1.(x + y2)_2y.y


(xl

+ y2)1

+ y2)2

We note that the so-called "mixed" partial derivative Inay be found in a <titTerent way, namely: 0 22 cJ2Z a( X) 1. (x 2+ y2)-2x.x x 2 _ y:! 2 2 ox ay ay ax = x + yl = (Xl + y2)1 = (x + y2)2

ax -

198

Functions of Several Variables


2.

[ell. 6

=/ (x,

Higher-order differentials. The second diUerential of a function


y) is the differential of the differential (first-order) of this function: d 2 z = d (dz)

We similarly define the differentials of a function z of order higher than two, for instance: and, generally,
dnz = d (dn-1z).

If z = t (x, y), where x and yare independent variables, then the second differential of the function z is computed from the fonnula
d
2

z= ax2 dx 2+ 2 ax ay

a~

a~

dx dy

+ ay 2 dyl.

a~

(1)

Gener all y, the followi ng symbolic fornl ula holds true:

dnz=( dX~ +dy a~rZi


eral independent variables, then
02Z d 2 d! z~ax2 x

it is fornlally expanded by the binomial law. If z = f (x, y), where the arguments x and yare functions of one or sev. y 02 + 2 oxoydXd Z 02 Z 2 az 2 2 +ay 2 d y +axdx+ayd Y

az

(2)

If x and yare independent variables, then d 2 x=O, d 2 y=O, and formula (2) becomes identical with formula (1). Example 2. Find the total differentials of the first and second orders or the function
z = 2x 2 -

3xy _ y2.

Solution. First method. We have


OZ 02 (jX=4x-3y, oy= -3x-2y.

Therefore,
dz iJz oz= ax dx+ ay dy= (4x-3y) dx-(3x+2y) dYe

Further \ve have

whence it follows that


dZ==ox2dx
2

a2 z

a z dx dY+a 02 + 2 axoy y

Z
2

dY =4dx -6dxdy-2dy.

Second method. Differentiating we find


dz=4x dx-3 (ydx+xdy)-2y dy= (4x-3y) dx-(3x+2y) dYe

Differentiating again and remelnbering that dx and dy are not dependent on x and y, we get d 2z= (4dx-3dy) dX-(3dx+2dy) dy=4dx 2 -6dxdy-2dy 2.

Seco 7]

Higher-Order Derivatives and DiOerentlals

199

1893. Find

O~2~y if
iJ~2;y if
z=arctan
t+y l-xy

1894. Find

1895. Find

~~ if
r =

V x 2+y2 +zlo

1896. Find all second partial derivatives of the function

u = xy + yz -+ zxo
97 F In 18.
o

ox dy
2

OIU

az
If

JOf

1898.

a r-'1 n d iJxayZ
o

z = sin (xy)

1899. Find

f:

(0, 0),

f:

r/

(0, 0), f~'l (0, 0) if

f(x, y)=(l-+ x)m(l +y)n.


1900. Show that
iJxiJy

02 Z

z if = iJLjiJx iJ2

z=arc sin
1901. Show that
iJxay

Yif

x- y x

2
2

ayox

02

z=xY
1902*. Show that for the function

f (x,

y)

= xy x --a-+ y

x 2 _y.
2

200

Functions of Several Variables

[Ch. 6

[provided that f (0, 0) = 0] we have f ~y (0, 0) = - 1, f~x (0, 0) =


1903. Find
OIZ o!z 02 ox ax oy , oy
2 ,

+ 1.

if

z = f (u, v),
where u =
Xl

1904. Find ::~ if u=f(x, y, z). where z = cp (x, y). . o!z 02Z O!Z. 1905. Find oxay , ay2 If

+ y., v = xy.

ax.'

z = f (u, v), where u = cp (x, Y), v = 'I' (x, y).


1906. Show that the function

u -= arc tan JL x

satisfies the Laplace equation alll

-+--0 ax! ay
2 -

a2 u

1907. Show that the function

u=ln, , where r=V(x-a)2+-(y-b)2, satisfies the Laplace equation


02 U

ox. -1- oy2 = o.

au
2

1908. Show that the function u (x, t) = A sin (a'At + cp) sin 'Ax

satisfies the equation of oscillations of a string


02 U
iJt2 =
2

0 !U

ax2
(X-XO)2+(y- YO)2 +(% -Zo)l 4a 2 t

1909. Show that the function


u(x, y, z, t)=
1 y_ e (2a 1T t)3

(where x o' Yo, zo' a are constants) satisfies the equation of heat conduction

Sec. 7]

Higher.Order Derivatives and DiOerentials

~Ol

1910. Show that the function


u = cp (x-at)

+ 'i' (x + at),

\vhere cp and ' are arbitrary twice differentiable functions, satisfies the eq uation of asci llations of a string 2 a2 u 2 au . at 2 =a ox l
1911. Sho\v that the function

z = xcp ( ~
satisfies the eq uation
Xl

)f- '!' ( ~ )
+ Yay! iJlz = 0 .
2

ax 2

a2z + 2x

Y iJxay

iJ2 z

J 912. Sho\v t hat the function

u = cp (xy) + l/ xy ( ~ )

satisfies the equation


1913. Sho\v that the function z = f [x -i- cp (y)] satisfies the equation az a2z az a2z
ax ax ay =

ay ax

1914. Fi 11 d u

--=:;

II

(x. y)

if

a -a a x y =0.
211

1915. Determine the fornl of the function u=u(x, y), \\'hich satisfies the equation 1916. Find d!z if 1917. Find d 2 u if
u = xyz. 1918. Find d1z if z=cp(t), where t=X 2_'t-yl.
x

1919. Find dz and d!z if


Z

= uf)

where u = y , v = xy.

202

Functions of Several Variables

[Ch.6

1920. Find d 2 z if
z=f(u, v), where
U=QX,

v=by.

1921. Find d1z if


Z

= f (u,

v), where u = xeY , v = ye". z = eX cosy.

1922. Find d'z if

1923. Find the third differential of the function z=xcosy-~y sinx.

Determine all third partial derivatives. 1924. Find df(l, 2) and d2f(1, 2) if f(x, y)=x l +xy+y2-4Inx-IOlny.
1925. Find d 2f (0, 0,0) if f (x, y, z) = Xl + 2y 2 -1- 3z 2 -2xy -t- 4xz + 2yz.
Sec. 8. Integration of Total Differentials
to. The condition for a total differential. For an expression P (x, y) dx + y) dy, where the functions P (x, y) and Q (x, y) are continuous in a simply connected region D together with their first partial derivatives, to be (in D) the total differential of some function u (x, y), it is necessary and sufficient that

+ Q(x,

ox

iJQ
====

iJP

oy .

Example 1. Make sure that the expression (2x -t- y) dx -f- (x + 2y) dy is a total differentIal of some function, and find that function.
Solution. In the given case, P=2x+y, Q=x+2y. Therefore,

~=:: =

= I,

and, hence,
(2x+ y) dx

au + (x+ 2y) dy = du = ou ax dx+ iJy dy,

where u is the desired function.

It is given that ax == 2x + y; therefore,


u= ~ (2x+y) dx=xl+xy+q> (y).
Buf on the other hand
iJu

ou

oy=x+Q>' (y)=x+2y, whence <p' (y)=2y, cp(y)=y2+C


u == x 2+xy

and

Finally we have

+ y2 + c.

(2x+ y) dx+(x+ 2y) dy=d (xl+xy+ y2+C).

Sec. 8J

'ntegration of Total DIfferentials

203

2. The case of three variables. Silnilarly, the expression P (x, y, z) dx+ Q (x, y, z) dy+ R (x, y, z) dz, where P (x, y, z), Q(x, y, z), R (x, y, z) are, together with their first p~rtial derivatives, continuous functions of the variables x, y and z, is the total differential of some function u (x, y, z) if and only if the following conditions are fu Ifi lied:

Example 2. Be sure that the ex pression


(3x 2

+ 3y-l) dx + (Z2 -t- 3x) dy + (2yz + 1) d2

is the total differential of some function, and find that function. Solution. Here, P=3x!+3y-1, Q =z2+3x, R=2I1z+ 1. We establish the fact that

and, hence,

(3x 2 -f-3y-l)dx+(z2+3x) dy+{2yz+ I) dz=du


\vhere it is the sought-for function. We have au ax =3x2 + 3/j-l, hence,

au
uX

au du dX+ - dY+- dz, ay az

u= ~ (3x 2 +3y-l) dx=x'+3xy-x+q> (y. z).


On the other hand,

- =

~=3x+a-=z +3x, uy y au aq> -=2yz--1-1, az iJz

au

a(j)

\vhence dq> =Z2 and aacp =2yz+ 1. The problem reduces fo finding fhe function y z of hvo variables cp (y, z) whose partial derivatives are known and the condition for total differential is fulfilled. We Hnd CI': q> (Y. z) = ~ zldy = yz2 + Ijl (z).

:: =2yz + Ijl' (z)=2yz+ I,


'1"(z)=I, 'I'(z)=z+C, that is, cp (y, 2)= yz 2+ Z+C. And finally,

u=x' +3xy-x+ yz2+ z+C.

204

Functions of Several Variables

I-laving convinced yourself that the expressions given below are total differentials of certain functions, find these functions. 1926. y dx+x dYe 1927. (cosx+3x1y)dx+(X 3 _y2)dy.
1928
(x+2y)dx+ydy
(x -1- y)2
II

x+2y 2x-y dx -,----11929. Z--+ x y2 x - y I dy. 1 x 1930. -II dx-dye y2

1931.

x x-t- y
2

dx+

X Y+ y
2

dy.

1932. Determine the constants a and b in such a manner that the expression
(ax 2

+ 2x Y -1- y2) dx - (x 2+ 2xy + by 2) dy (Xl + yl)2

should be a total differential of some function z, and find that function. Convince yourself that the expressions given below are total differentials of some functions and find these functions. 1933. (2x -t- y + z) dx + (x + 2y -r- z) dy + (x -r y -l- 2z) dz.
1934. (3x 2y 2 3z) dx (4xy 2y -z) dy + (3x - y - 2) dz. 1935. (2xyz-3yz 8xy 2 2) dx + (x 2 z- 6xyz 8x2 Y -t- 1) dy -1- (xly- 3xy 2 -l- 3) liz.
2

-r

1936.

(~'-;2)dx f-(~-:2)dY+(~-:')dz.
X
2

1937. x dx + y dy + z dz .
+y2+ Z 2

1938*. Given the projections of a force on the coordinate axes


X=-y(x+ y)2 ,

y=~ (x+ y)2 ,

where A, is a constant. What must the coefficient A be for the force to have a potential? 1939. What condition must the function f (x, y) satisfy for the expression

f (x,

y) (dx

+ dy)

to be a total differential? 1940. Find the function u if


du

= f (xy) (y dx -J- x dy).

Sec. 9]

DiUerentiation of / nlplicit Functions

205

Sec. 9. Differentiation of Implicit Functions


1. The case of one independent variable. If the equation f(x, y)=O, where x and y, defines y as a function of x, then the derivative of this irnplicitly defined function, provided that f~ (x, y) i= 0, may be found fronl the formula

I (x, y) is a differentiable function of the variables


dy dx = -

f~ (x, y) f~ (x, y)

(1)

Higher-order derivatives are


(I)

found by successive differentiation of formula


2

dy d y. Example 1. Flnd d- and -d 2 If x x


(x 2
th~

+ y~)' -

3 (x 2

+ y2) + 1 == O.

Solution. Denoting the left-hand side of this equation by f (x, y), we find partial denvatlves
I~ (x, y)

1;/ (x,

== 3 (x 2 + y2)2. 2x- 32x-= 6x [(x 2 + y2)2 -1], y) -= 3 (x 2 + y2)2. 2y-3 2y == 6y [(x 2 + y2)2_1].

\\lhenCl\ applyIng forll1ula (1), \ve get


dy f~ (x, y) 6x [(x 2 dx == --f~ (x, y) = - 6y [(x 2

+ !/)2_1] x + y2)2_1] = -y-

To find the second derivative, differentiate \vith respect to x the first derivatIve WhICh \ve have found, taking into consideration the fact that y is a functlun of x' dy l.y_x y_x{_'yt) d:!11 d ( x ) dx \ = _Y~ + x2
dx2=dx

--y

= -

y2

y2

y3

2. The case of several independent variables. Sirndarly, if the equation


F (x, y, z) =-=0, where F (x, y, z) is a differentIable function of the variables

x, y and z, defines z as a function of the independent variables x and y and F~ (x, !I, z) :p 0, then the partial derivatives of this irnplicitly represented function can, generally speaking, be found fro III the fonnulas

oz
at =

F~ (x, !It z)

az
oy =

F;/ (x,

y, z)

F ~ (x, y, z)'

F ~ (x, y, z)

(2)

Here is another way of finding the derivatives of the function z: differentiating the equation F (x, y, z) =0, \ve find

aF of of -dx+-dy+-dz=O. ox cJy iJz


Whence it is possible to detennine dz, and, therefore,

ax

iJz

and ay

QZ

206

Funct ions of Several Variable')


Example 2. Find

[Ch. 6

:~

and

:~

if

x 2 _2 y 2

+ 3z -yz + y== o.
2

Solution. First method. Denoting the left side of this equation by F we find the partial derivatives

(x, y, z),

F;(x, y, z)=2x, F;(x, y, z)=-4y-z+l, F;(x, y, z)==6z-y.

Applying formulas (2), we get


iJz ox =

F~ (x, y, z) 2x , ( x, y, z) = - 6z-y; Fz

dz ay =

F~ (x, !I, z) p' z (x, y, z)

1-4y-z 6z-y

Second method. Dlfferentiating the given equation, \ve obtain 2x dx-4y dy+6z dz-y dz-z dy+dy=O. Whence we detennine dz, that is, the total differential of the inlplicit function: dz =2xdx+(1-4Y-2)dy . y-6z Comparing with the formula

dl=:;dX+~dY.

we see that

az 2x az 1-4y-z ax = y-6z ' ay = y-6z

3. A system of Implicit functions. If a systelTI of two equations


F(x, y, u, v)=O, { o (x, y, u, v)=o

defines u and. v as functions of the variables x and y and the Jacobian D (F, 0) D (u, v)

then the differentials of these functions (and hence their partial derivatives as well) may be found from the following set of equations
dF aF iJF aF ax dx + ay dy + au du + av dv = 0,

aF of

au av :1= 0 ao ao ' au iJu

ao ao ao { iJO dx +-a du -+- v dv =0. x y dy +-a u

-a

a-

(3)

Example 3. The equations u+v=x+y, xu+yv=l

define u and

t1

. iJu au do av as functtons of x and y; find -iJ 'iJ--' ~ and x y ux ay .

Sec. 91
we obtain

DiOerentlation of Implicit Functions

207

Solution. First method. Differentiating both equations with respect to x.

\\'hence

Sim ilarl y

\ve

find
au v+y av v+x

dy=-x-y' ay=x-y Second nlethod. By differentiation we find two equations that connect the differentials of all four variables:
x du +u dx+y dv
Solvin~

du+dv==dx+dy, + v dy~.

this system for the dIfferentials du and dv, we obtain

du= _ (u

+ y) dx-t- (v -t- y) dy ,
x-y

dv ~ (u

+ x) dx + (v +x) dy
x-y

Whence

4. Parametric representation of a function. If a function z of the variables x and y is represented paralnetricatly by the equations
x=x(u, v), y=-=-y(u, v), z=z(u, v)

and D (x, y)
D (u, v)

then the differential of this function nlay be found froln the following system
of equations

ax ax dX-a-dll-f--a dv,
II

az du -1- az dz -== aiJ- dv. u v


ox=p and dy-=.q

ay dy = au da + av dv,

ay

(4)

az

Knowi ng the differential dz ~ p

az

d~

+q dy,

we find the partial derivatives

208

Functions of Several Variables

[Ch.6

Example 4. The function z of the arguments x and y is defined by the (lquations x=u+v, y=U 2+V 2 , z=ul+V S (u;6v). OZ . az Find ox and ay Solution. First method. By differentiation we find three equations that connect the differentials of all five variables: dx=du+dv, dv, { dy= 2u2du + 2v 2 dz -::::: 3u du + 3v dv. From the first two equations we determine du and dv:
du=- 2vdx-dy 2 (v-u) , dv==.dy-2udx. 2 (v-u)

Substituting into the third equation the val ues of du and dv just found, we bave: <lz = 3u 2 2v dx-dy + 3v 2 dy- 2u dx = 2 (v-u) 2 (v-u) = 6uv (u-v) dx+3 (V 2_U 2 ) dy:=.: -3uv dx -t-~ (u + v) dYe 2(v-u) 2 Whence oz az 3 -3uv, ay="2 (u+V).

ax=

Second method. From the third given equation we can find


az =3u 2 au +3v 2 av; ax ax ox aZ=3u 2 au +3v2aV . ay ay ay

(5)

Differentiate the first two equations first with respect to x and then \vith res pee t to y: l=au + av

ax ax'

au av { O=2Ll+2v ox ax'

From the first system we find au v av u ax=v-u' ax=u-v Fronl the second system we find au 1 av oy 2 (u- v)' ay

2 (v-u) .

'Substituting the expressions :: and :; into formula (5), we obtain


OZ 3 2 V 2 U iJx= U v_u+ 3v u-=v=-3uv,

iJy- U 2(U-v)+ v '2(v-u)

OZ _ 3

3 ( 2 u-i-v).

Sec. 9]

Dtfferentiation of Impltcit Functions

209

1941. Let y be a function of x defined by the equation


x2 y2 ~+1J2=1.
dy d 2y dx' dx2

FInd

and dxS 1942. Y is a function defined by the equation x!-t- y 2+2axy==O (a>l).

day

Show that ~~ = 0 and explain the result obtained.


1943. Find ~ if y = 1 -\- yX,
1944. Find dx and dx 2 If y = x -1- In y.
dy d2 y .

1945. Find

(lfJ!) dx

2y (d ) if dx x 2-2xy -l- y2 + x + y- 2 === O.


t=l

and

X=l

Taking advantage of the results obtained, sho\v approximately the portions of the given curve in the neighbourhood of the point x == 1. 1946. The function y is defined by the equation
In 'Vx 2 -1- y2 =-= a arc tan.11- (a=i=O).
x
d 2y dx 2

Find dX and

dy

d dy d y If an d -1947 F 111 dx dx 2

1 -+- xy-ln (e xy -t- e -xY) = o.


1948. The function z of the variables x and y is defined by the equation xa -1- 2y' -f-- Z3 - 3xyz-- 2y + 3 = O.

FInd

iJz az ax and ay

1949. Find ~ and

ax

az ay if

x cos y -1- y cos z + z cos x = 1.

l!)SO. The function z is defined by the equation Xl + y2_ zl-xy = o. Find


iJz

ax

and

az au for the systenl of values x = - 1,

y = 0, z = 1.

210

Functions of Several Variables

[Ch. 6

y, z)=O. Show that a-a-a-=-I. y z x 1953. z = cp (x y), where y is a function of x defined by the equation 'I\J(x, y)=O. Find ::. 1954. Find dz and d2 z, if x 2 + y 2 + Z2 == al.
1952. f(x,
t

ax ayaz

1955. z is a function of the variables x and y defined by the

equation
2x 2 + 2y2 -+- Z2 -- 8xz - z -t- 8 .= 0

Find dz and d2 z for the values x = 2, y ==- 0, z.:.:= I. 1956. Find dz and d2 z, if In z=x+U+z-1. What are the first- and second-order derivatives of the funct ion z? 1937. Let the function z be defined by the equation x 2+ y2 -1- Z2 = cp (ax + by +- cz), where (p is an arbitrary differentiable function and a, b, care constants. Show that
(cy- bz) ax

az + (az-cx) dY az == bx-ay.

1958. Show that the function z defined by the equation

F(x-az, y-bz)=O,

where F is an arbitrary differentiable function of t\VO argU111entS t satisfies the equation

==0. 1960. Show that the function z defined by the equation y = xcp (z) + 'P (z) satisfies the equation
JX2 ay
a2z(aZ)I_2~az

1959. F ( z'

X z y)

az ay= 1. az az Show that x-ax+Yay=z.


a ax + b
2 2

az

axayaxay

a z +a Z(aZ)I_ O
ay2 ax
-

1961. The functions y and z of the independent variable x are defined by a system of equations Xl +yl-zl =~O, Xl + 2y l + 3z1 =4. . dy dz d 2y d2z f Find ax, dx' dxl' dx l or x = 1, y = 0, Z = 1.

Sec. 10]

Change of Variables

211

1962. The functions y and z of the independent variable x are defined by the following system of equations:
xyz = a, x
2 2

+ y + z == b.

Find dy, dz, d y, d z. 1963. The functions u and v of the independent variables x and y are defined implicitly by the system of equations
u=x+y,

uv=y.

Calculate

au

iJu a2u a2a a2 u iJu au 02U a2 u a2u ~'aY'~2'~~'~2'~'~'a~2'axaY'~2

for x==O, y= 1. 1964. The functions u and v of the independent variables x and yare defined implicitly by the system of equations
u-f-v=x,
Find du, dv, d2 u, d 2 v. 1965. The functions u and

u-yv=O.

tJ of the variables x and yare defined implicitly by the system of equations

x = rp (a, v), y =- 'P (u, v).


V

au au au av

rln

a-' x -a Y ' a-' x a Y

1966. a) Find
b) Find

:~

and

~,if X=llCOSV,

y=usinv,
Z=llV.

z=cv.

x and y defined by the systenl of equations


x = , cos cp, y = , sin cp.

c) Find dz, if x=eu+t', y=eu-t', z=uv. 1967. z = F (r, q where rand q> are functions of the variables

~z and adz, if x=u+v, y=u-v, vX y

az d az Find ax an ay
1968. Regarding z as a function of x and y, find

~ and ~, if

x=acoscpcos,!" y=b sin cp cos, z=csin'1'.


Sec. 10. Change of Variables
When changing variables in differential expressions, the derivatives in them should be expressed in terms of other derivatives by the rules of differentiation of a composite function.

212

Functions of Several Variables

[Ch.6

1. Change of variables in expressions containing ordinary derivatives. Example 1. Transform the equation 2 2 d y dy a2 _ x -d 2+2xd-+2Y-O x x x

putting Solu tion. Express the derivatives of y with respect to x in tenTIS of the denvatives of y with respect to t. We have
dy dy

x=+.

dy = dt =

dx

dx dt

en

-72

= -t 2 dy

dt '

d y dt 2

=!!.(t!J!.) dx ,dx

d(dY) en dxdX =
dt

2 2y _ (2t dy + t 2 d 2 ) (_t 2 ) = 2t a dy + (-I d y dt dt dt dt 2

Substituting the expressions of the derivatives iust found into the given eiua tion and replacing x by we get
2y

+.
2

Y Y 1 t s (2 d 1 ( _ tf) 12 dt+ t d dt 2 ) + 2 "T .. d dt ) +a 2ft ~y~ 0

or

Example 2. Transfonn the equation


d y (d Y )' dy x dx 2 + .dx =0,

ax

taking y for the argument and x for the function. Solution. Express the derivatives of Y with respect to x in terrns of the derivatives of x with respect to yl
dy I {lX= dx ;

dy
dy d ( 2 dx = dx
2

1)

d ( = dy :;

1) dy

d 2x dy 2

dx =

- (:;

r:;1
1
dy

d 2x dy 2

= - (:;

Substituting these expressions of the derivatives into the given equation, we will have

::~] r -(dX)3 +(dX)3- ~=O, L


1
dy dy

Sec. 10]

Change of Variables

213

or, fina 11 y,
X

d:!x d y 2 - 1 + dy

(dX)2 == o.

Example 3. Transfonll the equation


dy x+- y

dx= x-y'

by passing to the polar coorciinates


X==

rcos q>,

y== r sin cpo

(1)

Solution. Considering r as a function of ep, frorn formula (1) we have


dx == cos <p dr - r sin ep dep, dy == sin ep dr -t- r cos (V d(j),

\vhence

ax

dy

. dr . sin (P - -I r cos ep sin <p dr r cos ep dq> d(p = cos q> dr - r sin cp d(p = dr ._-.cos q> dq> - r sin cp

Putting into the ~iven equatIon the expressIOns lor x, y, and ~. we will have
. q> iiq> d, + r cos q> sIn

cos <p or, after


51 nlpliHca Hons,

dr

dcp -

. r sin <p

r cos q> ~- r sin q> r COS cp - r SI n q> ,

dr dq> =-: r.

2 Change of variables in expressions containing partial derivatives. Exaillple 4. Take the equation of oscillations of a string
(a i= 0)

~=x-t-at.

and change it to the new independent variables a and

~,

where a=x-at,

Solution. Let us express the partial derivatives of II with respect to x and t in tenns of the partial derivatives of Ll with respect to a and ~. Applying the fonnulas for differentiating a conlpos1te function

au all aa
\Vl~

at = act at + a~

all

a~

au

at'

ax = oa ax + a~ ax'

au aa all

a~

get

214

Functions of Several Variables

lCh.6

a2 u aaa~ =0.
Example 5. Transfonn the equation x 2 aaZ
X

for the new independent variables, and w=l._J.. for the ne\v x Z x funetton. Gz.In t eftllS 0 f th e 1 denva . tIves az So Iution. Lc t us express tl Ie par t la ax an d ay
fI

== ~_-.!.-

Z + y2 aaY = Z2, takin~ II = X,

t'

==

partial deri,vatives

~~

and

:~.

To do this, differentiate the given relation-

ships between the old and new variables:

dII = d x,
On the other hand,

_ dy _ ~ _ dz . dv = dx , dw x 2 y~ x 2 Z2

ow Ow dw=-du+-dv.

Therefore,
iJw
or

au

av

-du+-dv=--au av x2 Zl

ow

dx

dz

Whence
dz=z

2( 1 dw I i)W) z aw ---- dx+- -dy


2

Xl

and, consequently,

au

av

y2

au

cJz 1 ow 1 OW) ax = z Xi - au - Xi iJv


I (

Sec. 10]

Change of Variables

21a

and

oz Zl ow ay= y2 OV
X2Z2

Substituting these expressions into the given equation, we get

(.!_ ow _.! Ow) all x ou


Xl
2

+Z2

au

oW =

Zl

or

aw=o
OU

1969. Transform the equation


x2 d y 2
dx
2

+ 2x ~ -f- y = 0 dx '
2

putti ng x === et
1970. Transform the equation

(l-x )d>..2 - - x dx -=0 '

d2 y

dy

putti ng x == cos t.

x
Fl~

69

1971

Transform the following equations, taking y as the are

gUlllent:
d y -t- 2y a) dx 2
2

(dY)1 =0
dx

2y _ 3 (d b) '!Jt dy 3 2

dx dx

)! = O. dx
y-X, y

'

1972. The tangent of the angle J! formed by the tangent line MT and the radius vector OM of the point of tangency (Fig. 69) is expressed as follows:

tan

~==-- 1 lL y'

+x

216

Funct ions of Several Variables

[Ch.6

Transform this expression by passing to polar coordinates: x=rcoscp, y=rsincp. 1973. Express, in the polar coordi nates x = r cos cp, y = r si n cp, the formula of the curvature of the curve

K=

[1

+ (y')2]S/2

y"

J974. Transform the following equation to new independent variables u and v: az az y--x-=O ax ay ,

if u=x, v=x!+y~. 1975. Transform the following equation to new independent variables u and v: az az x-+y--z=O ax ay ,
Or t

y tl=x, v=xo 1976. Transform the Laplace equation

a u 02 a ox -to ay =
2
2 2

to the polar coordinates


x= r

cos cp,

y = r sin cp.

1977. Transform the equat ion 2 2 az 2 02Z X dx z - Y a y 2 =0, putting u~xy and v=~. y 1978. Transform the equat ion
Y ax -Xay = (y-x) z,

az

az

by introduci ng new independent variables


u =xz + y2,
V

=~ +J.. x y

and the new function w=lnz-(x+y)o 1979. Transform the equat ion

az
2

OX 2 -

2ox ay

az
2

+ iJy

az
2

= 0,

taking u = x + y, v = JL for the new independent variables and z x w= -x for the new function.

Sec. 11]

The Tangent Plane and the Normal to a Surface

217

1980. Transform the equation


02 Z

ax 2

+ 2 ax 02 ay

Z _L

oy 2

02Z

0
,

putting u=x+U, v=x-y, W=xy-z, where w=w(u, v).


Sec. 11. The Tangent Plane and the Normal to a Surface
1n. The equations of a tangent plane and a normal for the case of exp 1fcIt representation of a surface. The tangent plane to a surface at a point 1\1 (point of tangency) is a plane in wh ich lie all the tangents at the point M to various curves drawn on the surface through this point. The normal to the surface is the perpendicular to the tangent plane at the point of tangency If the equation of a surface, in a rectangular coordinate system, IS gi\ten in explicit form, Z== f (x, y), where' (x, y) is a differentiable function, then th e eq ua t ion of the ta ngent pIa ne a t the poi nt M (x o, Yo, 2 0) 0 f the surf ace is
Z2 0 =-=-

f.~ (x o , Yo) (~X - xo) +,;/ (x o, Yo) (Y - Yo).

(1)

Here, 1 0 == f (x o, Yo) and X, Y, Z are the current coordinates of the point of the tangent plane. The equa bons of the nOrInal are of the fOrIn

f.\ (xu, Yo)


x 2" 2

X-x o

== - , - - - =

Y -Yo

'11 (xo, Yo)

Z-zo --I'
-

(2)

where 4Y, Y, Z are the current coordinates of the point of the nornlal. Example t. Write the equations of the tangent plane and the nortnal to the sur face
Z=

Y2 at t he po i 11 t ,\1 (2, - 1, I).

Solution. Let us lind the partial derivatives of the gIven function and their values at the point M

oz

ax=x,
-=-'>'/ cy -~ ,

OZ) At --2 , ( ax

dz

i!!) (ay

AI

=2

Whence, applying fornlulas (I) and (2), we will have z-1=2(x-2)+2(Y--1-I) or 2x-j-2y-z-I=O which is the equation of the tangent plane and x 2 2=
= y ~ 1= z

II, which is the equation of the normal.

2. Equations of the tangent plane and the normal for the case of implicit representation of a surface. When the equation of a surface is represented iIn pI icit Iy, F (x, y, z)=o.

and F (x o, Yo, zo) =0, the corresponding equations will have the fOrIn

F~(xo' Yo' zo) (X-xo)+F;(x o, Yo,

20 )

(Y-Yo)+F~ (X o, Yo' zo) (Z-lo)=O (3)

218

Functions

of Several Variables

[Ch.6

which is the equation of the tangent plane, and X-xo V-Yo Z-zo F~ (xo, Yo, zo) F ~ (x o, Yo' 2 0) F ~ (x o, Yo'

(4)
20)

which are the equations of the normal. Example 2. Write the equations of the tangent plane and the normal to the surface 3xyz-z'=al at a point for which x=O, y=a. Solution. Find the z-coordinate of the point of tangency, putting x=O, y=a into the equation of the surface: -z'=al , whence z= -a. Thus, the point of tangency is M (0, a, -a). Denoting by F lX, y, z) the Jeft-hand side of the equation, we find the partial derivatives and their values at the point M:
F~=3yz,
F;=3xz,

(F~)M= -3al ,

F~=3xy-3zl,

(F~)M=O, (F~)M= -3aI

Applying formulas (3) and (4), we get -3a1 (x-O) +0 (y-a)-3a 2 (z +a) =0 or x 1-z+a=0, which is the equation of thp tangent plane, x-O y-a z+a -3a 2 =-O-= -3a 2 or

,which are the equations of the normal. y=y 0 a=zt a

1981. Write the equation of the tangent plane and the equations of the normal to the following surfaces at the indicated points: a) to the paraboloid of revolution Z=X 2+y2 at the point
(1. -2,5);
c)

to the sphere X 2+y2+ Z 2=2Rz at the point (Rcosu R sin a, R). 1982. At what point of the ellipsoid

b) to the cone 16+9-8=0 at the point (4,3,4);

x2

y2

Z2

QI+/jZ+Ci=1
does the normal to it form equal angles with the coordinate axes? 1983. Planes perpendicular to the x- and y-axes are drawn through the point M (3, 4, 12) of the sphere r +!I + Zl = 169. Write the equation of the plane passing through the tan~ents to the obtained sections at their common point M. 1984. Show that the equation of the tangent plane to the central surface (of order two)

x2

u2

Z2

ax" + by" +czl=k

Sec. 11]

The Tangent Plane and the Normal to a Surface

219

at the point M (x o' Yo, zo) has the form axox + byoY -~- cZoz = k.
1985. Draw to the surface x2+2!1-~-3z2=21 tangent planes parallel to the plane x -t- 4y -to 6z = o. x2 y2 Z2 1986. Dra\v to the ell ipsoid a2 -~ lJ2 + CZ = 1 a tangent plane \vhich cuts off equal segments on the coordinate axes. 1987. On the surface X 2+y2'_ Z2 -2x=-=0 find points at \vhich the tangent planes are parallel to the coordinate planes. 1988. Prove that the tangent planes to the surface xyz = m3 form a tetrahedron of constant volume \vith the planes of the coord inates. 1989. Sho\v that the tangent planes to the surface Vx + ~/y + -1 V Z-== Va cut off, on the coordinate axes, segJnents \vhose Slun is constant. x2 y2 Z2 1990. Show that the cone 2-1--b 2 =-"i and the sphere a c
Xl -~ yl

-t- ( z -

- c-

b2

+ C2)2 = C2 b'l. (b I -1- c)

are tangent at the points (0, + b, c). 1991. The angle bet\veen the tangent planes dra\vn to given surfaces at a point under consideration is called the angle bet'lt'een 1u,10 surfaces at the point of their intersection. 1\t \vhat angle does the cyl inder Xl -t- y2 = R2 and the sphere
(X-R)2

Hl + z ~ R 2 intersect at the point M (~,

r:r, o)?

1992. Surfaces are called orthogonal if they intersect at right

angles at each point of the I ine of their intersection. Sho\v that the surfaces x 2 + !l 2 + Zl = rl (sphere), y = x tan (IJ (plane), and Z2 == (Xl +!I) tan 2 '1' (cone), \vhich are the coordinate surfaces of the spherical coordinates r, {fl, '1', are I11utually orthogonal. 1993. Show that all the planes tangent to the conical surface z =~ xt ( ~) at the point M (xo' Yo, zo)' where X o 7=- 0, pass through the coordinate origin. 1994*. Find the projections of the ellipsoid

x 2 -1-- y2 -I- zl-xy-l

on the coordinate planes. 1995. Prove that the nornlal at any point of the surface of revolution z = f <V x 2 -l- yl) (/' =F 0) intersect the axis of rotation.

220

F unct ions of Several Var iables

[Ch.6

Sec. 12. Taylor's Formula for a Function of Several Variables


Let a function f (x, y) have continuous partial derivatives of all orders up to the (n + 1) th inclusive in the neighbourhood of a point (a, b). Then Taylor's formula will hold in the neighbourhood under consideration: 1 , ,
j(x, Y)=f(a, b)+Tj [fx(a, b) (x-a)+fy(a, b) (y-b)]-t

1 " 2" " 2 +2! lfxx(a, b) (x-a) +2fx l/(a, b) (x-a) (y-b)+fY/J(a, b) (y-b) ]-t- ...
I + ni [ (x - a)

a+ (y ax

b) iJ y

aJn f (a, b) + R
(a

(x, !/),

(l )

where
R n (x, y)= (n

~ I)!

[(x-a) :x +
Y)+TI
1,

(y- b) ~r+J

+ e (x-a),
(0

b+ 0 (y-b)]
I).

I n other nota tion,


!(x+h, y+k)=f(x,

<

<

[hfx<x, y)+kfl/(x, y)]-t 2! [h fX;l (x, y)-f-

2 "

,. (x, y) + k2tyy " (x, y)] + ... + ti1 t [a 0JIl f (x, Y) -1+ 2hktXY h ax + k ay
or
M(x, y)= :, df(x,

+_1__), [h 1 a.x (n +.

a +k!ln+l f(x-I-ell;
uy

Y-1-0k).

(2)

Y)+~d2f(X, y)+ ...

... + ~! dnf (x, y) + (n ~ 1) I d n Hf (x+ 011; !I -+- Ok)


formula.

(:~)

The particular case of formula (1), when a=b=O, is called Maclaurin's Similar formulas hold for functions of three and a lar~er nUlllber of variables. Example: Find the increment obtained by the function f (x, y) = , \ 3 _ -2y'+3xy when passing from the values x= I, y=2 to the values x1--=-I-l-h,
YJ=2+k.

Solution. The desired increment may be found by applying fOrn1l11a (2). First calculate the successive partial derivattves and their values at the given point (1, 2):

f~ (x, y) = 3x 2 + 3y,

1:(1,2)=3.1+3.2=9,

t:

f:

f~ (x, y)= -6 y 2+3x,


x (x, y) =- 6x,
y (x,

f;/{I,2)=-6.4-!-3.!=-21,

t:.\ (1, 2) = 6 1 == 6,
f:y(l, 2) = 3,

y) c= 3, y) = -12y,

fuy (x

f~y(l, 2)=-12.2=-24,

I;;~ (x, y) = 6,

f:~x( 1, 2) = 6,

f~~y (x, y) == 0,

f~~~ (x, y) = 0, f~~1J (x, y) = - 12,

f~:~{l, 2) = 0, f~;J~( 1, 2) = 0, f~~y(l, 2) = -12.

Sec. 12]

Taylor's Formula tor a Function of Several VarIables

221

All subsequent derivatives are identically zero. Putt}ng these results into formula (2), we obtain: 1 Af(x, Y)=f(l+h, 2+k)-f(1, 2)=TI [h.9+k{-21)]+

+~ W6 + 2hk3 -I- k" (-24)] + ~!

(h"G + 3h"kO+ 3hk"O+ k l (-12)] =

=9h-21k -~ 31z 2

+ 3hk-12k

+h l -'2k l

po\ve rs

1996. Expand f(x-+-h,y+k) in a series of positive integral 0 f han d kif


f (x, y)

= ax2 -1- 2bxy -t- cy2.

the function f(x, y)=-x 2+2xy+3y 2_6x- 2y-4 by Taylor's fornlula in the neighbourhood of the point (--- 2, 1). 1998. Fi nd the increnlent received by the function f (x, y) = =-x 2 y \vhen passing frorTI the values x=l, y=l to
1997. Expand
x I =--~ 1 -t Iz, y 1 =-= 1 -t - k.

t 999. Expand the function f (x, y, z) == Xl --t- yl + z! + 2xy-yz--4x-3y-z-j-4 by Taylor's formula in the neighbourhood of the point (1,1,1). 2000. Expand f(x i h, y~ Il, z-l I) in a series of positive integral po\\'ers of h, k. and I, if
f(X,!I,Z)~--=X2
+

!/2-t-z 2 -2xy-2xz-2yz.

2001. Expand the follo\ving function in a i\\aclaurin's series up to terms of the thIrd order inclusive:

f (x,

y) ~ e"t; sin y.

2002. Expand the follo\\"ing function in a Maclaurin's series up tot e rIllS 0 for de r f0 uri 11 c Iusive :

f (x,

!I)

== cos x cos y.

2063. Expand the follo\ving function in a Taylor's series in the ne i gh bou rhood 0 f the poi nt (1, 1) 11 P toternlS of ordcrtwo i I1C IllSi \'e:

f (x,

y)

= y.'(.

2004. Expand the follo\ving function in a Taylor's series in the neighbourhood of the point (1, -1) up to ternls of order three inclusive:

222

Functions of Several Variables

(Ch. 6

2005. Derive approximate formulas (accurate to second-order terms in a and ~) for the expressions a ) arc t anl_~'
if
1+(1. b) ... /"'(l+a)m+(l+~Y' JI 2 '

lal and

I~I

are small compared with unity.

2006*. Using Taylor's formulas up to second-order terms,

approximate a) vr03; VO.98; b) (0.95)2.01. 2007. Z is an implicit function of x and y defined by the equation z'-2xz+y=O, which takes on the value z= 1 for X= 1 and Y== I. Write several terms of the expansion of the function z in increasing powers of the differences x--l and y-l.
Sec. 13. The Extremum of a Function of Several Variables
to. Deftnition of an extremum of a function. We say that a function has a maximum (minImum) f (a, b) a t the point P (a, b), !f for a 11 points P' (x, y) different from P in a suffici{'ntly sITlall neighbourhood of P the inequality f (a, b) > f (x, y) [or, accordingl y, f (a, b) < f (x, y is fulfilled. The generic term for maximum and minilnum of a function is ext retnu f11. In similar fashion we define the extremum of a function of three or t110re vanables. 2. Necessary conditions for an extremum. The points at which a differentiable function f (x, y) may attain an extremum (so-called stat,onary points) are found by solving the following system of equations:

t (x, y)

t~ (x, y) -==0,

1;/ (x,

y) -- 0

(1)

(necessary cClnditions for an extremutn). Systenl (I) is equivalent to a single equation, df (x, y) =0. In the general case, at the point of the extrel11tll11 P (a, b). the function f (x, y), or df (a, b) = 0, or df (a, b) does not exist. 3. Sufficient conditions for an extremum. Let P (a, b) be a stationary point of the function f (x, y), that is, df (a, b) _-.: o. Then: a) if d 2t (a, b) < 0 for dx 2 + dy 2 > 0, then f (a, b) is the nzaximunl of the function f (x, y); b) if d 2 f (a, b) > 0 for dx 2 + dy 2 > 0, then f (a, b) is the Inintmum of the function t (x, y); c) if d 2f (a, b) changes sign, then t (a, b) IS not an ex tremum of f (\", y). The foregoing conditions are equivalent to the folloWing: let f~ (a, b) :-.: = f~ (a, b) == 0 and A = f: x (a, b), B =--= y (a, b), C ~~ f~y (a, b). We fonn the dtscrimlnallt

t:

Then: 1) if ~ > 0, then the function has an extremunl at the point P (a, b), namely a maximuln, if A < (or C < 0), and a mininlunl, If A > 0 (or C > 0); 2) if Ii < 0, then there is no extremum at P (a, b); 3) if l\ == 0, then the question of an extremum of the function at P (a, b) rema ins open (which is to say, it requires further investigation). 4. The case of a function of many variables. For a function of three or nlore variables, the necessary conditions for the existence of an extrenlurn

Sec. 13]

The Extremum of a Function of Several Variables

223

are similar to conditions (I), while the sufficient conditions are analogous to the conditions a), b), and c) 3.

Example 1. Test the following function for an ex tremum: z=x+ 3xy l-15x-12y.
Solution. find the partial derivatives and fonn a system of equations (1):

oz ==3x + 3y2-15== 0; ax
2

oz =6xy-12 =0 dY

or

x!+y2-5=O, { xy-2=O.
Sol ving the systenl we get four sta tionary points: P J (I,2); P2 (2, I); P.(-I,-2); Pt(-2,-I). Let us find
t~le

second derivatives
2

az az 02 Z =6x -2=6x, a-a x x y =6y, ay a


2
2

and fortn the discrirninant t:. = AC - 8 2 for each stationary point.


1) For the pOInt PI:

A=(Oa2~)
\ X

=6, L\.=AC-B 2 =36-144 < O. Thus, there is no extremUI11 at the pOint PJ. 2) For the point P 2 : A -= 12, B ==6, C= 12; ~ -= 144-36> 0, A > O. At PI thp function has a mininlUnl. ThIs tniniJTIUnl IS equal to the valu' of the fune t ion for .\ - - 2, y =-= l' zmin -~8+G-30-12=-28.
3) For the point P,: A=--=-6, 8==-12, C=-::-6; A=36-144 <0. There extrelllUl1l. 4) For the point P4: A--=- -12, B = -6, C = -12; A = 144-36 > 0, A < n. At the point P t the function has a nlaxhnuln equal to zmdx = -8-6 -t- 30-t -t- 12 - ~ 28 5<t. Conditional extrenlum. In the shnplest case, the conditional extrenllJ/ll of a function f (.\, y) IS 8 111axinlUl11 or nlininlunl of this function which IS attained on the condition that its 3r~unlents are related by the equatioll (P (x, =() (coupling equation). To find the conditlonaJ extrE:'murn of a function (x, y), given the relationship cr (x, y) = 0 \\'C fonn the so-called Lagrarlge
1"6
110

p.

=6,

Z B=(oO!a =12, C=(002~) = ) x Y PI Y PI

y)

fUllction

F (x,

y)

-= f (x, y) -t- A.<p (x, y),

where A is an undeternllned t1lllltipticr, and we seek the ordinary extremUl11 of this auxiliary function. The neces~ary conditions for the extrel11UI11 red uce to a systelll of tluC'e equations:

aF
aF

ax -- ax
of

_~ of

+ iv ocp _0 ox - ,
otp
(2)

iv -a=O' d y ===a-+ y y
q> (x, y) -:: 0

with th ree unknowns x, y, A, fronl which it is, generall y speaking, possible to dete nnine these unknowns.

224

Functions of Several Variables

[Ch.6

The question of the existence and character of a conditional extremUln is solved on the basis of a study of the sign of the second differential of the Lagrange function: o2F a2 F olF 2 d F(x, y)= ox! dx2+2axoydxdY+oy2 d y 2 for the given systeln of values of x, y, A obtained from (2) or the condition that dx and dy are related by the equation

~ dx+ :: dy=O

(dx 2 +dy 2 =F 0).

Namely, the funchon f (x, y) has a conditional maxitnum, if d' F < 0, and a conditional minimum, if d!P > o. As a particular case, if the discriminant A of the function F (x, y) at a stationary point is positive, then at this point there is a conditional maximurn of the function f (x, y), if A < 0 (or C < 0), and a conditiona I minimunl, if A > 0 (or C > 0) In similar fashion we find the conditional extrelnum of a function of three or Inore variables provided there is one or several coupling equations (the number of which, however, nlust be less than the number of the variables) Here, we have to introduce into the Lagrange function as many undetenll1ned multipliers factors as there are coupling equations. Example 2. Find the extrelnum of the function
z==6-4x-3y

provided the variables x and y satisfy the equation


x2 -1- y2::= 1

Solution. Geolnetrically, the problem reduces to finding the greatest and least values of the z-coordinate of the plane z=6-4x-3y for points of its intersection with the cylinder ~2+y2= 1 We fonn the Lagrange function F (x, y) ~ 6 - 4x - 3y -1- A (x 2+ y! -1).

We have :: =-4+2J.x. C::;=-3+2'J..Y. The necessary conditions yield the following systenl of equations: -4+2AX==0, 3+2Ay-=O, { x 2 + y2= 1. Solving this system we find 5 1v 1 =2 ' and 5 'A - - 2' 2Since

It follows

tha~

Sec. 13]
If A-==2'
5

The Extremum of a Function of Ser.'eral Variables

225

x"=5

and Y=r;' then d2 F > 0, and, consequenHy, the function

has a conditional minimum at this point. If A=- {. x=- : and y=- ~ then d2 F < 0, and, consequently, the function at this point has a conditional Inaxilnunl. Thus 1

zmax=6+ 5
Zmin =6-

16

9 +5 =11,
9
= 1.

5- 5

16

6". Greatest and smallest values of a function. A function thaf is differentiablc in a limited closed region attains its greatest (smallest) value either at a stationary point or at a point of the boundary of the region. Example 3. Determ inc the grea test and sll1allest values of the function z-=x 2+ y2_ xy +x+ y in the region
x~o, y~O, x-t-y~-3

Solution. Thc indicated re~ion is a triangle (Fig. 70). 1) Let us find the stationary pOints:

I
'\ \

z't == 2x-y
l' y

-to 1 =0,

~ 2lJ-x+ 1.=.0; .

Fig. 70

\\'hence x-=-l, 11--==-1; and \\'e get thc point M (-I, -1) 1\t "1 the valuc of the function zl'1=-1 It is not absolutely necessary to test for an extrenlunl 2) Let us investt~ate the function on the boundaries of the region. When x-o \ve have Z=y2+ y , and the problenl reduces to seeking the greatest and sll1allec;t valucs of thiS function of one argulIlent on thE' interval -3 ~ y ~ O. Investigatin~, \\'e find that (Zgr)x=o= 6 at the point (0, -3);
(Zsm)",=o=--{- at the POl\1t (0.

_1/ 2 )
Shnilarly, we find that (Zgr)v=o=6 at the will have z=3x 2 +9x+6. Similarly

When y=:...Q \ve get

Z==X2~-X.

point (-3.0); (Zsm)y=o=-

at the point (_1/ 2, 0)


\VC

Whl'1l x-ry=-3 or y-==-3-x


\VC

find that (zsm)x+v=-a=- -:[ at the pOint \ -

. (3 :~ ) 2" - 2 ; (z~r)x+v=-a=6

nlctres coincides with (Zgr)x=o and (lgr)..v.=oo On the straight line x+ y=- 3 we could test the function for a conait ional extremUlll without reducing to a function of one argunlcnt. 3) Correlating all the values obtained of the function z, we conclude that zJlr= 6 at the points (0, -3) and (-3, 0); z5m = -1 at the stationary point M.
8-1900

226

Functions of Several Variables

[Ch. 6

Test for maximum and minimum the follo\\ring functions of two variables: 2008. z = (x-I)2 + 2y2. 2009. Z = (X-I)2 -2y'. 2010. 2 =x2 +xy+ y2 -2x-y. 2011. 2=X 8 y 2(6-x-y)(x>O, y>O). 2012. z = x4+ y4-2x + 4xy-2y .
1 -7-1)2 2 2014. Z= 1_(X +y2)2/ a 2 2015. z=(x 2+y2)e-ex +y2 ).
2013. z=xy
2016. Z =
.. /

x2

y2

y 1 +X 2+y2
y2
02 2

I +x-y

Find the extrema of the following functions of three variables: 2017. U=X 2+y2 +z2- xy + x -2z.
2018.

u=x+y>O, z>O). y z 4x +-+-(x>O,

Find the extrema of the following implicitly represented functions: 2019*. x 2+y2+ zl-2x+4y-6z-ll =0. 2020. x'-y2--3x+ 4y + Z2 + z-8=0.
2021. z=xy 2022. z.=x+2y
2023. Z = x
2

Determine the conditional extrema of the following functions: for x+y= 1. for X 2 +yl=5.

+y

f or "2 x l/ -I- "3 = I.


for

2024. z = cos l x+ cos l y


2025. u = x- 2y 2026.
U=

+ 22

y-x = ~ . for Xl + y2 + Z2 = 9.
x2
y2

x -t- y2 -J- Z2 for 2 +-b 2 + -c2 = I (a >b > c > 0). a 2027. U= xy 2 z 3 for x+y+z=12(x>O,y>0,z>O). 2028. u = xyz provided x -f-y+z=5, xy+yz+zx=8. 2029. Prove the inequality
x+y+z
3
~

Z2

V-xyz,

if

x~o, y~O, z~O.


Hint: Seek the maximum of the function u=xyz provided x+y+z=S.

Sec. 141

Finding the Greatest and Smallest Values of Functions

227

2030. Determine the greatest value of the function z = 1 +x+ 2y in the regions: a) x~O, y~O, x+y~ 1; b) X~O, y<:O,
x-y~ 1.

2031. Determine the greatest and smallest values of the functions a) z=x1y and b) Z=X I _ y 2 in the region x'+y2~1. 2032. Determine the greatest and smallest values of the function z=sinx-l-siny-t-sin(x+y) in the region O~x~ ~.
O~Y~2
31

2033. Determine the greatest and smallest values of the function z=x'+y'-3xy in the region O~x~2, -1~y~2.
Sec. 14. Finding the Greatest and Smallest Values of Functions
Example 1. It is required to break up a positive nunlber a into three nonnegative nUIIlbers so that their product should be the greatest possible. Solution. Let the desired ntllnbers be x, y, a-x-y. We seek the maximunl of the function f (x, y) ==-:xy (a-x-y). According to the problell1, the function f (x, y) is considered inside a closed triangle x~o, y~O, x+y~a (Fig. 71).

x
Fig. 71 Solving the systenl of equations
f~(x. y)=ay(a-2x-y)=O,
{

fy

(x, y)==x(a-x-2y)=O,

we will have the unique stationary point (; for the tntel'lor af the triangle. Let us test the sufficiency conditions. We have

,i)

8*

228 Consequent\ y,

Functions of Several Variables

[Ch. 6

2 A= 'xx 3' "'3 =-3" a,

"(a a)

B=f xy
C=fuu

" (a a) = - 3I a, 3' 3"

a ) =-3 2 a and 3 L\=AC-B 2 > 0, A < o.

" (a 3'

And so at

the function reaches a maximum. Since f (or. y) = 0 on the contour of the triangle, this maximunl will be the greatest vdlue, which

(~. ~)

is to say that the product will be greatest, if X= Y =a-x-y = a ,and the 3 aB greatest value is equal to 2f' Note The ploblenl ran also be solved by the l11ethods of a conditional extremum, by seeking the maximum of the function u ==xyz on the condition that x+y+z==a.

2034. From among all rectangular parallelepipeds with a given volume V, find the one whose total surface is the least. 2035. For what dimensions does an open rectangular bathtub of a given capacity V have the smallest surface? 2036. Of all triangles of a given perimeter 2p, find the one that has the greatest area. 2037. Find a rectangular parallelepiped of a given surface S with greatest volume. 2038. Represent a positive number a in the form of a product of four positive factors which have the least possible stun. 2039. Fil1d a point M (x, y), all an xy-plane, the sunl of the squares of the distances of which from three straight lines (x=O, y==O, x-y+ 1 =0) is the least possible. 2040. Find a triangle of a given perimeter 2p, \vhich, lipan being revolved about one of its sides, generates a solid of greatest volunle. 2041. Given in a plane are three nlaterial points PI (xl' !Jt)' PI (XI' Y2)' P a (x a, Ya) with masses m m 2 , rna For \vhat position " of the point P (x, y) will the quadratic moment (the moment ()f inertia) of the given system of points relative to the point P (Le., the SUITI m l P t P2 i-m2P2P2 +m aP ap2 ) be the least? 2042. Draw a plane through the point M (a, b, c) to [orIn a tetrahedron of least volume with the planes of the coordinates. 2043. Inscribe in an ellipsoid a rectangular parallelepiped of greatest volume. 2044. Determine the outer dimensions of an open box \vith a given wall thickness ~ and capacity (internal) V so that the smallest quantity of material is used to make it.

Sec. 141

Finding the Greatest and Smallest Values of Functions

229

2045. At what point of the elli pse x2 y2

i2+V=l
does the tangent line to it form with the coordinate axes a triangle of sIl1allest area? 2046*. Find the axes of the ellipse 5xl + 8xy + 5y 2 = 9.
2047. Inscribe in a given sphere a cylinder having the greatest total surface. 2048. The beds of two rivers (in a certain region) approxinlately represent a parabola y=x 2 and a straight line x-y-2=O. I t is required to connect these ri vers by a straight canal of least length. Through \vhat points will it pass? 2049. Find the shortest distance fronl the point 1\;1 (1, 2, 3) to the straight line

T= -3=2
2050*. The poi nts A and B are situated in different optical 111e(lla separated by a straight line (Fig. Z2). The Yclocity of

13

I
Fig. i3

light ill the first 111CdiuI11 is vI' in the second, v 2 Applying the Ferlllat principle, according to which a light ray is propagated along a line A,\1B \vhich requires the least tillle to cover, derive the la\v of refraction of light rays. 2051. Using the Ferlnat principle, derive the la\v of reflection of a light ray frolll a plane in a homogeneous medium (Fig. 73). 2052*. If a current J Hows in an electric circuit containing a resistance R, then the quantity of heat released in unit tinle is proportional to J2 R. Deterilline how to divide the current I into

236

Functions of Several Variables

[Chi 6

currents I l' 12 , I a by means of three wires, whose resistances are R1 , Ra, R a , so that the generation of heat would be the least possible?
Sec. 15. Singular Points of Plane Curves

f (x,

to. Definition of a singular point. A point M (x(\' Yo) of a plane curve


y)=O is called a singular point if its coordinates satisfy three equations

at once:

f (xo,

Yo) = 0,

f~ (x o, Yo) =

o.

f~ (xo' Yo) = O.

2. Basic types of singular points. A t a singular point M (xo, Yo), let the second derivatives

A ==
B=

f:

':x
y

(xo' Yo)'
(xo' Yo)'

C = f~y (xo, Yo)

be nof all equal to zero and


then:
a) if Ii> O. then M is an isolated point (Fig. 74); b) if Ii < 0, then M is a node (double point) (FIg. 75); c) if Ii = 0, then M is either a cusp of the first kind (Fig. 76) or of the second kind (Fig. 77), or an isolated point, or a tacnode (Fig. 78).

Fig. 74

Fig. 75

When solving the problems of this section it is always necessary to draw the curves. Example 1. Show tha t the curve y2 = ax! + x has a node if a > 0; a 11 isolated point if a < 0; a cusp of the first kind if a = O. Solution. Here, f (x, y) ax 2+x-y2. Let us find the partial derivatives and equate them to zero:

==

f~(x, y)=2ax+3x2 =0,

f~ (x, y)=- 2y=O.

Singular Points of Plane Curves

231

This system has two solutions: 0 (0, 0) and N ( - : a, but the coordinates of the point N do not satisfy th~ equation of the given curve. Hence, there is a unique singular point 0 (0, 0).

0):

x
Fig. 76 Fig. 77 Fig. 78

Let us find the second deriva tives and their val ues at the pOint 0:

,:% (x, ':y(X,


f;,/(X,

y)

= 2a + 6x,

= 2a,

y)=O, y)=-2,

8=0, C=-2,

~=AC-B2=-4a.

y
v

o
Fig. 79

Fig. 80

Fig. 81

Hence, if a > 0, then A < 0 and the point 0 is a node (Fig. 79); if a < 0, then 1\ > 0 and 0 is an isolated point (Fig. 80); if a::--::O, then ~-=O. The equation of the curve in this cas~ will be y2=X' or Y= YX'; y=exists only when x~O; the curve is symmetric about the x-ax is, which is a tangent. Hence, the point M is a cusp of the first kind (Fig. 81).

232

Functions of Several Variables

[Ch.6

Determine the character of the singular points of the following curves: 2053. y" = - x" +x".
2054. (Y_X I)2 = Xl.

2055. a 4 y!
2056. X
2

y 2 _x 2 _ y" = o. 3 2057. x +yl-3axy=O (folium of Descartes).

= a 2 x' -

Xl.

2058. y2 (a-x) = Xl (cissoid). 2059. (x 2+y!)I=a 2 (x"_y2) (lemniscate). 2060. (a+x)yl=(a-x)x! (strophoid). 2061. (Xl y2) (x-a)1 = b2x 2 (a> 0, b > 0) (conchoid).

Consider three cases:


1) a>b, 2) a=b, 3) a<b.

2062. Determine the change in character of the singular point of the curve y2=(x-a)(x-b) (x-c) depending on the values of a, b, c(a~b~c are real).
Sec. 16. Envelope
10. Definition of an envelope. The envelope of a family of plane Cllrvefi is a curve (or a set of several curves) which is tangent to all tinps of the given family, and at each point is tangent to sonle line of the given faillily. 2. Equations of an envelope. If a family of curves

f (x,

y, CL):::::O

dependent on a single variable parameter a has an envelope, then till' Ilarametric equations of the latter are found fronl the system of equations
{ ',(X. y. a) =~' fa (x, y, a)-D.
(1 )

Eliminating the parameter a fronl the system (1), we get an equation of the f onn D (x, y) =0. (2)
It should be pointed out that the formally obtained curve (2) (the ~o called "discriminant curve") may contain, in addition to an envelope (if there is one), a locus of singular points of the given farnily, which locus IS not part of the envelope of this f am j} y. Whfn solving the problems of this section it is advisable to Illake

drawings.

Exanlple. Find the envelope of the family of curves

x cos a+ y sin a-p =0 (p = const. p > 0).

Sec. 16]

Envelope
Q.

233
Form

Solution. Th~ given family of curves depends on the parameter the system of equations (1): xcosa+ysina-p=O, { - x sin a 1- Y cos a -= O.

Solving the system for x and y, we obtain parametric equations of the envelope x=pcosu, y==psina. Squaring both equations and adding, we eliminate the parameter a:
x 2 -1- 1/ 2 == p2

Thus, the envelore of this family of strai~ht lines is a circle of radius p with centre at the origin. This particular family of straight lines is a family of tangent lines to thIs circle (Fig. 82).

2063. Find the envelope of the family of circles


(x-a)t

+ yt =

a; .

2064. Find the envelope of the family of straight lines

-kx+.P.. Y2k (k is a variable parameter). 2065. Find the envelope of a family of circles of the same radius R whose centres lie on the x-axis. 2066. Find a curve which forms an envelope of a section of length 1 when its end-points slide along the coordinate axes. 2067. Find the envelope of a family of straight lines that form with the coordinate axes a triangle of constant area S. 2068. Find the envelope of ellipses of constant area S whose axes of synlnletry coi Heide.

234

Funct ions of Several Varl abies

[Ch. 6

2069. Investigate the character of the "discriminant curves" of families of the following lines (C is a constant parameter): a) cubic parabolas y=(x-C)'; b) semicubical parabolas 11 = (x-C)'; c) Neile parabolas yl = (X-C)I; d) strophoids (a+x) (y_C)2 =r (a-x).

o
Fig. 83

2070. The equation of the trajectory of a shell fired from a point 0 \vith initial velocity V o at an angle a to the horizon (air resistance disregarded) is

Taking the angle a as the parameter, find the envelope of all trajectories of the shell located in one and the sanlC vertical plane ("safety parabola") (Fig. 83).
Sec. 17. Arc Length of a Space Curve
The diOerenttal of an arc of a space curve in rectangular Cartesian coordinates is equal to ds = V dx 2 + d!l2 + dz 2 , \vhere x, y, z are the current coordinates of a roint of the curve.

If

x=x(t),

y=y(t),

z=z(t)

are parametric equations of the space curve, then the arc length of a section of it from t=t l to t=t 2 is
S=

tr, I" ( dx ) J V lIt +


2

df

dy ) 2

+ (dZ) Cit

dt.

t1

S~c.

18]

The Vector Function of a Scalar Argument

235

In ProbletTIs 2071-2076 find the arc length of the curve:

2071. x=t, y=t', Z=2~'

from
3

t=O to t=2.

2072. x=2cost, y=2sint, z=-t 3t


2073. x=etcost) y=etsint,

from t==O to t='Jt. from t=Otoarbitraryt.

z=et

2074. Y=T' z=6 2075. Xl = 3y, 2xy = 9z


a

x2

Xl

from x=O to x=6. from the point 0 (0,0,0) to M (3,3,2).


4

2076. y=aarcsin~, z= a In ~ from the point 0(0,0,0)


a-x

to the point M (x o, Yo, zo). 2077. The position of a point for any time t (t >0) is defined by the equations x=2i, y=lnt, z=t l Find the mean velocity of motion between times t = 1 and t = 10.
Sec. 18. The Vector Function of a Scalar Argument
tUtlct ion a == a (t) may be defined by specifying three scalar functions ax (t), ay (t) and az (t), \\'hich are its projections on the coordinate axes:

10 The derivative of the vector function of a scalar argument. The vector

a=a x (t) I +ay (t)J+a z


~ _ lim a (I -1- Llt) -a (t)
dt - At ~ 0 llt

<t) k.

The derivative of the vector function a::::: a (t) with respect to the scalar argument t is a new vector function defined by the equality
da x (t)

i -1- day

(t)

J + dar (t) k
dt

dt

dt

The modulus of the derivative of the vector function is

. l(dax)2 + (da,v) + I 1= .V
~
dt
2

dt

dt

z)1 da dt

The end-point of the variable of the radius vector r=r(t) describes in space the curve

r=x (t) 1+ y (t) J+ z (t) 11,


which is called the hodograp1J of the vector r.
The derivative :~ is a vector, tangent to the hodograph at the corre sponding point; here,

Itit I=dt,
dr ds

where s is the arc length of the hodograph reckoned from some Initial point.
For example,

I~~ 1= 1.

236

Functions of Several Variables


If the

[Ch.6

parametl~r t

is the time, then

extremity of the vector r, and :;~= ~~ is the acceleration vector of the extremity of the vector r. 2. Basic rules for differentiating the vector function of a scalar argument.
1)

=tv

~ =(/

is the velocity vector of the

dt (a+b-c)=dT+(jf-df ;

da

db

de

2)

~
d d

(ma)=m

~~

, where m is a constant scalar;

3) (if (cpa)=(jf

dq>

da a+ cp dt' db

where cp (i) is a scalar function of t;

4) (j[

(ab)=Fb+adt;
da db

da

5) (if (aXb)=dtXb+aXdt;

6)
7)

F
a

a [ep (i)] =dfP

da

dq>. F'

~~

=0, if lal=const.

Example t. The radius vector of a moving point is at any instant of Hnle defined by the equation

r= i-4t 2j+3t 2k.


Detennine the trajectory of motion, the velocity and accelera tion. Solution. From (1) we have: x== 1, y==-4i 2 , 2=3i 2

(I)

Eliminating the time i, we find that the trajectory or nlotion is a straight line: x-I y z -0-=-4=3"
Frolll equation (1), differentiatin~, we find the velocity

([[=- 8ij+6tk

dr

and the acceleration


d2r di 2 =-Bj+6k.

The nlagnitude of the velocity is

I~~ 1= Y(-

81)2+ (61)2= 10 I t I

We note that the acceleration is constant and l~

I::~I= Y(-8)2+ 6 =10.


2

Sec. 18]

._~---------~----..::::....--------

The Vector Function of a Scalar Argument

237

2078. Show that the vector equation r..:.-r t = (rl-r 1 ) t, where r 1 and r l are radius vectors of two given points, is the equation of a straight line. 2079. Determine which lines are hodographs of the following vector funct ions: a) r=at+c; c) r = a cos t + b sin t; l b) r = at -t- bl; d) r=acosht-t-bsinht,

where a, b, and c are constant vectors; the vectors a and b are perpend icular to each other. 2080. Find the derivative vector-function of the function a (/)::= a (I) a O (I), where a (t) is a scalar function, \vhile aO (t) is a unit vector, for cases \vhen the vector a (t) varies: 1) in length only, 2) in direction only, 3) in length and in direction (general case). Interpret geolnetrically the results obtained. 2081. Using the rules of differentiating a vector funct i::>n with respect to a scalar argument, derive a formula for differentiating a fnixed product of three vector functions a, b, and c. 2082. Find the derivative, with respect to the paranleter t, of the volunlc of a parallelepiped constructed on three vectors: a = i + tj+ t 2 k; b=2ti-j+ t 3 k; c = - tli -f- tSj+ k.
2083. The eq uat ion of nlot ion is r =---= 3i cos t --~- 4j sin

t,

where t is the tinlC. Deterlnine the trajectory of ITIotion, the velocity and the acceleration. Construct the trajectory of nlotion and the vectors of velocity and acceleration for times, t = 0,

'-0

-T

and t = ~

r = 21 cos t

2084. The equation of nlotion is

+ 2j sin t -J- 3kl.

Dctcrll1ine the trajectory of nlotion, the velocity and the acceleration. \Vhat are the nlagnitudes of velocity and acceleration and \vhat directions have they for time t = 0 and t = ~ ? 2085. The equation of nl0tion is r = I cos a cos rot -f- j sin a cos rot + k sin rot, where a and (t) are constants and t is the tilne. Determine the trajectory of motion and the lTIagnitudes and directions of the velocity and the acceleration.

238

2086. The equation of motion of a shell (neglecting air resistance) is

Functions of Several Variables

(Ch.6

where V o {vox, v oy ' v oz } is the initial velocity. Find the velocity and the accelerat ion at any instant of time. 2087. Prove that if a point is in motion along the parabola y =~, z == 0 in such a manner that the projection of velocity a
on the x-axis remains constant (:; = canst). then the acceleration remains constant as well. 2088. A point lying on the thread of a screw being screwed into a beam describes the spiral x=acos6, y=asin6, z=h6,

where 6 is the turning angle of the screw, a is the radius of the screw, and h is the height of rise in a rotation of one radian. Deternline the velocity of the point. 2089. Find the velocity of a point on the circumference of a wheel of radius a rotating with constant angular velocity ro so that its centre moves in a straight line \vith constant velocity v O
Sec. 19. The Natural Trihedron of a Space Curve
At any nonsingular point M (x, y, z) of a space curve r==r(t) it is possible to construct a natural trihedron consisting of three ITIutually perpendicular pI anes (Fig. 84): 1) osculating plane MM t M 2

containin~

the vectors

c:;;

and

~:~;
and

2) normal plane MM:M which is perpendicular to the vector '::;

3) rectifying plane MM l """ which is perpendicular to the first two J1lanes. At the intersection we obtain three straight lines; 1) the tangent MM l ; 2) the principal normal MM z ; 3) the buzofltlul MM" all of which are defined by the appropria te vectors:

1) T=

~~
dr

(the vector of the tangent line);

d 2r . 2) B=dfX dt2 (the vector of the blllormal); 3) N = Bx T (the vector of the principal normal);

The corresponding unit vectors

-r=iTi;

~=iBi;

\'=iNi

Sec. 19]

The Natural rrihedron of a Space Curve


d'f

239

may be computed from the formulas

~=dS ;

dr

v=

I:: I;

tiS

1J=~Xv.

If X, Y, Z are the current coordinates of the point of the tangent, then the equations of the tangent have the form

X-x
TJI:

=r;=r-;-'

Y-y

Z-z

(1)

Rectlfying plane

T=at
Fig.
~4

dr

wh('rc T x -= :; ; T y= :~-. T z= :: ; from the condition of perpendicularity of the hne and the plane \ve get an equation of the normal plane:
Tx(X-X)1-Ty(Y-y)+Tz(Z-z)=O. (2)

If in C'quations (1) and (2), \\re replace T x ' Ty , T z by B.'(, B", Bz and N". Ny, N z , we get the equations of the binornlal and the princij'al normal and, respectively, the osculating plane and the rectifying plane. Example 1. Find the basic unit vectors T, \' and Ii of th(l curve at the point t=l. Write the equations of the tangent, the principal nornlal and the binor n1al at this point. Solution. We have r=tl+tIJ+tl/l and

x=t, U=t 2 , z=t'

:~ =1+2tj+3t 1k.

d1r

dtl == 2J

+6tll.

240

Functions of Several Variables


dr

[Ch.6

Whence, when t = 1, we get

T=d[ =1+2J+3k;
B= d d~
d Xdt~=
2

N=BXT=I~ JO ~1~_221-16j+18k.
123

I'

1 j 2 k 3 =6i-6j+2ki

Conseq uentl y,
l'

1+2j+3k Vl4

p __ 31-3j+k
-V19 ,
v=

-111-81+911 V266

Since for t= 1 we have x= 1, y= 1, z= 1, it follows that x-I y-l z-1 -1- = -2- = -3are the equations of the tangent,
x-I y-l z-1 -3-=-3 = - 1

are the equations of the binornlal and


x-I -11

11-1 2-1 -8 =--9-

are the equations of the principal normal. If a space curve is represented as an intersection of two surfaces F (x, y, z) =0, a (x, y, z) ==0, then In place of the vectors (j[ and dt 2 we can take the vectors dr {dx, dy, dz} and d2r {d 2x, d2y, d2z}; and one of the variables x, y, 2 nlay be considered independent and we can put its second differential equal to zero. Example 2. Write the equation of the osculatin~ plane of the circle X2+y2+ Z2=::6, X+Y+2~-=0 (3) at Its pOInt M (1, 1, -2). Solution. Differentiating the systenl (3) and considering x an independent variable, we will have xdx+ydy+zdz=O, dx+dy+dz=O and dx + dy + Y d 2y + dz 2 + Z d 2z -= 0, d 2 y+d 2 z=O. Putting x= 1, y== I, 2==-2, we get
dy==-dx; . . dr d 2r

dz=O;
d2z =

d y == -

3' dx

"3 dx 2

Sec. 19]

The Natural Trihedron of a Space Curve

241

Hence, the osculating plane is defined by the vectors

{dx. -dx, o}
or

and {D. and

~ dx, idX.}
-1, I}.

{I, -I, O}

{a,

Whence the normal vector of the osculating plane is

B=I~a -{ ~I=-I-j-k -1 I
and, therefore, its equation is
- I (x-l)-{y-I)-(2+2)==O,

that is,

x+y+z=o,
as it should be, since our curve is located in this plane.

2090. Find the basic unit vectors

T,

v,

of the curve

x==:l-cost, y=sint,

z=t

at the point t = ~ . 2091. Find the unit vectors of the tangent and the princi pal

norillal of the conic spiral r = et (I cos t

+j

sin t i- k)

at an arbitrary point. Determine the angles that these lines Illake \vith the z-axis. 2092. Find the basic unit vectors T, v, p of the curve
at the point x

== 2.

y=x 2 ,

z= 2x

2093. For the screw line

x = a cos t,

Y == a sin t,

= bi

write the equations of the straight lines that forln a natural trihedron at an arbitrary point of the line. Deternline the direction cosines of the tangent line and the principal normal. 2094. Write the equations of the planes that form the natural trihedron of the curve x" -1- y2 -t- Z" = 6, x 2- y2 -}- Z2 == 4

at one of its points 1\1 (1, 1, 2).


2095. Fornl the equations ot the tangent line, the normal plane and the osculating plane of the curve x=t, y=t', z=t' at the point M (2, 4, 8).

242

Functions of Several Variables

[Ch.6

2096. Form the equations of the tangent, principal normal, and binormal at an arbitrary point of the curve tC t' t2

X=4' Y=3'

Z=2

Find the points at which the tangent to this curve is parallel to the plane x+3y+ 2z-10=0. 2097. FOrlTI equations of the tangent, the osculating plane, the principal normal and the binormal of the curve

x=t, y=-t, z=2


at the point t = 2. Compute the direction cosines of the binormal at this point. 2098. Write the equations of the tangent and the normal plane to the following curves:
a) x=Rcos1t, y=R sintcost, z=R sint for t= ~ ; b) Z=X 2 +y2, x=y at the point (I, 1,2); c) X2 +y2+ Z2=25, x+z=5 at the point (2, 2V3, 3).
2099 Find the equation of the normal plane to the curve Z=X 2 _ y 2, y=x at the coordinate origin. 2100. Find the equation of the osculating plane to the curve x=et , y=e- f , z=tV2 at the point t=O. 2101. Find the equations of the osculating plane to the curves: a) x2_l-Y~+Z2=9, X2_ y 2=3 at the point (2,1,2); b) x 2 = 4y, x' = 24z at the point (6, 9, 9); c) x 2 + Z2 = a 2 , y2 t Z2 = b 2 at any point of the curve (x o' Yo. zo). 2102. Form the equations of the osculating plane, the principal normal and the binormal to the curve y2=X, x 2 =z at the point (I, I, 1). 2103. Form the equations of the osculating plane, the pal normal and the binormal to the conical scre\v-line x = y=tsint, z=bt at the origin. Find the unit vectors tangent, the principal normal, and the binormal at the

t2

princit cos t, of the origin.

Sec. 20. Curvature and Torsion of a Space Curve


1. Curvature. By the curvature of a curve at a point M we mean the number 1 1 cp- . K=-= Im R ~s-+o bas

Sec. 20)

Curvature and Torsion of a Space Curve

243

where cp is the angle of turn of the tangent line (angle of contingence) on a segment of the curve MN, f!s is the arc length of this segment of the curve. R is called the radius of curvature. If a curve is defined by the equation r=r (s), where s is the arc length, then

~=I~~I
For the case of a general parametric representation of the curve we have
1

Icn

dr

([t2

d2rl
(1)

R=
2. Torsion. By mean the number

I~~r
~s '

torsion (second curvature)

of a curve at a point M we

1 1. 0 T=-= 1m Q
~S-t>O

where 0 is the angle of turn of the binormal (angle of contingence of the second kind) on the segment of the curve AT.V. The quantity Q is called the radIus of torsion or the radius of second cart'at ure. If r= r (5), then

~= I ~1=dS(fS2(fSi 1
Q
ds
2

drd 2rd ar

(ddsr)2'
'Y

where the minus sign is taken when the vectors ~~ and direction, and the pi us sign, when not the same. If r= r (t), \vhere t is an arbitrary parameter, then
dr d 2r dar

have the same

Q=

(dr

di (fl2 lfii
d2
dt X dt2

r)2.
(a

(2)

Example 1. Find the curvature and the torsion or the screw-line

r= I a cos t + j a sin t + k
Solution. We have
dr

bt

>

0).

di = - I a sin t +J a cos t +kb,


d 2r
dt a
Whenc~
2

dt 2 = - I a

cos t - j a sin I,

dar

= - I a sin t - j a cos t.
Jcos t kb \ = I ab sin t -Jab cos t +a2i a -a sin t 0

dt X dt 2 =

dr d r

I -a I sin t
-a cos t

244
and

Functions of Several Variables

[Ch.6

d d 2 dB sin t t b 2 ; dt~ dt~= -a c~s t -a sin t 0 = a b.

I-a
a

a sIn t -

acos I a cos t 0
a
b
2

Hence, on the basis of formulas (1) and (2), we get


1

yaz-tbi
a2b
(a 2

R
and

(a2+b2)3/2 = a 2 +b2

Q=
dT _" ([S-7['

a2

+b = a
2)

-t- b2

Thus, for a screw-line, the curvature and torsion are constants. 30 Frenet formulas: dv _ T+~ ds--7[ Q' dP v ds=-Q"

2104. Prove that if the curvature at all points of a line is zero, then the line is a straight line. 2105. Prove that if the torsion at all points of a curve is zero, then the curve is a plane curve. 2106. Prove that the curve x=1+3t-t-2t 2 , y=2-2t-t-5t 2 , z=l-{1

is a plane curve; find the plane in which it lies. 2107. Conlpute the curvature of the following curves: a) x=cosf, y=sint, 2=cosh t at the point t==O; b) x 2- y2 -t- Z2 == 1, y2 - 2x -~- z = 0 at the poi nt (1, 1, 1).
2108. Compute the curvature and torsion at any point of the
curves:

b) x=a cosh t, y=a sinh

a) x = et cos t, Y = e' sin t,

t.

==

et ;

z=at (hyperbolic screw-line).

2109. Find the radii of curvature and torsion at an arbitrary point (x, y, z) of the curves:
a) x 2

= 2ay,

Xl

b)

X =

3p2y , 2xz = p2.

= 6a

z;

2110. Prove that the tangential and normal components of acceleration ware expressed by the formulas
UJT = dtt',

dv

'W y

= R \',

v2

where v is the velocity, R is the radius of curvature of the trajectory, ~ and v are unit vectors of the tangent and principal normal to the curve.

Sec. 20]

Curvature and Torsion of a Space Curve

245

tion w.

= ia cos t -1- ja sin t + btk with velocity v. Compute its accelera2112. The equation of mot ion is

2111. A point is in uniform motion along a screw-line , =

r= tl-t- t j+ tSk.
2

DetermiRe, at times t~O and t=l: 1) the curvature of thetrajectory and 2) the tangential and normal components of the. acceleration.

Chapter VII

MULTIPLE AND LINE INTEGRALS

Sec. 1. The Double Integral in Rectangular Coordinates


~inuous

1. Direct computation of double integrals. The double integral of a confunction f (x, y) over a bounded closed region S is the limit of the corresponding two-dimensional integral sum

~ f (x, y) dx
(S)

dy=

lim
max dXI ~ max dYl' -?
0

~ ~f (xi. Yk) !J.x,l!Yk.


i
k
0

(I)

where ~xi=xi+.-xi' ~Yk=Yk+l-Yk and the sum is extended over tho~e values of i and k for which the points (xi, Yk) belong to S. 2. Setting up the limits of integration in a double integral. We distinguish two basic types of region of integration.

D
I

A
X,

c
x

x
Fig. 85

o
Fig. 86

1) The region of integration S (Fig. 85) is bounded on the left and right by the straight lines x=x. and x=x (XI> x.), from below and from abovE by the continuous curves Y= (J). (x) (AB) and y = (J)I (x) (CD) [<PI (x) ~ q>1 (x), each of wh ich intersects the vertical x = X (x .-; X < XI) at only one point (seE Fig. 85). In the region S, the variable x varies from Xl to XI' while the va riable y (for X constant) varies from Yl = q>l (x) to Ya = q>1 (x). The integral (1) ma~

Sec. 1]

T he Double Integral in Rectangular Coordinates

247

be computed by reducing to an iterated integral by the formula

H
(5)

X2

f(x, y)dxdy=

~ dx ~
Xl

CPI CPI

(x)

f(x, y)dy,

(X) q>2 (x)

where x is held constant when calculating

~ f (x,
CPI (x)

y) dy.

2) The region of integration S is bounded from below and from above by the straight linE's Y=YI and Y=Y2(Y. > YI)' and fronl the left and the right by the cont inuous curves x = '1'1 (y) (AB) and x = '1'2 (y) (CD) ['1'2 (y) ~"" (y) l. each of which intersects the parallel Y = Y (y, E;;; Y ~ Y2) at only one point (Fig. 86). As before, we have

Hf
(5)

y.,

(x, y) dx dy =
(Y)

~ dy ~
1J'1
(II)

1J'2

(Y)

f (x, y) dx,

'It

y) dx we consider y constant. '1'1 (y) If the region of integration doe~ not belong to any of the above-discussed types, then an attempt is luade to break it up into parts, each of which does belong to one of these two types. Example 1. Evaluate the integral
1 1

her~, in the integral

'1'3

~ f (x,

1=

~ dx ~
o
y

(x+y)dy.

-2

Fig. 87 Solution.

Example 2. Deternlinc the lilnits of integration of the integral

H
(S)

f(x, y)dxdy

'248

Multiple and Line Integrals

[Ch.7

if the region of integration S (Fig. 87) is bounded by the hyperbola y2_ X 2= 1 and by two straight lines X= 2 and x=- 2 (we have in view the region containing the coo rdinate origin). Solution. The region of integration ABeD (Fig. 87) is bounded by the straight lines x-= -2 and x=2 and by two branches of the hyperbola

y=1+x2

and

y-==-Yl+x2
2

that is, it belongs to the first type. We have:

H
(S)
2 I

f (x. y)dxdy=

~
- 2

YI+X 2

dx
_

f(x, y)dy.

YI+X 2

Evaluate the follo\ving iterated integrals:


2113. ~ dy ~ (Xl
o
4,

+ 2y) dx.

2117. ~dY ~ (x-l-2y)dx.


-I y2-4

0 2

2114

55
dx
a
I

(x

+ Y/&
2

dy

In

2118. ~ dcp
o

~ r dr.
a sin q>

1 51 x dy 2115. 5 dx 1 + y2
o
0
!

n
2

a cos q>
0

2119.

~ dcp ~ r l sin l (p dr.


n -!

2116.

55
dx
1 I
X

7.

x2dy

11' I-X:'

2120. ) dx
o

Vi-Xl_yl dy.

Write the equations of curves bounding regions over which the following dduble integrals are extended, and draw these regions:
2

2-y

2X

2121. ~ dy ~ {(x, y) dx.


-0
1

2124. ~ dx ~ {(x, y) dy.


I
1
X

!!~_ I
4

X+8

2122. ~ dx ~ {(x, y)dy.


I 4

2125. ~ dx
0 2

V 2S-X"

~ {(x, y)dy.

x2
IO-Y

2123. ~ dy ~ {(x, y) dx.


o
Y

2126. ~ dx ~ {(x, y) dy.


-I

X+2
X2

Set up the limits of integration in one order and then in the other in the double integral

H{(x, y) dxdy
(S)

for the indicated regions S.

Sec. I]

The Double Integral in Rectangular Coordinates

249'

2127. C(O, 1). 2128. 2129. C(O, 1). 2130. C(2, 7), 2131. o (0, 0),

S is a rectangle with vertices 0 (Ot 0), A .(2,0), B (2, 1),


S is a triangle with vertices 0(0,0), A(l, 0),8(1,1).

S is a trapezoid with vertices 0 (0, 0), A (2, 0), B (1, 1),


S is a parallelogram with vertices A (1, 2), B (2, 4), D(I, 5). S is a circular sector DAB with centre at the point \vhose arc end-points are A (1, 1) and 8 (--1, 1) (Fig. 88).

B(-I,f)

A(~f)

o
Fig 89

21 a2. S is a rIght para boIic segnlent AOB bounded by the parabola BOA and a segment of the straight line BA connecting the poi nts B (--- 1, 2) and A (1, 2) (F~ig. 89). 2133. S is a circular ring bounded by circles \vith radii r == 1 and R == 2 and \\'ith cotTIITlOn centre 0 (0, 0). 2134. S is bounded by the hyperbola y2_x- = 1 and the circle x 2 I 1/ 2 == 9 (the region containing the origin is meant). 2135. Set lip the linlits of integration in the double integral

~ ~ f (x, y) dx dy
(S)

if the region S is defined by the inequalities a) x;~~O; y~O; x-{-y~l; d) y;-..~x; x~-l; b) x2-t-y2~a2; e) y~x~y-+ 2a;
c)

y~l;

x 2 -t- !l 2 ~ x;

0 ~ y ~ a.

Change the order of integration in the follo\ving double integrals:


2136. ~ dx ~
o
" J2,X
I

x
2X

f (x,

y) dy.

2137. ~ dx ~ f(x. y)dy.


Q

3x2

250
a

Multiple and Line Integrals

[Ch. '1
l-y

2138. ~
o

Y a2 -x2 dx S f(x, y)dy.


a -x
2 2

2141.

~ dy
1

~
-Yl-y2

f (x, y) dx.

2a

Y 2ax-x2

Y a _y2

2139. ~ dx
a
2

f (x, y) dy.

2142. ~dy
o

~ f(x, y)dx.
y2
:&

2a

2140.

~dx
!

~ f(x, y)dy. Y 2ax-X 4


X

Ym

YR2_X 2

2143.
n

~ d.x Sf
o
0

(x,

y) dy+

~
R V2
2

dx

~
0

f (x, y) dy.

sin x

2144. ~ dx ~ f (x, y) dy.


o
0

Evaluate the following double integrals: 2145. xdx dy, where S is a triangle with vertices 0 (0, 0),

H
(5)

A(I, 1), and 8(0. I).


y

y
(0,2)

B(O'1)a...............~A(f,l)

e(G, 1)

A(2,O)X
Fig. 90

o
Fig. 91

2146.

Hxdx dy, where the region of integration S is bounded


(5)

by the straight line passing through the points A (2, 0), B (0, 2) and by the arc of a circle with centre at the point C (0, 1), and

radius 1 (Fig. 90).

Sec. 1]

The Double Integral in Rectangular Coordinates

251

2147.

SSV :xd~
(5)

a -x-y

I'

where S is a part of a circle of radius

a with centre at 0 (0, 0) lying in the first quadrant.

2148.

HV
(5)

Xl -

yl

dx dy, where S is a triangle with vertices

0(0,0), A(I, -1), and 8(1, 1). 2149. HVxy-yldxdy, where S is a triangle with vertices

o (0,

(5)

0), A (10, 1), and 8 (1, 1).

2150.

HeYdxdy,
(5) (5)

where S is a curvilinear triangle OAB bound-

ed by the

parabola y"=x and the straight lines x=O, y= 1 (Fig. 91). 2151. S S:Id~~, where S is a parabolic segment bounded by
the parabola y= ~ and the straight line y=x. 2152. Compute the integrals and draw the regions over which they extend:
11:
I

+cos x

11:
2

a) ~ dx
o
11
2 1

S if sin xdy;
0
4

a cos y
Xl

C)

S dy S
11

sin l ydx.

b)

S dx S y
o

dy;

"

cosx

When solving Problems 2153 to 2157 it is abvisable to make the drawings first. 2153. Evaluate the double integral

~ Sxy dxdy,
(S)

if S is a region bounded by the parabola yl = 2px and the straight line x=p. 2154*. Evaluate the double integral

~ ~ xydxdy,
(S)

extended over the region S, which is bounded by the x-axis and an upper sem icircle (x - 2)2 y" = 1.

252

Multiple and Line Integ rals

[ChI 7

2155. Evaluate the double integral

SS Y2a-x'
(S)

dxdy

where S is the area of a circle of radius a, which circle is tangent to the coordinate axes and lies in the first quadrant. 2156*. Evaluate the double integral

Hydxdy,
(S)

-where the region S is bounded by the ax is of a bsc issas and an "arc of the cycloid

x=R (t -sin t),


y=R(l-cost).
2157. Evaluate the double integral

~ ~ xydxdy,
(S)

-in which the region of integration S is bounded by the coordinate axes and an arc of the astroid

x=R cos s t, y=R sinS t (O~ t~ ;).


2158. Find the mean value of the function

f (x,

y)

== xyl in the

region

S{O~x~

1,

O~y~

I}.
rC~IOl1

Hint. The ,nlean value of a function I (x, y) in the

S is the nUIuber

7 = ~ SSf (x,
(~)

y) dx dy.

2159. Find the mean value of the square of the distance of a point M (x, y) of the circle (x-a)2+yl~R2 from the coordinate origin. Sec. 2. Change of Variables in a Double Integral
to. Double integral in polar coordinates. In a double integral, when passing fronI rectangular coordinates (x, y) to polar coordInates (r, cp), which are connected with rectangular coordinates by the relations x = r cos cp, y = , sin tp, we have the formula

Hf
~)

(x, y) dx dy=

~~
~)

(r

cos '1', r sin Ip) r dr dip,

(I)

Sec. 2]

Change of Variables in a Double Integral


==Q

253
and

r =- ~ (a <~) and the curves , == r) (q and r = r2 (q, where '. (q and '2 (q [r 1 ({p) ~ r 2 (cp)] are single-valued functions on the interval a ~ rp ~ p,

If the region of integration (S) is bounded by the half-tines r

then the double integral nlay be evalualed by the forrnula


f3
rJ
(CM

~ ~ F (cp. r) r dr dcp = ~ dcp ~ F (cp. r) r dr,


(S)

'1 (q

'2 (cp)
where F(cp. r)=f(rcoscp. r sin cp). In evaluating the integral

~ F(cp. rlrdr

'\ (cP)

we hold the quantity cp constant. If the region of integration does not belong to one of the kinds that has been exalnined, it is broken up into parts, each of which is a region of a given type. 2. Double integral in curvilinear coordinates. In the J110re gener al case, if in the double integral

~ ~ f (x.
(~)

y) dx dy

it is required to pass froln the variables x, y to the variables ll, (I, which are connected with x, y by the contInuous and differentiable relationshlOs
x:...=. {f' (a, tI), Y-=(ll, u)

that establ1sh a one-to-one (and, in both directions, continuou~) correspondence behveen the points of the region S of the -,"y-plane and the points of SOllIe region S' of the UV-plane, and if the Jacoblan

ax
I-==D(x, y)== JU D (ct, v) iJx

ay

au

avav

ay

retains a constant sign in the region S, then the fonnula

~ ~ f (x.
(~)

y) d.t dy;=

~ ~ f [rr (u,
(~')

v). '" (u. v) I III

d Il d v

holds true The lilnits of the new integral are detef111incd fronl general rules on the basis of the typc of region S' Exarnple 1. In pas~ll1g to polar coordinates, evaluate

) ~ Vl-x
(S)

-1J 2 dx dy,

where the region S is a circle of radIUS R == 1 \vith centre at the coordinate origin (Fig 92). Solution. Puttin~ x-=r~oscp, y::..::rsincp, \v~ obtain:

l'1" l-x 2 - y 2 -=

VI -

(r cos cp)2 - (r sin tp)2 =

Vi -

(I

254

Multiple and Line Integrals

[eh. 7

Since the coordinate r in the region S varies from 0 to 1 for any cp, and cp varies from 0 to 231, it follows that

55 Yl-xZ-yZdxdy= 5 Sr Yl-rZdr=i n.
dq>
(5)
0 0

21t

Pass to polar coordinates rand cp and set up the limits of integration with respect to the new variables in the followin~ integrals:
1 1
2

2160. ~ dx ~ f (x, y) dy.


o

2161. ~ dx ~ f eVxz+yZ)dy.
o

2162.

Hf (x,
(5)
1 1

y) dx dy,
t

where S is a triangle bounded by the straight lines Y=X, y=-x y=l.


2163.

Sdx Sf ( ~) dy.
-I

x2

2164.

Hf (x, y) dx dy, where S is bounded by


(s)

the lemniscatE

{x 2

+ y2)2 = a2 (Xl _
y

y2).

x
Fig. 92 Fig. 93

2165. Passing to polar coordinates, calculate the double inte gral

~ ~ ydxdy,

(5)

where S is a semicircle of diameter a with centre at the poin C ( J 0) (Fig. 93).

i-

Sec. 2]

Change of Variables in a Double Integral

255

2166. Passing to polar coordinates, evaluate the double inte

gral

~ ~ (Xl + yl) dxdy,


(5)

extended over a region bounded by the circle Xl + y. = 2ax. 2167. Passing to polar coordinates, evaluate the double integral ~ ~ Val-xl_yl dxdy,
(5)

where the region of integration S is a semicircle of radius a with centre at the coordinate origin and lying above the x-axis. 2168. Evaluate the double integral of a function f(r, qJ) = r over a region bounded by the cardioid r = a (1 + cos cp) and the circle r = a. (This is a region that does not contain a pole.) 2169. Passing to polar coordinates, evaluate
a

11 a 2 -x2

~ dx
o

~
0

VXI+y1dy.

2170. Passing to polar coordinates, evaluate

~ ~ Val-xl_yldxdy,
(S)

where the region S is a loop of the lemniscate (x 2 + yl)1 = at. (xt. - yl) (x ~ 0).
2171*. Evaluate the double integral

'CS)

f f Vl-~-~dXdY,
Xl

extended over the region S bounded by the ellipse aZ+/ii= 1 by passing to generalized polar coordinates:
x a =

y2

y. r cos CPt b = r sin qJ.


c

2172**. Transform

~ dx ~ f (x, y) dy
ax

f3x

uv=y.

(O<a<p

and c>O) by introducing new variables u=x+Y,

256

Multiple and Line Integrals

[Ch. 7

2173*. Change the variables u=x+y, v=x-y in the integral


t t

~ dx ~ f (x, y) dy.
0

2174**. Evaluate the double integral

~ ~ dxdy,
(S)

where S is a region bounded by the curve


( X2 a2

+b

Y2)2 _ 2 -

_ y2
h2

k2

Hint. Make the substitution

x == ar cos CPt

Y = br sin (p.

Sec. 3. Computing Areas


1. Area in rectangular coordinates. The area of a plane region S is

S=~~dXdY.
(S)

If the region S is defined by the inequalities a ~ x ~ b, cP (x) then


b
'\I> (x)

~y~

"I' (x),

S=

~ dx ~
a q> (x)

dy.

is defined by the inequalities a


S

2. Area in polar coordinates. If a region S in polar coordinates rand q> ~ cp ~~, f (cp) ~ r ~ F (cp), then
~

F (cp)

=~~
(S)

r dcp dr =

~
a

dcp
f

~
(q

r dr.

2175. Construct regions whose areas are expressed by the in-

tegrals
2

X+2

Va 2 _y2

a)

~ dx ~ dy;
-] x2

b) ~ dy

~ dx.
a-lJ

Evaluate these areas and change the order of integration. 2176. Construct regions whose areas are expressed dy the integrals
n

arc tan

sec cp

a)

~
4

dcp

, dr;

b)

~ drp
n

a (J+coc;
a

cp)

r dr.

Compute these areas.

Sec 8]

Conzpu tintZ Areas

257

2177. Compute the area bounded by the straight lines x = YI x=2y, x+v=a, x+3y=a(a>O). . 2178. Conlpute the area lying above the x-axis and bounded by this axis, the parabola y2=4ax, and the straight line x-!-y=3a. 2179*. Compute the area bounded by the ellipse
(Y_X)2
-~r =

1.

2180. Find the area bounded by the parabolas

y2 = lOx -J- 25 and y2

= -6x + 9.

2181. Passing to polar coordinates, find the area bounded by t he lines x2 _t y 2==2x, x2 -i- y 2=4x, y=x, y=O.

2182. Find the area bounded by the stra ight 1ine r cos rp = 1 a nd the circle r == 2. (The area is 110t to conta ina pole.) 2183. r:ind the area bounded by the curves r = a (1
-~

cos (r) and r =-= a cos fl (a> 0).


l

2184. Find the arpa bounded by the line


(

y2 ) -+-4 9
X2

2 _

-- 4

x2 ----a
y2

2185*. Find the area bounded by the ell ipse


(x- 2y t- 3)2

+ (3x -1- 4y- 1)2 =-== 100.

<a<b,

2186. Find the area of a curvilinear quadrangle bounded by the arcs of the parabolas x 2=alj, x 2=by, y2=ax, y2=~X(O<
O<a<~).

Hint. Introduce the new variables u. and tI, and put x 2= uy, y2 =-= vx.

2187. Find the area of a curvilinear quadrangle bounded by the arcs of the curves !J2 = ax, y2 === bx, xy === a, xy = ~ (0 < a < b,
O<a<~).
Hint. Introduce lhe nc\v variables u and
tI,

and put

XY=U,
9 -1900

y2=VX.

258

Multiple and Line Integrals

[Ch. 7

Sec. 4. Ccmputing Volumes


Thp volume V of a cylindroid bounded above by a continuous surface z = f (x, y), b 10\\' by the pie' ne 2 =- 0, and on the sidrs by a righ1 cylindrical surface, which cuts out of the xl/-plane a region S (Fig. 94), is equal to
V=

~ ~
tS)

t(x, y) dx dy.

2188. Use a double integral to express the volume of a pyramid wilhvertices 0(0,0 0), A(I,O,O), 8(1, 1,0) and C(O,O, 1) (Fig. 95). Set up the limits ot integration.

B (~I,O)
Fig. 95

x
Fig. 94

In Problems 2189 to 2192 sketch the solid whose volume is expressed by the given double integral:
1 I-X 2

V 1- x- 2

2189. ~dX~(l-x-Y)dY,
o
2 0 2-X

2191.~dX
0 2 2

~ (l-x)dy.
(4 - x - y) dy.

2190.
a

~ dx ~
o n l/(l'2.:Ki

(4 - x - y) dy.

2192.

~ dx ~
0

2-X

2193. Sketch the solid whose volume is expressed by the in-

tegral ~ dx
o

~
0

Va! -r-y

dy; reason geometrically to find the

value of this integral. 2194. Find the volume of a solid bounded by the elliptical paraboloid z=2x 2+y2+t, the plane x+y=l, and the coordinate planes. 21i:S. A solid is bounded by a hyperbolic paraboloid

z=xl_Jt

and the planes y=O, z=O, x= 1. Compute its volume.

Sec. 5)

Com'Ju,tin,! th~ Afea~

of

S'1.f faces

259

2196. A solid is bounded by the cylinder x 2 +-z~ =a3 and the planes y=O, z==O, y=x. C)mpute its volume. Find the voluln~s bounded by the followiilJ surfaces: 2197. az = y2, x 2 +- y2 == ,..2, Z = O. 2198. Y=Vx, y==2 J,tlx, x+z=6, z=O. 2199. z = r -r- y2. Y = x2, y = 1, Z = o. 2200. x+y-j-z=a, 3x+y=a, {x+-y=a, y=O, z=O. 2201. 2 a 2202. x 2
x2

+c 2

= 1, y = -a x, y = 0, z =0. +- y2 = 2ax, Z = ax, Z = px (a > Pl.

Z2

In Problems 2203 to 2211 use polar and generalized polar coordinates. 2203. Find the entire volume enclosed between the cyl inder 2 2 Z 2 X + y2 = a and the hyperboloid x + y3 - z! = _ a . 2204. Find the entire volume contained between the cone 2(x 2 tif)-Z2=O and the hyperboloid x 2 +_y2_ ZJ. =_a 2 2205. Find the volunle bounded by the suriaces 2az = x 2 + y'l. t
Xl

-t- y 2 _ z2 = a2 z=o. 2206. Deterlnine the volume of the ellipsoid

l2 -1- IJ2 -t- C2 = 1.


2az

x2

y'.!.

Z2

2207. Find the volume of a solid bounded by the paraboloid == x 2 -+ y'l and the sphere x 2 .,- yl -t- Z2 = 3a 2 (The volume lying inside the parclboloid is nleanL) 2208. Cornpute the volurne of a solid bounded by the xy-plane, the cylinder x 2 -t y2==2a~t and the cone Xl +y2=Z2. 2209. C0l11pute thp VOlUI11e of a solid bounded by the xy-plane, the surface z =- ae- (x:l-f l/~), and the cyl i.nder x 2 + y2 = R J 2210. Compute the volunlc of a solid bounded by the xy-plane, ,,\ 2 !l 2 x2 y2 x the paraboloid z=-2-1-2' and the cyltnder 2+-b2=2-. a b a a 2211. In what ratio does the hyperboloid x 2 +y2_ z2=al divide the volUIlle of the sphere x 2 -+ yl + Z2 ~ 3a2 ? 2212*. Find the volunle of a sol id bounded by the surfaces z=x+y, xy= 1, xy=2, y=x, y=2x, z=O{x>O, y>O).
Sec. 5. Computing the Areas of Surfaces
The area
0

on the xy-plane is

of a srnooth single-valued surface z = f (x. y). whose projection th~ region S, is equal to

9*

260

Multiple and Line Integrals

[Ch. 7

2213. Find the area of that part of the plane ~

+ f+ f

which lies between the coordinate planes. 2214. Find the area of that part of the surface of the cylinder x 2 + y2 = R 2 (z ~ 0) \\'h ich lies bet\veen the planes z = mx and
z = nx (Tn> n > 0).

2215*. COOlpute the area of that part of the surface cone x 2- y2 = Z2 \vhich is situated in the first octant bounded by the plane y -1- z = a. 2216. Compute the area of that part of the surface cylinder x 2 + if = ax which is cut out of it by the x 2 -t- y2 + Z 2 =_ a 2 2217. Compute the area of that part of the surface x2 11 2 sphere x 2 -+ y2 + Z2 = a 2 cut out by the surface l2 + fj2 = 1.

of the and is of the sphere of the

2218. Compute the area of that part of the surface of the paraboloid y2+ z2=2ax which lies between the cylinder y2=ax and the plane x = a. 2219. Compute the area of that part of the surface of the cylinder x 2+ y2 = 2ax which lies between the xy-plane and the cone x 2-~ y2 = Z2. 2220*. Compute the area of that part of the surface ot the cone X 2_!J2=Z2 which lies inside the cylinder x 2 +y2::::::2ax. 2221*. Prove that the areas of the parts of the surfaces of the paraboloids x 2 +y2='2az and x 2_ y 2=2az cut out by the cylinder x2 -1- y2 == R2 are of equivalent size. 2222*. A sphere of radius a is cut by two circular cylinders whose base diarlleters are equal to the radius of the sphere and which are tangent to each other along one of the dialneters of the sphere. Find the volurne and the area of the surface of the re111a in i ng part of the sphere. 2223* An opening with square base whose side is equal to a IS cut out of a sphere of radi us a. The ax is of the open iug coincides with the diameter of the sphere. Find the area of the surface of the sphere cut out by the opening" 2224*. C0111pute the area of that part of the hel icoid z=carctan JL \vhich lies in the first octant between the cylinx 2 ders x ..,.y2=a 2 and X 2+y2=b 2,

Sec. 6. Applications of the Double Integral in Mechanics


to. The mass and static moments ot a lamina. If S IS a region in an xy.plane occupied by a lanlina, and Q (x, y) IS the surface density of the laIn ina at the point (x, Y), then the mass M of the lamina and its static

Sec. 6]

Applications

of the Double Integral in Mechanics

261

double integrals

moments Mx and My relative to the x- and y-axes are "rxpressed by the


M=

~ ~
(S)

Q(x, y)dxdy, M X =

~ ~ YQ(x,
(~)

y)dxdy, (1)

My= ~ ~ xQ
(5)

(x, y) dx dy.

If the lamina is homogeneous, then Q (x, y) = const. 2. The coordinates of the centre of gravity of a lamina. If C

centre of gravIty of a lamina, then My

<x: Y>

is the

x=M' Y=M'

Mx

where M is the tnass of the lamina and At x' My are its static moments relative to the coordinate axes (see to). If the lamina is hornogeneous, then in fonnulas (1) we can put Q = 1. 3. The moments of inertia of a lamina. The ITIOments of inertia of a laluina relative to the x- and y-axes arc, respectively, equal to

1x=

~~
(5)

y2 Q(x, y) dx dy,

/ y=

~ ~
(5)

x Q (x, y) dx dy.

(2)

The l110ment of inertia of a lanlina rl'lative to the ongin is


10 =

~ ~
(.."

(X 2 +y2)Q(X, y)dxdY=/x+1y.

(3)

Putting Q (x, !/) -== 1 in fornlulas (2) and (3), \ve get the geolnetric moments of lncrtJa of a plane figure.

2225. Find the nlass of a circular lanlina of radius R if the density is proportional to the distance of a point fronl the centre and is equal to () at the edge of the lalnina. 2226. A laIllina has the shape of a right triangle \vith legs 08 === a and OA == b, and its density at any point is equal to the distance of the point froln the leg 0 A. Find the sta t ic IIl0I11ents of the lamina relative to the legs OA and DB. 2227. COlnpute the coordillat{)s of the centre of gravity of the area GlnAnO (Fig. 96), which is bounded by the curve !J= 5inx and the straight 1ine OA that passes through the coord inate origin and the vertex A ( ~, 1) of a sine curve. 2228. Find the coordinates of the centre of gravity of an area bounded by the cardIoid r=a(l-t-cos(p). 2229. Find the coordinates of the centre of gravity of a circular sector of radius a with angle at the vertex 2a (Fig. 97). 2230. Compute the coordinates of the centre of gravity of an area bounded by the paraholas 1}2 = 4x 1- 4 and y2 = - 2x +4. 2231. C01l1pute the rnoment of inert ia of a triangle bounLied by the straight linesx+y=2, x=2, y.=2 relative to the x-axi~~.

262

Multiple and Line lnfe/!ra!s

[Ch. 7

2232. Find the monlent of inertia of an annulus with diameters d and D (d <D): a) relative to its centre, and b) relative to its diameter. 2233. Cumpute the moment of inertia of a square with side a relative to the axis. passing through its vertex perpendicularly to the plCine of the square. 2234i:. C )mpute the moment of inertia of a segment cut ofT the parabola y2 ==ax by the straight line x=a relative to the straight line y=-a.

o
Fig. 96

x
Fig. 97

2235*. Compute the moment of inertia of an area bounded by the hyperbola xy=4 and the straight line x-t-y==5 relative to the straight line x=y. 2236;;. In a square la~in1 with side a, the density is proportional to the distance from one of its vertices. C)mpute the nlOment of inertia of the lamina relative to the side that passes through tftis vertex. 2237. Find the moment of in~rtia of the cardioid r=a(l +cos<p) relat ive to the pole. 2238. Compute the moment of inertia of the area of the lernn iscate (2.=.-:. 2a 2 cos 2(J) rela tive to the axis perpendicular to its plane In the pole. 2239*. C3mpute the moment of inertia of a homogeneous lamina bounded by one arc of the cycloid x=a(t-sint), y=a(l-cost) and the x-aXIS, relative to the x-axis.

Sec. 7. Triple Integrals


to. Triple integrals in rectangular coordinates. The triple inte~ral of the function !(x, y, ~) extended over the region V is the limit of the corresponding threefold iterated sum:

~ ~ ~ ,(x, ~',
V

z)dxdy dz=lim
m;JX

max h") max ~zk

'Xi -+

0 -+ 0

~~~f(Xtl YJI
i

z")AXtAYJAz,,.

-+ 0

!jec. 7]

Triple

'nte.~,als

Evaluation of a triple Integral reduces to th~ succeSC;;lve comput'.Jt1on of th~ three ordinary (on~fold iterated) integrals or to the COinputatlon of one double and one sin~le integral. Example I. COlllpute J = ~ ~ ~ X 3y 2 z dx dy dz,

where the region V is defined by the inpqualities O~x~ 1, O~Y~x, O~z~xy. Solution. We have

1=

j dx 5dy Jx3y2z dz 5dx 5xV f Idy=


=
.. 0 0 0 0 0

XII

Jt.

XII

=JdX.
o
0

xI'

2- dY =J 25
0

X'II"

Xl 1/' xI
0

\1. x 10 1 dX-=tj lod~=lll)'


0

Example 2. Evaluate

~~~
(V)

x d x dy dz,
x2
11 Z2 b 2 +- C2 = a
2

t~xtel1drd

over the volulne of the ellipsoid Q2 -ta

1.

Solution.

~ ~ ~x
H')

dx dy dz -=

~
-a

x dx
2

~~
(5

dy dz =
2

u.>

~
-a

x2 S yz dx,
const, and is equal lo

\\ here S JlZ IS the area of the el1i pse

1/ Z~ x lJ2 + (2 -= 1- Q2 ' x =

Syz=nb
\Ve therefore ti lla 11 y

-V l-~.C -V l-~=nbc
~el
2

l-~).

SSSx
(V)

dxdydz=,nbc

-a

Sx l-~) dx=~na3bC,
2 (

2. Change of variables In a triple integral. If in th~ triple il1h"\~rat

~ ) ) f (x, y.
( /)

z) d.t d y d z

it is required to pass from th~ variables x, II, 2 to the variables u, v, w, which arl~ conncct~'d with X,!I, z bv the rl-'I at ion~ x = q> (u. v, w), y = 'I' (u, v, w), z =X (u, v, w), whlr~ the functlon'\ q>. '1'. X ar~: 1) continuous together with their pal tial fir~t derivatives; 2) in one-to-one (lnd. In both direLtiolls. contil1uJu;;) correc;;pondence bet\veen the poi.,ts of the region of integratlon V 01 Xyz-spdce and the pOints of sOlne region V' of UVW -space;

264

Multlple and Line Integrals


3) the functional deternlinant (Jacobian) of these functions

[ell. 7

au
/=
D (x, y,!l.= D (u, v, w)

ax

iJx

ax
iJy

iJu ath

au av iJw az az az
au au

ay ay

dw

retains a constnnt sign in the regIon V, then \\'e can lnake use of the formula

~ ~ ~ f (x, y,
(V)

z) dx dy dz =

~ ~ ~ f Irr (u,
(\I')

v, w), (u, v, w), X(u, v, w))

III du dv dw.

o~

__-+

h
_

x
Fig. 98

x
fIg. 99

I n part Icular, 1) for cylindrical coordinates r, cp, Iz (FIg. 98), where x == r cos (P, y sin (P, z ::::- II , we get I -= r; 2) for spherical coordinates cp, ,~, r (cp is the longitude, r the radius vector) (FIg. 99), \vhere

=,

'p

the latitude,

x = r cos cos q>,


2

y= r cos 'P sin q>,

= r sin ~"

we have 1::-:: r cos 'i'. Example 3. Passing to spherical coordinates, compute

~ ~ ~ yX 2 +y2+ Z 2dxdydz,
(V)

\\there V is a sphere of radius R. Solution. For a sphere, the ranges of the spherical coordinates (p (longItude), 'I' (latitude), and r (radius vector) will be

SPC

7J

Triple Integrals
n
2";

265

We therefore have

~~ ~
H')

y x2-+!l -I- 22 dx dy dz =

~ drp ~ d1Jl ~
0

r r 2 cos 1Jl dr=1tR",

n
2

3. Applications of triple integrals. The volU1ne of a region of three-dimensiona I xyz-space is

V=~~~dXdYdZ.
(\/)

The

1!laSS

of a solid occupying the region V is

Mc-~ ~ ~ ~
(V)

y(\'. Y. z) dxdydz.

where y (x, !/, z) is the density of the body at the pOint (x,!I, z). The static l1Z011zenls of the body relative to the coordinate planes are

M.w ~=
Ml'z=
Mz.\"=

~ ~ ~ ,. (x. Y. z) z dx dy dz;
(V)

~~ ~
0')
(V)

,,(x. Y. z):.dxdydz; z) ydxdydz.

~ ~ ~ y(x. y.

The coordinates of tire celltre of gravlty are i\lyz A'zx -

x="M-'

!I:::-:::~, z=~.

A-1 XI'

If the solid is homogeneous, then \ve can put y (x, y, z) == 1 in the fornlulas for the coor(ltnates of the centre of grav1ty. The l1Z011Zettts of inertia relative to the coordinate axes are
I x= ly= lz=

~ ~ ~ (y2+ Z2) Y (x. Y.


(V)

z) dx dy dz;

~ ~ ~ (Z2+ X2) Y(x. Y. z)dxdydz;


(\')

~ ~ ~ (xt+yt)y(x. y. z)dxdy dz.


(\')

Putting 'V (x. y, z)= 1 in these fornlulas. \\'e get the geoilletric Jnomen{s of inertia of the body.

A. Evaluat ing triple integrals Set up the linlits of integration in the triple integral

~ ~ ~ f (x.
(V)

y. z) dx dy d z

for the indicated rl'giolls V.

266

Multinle arrd Lire Integrals

(Ch. 7

2240. V is a tetrahedron bounded by the planes

z=O. 2241. V is a cylinder bounded by the surfaces x 2 +y" =R 2 , z=O, z=H. 2242*. V is a cone bounded by the surfaces
x=O, y=O,
x2
-2

x+y+z= 1,

+ b = 2"'
2

,,2

Z2

= c.

2243. V is a volume bounded by the surfaces z = 1- x2- y2, Z = o.

Compute the following integrals:


2244.

J J JVx+y+z+l 000
2

r dx rdy r
"V x
o

dz

V-22245. ~ dx ~ dy xdz. ~
a

.. f

4X-I/2

VQi":X2

V-U2-_-X'l-_~y3

2246.

Sdx
I

.\
o
I-X

dy
I-X-II

j'
0

dz

2247.

~ dx ~ dy ~

xyzdz.

2248. Evaluate

555
(\I) (V)

(x

-t- II + Z + 1)3 t

d"ldydz

\\lhere V is the region ot Integration bounded by the coordinate planes and the plane x+y+z=l. 2249. Evaluate

~ ~ ~ (x + Y + Z)I dx dy dz,
\\,here V (the region or integration) is the common part of the palabolold 2a,~x2+y2 and the sphere x'+y2+z2~3a2. 2250. Evaluate

where V (region ~ integration) IS the common part of the spheres x2+ y2 t- Z2 ~ R~ and x 2 + y2 + Z2 ~ 2Rz

Sec. 7]

Triple Integrals

267

2251. Evaluate

~H zdxdydz,
n')

where V is a volume bounded by the plane z = 0 and the upper . . x2 11 2 Z2 half of the ellIpsoId -2-t':-b = 1. 2 +-2 II C 2252. Evaluate

SJ
2253. Evaluate

rs (i2 +
x
2

{j2

y2

Z2 ) -f- C2 dx dy dz,

(V)

"There l! is the interior of the ell ipsoid

11 x: + b : + 2: = c

1.

~ ~ ~ z dx dy dz,
n')

\vhere V (the region of integration) is bounded by the cone


z! == R~ (x 2 t- y2) and the plane z == h. 2254. Passing to cylindrical coordinates, evaluate
1z2

~~~ dxdydz,
(\ ')

\vhcrc V is a region bounded by the surfaces x:! -\- y2 ix 2 -I y2 -=-= Z2 and contaIning the point (0,0, R). 2~55. Evaluate
2

Z2

= 2Rz.

1":! \. - x'

~ tlx

~ dy ~ z V x 2 + y2 tiz,

OliO

first transfortning it to cylindrical coordinates. 2256. Evaluate


2r

lr 21x -

A .I

4f~J_y~

dx

-V~

dy

dz,

first tra nsformi ng it to cy Ii ndrical coordi nates. 2257. Evaluate


R V R.I- x 2

V R2 -

x.l -

y2

~ dx
-R

dy

(Xl -I- yl) dz.

-VR.I-x2

first transforming it to spher ical coordinates.

268

Multiple and Line Integrals

[Ch. ~

2258. Passing to spherical coordinates, evaluate the integral

~ ~ ~ V Xl +y2 + Zl dx dy dz,
(V)

where V is the interior of the sphere x2 -I- y2 -1- Z2

x.

B. Computing voluflles by means of triple integrals


2259. Use a triple integral to compute the volume of a solid bounded by the surfaces y2=4a2 -3ax, y" = ax, z==h. 2260**. Compute the volume of that part of the cylinder x 2 + y2 = 2ax which is contained between the paraboloid r + y2 = 2az and the xy-plane. 2261*. Compute the volume of a solid bounded by the sphere x 2 + y2 + Z2 = a2 and the cone Z2:.-= x2 + y~ (external to the cone). 2262"'. Compute the volume of a solid bounded by the sphere 2 2 X +y2+ Z 2=4 and the paraboloid X +y2=3z (internal to the paraboloid). 2263. Compute the volutne of a solid bounded by the xy-plane, the cyli nder x 2 -~- y2 = ax a nd the sphere x 2 + y2 -t- Z2 == a2 (i nterna 1 to the cylinder). 2264. Compute the volume of a solid bounded by the paraboloid

Z~
C

b!+ CZ = 2 a and the plane x=a.


C. Applications of triple integrals

y2

z~

to mechanics and physics


2265. Find the rnass M of a rectangular parallelepiped 0 ~ x ~ a, O~y~b, O::=;;z~c, if the denv sity at the point (x, y, z) is (l(x, y, z)=x-~y-~-z. 2266. Out of an octant of the sphere x 2 + y2 -I- Z2 ~ c2 , X ~~ 0, y~O, z~O cut a solid OABC bounded by the coordinate planes 1 (a~c, b ~ c) (Fig. 100). Find the mass

x
Fig. 100

and the plane : + : = of this body if the density at each point (x, y, z) is equal to the zcoordi nate of the poi nt. 2267*. In a solid which has the shape of a hemisphere 2 Xl + y2 Z2 ~ a t Z ~ 0, the density varies in proportion to the

Sec. 8]

Improper Integrals Dependent on a Paranleler

269

distance of the point from the centre. Find the centre of gravit~' of the soli d. 2268. Find the centre of gravity of a solid bounded by the paraboloid y2 +2z 2 == 4x and the plane x= 2. 2269*. Find the nlonlent of inertia of a circular cylinder, \vhosc altitude is h and the radius of the base is a, relative to the axis which serves as the diameter of the base of the cylinder.. 2270*. Find the mOI1lcnt of inertia of a circular con~ (altitude, Iz, radius of basc, a, and density Q) relative to the diaIllcter of the base. 2271**. rind the forcc of attraction exerted by a homogeneous cone of altitude It and vertex angle t (in axial cross-section) on a l11aterlal point containing unit Inas~ and located at its vertex. 2272**. Sho\v that the force of attraction exerted by a honl0geneous sphere on an external nlaterial point does not change if the entire I1laSS of the sphere is concentrated at its centre.
Sec. 8. Improper Integrals Dependent on a Parameter. Improper Multiple Integrals
1. Difl'erentiation with respect to a parameter. In the case of certain f('stnctlons iJllpOscd on thp functIons f (x. a), f~ (x, a) and on the correspondIn~ illlpro!Jcr Integrals \\TC have the Lelb111Z rule
r:r:

d da

Cf (x, J
II

et) dx ==

r fa' (X, J
'I

00

a) dx.

Example 1. By diff<?rentiatinl! \vith respect to a paranletcr, evatuate


00
t

J
Solution. Let
Then

e-l}x2 _e-;,o'\:l - - - - - dx

(a

> 0,

> 0).

~ - - - - dx==F(a,
e-l'Jo~1 _e-~xl

00

~).
~

00

aF (a,
Whence F(a,

aa

t\) = _

r xe-'lX I dx= 2a -!- eJ


o

lJx1

! :::z
0

_J.-.
2a

~)==- ~ Ina+C(~). To find C(~). we put a=~ in the latter

equation. We have 0= 1

In

~ + C (~).
1

Whence C (P) ="2 tn~. Hence,

F {a, ~):.=

-2 111 a+2"ln p=2"ln

~ a.

270

Multiple and Line Integrals

[Ch. 7

20 Improper double and triple integrals.

a) An infinite region. If a function region S, then we put

f (x,

y) is continuous in an unbounded
(x, y) dx dy,

Hf
(S)

(x, y) dx dy= 11m


(J

Hf

(1)

-'s(O)

where (J is a finite region lying entirely within S, where 0 -+ S signifies that we expand the region 0 by an arbItrary law so that any roint of S should enter it and rel11ain in it. If there is a limit on thp right and if it does not depend on the choice of the rpl!ion 0, then the corresponding itnproper integral IS called cOllverR,eHl, othcr\vise it is dIvergent. If the inh'gratHI !(x, y) i~ nonnegative l{(x, y)~Ol, then for the con ver~ence of an t1l1rJop~r integral it IS nt'ce~sary and sufficient for the limit on the right of (I) to exist at least for one system of regions a that exhaust the region S. b) A di\continuous function. If a function f (x, y) is every\\'here continuous in a bounded closed region S, excppt the point P (a, b), then we put

Hf
(S)

(x, y) dx dy= lim


e -+

Hf
0 (5
E

(x, y) dx dy,

(2)

where Se is a region obtained from S by elinl1nating a snlall region of dla metfr E ihat contaIns thp roint P. If (2) has a limit that dops not depend on the tYre of snlall r(3gions elimira t (3d from 5, the in1rroper integral under cons\deration is called (onverr..enl, oth( rWlse it is divergent. If f (x, y)~O, UHn the limit on the ri~ht of (2) is not dependent on the type of regions elitninated from S; for instance, such reg:ons may be circles of radius

with ccntrp at P.

The concept of improper double integrals is readtly extended to the case of trirle integrals. Example 2. Test for convergence
(3)

where S is the entire xy-plane. SotuHon. Let a be a circle of radius Q with centre at the coordinate origi n. Passing to ~olar coordi nates for p :1= 1, we have

1(0)=

55
(~

(1

+~~ yElP = Sd<p


0

21t

5 ~d;l)p =5
(l
0
21t

2 P 1 2 (1+r )I- IO I-p 0

dcp=~ [(1+Q2)I-P_I].
I-p

If p < 1, then Urn 1 (0)= lim


o
-to S

1(0)= 00

and the integral diverges. But if p> 1, converges. For

~ e:t:

then

Q-+fD

lim I (0) = ~l

p-

and

the

integral

p = 1 we have

Sec. 8]

Improper Integrals Dependent on a Parameter


that is, the

271

1 (0)= ISn dlpSQ I'd' 2=rtln(1+Q2);


o
0

+r

O~CI)

lim 1(0)=0),

integrat

diverges. Thus, the integral (3) converges for p

>

1.

2273. Find

I' (x), if
Ql)

f (x) = ~ e- xy' dg
x

(x

> 0).

2274. Prove that the function


LJ

C xf (z) = J x 2 + (y-zr' dz
-00

+~

satisfies the Laplace equation


iJ2 u iJx2

+ d!J2 = o.
f (t)
c)CI

iJ!ll

2275. The La place transfornlatlon F (p) for the function is defined by the fornlula
F (p) = ~ e-t.tf (t) dt.
u

d)

f (I) == cos ~t.

Find

F (p),

if:

a)

f (I) =
1

1:

b)

t (t) = eat;

c)

f (t) =

sin ~t;

2276. l"aking advantage of the rornlula

5xn-'dx
o

~ (n > 0),

compute the integral


1

~ xn -'1nxdx.
I)

2277*. Using the formu la


Ql)

5eo

pt

dt

(P > 0),

eval uate the integral

272

M uLUpie and Line Integrals

rCh. 7

Applying differentiation \vith respect to a parameter) evaluate

the following integrals:


2278. 2279.

Se-'
o
eX)

00

;e-;i dx (u

> O.

> 0).
~

Se-,x-;e-;;x sin mxdx (u > O.


o
eX)

>

0).

2 280.

J
o
1

C arc tan ax d
x (1

+x

2)

x.

2281. Sln(I~)dX 2
x YI-x 2
o
00

(/u/< 1).

2282. ~e-ax Si:~X dx (u~ 0).


o

Evaluate the following improper integrals:


ex> cr.,

2283. ~ dx ~
o
1
0

e-(HIJ)

dy.

2284.
2285.

~ dy ~ eu dx.
0

y2

rr dxdy J j 4 _j- y2 , \vhere S is a region defined by the inequali(5)

ties

x~

1,
00

Y~X2.
00

Cdx S {x 2 +y2_I_a dr 2 )2 (a>O). 2286*. J


o
0

2287. The
00

Euler-Poisson integral defined by the forl11ula


00

J= ~
o

e- x'

dx may also be written in the form I = ~ e- Y ' dy. Eval0

uate / by multiplying these formulas and then passing to polar coordinates. 2288. Evaluate
00 00 00

Sdx ~ dy ~ (x2 + y2 ~ Z2 + 1)2


o
0 0

SefJ. 9]

Line Integrals

273

Test for convergence the improper double integrals: 2289**. ~ ~ In V Xl -+ y2 dxdy, where S is a circle x 2 + y2 ~ 1.
2290.

55 (X~~d:2)'1.'
(5)

(S)

where S is a region defined by the ine

quality x 2 -1 y2
2291*.

1 ("exterior" of the circle).

where S is a square Ix!:=;;;; 1, Iyl:=;;;;l. 5 2292. 555 (X2::'Y:~2)'1. , where V is a region defined by the
JV(X~y)21
(S)
0')

dx dll

inequality x 2 -t- y2 -1Sec. 9. Li ne Integrals

Z2 ~

1 ("exterior" of a sphere).

1. Line integrals of the first type. Let f (x, y) be a continuous function CJnd y = cp (x) [a ~ \ ~ b) be the equation of some snlooth curve C. Let us construct a system of points 1\1 , (xi, y,) (i == 0, 1, 2, ... , n) that break up the curve C into elenlentary arcs A1 ,':-;M i == ~si and let us fornl the
Il

integral
lliax

SUln

Sn -== ~ f (xi, y,) fls ,. The ltmit of this sunl, \vhen n ~


l-I

00

and

f1s i ~ 0,

IS c~lled

a ILne I1ltegral of the fIrst type


lim

Il

{(xi. Yi) !'lsi =

1l"\X;i=1

~ f (x,

y) ds

(ds

IS

the arc differential) and is evaluated fronl the fonnula


b

~ f (x,
C

y) ds =

~
a

{(x,

er (x Y 1 + (cp' (xW dx.

In the case of paranletric representatIon of the curve C: x==<p (t), Y --= 'l' (t) [a ~ t ~~], \ve have

~ f (x,
C

y) ds =

~ f (cp (t), ~, (t Y <p'l (t) + 1\J'1 (t) dt.


a

Ii

Also considered are line integrals of the first type of functions of three variables f (x, y, z) taken along a space curve. These integrals are evaluated in li'<e fashion A line integral of the first type does not depend on the direction of the path of il1terzratton; if the integrand f is interpreted as a linear density of the curve of integration C, then this integral represents the mass of the curve C.

274

Multiple and Line Integrals


Example 1. Evaluate the line integral

(Ch. 1

~ (x+ y) ds,

where C is the contour of the triangle ABO wi th vertices A (1, 0), B (0, 1), and 0 (0, 0) (Fig 101). Solution. Here, the equation AB is y= I-x, the equation DB is x=O, and the equation OA IS Y = O. We therefore have

(x+y)ds=

~ (x+y)ds+ ~ (x+y)ds+ ~ (x+y)ds=


AB
80
I

oA
I

=) Y2dX+) ydY+) xdx= Y2 + I.


o
0 0
t

2. Line Integrals of the second type. If P (x, y) and Q (x y) are continuous functions and y = (f) (,\) 1S a sInoa th curve C that runs from a to b as

y
B

A
Fig. 101

x varies. then the corresponding ltne tntegral of the second type is expressed as follows:
b

) P (x, If) dx C

+ Q (x,

y) dy = ) (P (x, q> (x + q>' (x) Q (x, q> (x1 dx.


a

In the more genera) case when the curve C is represented parametrically: %=<p(t), y=lj)(t), where t varies (roln a to p, we have p ~ p (x, y) dx + Q (x, y) dy + ) (P (q> (t), (t)) q>' (t) + Q (q> (t), (t ' (t tIt.
C
a

Similar formulas hold for a line integral of the second type taken over a space curve. A line Integral of the second type clzanaes stgn when the dtreclton of the path of tl1teg ratton lS reversed. This integral may he interpreted nlE.'chanically 8S the work of an appropriate variable force {P {x, y), Q (.x, y)} along the curve of integrdtton C Example 2. Evaluate the line integral

~ y 2 dx+x 2 dy,
C

Sec. 9)

Line III feR! als

275

where C is the upper half of the ellipse x=a cos t, clockwise. Solution. We have
o

y=b

sin t traversed

~
c

y2 dx

+ x 2dy =

~
n

{b 2 sin 2 t ( - a sin t)

+a

cos 2 t b cos t] dt =

ab". n n 3. The case of a total dift'erentfal. If the inte~rand of a line integral of the second type is a total differential of some sIngle-valued function U ~ U (x, y). that is, P (x, y) dx+ Q (x. y) dy-=dU (x. y). then this line integral IS not dependent on the path of integration and we have the Newton-Leibn iz formula
(xz. Y2)

= -abo

sill' t dt

+ a'b Scos' t dt = ~

~
(XI' YI)

P (x, y) dx

+ Q (x,

y) dy == U (x 2 , Yl) -U (XI' YI)'

(1)

\vhere (XI' YI) IS the initial and (x 2 y~) is the terminal point of the path In particular, if the contour of inte~ratioll C IS closed, then

~p
C

(x. y) dx -\- Q (x. y) dy

=0

(2)

If ]) the contour of inteL!r:ltion C ic; cont:lined entirely within some simnlv-connected re!iO,l ~ Clnd 2) the functions P (r. l/) and Q (x, y) totfether \\'Ith th . .' ir partial derlv~ttivcs af the first areier are continuous in S, then a neCCS"iarv and sufficiel1t co Hhtio 1 fa, tho? exi~tellce of the function U is the i,lenticai fulJilrllent (In S) of the equality iJQ uP (3) ax =dy

If conditions one and hvo are not ruldocs not guarantee the existence of a sll1l!le-vall1(,o 'unction U, and fOrlnulas ~ 1) and (2) may prove wrong (s('e Problenl 2.t 2) v.,'e give a 111ethod of Bnding a function U (x, 1/) lronl its total diflcrcn1ial based on the lise of line integrals (\vhich is yet ~1t-:other method of inte~ratil1g a total differential). For the contour of Integration C let us take a brok('11 11l1e P(,P I A1 (FIt! }(2). where p(\ (X(I' Yo) is a fixed f oint and M (x, y) IS a variable pOI nt. Then a long PoP I \ve have y =. Yo and dy -= 0, and along P 1"'1 \ve have dx:.::;: 0 We get:
JiUed, the." prcscllcc of conditton (3)
(x. y)

(see

jnte~ration

of total difrl"rentials)

U (x. y)- U (x o' Yo) =-=

~
(X Ut Yo)

p (x, y) dx + Q (x, y) dy =
x Y

~ p (x.
Xo

Yo) dx+

~ Q (x.
Yo

y) dg.

SimIlarly, integrating with respect to PtP"M, we have


y

x
%0

U (x. y)-u (.Ko Yo) = ~ Q (x o' y) dy + ~ p (x. y) dx.


Uo

276

Multiple and Line Integrals


Example 3. (4x+2y) dx+ (2x-6y) dy=dU. Find U. Sol ution. Let Xo = 0, Yo = O. Then
X

[CIl.7

1/

V (x, y) = or
y

S4x dx+ S (2x-6y) dy+C=2 x2 +2xy-3y2+C


o
0

V (x, 1/)= S -6ydy+


o

S(4x+2y) dx+C=-3 y 2+2.t 2+3xy+C,


0

where C ==- U (0, 0) is an arbitrary constant.

M(x,y)

llo

,Po (;xo, Yo) I P, (x, {Jo)


Xo

X
Fig. 102

4. Green's formula for a plane. If C is the boundary of a region Sand the functions P (x, y) and Q (x, y) are continuous together with their firstorder partial derivatives in the closed region S-t-C, then Green's fOrf1lula holds:

1P dx+Q dy= 55 (~~ -:) dxdy,


C (5)

here tl'e circulation about the contour C is chosen so that the region S should remain to the left. 5. Applications of line integrals. J ) An area bounded by the closed contour C is

S=

- ydx= xdy
c c

(the direction of circulation of the contour is chosen counterclockwise). The fo lowing formula for area is more convenient for application:

S=

~ (xdy-ydx)= ~
C

x
C

(~).

z= Z (x,

2) The work of a force, having projec!ionsX=X(x, y, 2), Y=Y(x, y, Z), y, z) (or, accordingly, the work of a force field). along a path C is

Sec. 91
expressed by the integral
A=

Line Integrals

277

X dx

+Y dy +Z dz.
ij, z)

If the force has a potential, i.e., if there exists a function U ==U (x, (a potential functIon or a force function) such that
iJU
-=)(, ax

iJU -=-=Y, -=--=z, iJy dz


z.d

au

then the wort" irrespective of the shape of the path C, is equal to


(X J II'!" Z,:) (X,:. th.

A=

)
(\"1. Vi'

Xdx-j-Ydy-!-Zdz=
zd
Zl)

~
(t 1 , 111'

dU=U(X 2'Y2' Z2)-U(X"y. ".), zd


Z2)

where (\"1' Yl'

is the Initial ano (x 2, Y'2.'

is

th~

tefl111nal point of the path.

i\. Lille Integrals of the First Type


Evaluate the follo\ving line integrals: 2293. ~ xy ds, where C is the contour of the square I x I + I y I = a c 2294. \ .r -2 ds ;;-~-, where C is a segment of the straight line

(a

> 0).

cr
c

i- y

t-

,t

connecting the points 0 (0, 0) and A (1, 2). 2 II r x 2295. J xy ds, \\'here C is a qua rter of the ellipse i2 + ~ = 1

1ying in the first quadrant. 2296. ~ y'ds, where C is the first arc of the cycloid x = a (t - sin t).
y = a (1 - C()~ f). 2297. ~ V-X2-~-'_-y-2 lis, \vhere C is an arc of the involute of the
c

circle x::-:-a(eost i-tsint), !J=a(slnl-tcost) \0~t~21t]. 2298. ) (x 2 + y2)2 ds, where C is an arc of the logarithmic spi c ral r ~ ae l1lrp (Ill> 0) froln the point A (0, a) to the point 0 (- 00, 0). 2299. ) (x +y) ds, where C is the right-hand loop of the lemc

niscate r" = a" cos 2<p. 2300. ) (x 1- y) ds, where C is an arc of the curve x = t. c Y = ; ; , z = t ' (0 ~ t ~ 1].

~78

Multiple and Line Integrals

[eh. 7

2301.
X

~ I + d~ + I ' where C is the first turn of the screw-line cx y z


Z=
1

= a cos t, Y = a s j nt,
c

bt . 2302. ~ -V 2g -1- Zl ds, where C is the circle Xl + gl +Zl = a"

x=y.
2303*. Find the area of the lateral surface of the para bolic cylinder g=iX- bounded by the planes z=O, x=O, z=x, y=6. 2304. Find the arc length of the conic screw-line C x=ae t cost, y = ae l sin t, z = ae t from the point 0 (0, 0, 0) to the point A (a, 0, a). 2305. Deterlnine the Blass of the contour of the ellipse ~~ = I, if the linear density 01 it at each point M (~, y) is

:: +

equal to

2306. find the mass of the first turn of the screw-line x = a cos I,

'y ,.

y=asint, z=bt) if the density at each point is equal to the radi us vector of this point. 2307. Determine the coordinates of the centre of gravity of a half-arc of the cycloid
x=a(t-sint), y=a(l-cost)
[O~t~jtl.

2308. Find the moment of inertia, about the z-axis, of the first 1urn of the screw-l ine x ==- a cos t, Y =- a sin t, Z = bt. 2309. With what force will a mass M distributed with uniform density over the circle x 2 -+ y2 == a 2 , Z == 0, act on a Blass nl located at the point A (0, 0, b)?

B. Line Integrals of the Second Type


Evaluate the follo\ving I ine integrals: 2310. ~ (x l -2xg)dx+ (2xy+y2)dy, where AB is an arc of the parabola Y= x 2 from the point A (1, 1) to the point B (2, 4). 2311. ~ (2a- y) dx \. x dy, where C is an arc of the first arch of the cycloid
x=a(t-sint), y=a(l-cost)
AB

which arc runs in the direction of increasing parameter t. 2312. ~ 2xgdx-x!dy taken along different paths emanating
VA

from the coordinate origin 0 (0, 0) and terminating at the point A (2, 1) (Fig .l 03): a) the straight line OmA;

Sec. 9]

Lin~

I nte~ralf;

2i9

b) the parabola OnA, the axis of symnletry of which is the y-axis; c) the parabola OpA, the axis of symmetry of which IS the x-axis; d) the broken line OBA; e) the broken line OCA.

2313. ~ 2xydx +-x'dy as in Problem 2312.


OA

:Y counterclocl{wise.

2314*. th (x+ u) dX-(X-L/) dy 2


x

taken alonQ the circle x 2


.....

y2

y2

==al

cl~ 1) t-----II-~ A (2, 1)

o
Fig. 103

x
B(2,O)

2315. ~ y'dx+x'd!l, where C is the upper half of the ellipse


r:

x ~ a cos t, Y = b sin t traced clocl<wise.


2316.
bi~ector

~ cosy dx- sin xdy taken along the segment AB of the


AH

of the second quadrantal angle,

if the abscIssa at the

point A is 2 and the ordinate of B


2317.

where C is the right-hand loop 01 the lenlnlscate ,z =- 0 COS 2(p traced counterclocl<wise. 2318. Evaluate the line integrals \vith respect to expressions. which are total ditTerentials:
dIll,
2

#X!l(llt~~~:

IS

2.

(2, 3)

(3, oi)

(I. I)

a)

)
( - l~ 2)

xdy+ ydx,

b)

)
(0, I)

xdx+ydy, c) ) (x+y)(dx+dy),
(0, 0)

(2, 1)

d)

(I, 2)

,J

y dx- x dy

y2

(along a path that does not intersect the

x-ax is),

~80

Multiple and Line Integrals


(x, y)

[CIL.7

e)

(~.+)
straight line
(x 2 Y2)

dx-t-dy (along a path that does not intersect the x+y

x-~y=O),

f)

cp (x) dx -\- 'i' (y) dy.

(x", Yl)

2319. Find the antiderivative functions of the integrands and evaluate the integrals:
(a, 0)

a)

~
(-2, -1) (I, 0)

(x t

+ 4xy 8) dx-+ (6X 8y l_5y t) dy,

b)

S
(0, - I )

x ~~=~)~x (the integration path does not intersect the

straight line y = x),


(a, I)

c)

S (x+~;~;)tYdY (the integration

path does not intersect

(I, I)

the straight line y=-x),


(1,1)

d)

(0, 0)

S (v x2x + y2 + Y) dx+ (-V x2Y+ y2 -I- x) dYe


Co~pute

2320.

taken clockwise along the quarter of the ellipse 02+ fi!= 1 that 1ies in the first quadrant. 2321. Show that if f (u) is a continuous function and C is a 'Closed piecewise-smooth contour, then

Xl

y2

1 f (Xl + yl) (x dx+ y dy) = O. c


2322. Find the antiderivative function U if:
a) du=(2x+3y)dx+(3x--4y)dy; b) du = (3x l -2xy y2) dx _(Xl -2xy+ 3y2) dy; c) du = eX - Y [(1 + x -t. y) dx (l-x-y) dy];

d) du= ~

!!JL. x+y+x+y

Sec 9)

Line Integrals

281

Evaluate the line integrals taken along the following space curves: 2323. ~ (y-z)dx+(z-x)dy+(x-y)dz, where C is a tum
c

of the screw-l ine

= a C?S i, f xy=a slnt, l z bt,


=

corresponding to the variation of the parameter t froln 0 to 2n.. 2324. Y dx +- z dy +- x dz, where C is the ci rc1e

p c

J Y = R cos a sin i,

x = J? cos a cos t ,
= const),

l z = R sin a. (a.
x 2 -1- y2 --1Z2 =

traced in the direction of increasing parameter. 2325. ~ xydx+yzdy+zxdz, where OA is an arc of the
0.4

circle

2Rx,

= x,

situated on the side of the xz-plane \vhere !I> O. 2326. Evaluate the line integrals of the total differentials:
(6, .1, 8)

a)
b)

~
(1,0, -3) (cr, b. c)

xdx-'r-ydy-zdz. yzdx+zxdy-i-xydz,
x cl~ I_-_~~ , Y .\2 _1_ y2 -1- Z2

~
(1, 1, 1)

(:s, -I, s)

c)

(0, 0, 0)

( x.

1/,

d)

2..)
Xli

liZ dx + Zt dr/-I- xy dz (the

xyz

integration pa th is si tun teo

(1. 1,1)

i 11 the fi rs t oc tan t).

C. Green's Fornlula
2327. Using Green's forlnula, transfOrlTI the line integral l=p1lx2 -1 y 2dx+y[xy+ln(x-+ VX 2+y2)] dy,
C

where the contour C bounds the region S.

282

Multiple and Line Infellfals

[Ch.7

2328. Applying Green's formula, evaluate

J=

ff 2 (r -t- if) dx + (x +y)Z dy, c ff _xZy dx + xyz dy, e

\vhere C is the contour of a triangle (traced in the positive direction) with vert ices at the points A (1, 1), B (2, 2) and C (1, 3). Verify the re~ult ob1ained by conlputin~ the inteeral directly. 2329. Applying Green's forlTIula, evaluate the inLcgral

where C is the circle x 2 + y" = R 2 traced counterclockwise. 2330. A parabola AmB, whose aXIs is the y-axIs and whose chord is AnB, is drawn through the points A (1, 0) and B (2,3). Find (x + y) dx-(x-y) dy directly and by applying Green's

ff

AmBnA

fornlula. 2331. Find

~ eXY[yZdx 1-(1 +xy)dy], if the points A and B


AmB

lie on the x-axis, while the area, bounded by the integration path AnlB and the segrr~ent AB, is equal to S. 2332*. Evaluate tI; x dl~-!l2d!. Consider t\VO cases: +b a) when the origin is outside the contour C, b) when the contour enrircl(\s the origin n times. 2333**. Show that If C is a closed curve, then

l ).

ff cos (X, c
I =

n)ds=O,

-where s is the arc length and n is the outer norl11al. 2334. Applying Green's formula, find the value of the integral

ff [x cos (X, n) +- y sin (X, e


::f x+y ,
c
~dx-dy

n)] ds,

\vhere ds is the differential of the arc and n is the outer normal to lhe contour C. 2335*. Evaluate the integral

taken along the contour of a square with vertices at the points A (1,0). B(O, I), C(-l, 0) and D(O, -1), provided the contour is tra<.:ea countercl()~kwi~e.

Sec. 9]

Line integrals

28~

D. Applications of the Line Integral

Eva1 uate the areas of figures bounded by the following curves: 2336. l'he ellipse x=aLost, y=bsint. 2337. The astroid x=acos 3 t, y-==asin 3 t. 2338. The cardioid x=a (2cost--cos2t), y=a (2sintsin 2/). 2339*. A loop of the folium of Descartes x 3 +y:l-3.2xy=O

(a> 0).

2340. The curve (x -t- y)3 = axy. 2341*. A circle of radius r is rolling \vithout sliding along a fixed circle of radius R and outside it. Assuming that ~ is an integer, find the area bounded by the curve (epicycloid) described by some point of the nlc>villg circle. Analyze the particular case of r == R (cardioid). 2342*. A circle of radius r is rolling without s1idin~ along a fixed circle of radius R and inside It. Assunling that !i , is an integer, find the area bounded by the curve (hypocycloid) described by some point of the nloving circle. Analyze the particular case when , = ~ (astroid). 2343. A field is generated by a force of constant nlagnitude F in the positive x-direction Find th(."\ \vork that the fIeld does when a Inaterial poi I1t traces clock\V1Se a quarter of the circle x 2 -t- y2 == RJ. I yi ng i 11 the first quadra nt. 2344. Find the \vork done by the forLe of rravity when a material point of nlass nl is Il10ved lrOll1 position ,1 (xl' !it' z,). to position B (x 2 , Y2' Z2) (the z-axis is directed 'vertically upwards). 2345. Find the work done by an elastic force dIrected towards the coordinate origin If the l11agnltudc of the force is propJrtional to the distance of the point flOIll the origlll and if the point of application of the force traces counterclock\vise a quarter of

the ellipse ~+~~= 1 Iyin~ in the first quadrant. 2346. Find t:le p)tential rUl1~tion of a for~c R {X, Y, Z} and deternline the work don~ by the force over a given path if: a) X=O, Y=-=O. Z-=-I1Zg (force of gravity) and the material point is nl0ved fronl p0sitiol1 A (Xl' y" Zl) to position Z=- , where J1 = canst and , =-= V Xl + y" + z" (Newton attractive force) and the material point moves from position A (a, b, c) to infinity;
~-~--=.-

B (x~p Y2' zs); b) X ~ _~t:, r

y = _Jl~. ,

tt:_,

~84

Multiple and Line Integrals


c)

[Ch.7

X = - k 2 x, Y = _k 2 y, Z = - k 2 z, where k = canst (elastic force), and the initial point of the path is located on the sphere 2 Xl + y2 + Z2 = R , while the terminal point is located on the sphere
x2

+ + Z2 =
y2

r 2 (R

> r).

Sec. 10. Surface Integrals


to. Surface integral of the first type. Let f (x, lj, z) be a continuolls function and z=-=q> (x, y) a smooth surface S. The surface integral of the fIrst type is the liruit of the integral surn
n

SS f (x, Y, z) dS = lim .~ f (Xi, S n-+oot=J

Yi' Z,) Mi.

where ~Si is the area of the Ith element of the surface S, the point (xi, Y P Zi) belongs to this elelnent, and the mctximum diameter of elements of partition tends to zero. The value of this inte~ral is not dependent on the choice of side of the surface S over which the integration is performed. If a projection a of the surface S on the xy-plane i~ single-valued, that is, every strdight line parallel to the z-axis intersects the surface S at only one point, then the appropriate surface integral of the first type Inay be calcula ted froln the formula

SSf (x,
S

y, z) dS

=- SSf [x,
(0)

y, q> (x, y)]

VI + cp: (x, y) + CP~' (x, y) dx dy.

Example t. COlnpute the surface integral

H s
H
00 J 1 J 1

(x+y+z)dS,

where S is the surface of the cube O~x~l, O~y~l, O~z~l. Let us comput(3 the sum of the surface integrals over the upper edge of the cube (z = 1) and over the lower edge uf the cuue (z = 0):
(x+y+ 1) dx dy+

SS (x+ y) dxdy=
00

H
00

11

(2x+2y+ 1) dxdy=3.

The desired surface integral is obviously three tilnes greater and equal to

~~
s R = R (x, y,

(x

+ y + z) dS = 9,

2. Surface integral of the second type. If P = P (x, !I, z), Q = Q (x, lj, z), z) are continuous functions and S+ is a side of the snJooth surface S characterized by the direchon of the normal n {cos Cl, cos ~, cos y}. (hen 1he correspondinf..! surface tntegral of the second type is expressed a~ follows:

S~ P dydz+ Q dz dx+ R dx dy=


s+

H
s

(P cosa

+Q cos ~ + R cos V) dS.

Sec. 10]

Surface Integrals

285

When we pass to the other side, S-, of the surface, this integral reverses sign. If the surface S is represented implicitl y, F {X, y, z) = 0, then the direction cosines of the nonnal of th is surface are determ ~ned fron] the form ulas 1 1 iJF 1 cos a = 15 ax ' cos ~ = D ay , cos y = D oz '

of

iJF

where D=
\v ith

Y(~~r+(~:r+(~~y.

1 c

and the choice of sign before the radical should be brought into agreemenf the si de of the surf ace S. 3. Stokes' formula. If the functions P = P (x, y, z), Q = Q (x, y, z), R == R (x, y, z) are continuously differentiable and C is a closed contour bounding a hvo-sided surface S, \ve then have the Stokes' fornzula
P dx

+ Q dy + I? dz =

=55 [(iJR - iJ s
iJy

iJz

Q ) cosa

Q + (ap cos ~ + (a - iJP) cos v1 dS az _ aR) 0'" ax iJy ,

\vhere cos a, cos~, cos V are the direction cosines of the norrnal to the surface S, and the dircctton of the nonna) is defined so that on the side of the nornlal the contour S IS traced counterclockwise (In a right-handed coordinate
~)steln).

Evaluate the follo\ving surface integrals of the first type: 2347. ~ ~ (x' -+ y', dS, where S is the sphere x + y2 -1- Z2 = a 2
~

cone 2-1-'2--l2=-=0 [O~z~b). Evaluate the follo\ving surface integrals of the second type:
(I
(4 ')

2348. ~ ~ V x 2 -I- y' dS where S is the lateral surface of the s ~\ 2 t/!' z'.! __

2349.
y = 0,
Xl

side of the surface of a tetrahedron bounded by the planes x = 0, Z = O. x -I- y -t- z =-.:= a. 2350. zdxdy, where S is the external side of the ellipsoid s

H yzdydz+xzdzdx-l-x!Jdxd!J, s

where S is the external

H
Z2

Qi-I- b2+ CZ = 1.
2351.

yl

side of the surface of the henlisphere x" + !It. + Zl = a 2 (z ~ 0). 2352. Fi nd the l1lass ot the surface of the cu be 0 ~ x ~ 1 o ~ y ~ 1, 0 ~ z ~ 1, if the surface density at the point M (x, y, z) is equal to xyz.
J

H s

Xl

dy dz

-+ yl dz dx + Zl dx dy,

where S is the external

286

Multiple and Line Integrals

[Ch.7

2353. Determi ne the coordi nates of the centre of gravity of a homogeneous parabolic envelope az=x' +y2(0~z~a). 2354. Find the mOinent of inertia of a part of the lateral surface of the cone z = V x 2 + y2 [0 ~ Z ~ h] about the z-axis. 2355. Applying Stokes' formula, transform the integrals:
a) p (Xl - yz) dx (yl - zx) dy c b) y dx + z dy -+- x dz. c

+ (Zl -

xy) dz;

:ft

Applying Stokes' formula, find the given inte~rals and verify the results by direct calculations: 2356. (y + z) dx + (z + x) dy + (x + y) dz, where C is the circle c x Z+y2+ z2=a 2, X-i"-Y+z==O.

:I

2357.

:I (y-z)dx+ (z-x)dy+<x-y) dz, where C is the ellipse c


Xl

-1- yZ = 1,

x -1- z = 1.

2358.pxdx+(x+Y)dy+(x+y+z)dz, where C is the curve c x=asint, y=acost, z=a(sint+cost)[O~t~2Jt]. 2359. ,cP y 2dx-t z2dy-t-x 2dz, where ABCA is the contour of
ARf;.4

ABC with vertices A (a, 0, 0), 8 (0, U, 0), C (0, 0, a). 2360. In what case is the line integral 1=:1 Pdx+Qdy+Rdz c

over any closed contour C equal to zero?


Sec. 11. The Ostrogradsky-Gauss Formula
If S is a clo'ied smooth surface bounding the volume V, and P -= P (x, y, z), Q = Q (x, Ij, z), R = R (\', y, z) dre functions that are continuous togpther with th-'ir first partial denvatlves in the closed region V, then we have the OstrogrlJdsky-Gauss forlnula

SS (P cos a+ Q cos ~ + R cos


oS

y) dS=

55S(::
(V)

~~ + ~:) dxdydz,

surrac~

where ers a, cos S

p.

cos yare the direction cosines of the outer normal to the

Applying the Ostrogradsky-Gauss formula, transform the following surface Integrals over the closed surfaces S bounding the
\

;ec. 11]

The Ostro[!radsky-Gauss Formula

287

/olume V (l'osa, cos~, cos)' are direction cosines of the outer lormal to the surface S).
2361.

H xy dx dy -I- yz dy dz -+ zx dz dx. s
Xl

2362. ))
~

dy dz

+ yl dz dx -I- Zl dxdy.
Z2

2363.

J. S

rIll xc('sa+~~ ~~c()S Y dS. V + y2 +


x2

2364.

55 (~cosa + :~ cos ~ + ~cosy) dS. s

Usin~ the Ostrogradsky-Oauss formula, compute the following surface integra Is:
Xl dy dz -I- yl dz dx + Zl dx dy, where S is the external s side of the surface of the cube O~x~a, O~y~c, O~z~a. 2366. ~~xdydz+ydzdx+zdxdy. where S is the external side s of a pyranlid bounded by the surfaces x-+-y-l-z=a, x=O, y=O, z =- O. 2367. ~ ~ Xl dy dz -1- yl dz dx = Zl dx dy, where S is the external

2~63. ~ ~

side of the sphere x!_t-y2--1-z2=a2. 2368 ~ ~ (Xl cos a -j yl cos ~ -I- Zl cos y) dS, where S is the exters na I total surface of the cone
[O~z~b).

236ft Prove that if S is a closed surface and direction, then

is any fixed

H cos (n, I) dS = 0, s

where n i5 the outer nOftTIal to the surface S. 2370. I)rove that the volunle of the solid V bounded by the sL1rface S is equal to

V = -}

s where (~oSC, cosp, cosy are the direction cosines of the outer normal to the surface S.

55(x cos a + y cos ~ + z cos v) dS,

28R

Multiple and Litle Integrals

[Ch, 7

Sec. 12. Fundamentals of Field Theory


10. Scalar and vector flelds. A scalar field is defined by the scalar function of the point u = f (P) = f (x, y, z), wh~re P (x, y, z) is a point of space. The surfaces f (x, y, z) = C, where C = const, are called level surfaces of the scalar field. A vector field is defined by the vector function of the point a == a (P) == =: a (r), where P is a point of space and r= xi + yJ+ zk is the radius vector of the pOint P. In coordinate form, a~axi+avJ+azk, where ax == ax (x, y, z), ay = ay (x, y, z), and az = at: (x, lj, 2) are projec.tions of the vector a on the coordInate axes. The vector lines (force lines, flow lines) of a vector field are found from the following system of differential equations dx dy dz -=-=ax ay az

A scalar or vector field that does not depend on the time t is called stationary; if it depends on the time, it is called nOttstationary. 2. Gradient. The vector
grad U (P) = ox l-~
vX' vy

au

au. au

ay J + az k.=== \jU,

where \l=i ~ +j:+kO is the Hamiltonian orerator (del, or nabla), is called the gradient of the field U = f (P) at the given point P (cf. ChI VI, Sec. 6). The gradient is in the direction of the normal tl to the level surface at the roint P and in the direction of Increasing function U, and has length equal to
dz

an

au === -./"'(aU)2 + (aU)2 + {UU):!.


Y
ax
ay

\dz

If thr directIon is given by the unit vector I {cos a, cos ~, cos V}, then

au au au cos~+dz au cosV ar=gradU.I=gradtU=ax cosa+


dy

(the derivative of the function U in the direction I). 3. Divergence and rotation. The divergence of a vector field a (P) == a>.i t. k tl I d iJel x day iJa z ~"7 +ayJ+a IS lescaar Iva=-iJx+ay-+az==:va.
z

The rotation (curl) of a vector field a(P)=axl+ayJ+azk is the vector

rot a=

z _ aay ) l + (aa. _ aaz ) J + (oay _ aa x ) (iJa dy ~ ~ ax ax ay

== \!Xa.

4. Flux of a vector. The flux of a vector field a (P) through a surface S in a direction defined by the unit vector of the normal 11 {cos U, cos p, cos V} to the surface S is the integral

~~ s

an dS =

~~ s

an dS =

SS(ax cos a -I- ay cos ~ -+ az cos y) dS.


s

If S is a closed surface bounding a volume V, and n is a unit vector of the outer normal to the surface S, then the OstrogradskyGaus:s formula holds,

Sec. 12,]

Fundamentals of Field Theory

289

wh ich in vector form is


tan dS= s

H~
(V)

diva dxdy dz.

5. Circulation of a vector, the work of a fteld. The line ultegrai of the vector a along the curve C is defined by thl: formula

~ adr= ~ Qsds= ~ axdx+Qvdy+azdz


C C C

(I)

and represents the work done by the field a along the curve C (as is the projection of the vector a on the tangent to C). If C is closed, then the line integral (I) is called the clrculuilon of the vector field a around the contour C. If the closed curve C bounds a twosided surface S, then Stokes' IOrtrlula holds, which in vector form has the form

1 a dr= ~ ~ c s

n rot adS,

where n is the vector of the normal to the surface S; the direction of the vfctor should be chosen so that for an observer looking in the direction of n thf circulation of the contour C should be counterclock\vise in a right-handed coordinate systelll. 6. Potential and solenoidal fields. The vector tiel d a (r) is called potentlal if a=grad U. where U =1 (r) is a scalar function (the potellttu/ of the field). For the potentiality of a field a, given in a sirnpl yconnected domain, it is necessary and sufficient that it be non rotational, that is. rot a=O. In that case there exists a potential U defined by the equation
dU = ax dx + a v dy + a z dz.

If the potential V is a single-valued function, then ~ a dr


AB

= V (B)-V (A);

in particular. the circulation of the vector a is equal to zero:

A vector field a (r) is called solenoidal if at each point of the field div a =0; in this case the flux of the vector through any closed surface is zero. If the field is at the same tinle potential and solenoidal, then div (grad U) ==-0 and the potential function U is harnlonic; that is, it satisfies the Laplace 2 82U a u a2 u a2 a2 a2 equation O~I + ay 2 + az" =0, or tlU =0, where tl = V2 = ch,a+ ay2+ az 2 is the Laplacian opera"tor

1 c

a dr=O.

2371. Determine the level surfaces of the scalar field U = f (r)t -= y 2 -t-Z 2 What will the level surfaces be of a field U = F (Q), where Q = y2?

where

r Vx +

Vx. +

10-1900

290

Multiple and Line Integrals

(Ch. 7

2372. Determine the level surfaces ot the scalar field

.. r z U = arc sin y x2 + y2
2373. Show that straight lines parallel to a vector c are the vector lines of a vector field a (P) = C, where C is a constant vector. 2374. Find the vector lines of the field a = - royl-~ roxj, where C1l is a constant. 2375. Derive the formulas: a) grad(C I U+C 2 V)=C 1 gradU+C a gradV, where C1 and C1 are constants; b) grad (UV) = U grad V -t- V grad U; c) grad (V 2 ) = 2U grad U; grad V . d) gra d ( V - V grad U -U V2 , e) grad cp (V) = (P' (U) grad U. 2376. Find the magnitude and the direction of the gradient of the field U = Xl + y3 --1- Z3 - 3xyz at the poi nt A (2, 1, I). Determine at what points the gradient of the field is perpendicular to the z-axis and at \vhat poi nts it is equal to zero. 2377. Evaluate grad U, if U is equal, respectively, to: a) r, b) r', c) J., d) f (r) (r = V x 2 -t- y2 + Z2). r 2378. Find the gradient of the scalar field U = cr, where c is 8 constant vector. What will the level surfaces be of this field, and what will their position be relative to the vector c? 2379. Find the derivative of the function U = a + Y -; +:;. at a c b given point P (x, y, z) in the direction of the radius vector r of this point. lIn what case will this derivative be equal to the magnitude of the gradient? 2380. Find the derivative of the function U = ~ in the dir rection of I {cos a, cos~, cos V}. In what case will this derivative be equal to zero? 2381. Derive the formulas: a) div (C1a l + C2 a 2 ) = C l div a l + C2 div a 2 , where Cl and Cz are constants; b) div (Uc) = grad U c, where c is a constant vector; c) div(Ua)=grad Ua+ Udiva. 2382. Evaluate div (

l:!-) _

x:

f) .

2383. Find diva for the central vector field a (P) = f (r)!... where r = VXl + y2 + Z2. r

Sec. 12]

Fundamentals of Field Theory

291

2384. Derive the formulas: a 2) = C1 rot at + C,. rot aI' where C 1 and C2 are a) rot (Cta, + C 2 consta nts; . b) rot (Uc) = grad U c, where c is a constant vector; c) rot (Va) =grad Ua + U rot a. 2385. Evaluate the divergence and the rotation of the vector a if a is, respectively, equal to: a) r; b) rc and c) f (r) c, \vhere c is a constant vector. 2386. Find the divergence and rotation of the field of linear velocities of the points of a solid rotating counterclockwise with constant angular velocity (0 about the z-axis. 2387: Evaluate the rotation of a field of linear velocities ttJ = (t}r of the points of a body rotating with constant angular velocity ro about some axis passing through the coordinate origin. 2388. Evaluate the divergence and rotation of the gradient of the scalar field U. 2389. Prove that div(rota)=O. 2390. Using the Ostrogradsky-Gauss theorem, prove that the flux of the vector a = r through a closed surface bounding an arbitrary volume v is equal to three times the volume. 2391. Find the flux of the vector r through the total surface of the cylinder X2+y2~R2, O~z~H. 2392. Find the flux of the vector a=x 3 i+y3j-t-z sk through: x 2 + 11 2 Z2 a) the lateral surface of the cone ~ ~ H2' 0 ~ z ~ H; b) the total surface of the cone. 2393*. Evaluate the divergence and the tlux of an attractive force F = - r of a point of mass ln, located at the coordinate origin, through an arbitrary closed surface surrounding this point. 2394. Evaluate the line integral of a vector r around one turn of the screw-l ine x = R cos t; Y = R sin t; z = hi from t = 0 to t = 231. 2395. Using Stokes' theorem, evaluate the circulation of the vector a == X 2 y 3; + i+zk along the circumference x2 + y2 = R2~ Z == 0, taking the hemisphere z == VR 2 _x2 - y2 for the surface. 2396. Show that if a force F is central, that is, it is directed to\vards a fixed point 0 and depends only on the distance r from this point: F=f(r)r, where f(r) is a single-valued continuous function, then the field is a potential field. Find the potential U of the fiel d. 2397. Find the potential U of a gravitational field generated by a material point of mass nz located at the origin of coordinates: a = - ~ r. Show that the potential U satisfies the Laplace equation ~U = o.

Ill:

10*

292

Multiple and Line Integrals

[eh. 7

2398. Find out whether the given ,vector field has a potential U. and find U if the potential exists: a) a= (5xy-4xy) I + (3x l -2y)j; b) a=yzi+zxj+xyk; c) a = (y+ z) 1+ (x+ z)j+ (x + y) k. 2399. Prove that the central space field a = f (r) r will be solenoidal only when f (r) = r~ t where k is constant. 2400. Will the vector field a = r (c x r) be solenoidal (where c 15 a constant vector)?

Chapter VIII

SERIES

Sec. 1. Number Series


It). Fundamental concepts. A number series
~

al +a2 + +an + = ~ an
n=1

(1)

is called convergent if its partial sum

Sn=al+ a2+
has a finite limit as
n --+ 00.

.. +an
=
n -+

The quantity S

lim Sn is then called the


00

Sllln

of the series, \vhile the number

Rn == S-Sn ==a n + 1 + all + 2

+...
n~ lX)

is called the remaInder of the series. If the limit lim Sn does not exist (or is infinite), the series is then called divergent. If a series converges, then Iirn an = 0 (necessary condition for convergence).
n-+<r>

The converse is not true. For conv~rgencc of the series (I) it i-; necessary and sufficient that for any positive nUI11ber e it be possible to choose an N such that for n > lV and for any posi tivE' p the following inequality is fulfilled:

\a n +l +a n + 2 + ... +an+pl <8


(Cauchy's test). The convergence or divergence of a serit's is not violated if we add or subtract a finite nunlber of its terms. 20 Test~ of convergence and divergence of positive series. a) Comparison test I. If 0 ~ all ~ bn after a certain n = no, and the series
00

bl +b 2

+ +bn + = n=1 ~ bn

(2)

converges, then the series (1) also converges. If the series (J) diverges, then (2) diverges as well. It is convenient, for purposes of cOlnparing series, to take a geometric progression:
{a 0).

294

Series

(Ch.8

which converges for I q I < 1 and diverges for I q I ;=:: 1, and the harrnontc sert es 00 1

L. n'

\vhich is a divergent series. Example t. The series 1 1 1 t:2 + 2.2 2 + 3.2 3 -1- ... converges, since he re
1 atZ == n2 n

+ 11 2n +
1

< 2'i'

while the geometric progression

' IS q = 2 1 ' converges. whose ra t 10


Example 2. The series

In2+~+ 2 3 ... +~+ 11


d iv~rges, sInce t I s genera I t ertn In n.IS grea t er th an th e correspon d ing ternl

.!.11

of the hannonic series (which diverges).

b) Comparison test II. If there exists a finite and nonzero linlit Hln an H ~ 'X btl (in particular, if an "- bn), then the series (I) and (2) converge or diverge at the sanle time. Example 3. The series
1
1 1 1

+ 73 + "5 + . + 2n-1 ~- ..

o Iverges,

SI nce

n -+

(1 1) =-:1=0 1 1 1m --:2n -1 n 2 '


aJ

whereas a series \vith general term


Example 4. The series

.-!.n

diverges. 1

1 1 1 2-1 +2 2 -2 +2 3 -3 converges, since


n -+

+ ... +2 n -n + ...
I

lim
QO

(_1_ ..-!.-) =1 2" - n 2


n

I e ,. .

while a series with general terln

cOllverges.

Sec. J1

Number Series

295

c) D' Alembert's test. Let an

a limit

>0
(X)

(after a certain n) and let there be

lim an+t=q.
n -+ an

it is not known whether the series is convergent or not. Example 5. Test the convergence of the series

Then the series (1) converges if q < 1, and diverges if q > 1. If q = 1, then
1 3 5

2" + 22 + 23 + + ~ + ..
Solution. Here,

2n-1

a,,=--pr-'
and lint an + t =
t1 -.

2n-l

0n+l= 2n + 1

2n+ 1

lim
t1 -. 00

atl

2n ~- I (2n - 1)

(2n -t- 1) 2

- hnt - - = 2 n-.oo 1-.!2 '2n


be a Ii mif

I + 2n

Hence, the given series converges. d) Cauchy's test. Let a" ~ 0 (after a certa in n) and let there
rz-.::sJ V

lim

n/

a =q.
tz

Then (1) converges if q < 1, and diverJ.!:es if q > 1. When q= 1, the question of the convergence of the seri~s relnains open. c) Cauchy's integral test. If atl = f (n), where the function f (x) is posi ti ve, rnollot0111Cally decreasin~ and continuous for x:;:::a~l, the series (I) and the integral
(X)

~ f (x) dx
a

converge or diverge at the saine time. By l11eans of the integral test it may be proved that the Dtriclllet sertes
(3)
convcn~es

if p > 1, and diverges if p ~ I. The convergence of a large nllIuber of senes l11ay be tested by conlparing \vith the corresponding Dirichlet series (~3) Example 6. Test the following series for convergence

r:2 +3 4 + 5 6 + .. + (-2'-1--1)-2-n +
Solution. We have

an :.:: -(2-n---1 )-2-/~

1
1-2n

1
'

4n 2 - - 1 - 4n 2

296

Series

(Ch.8

Since the Dirichlet series conver~es for p=2, it follows that on the basis of comparison test I I we can say that the given series likewise converges. 3. Tests for convergence of alternating series. If a series

I aJ '+' a2 1+ ... + I an 1+ ... , (4) composed of the absolute values of the terms of the series (I)t conver~es, then (1) also converges and is called absolutely converJIent. Bu t if (1) converges and (4) divE:'rges t then the series (1) is called conditionally (not absolutely) convergent. For investigating the absolute convergence of the series (l)t we can make use (for the series (4)] of the familiar convergence tests of positive series. For instance t (1) converges absolutely if
lilll t1 n + 1 n -+ (X) an

I
(X)

I<

1 or

n -+

lim
(Xl

;/1 a I <
ri

1.

In the general case, the divergence of (1) does not follow from the diver an , > It then not only does gence of (4). But if Jim an +1 I > 1 or lim
n -+

an

n -+

VI

(X)

(4) diverge but the series (1) does also. Lei bniz test If for the aIterna t i ng series
bt -b 2

+ ba-b~+...

(btl ~O)
n
-+
~

(5)
lim b'J = 0,

the following conditions are fulfilled: 1) b 1 ~ b2 ~ b,~ ; 2)

then (5) converges. In this case t for the remainder of the series Rn the evaluation holds. Example 7. Test for convergence the series
1this series:
IRn'~bn+1

2)2 - (3)3 n )n "5 + (4)4 7" + ... +(-1) ~ (211,-1 + ... (3


2

Solution. Let us form a series of the absolute values ot


1+

th~

terms of

, (3" 2)2 + (3)' (4)4 (n)n "5 + -::;- + .. + 2n -- 1 +

Since

lim
n -+ (Xl

V(_n_)n = 211, - 1

n -+

lim _11,_= Jim _l_=~ ex> 2n - 1 11 -+ ~ 2 _ ~ 2'


n

the serieR converges absolutely. Example 8. The series

l-~+~- ... +<_l)n+I.*+ ...


converges, since the conditions of the Leibniz test are iulfilled. This series converges condi tionally, since the series I I 1

1+"2'-3++/1+

diverges (harmonic series).

Sec

I)

Number Series

297

Note. For the convergence of an alternating series it is not sufficient that Its general term should tend to zero. The Leibniz test only states that an alternating series converges if the absolute value of its general term tends to zero monotonically. Thus, for example, the series

1-5"+"2- 52 +"3- ... +k- 5k + ...


diverges despite the fact that its general term tends to zero (here, of course, the monotonic variation of the absolute value of the general term has been violated). Indeed, here, S2k=S~+S~, where

Sk=1+ 2

11 1 I) + 3 ++ k1, Sk=- (1 5+S;++5


II

'

and tiln S~ =
k -.
7J

00

(S~ is a partial SUITI of the harrrtonic series), \vhereas the


Of

limit litll S~ exists and is finite (S~ is a partial sum


k -.
Xl

the convergent geo-

metric progression), hence, lim


k -+
00

S2k = 00.

On the other hand, the Leibniz test is not necessary for the convergence of an alternating series: an alternating series may converge if the absolute value of its general ternl tends to zero in nonmonotonic fashion Thus, the senes

1 1 1 I - 22 + 3i - 4i + ...

+ (2n -

1)3

converges (and it converges absolutely), although the Leibniz test is not fulfilled: though the absolute value of the general term of the series tends to zero, it does not do so monotonically. 4. Series with complex terms A series with the general term cn =an
00

-1- Ib n (i2 == - I) converges if, and only if, the series with real terms ~ an
n=l
:')

and ~ b'l converge at the same time; in this case


11=1
00 00
r:IJ

n=l

C"

= ~ an + i ~ bn .
"=J

(6)

n=1

The series (6) defini tely converges a nd is called absolutely convergent, if the series

I1=J

~ I en I = ~ V a~ + b~l'
n=J

whose trrlllS are the moduli of the terms of the series (6), converges. 50. Operations on series. a) A convergent series may be multiplied terlnwise by any number k; that lc), if

a,+ a +... +an + ... =S,


2

then
ka.

-t- ka 2 +... + ka n + ... = kS.

298

Series
b) By the sum (diUerence) of two convergent series

(Ch.8

a J +a 2 + +a n =8 1 , bl +b 2 + +b,J + ... =8 2

+..

(7) (8)

we mean a series
(at btl (a 2 b2) (an bn ) -1- = 8 1 c) The product of the series (7) and (8) is the serifs
c1

+ +
2

S2
(9)

+ c + ... + cn + ,

\vhere cn=albn+afGbn_t+ ... +anbl(n=l, 2, ). If the series (7) and (8) converge absolutely, then the series (9) also converges absolutely and has a sunl equal to SIS2. d) If a series converges absolutely, its SUITI remains unchanged \vhrn the terms of the series are rearranged. This property is absent if the series COIlverges conditionally.

Write the simplest formula of the nth term of the series using the indicated terms:
2401.1-+-3"-1- 5 2402.
1 1 1 I I +7 + ... 1 1

2+4+6+ 8+
234

2403. 1+2+4+8+ ... 2407. 2" ..t- "6


1 I 1 I 1 I +12 -1- 20 ,- 30 + 42 +
I

.
...

13 135 1.357 2408. 1+r:4+1.4.7+1.4.7.10i -

2409. 1- 1 1- 1 - 1 -t 1 - 1 -~-

2410. 1+ .~ +3+++5+~ +3n-2

.
2414. an =[3+(_1) /J J".
( . 2+8111 1

In Problems 2411-2415 it is required to write the first 4 or 5 terIllS of the series on the basis of the known general term an.
2411. an =n 2 +1 2412. (-l)nn -2n 2-t-(-I)n

2413 an =

2415. alJ=

nn) cos nrt

n!

n2

Test the following series for convergence by applying the cOlnparison tests (or the necessary condition): 2416. 1-1 + 1-1 + ... +(_I)n-J + ...
n

~417. 5" + 2" 5"

(2)2 +"3 1 (2)3 I (2)n 5 + ... + Ii "5 +..

S c. I]

Series - - - - - - - - -Numb:!r --- - - - - - - - - - -299 -

24 t 8. 3" -1- 5 -:1-. 7" 2419. 2420.

+ .. + 2n t- 1 -t- (_1)" r 1 1 1 VTo- VlO + Vllj- -j- VTO + ..


l
I

n+l

'1+

1 1 1 2+4+6+

1 -l-2n

2421.

h+~ +~I +
y

-+- ... 1 +.10/1 +1 +


1l

...
f 11

2422. ) 2423. 2424.

+ V~+.)34 + ., -I-. I + 23 . r (Il + 1) 2 2 2 2 -J- 2 -~- 3 -1- -r- n + 1-+- ';_+'; + ... + .;-_+ ... 2 3
12
f
2 3

f tt

1 ,1 2425. 22 -r 52

+ 82 -1- . + (:3n _

1
1)2

-~

I' 1 2426. 'T -J- ---V -J- ~ -~...


~3

V2

V3
f

. +- - . ; r -1(Il 1- 1) r 11
follo\vin;~

VIl

\ Using d'Alenlbert's test, test the


gence:
1 t- 3 2427. y"2 - 2

~crles

for conver-

+-2 Y-25 + ,2r1 - 1 .. -r (V2)n


+

,
j

') 8. 2 I 25 + 2 5. 8 1 I 2 5 8 r31l - 1) I 2 4_ -.- -- 15 r:S:-9 -- .. -- 159 (411-3) - - .. l'est for convergence, llsing Ca uchy 's test: 2 ( ;3 ) 2 ( 4 ) 3 ( 11 -1- 1 ) It 2429. T+ :3 5" -f- ... 2n -1 t-.
I

2430.

+ 1 2 )3 3 )' 2 -}- ( 5 + (-8-11 1


....

fl )2/1-1 + .. ... -1- ( 311-1

Test for convergence the posit ive series:


2431. 1 + 2'.
1

+ -3' + . -t- -, -1- .


II

2432. 2433. 2434.


r: 243 n.

3 -1- R + is -t- -}- (n -f- 1)2 2

1 I"

1~4+4\+/1O+ '"+(311-2)\311+1)+''' 1 4 9 11 3 + 9 -t- 19 + + 2n + 1 -t- 1+ 2 + 3 + n '2 5 10 -r n +1-r ...


2
I

2436. 22.33

+32.42 + 42.52 + + (Il + 1)2 (11 +2)2 +

2n,- 1

300

SerifS

(Ch.8

2437 : 2438.

+( ~ ).+( l~ ). + + (3n~ I ) n +..


I I

2439.

2440.
2441_ 2442.

(i)I+~+(l~)t+ ... +G:~:ri+ ... 1 8 27 n' e + e + e' + + en + 2 4 2n 1 +2 +3' + ... + 7 + - . 11 2! 3! nl 2+1 +2 +1 +2'+1 + ---+2 n + 1 + ... 2 4 2n 1 +rr +2f + .. -+ (n-l)! + --.
2 1

1 13 13-5 1-3-5. _. (2n-l) 44 23"4+4.8+4.8.12+-+ 4-8-12 .. _4n

+ ...

2444.

"21+41" +61+ ... +(2n)1 + .


.

(U)2

(21)2

(31)2

(n!)!

2445. 1000 + lOO~:~002 + lOOO11~~.2; 1004 +

10001002-1004 ... (998+ 2n) 14-7 ... (311-2)

2446.

T+ 1.5-9+
1

2-58

-. -

+ 1.5-9

2-5-8

(fin -7) (6n-4)

(an-Il) (8n-7)
1-5 ... (4n -3)

+. _.

2 447 2 2-4-6 + -- + 2-4-6 .. . (4n-4) (4n -2) + -- +1-1l-21 ... (10n-9) + 24 48 ~-I_~+1.11-21+

1.5

- I!

24 49.

3! 5' --(2n -1 ) r - 1-4 14.9 149 n2 1 +1-3-5+ 1.3-5-7-9+ +1-3-5-1-9 (4n-3)

+...

2450_

n=l
00

L arc sin
~.
~

'IJ

(X)

.;._.
,

2455.

n ~2
co

L n lin n I.
J

2451. L sin
n=J

2456_ ~
(X)

n=2

~nln2n

2452.

n=J
(X)

L In ( 1 +{ ) .
2 ,;;

2457_ ~
co

n=a

~n-lnn-lnlnn
n

2453. Lin n
n=1
00

2458.

n:;:;2
~

n'

2454_ ~ 1

~rnn n=2

2459_

L. I + n=l Y n
(n

1)

Sec. 1)
CJ)

Number Series

301
00

2460. 2461. 2462.

L n=1 Yn(n+l)(n+2)
1

2465. 2466. 2467. 2468*.

I: 1 n=2nlnn+Ylnln
n=J
eX)

CJ)

n1 I: nn n=1 1: 2"nl nn
CJ)

n=1
eX)

L n V n 1-

CJ)

YiI-

'1=1

L. 3nil "nl
~

2463. 2464.

n =1 (2n eX)

Vii V- 1) (5 ~ n -1)
cos

~e"nl
n=1
fIn

I: (1n=1

~)

CJ)

2469. Prove that the series ~ '\.~ fiPTiiQIi 1


n=2

1) converges for arbitrary q, if p>l, and rorq>l, if p=l: 2) diverges for arbitrary q, if P < 1, and for q ~ 1, if P = 1. Test for convergence the following alternating series. For convergent series, test for absolute and conditional convergen~e.
2470. 1-3"+5-f- 2n-1
1 1 1 )
(_1)"-1

+ ...

2471.

l-yr2 + Y3- ... + Yn1

(_1)1l-1

+...

2472. 1- 4

+ 91 + 13 5
3

...

2473.1- 7
2474
3

+ (-1)"-1 n"l. + ... (-I)n-I n + 6n-5 + ...


~r

[:2-2.3

+ 3.4- 7
-to 1f + 16 -

L(

l)n-1 2n+1
11 2 +11

n(n+1)

2475. - "2 - 4 2476.

1234

-z-n, (-1 ) 2" -r

234 2 yf-I 3 3-I 4 Y4 - 1

+ Y
2

+ ... +
n+l
(n+l) Vn+l-l ( 211 -t 1) n
311

+(_l)n
3 2477. - 4"
2

+ ...

5) + ( ""1
2 5
1 4

(7 ) a

10

+ ... +(-1 ) n
n_
J

-t- 1

+
2)

2478. ~_3.5+3.5.7_
1

2479. 1-1.9+7.9.11- +(-1)

2 5 . ~ 1 4. 7

+(_I)n-l~~~':+I)+ 2 5 8.. (In - 1)


I 4 7 l.U.1I
(~in -

(2n+5)

+ ...

302

Series

[Ch. 8

00

~(_I)nlnnn. 2481. ~
n=1

2482 ~ ,~( - l)n - It an ..1 r-.


n=1

J' It

2483. Convince yourself that the d' Alembert test for convergence does not decide the question of the convergence of the
(X)

series ~ an' where


1l=1

(k=l, 2, ... ),

whereas by means of the Cauchy test it is possible to establish that this series converges. 2484*. Convince yourself that the Leibniz test cannot be applied to the alternating series a) to d). Find out which of these series diverge, which converge conditionally and which converge absolutely:
a)

Y2~1-Y2+1+Y3-1-Y3+1+Y4( a 2k-l-

Yk+l-l'
2

_
2k--

Yk+l+l '

1).
'

l-Y4+1+

b) 1- 3

1 1 I 1 + 2-33 + 2 -35" + ...

(
1 1

a
1

2k-l -

2k -

_1_

a _ _
2k -

1 '

32k -

1).
J

c) J -3 +~-32 -t-

1 1 5-33
1

+ ..

(a
1 1

zk -

=
1

2k~1'
1
1

2k = -

~k) ;

d) 3-1--1-7-5+TI-g+ ...

( a2k - 1 = 4k l ' a2k = - 4k~3 ) Test the following series with complex terms for convergence:
2485.
2486:
2487.
n=J
00

L n (2+i~~ 2
f7J

00

2488. 2489.
2490.

n=J
f7J

L~'
f7J

L n (2/-1)n
n=1
~

3'Z

I1=J

Ly~ n +I

n;;:l

L n (3~i)n.

n=l

L (n+i) 1 Yn

Sec. 1]
~

Number Series

303

4 1 ~ 1 29. ~[n+(2n-l)i]2.
n=1

2492

n=1

~ [ n (2 - i) + 1 ]
n{3-2i)-3i

II

2493. Between the curves y = da and y =

~ and to the ri~ht

of their point of intersection are constructed segments parallel to the y-axis at an equal distance from each other. Will the sum of the lengths of these segments be finite? 2494. Will the sum of the lengths of the segments mentioned is replaced by the in Problem 2493 be finite if the curve y = -; x curve y==-? x
2495. Form the sum of the series
I

L 1t nand L <-11:- n
n=1
n=J
~

rSJ

r:n

Does this sum converge?

2496. Form the difference of the divergent series ~


(L

n=l

~ 21l-1

and
~

L 2~
II=J

and test it for convergence. Does the series forlned


00

2497.

by subtracting the

series

L
n=l

'ill ~ I from the series

L
ft=-J

~ converge?
\\yh

2498. Choose two series such that their sunl converges their difference diverges. 2499. Form the product of the series

ile

L
Il=J

r:r:

11 Y 11

.~-- and

n=1

L LJ .
2

a;

Does this product converge? 2500. Form the series (1 + ~ +{ + ... + 2 + ... Does this series converge? 1 2501. Given the series 1 -t- 2 , - 3\ + ... + <_:>n + ... Estimate . 11.

LJ

)2.

the error committed when replacing the sum of this series with the sum of the first four ternls, the sum of the first five terms. What can you say about the signs of these errors? 2502*. Estimate the error due to replacing the sum of the series

~ -I- 2 , ( ~ ) -I- ~! ( ~ ) a + ... + ~ ({ )n +...


1
2

by the sum of its first n terms.

304

Series

[Ch. 8

2503. Estimate the error due to replacing the sum of the

5eries
1 + 2! + 3! + ... + ;Ii + by the sum of its first n terms. In particular, estimate the accuracy of such an approximation for n= 10. 2504**. Estimate the error due to replacing the sum of the series
1
1 1 1

+ 2i + 3 + .. -I- n2 +
2

by the sum of its first n terms. In particular, estimate the accuracy of such an approximation for /1, = 1,000. 2505**. Estimate the error due to replacing the sunl of the 5eries 1+2 ( ~ + 3 ( + ... +n ({ + ...

r i- r

yn-I
n

by the sum of its first n terms.


2506. How many terms of the series

L
n=l

00

(_l)n-l

does one have

to take to cOInpute its sum to t\VO decilnal places? to three decimals?


2507. How many terms of the series ~
n=l

~:~

(2n

n + I) sn does one

have to take to compute its sum to two decimal places? to three? to four? I 1 1 2508*. Find the sum of the series r:2~-2.3+3.4 + ... +
1

+n(n+l)ot ..

2509. Find the sum of the series

Vi+(Vx-Vx) +<Vx- Vi)+ ... +(2k+ VX_ 2k - Vx)+ ...


Sec. 2. Functional Series
1. Region of convergenc~. The set of values of the argument x for which the functional series
fl(x)+f2(x)+.+ftl(x)+...
(1)

converges is called the region of convergence of this series. The function S (x) = lim Sn (x),
n-+~

where Sn (x) = f I (x) + f 2 (x) + ... + fn (x), and x belon~s to the region of convergence, is call(lld the surn of the series; Rn (x) = S (x) -Sn (x) is the remainder of the series.

Sec. 21

Functional Series

305

I n the simplest cases, it is sufficient, when determiping the region of convergence of a series (1). to apply to this series certain convergence tests. holding x constant.

Diverges

Converges
i,(1f(~9

Diverges

-J

-1
Fl~.

0
1~4

x
(2)

Example 1. Determine the region of convergence of the series


x I 1 2

+ + (x + 1)2 + (x + 1)1 +
2 . 22 3 2'

...
n

+ (xn+ 1)n + . 2n ..
of the series, we will have
J

Solution. Denoting by u" the general

t~rm

n lim Itln+11_lim Ix+ll + 2 n _lx+ll n-+ etJ I un' - n-+ etJ 2n+ J (n + 1) I x In 2

absolutely), if

Using d' Alembert's test, we can assert that the series converges (and converges I 11< 1, that is, if -3 < x < I; the series diverges, if

X!

IX!Il> I,

that is, if -co

<x<-3 or I <x< co (Fig. 104). When x=1

we get the harmonic series I we have the series

+ + + ... , which diverges, and when X= - 3 -I + + ... , which (in accord with the Leibniz
+C
(x-a)2 + ...

i ii - i2

test) converges (conditionally). Thus, the senes converges when - 3 ~ x < 1. 2. Power series. For any power series
Co

-1- C1 (.\ -

a)

+ Cn (x-a)n + ...

(3)

(C n and a are real nunlbers) there exists an interval (the interval of convergence) I x-a I < R with centre at thr point x= u, with in which the series (3) converges absolutely; for I x-a I> R the series diverges. In special cases, the radius of convergence R may also be equal to 0 and 00. At the end-points of the interval of convergence x=a :1: R, the power series may either converge or diverge. The interval of convergence is ordinarily determined with the help of the d'Alembert or Cauchy tests, by applying them to a series, the terms of which are the absolute values of the terms of the given series (3). Applying to the series of absolute values

I Co I + I c1 II x-a 1+ ... + I cn II x-a In + ...


the convergence tests of d' Alembert and Cauchy, we get, respectivel y, for the radius of convergence of the power series (3), the formulas

R= lim
n~(f)

jYlc" I

and

R=liml~1 n
n-+oCD

+1

However, one must be very careful in using them because the limits on the right frequently do not exist. For exanlple, if an inflniture of coefficients clI

306

Series

(Ch.8

vanishes [as a particular instance, this occurs if the series contains terms with only even or only odd powers of (x-a)], one cannot use these formulas. It is then advisable, when determining the interval of convergence, to apply the d' Alembert or Cauchy tests directly, as was done when \ve investigated the series (2), without resorting to general formulas for the radius of con vergence. If z=x+ ty is a complex variable, then for the po\ver series

CO+c 1 (z-zo)+c 2 (Z-ZO)2+ ... +c n (z-zo)n+...

(4)

(cn=an+ib n, zo=xo+iyo) there exists a certain circle (circle of convergence) I Z-Zo 1< R with centre at the point z=zo' inside which the series conver~e5 absolutely; for I z-zo I> R the series diverges. At points lying on the cir cumfprence of the circle of convergence, the series (4) may both converge and diverge. It is custoInary to determine the circle of convergence by nlcans of the d' Alembert or Cauchy tests a pplied to the series

I Co I + I ct '1 z-zo I+ I c2 11 Z-Zo 12 +... + I en 11 z-zo ,n+ ... ,


whose terms are absolute values of the tenns of the given series. Thu~, for exaInple, by means of the d' Alelnbert test it is easy to see that the Circle of convergence of the series

z+ 1 +(Z+ 1)2+(z+ 1)3+ 1.2 2.2 2 3.23

+(Z+ l)n+ n2 rl

is determined by the inequality I z+ 11 < 2 [It is sufficient to repeat the cal~ culations carried out on page 305 which served to deterIlline the interva I of convergence of the series (2), only here x is replaced by z]. The centre of the circle of convergence lies at the point Z === -1, while the radius I? of thl~ circle (the radius of convergence) is equal to 2. 3. Uniform convergence. The functional series (1) converges unifonnly on some interval if, no Inatter \vhat e > 0, it is possible to find an N such that does not depend on x and that when n > N for all x of the given Intervdl we have the inequal ity I Rn (x) I < 8, where RIl (x) is the renlai nder of th~ given series. If Ifn(x)l~cn (n=l, 2, ... ) when a~x~b and the nunlber series
00 '

~ en converges, then the functional series (I) converges on the Interval


n=t

lat h' absolutely and uniformly (Weierstrass' test).

The power series (3) converges absolutely and uniformly on any interval lying within its interval of convergence. The power series (3) may be ternlwise differentiated and integrated within its interval of convergence (for , x-a I < R); that is, if

Co

+c (x-a) +
1

C2

(x-a)!

+ ... +en (x-a)n + ... = f (x),

(5)

then for any x of the interval of convergence of the series (3), we have

c1+ 2c 2 (x-a)
x x x

~ codx + ~ c1 (x-a) dx+ ~ c2


%0
Xo
Ko

+... + nCn (x_a)n-l + ... =1' (x), x (x-a)2 dx+ . + ~ cn (x-a)n dx+ ... =
Xo
00

(6)

= ~ en
n=:o

(x_a)n+t (x o-a)n+l

+1

x S = f (x) dx
Xo

(7)

Sec. 2]

Functional Series

307

(the nunlber X o also belongs to the interval of convergence of. the series (3). I-Iere, the series (6) and (7) have the same interval of convergence as the series (3).

Find the region of convergence of the series:


00

rn

2510.

n=1
00

L
YJ

t~x,
n

2518. 2519. 2520. 2521. 2522. 2523 .

L
"f)

n! x n

'1=1

2511. L(-l)"+I~. X
11=1

11=1
YJ

L (2tl~
. . VOn
"f)

I) X"

~~ 2512. ......
11=1
'Y)

(_1)"+1

_1_ Inx n

IZ .= 1

2513. 2514.

L. sin(~-l~ (211-1)2
11=1

II='J

L (x--2),i' .L. (11 2a+ 1 -t- 1)5


L.
~

X 2"

.L. 2 sln:)ij x .
n '
d

YJ

(_1)"-1

1l:3 1l (x-5)"
11=1
OCJ

n=o

2515**.

L c:;,.\~ .
11=0
Cfj Cfj

'r.

......

~.,~
00

"rz

11=1 X

2516. L.(-l)l1tJe-Il~ln.~.
1l::0

2524*. 2525.

11=1
00

L (XII +2'!X II )
x
n

2517.

11=1

L~ x"

L
11=-1

Find the interval of convergence of the power series and test


the convergence at the end-points of the interval of convergence:
Yl

2526. 2527. 2528. 2529. 2530.

~.., " .... X.


'1=0

2531 . 2532. 2533. 2534. 2535.

L
Y.)

(tl

11 =0
00

+ 1)5 X2n 2n -t- 1


Xr&.

11=1
'J:)

L n2" ~" L
x~n-I

-n

11=0
00

11=1
00

211-1 x - 1 (4n-3)2
XI i .

L (_l)n (2n + 1)2 L n!


x"
11=1
00

L. 2
n=1
00

1 W

L n!x
n==1
00

L(_I):-I
n=l

L'~ nn
n=l

308
2536. ~ 2nl
II=J

Series
~~
co

(Ch.8

)2n-.

+
n1

2551.

n=l
CX)

L(
co

X+5)211-1
~n4n

2537.
2538.

L 3/1 x
2

2552. ~

/1=0

(x- 2)11 ~ (2n-I)2 n

co

~ n~l (~
co

2553.

L (_I)n+1 X
n=l

2539. ~ nlx
n=J

X
co

(2n_I)2n (x_I)n
(3n-2)2fJ

~
ao

nn
lI

2540. ~

11=2

x ~ n3 n lnn

2554. ~ nl (x+ 3)11


n=1
~
co

nn

2555.

2541. ~
n=l

L
n=1
CX)

x U1

(n+ I) In 2 (11+ I)
(x-3)Z"

(x+ I)n

2542**. 2543*. 2544*. 2545. 2546.

L n!
n=1
crJ crJ

2556.
x nl

n=l

L (n+l)ln(n+I)
(_l)n+1

L 2n~n:n" n=J L x::.


n=l n=l

2557.

L
n=J

(x-2)n

X
CIJ

(n-r I) In (n+ 1)

L. (_1)"-1 (x-S)" n3
n

~558.

(x+2)n'
nn

fl.=J

L
'1 ==J
00

co

(X n S

3/

2559*. 2560.

t:1 ( + ~ )"I
CD

(x - 1)".

2547

~ (X_I)211

n=l
GO

ng"

2548.

L
n=l
CD

~561.
2n

i: (n=o
co
ClO

1)"

V~2 X
n

(_1)/1-1 (x_2)zn

+1

X (x-2)n.

2549. ~~ (x+3)n
~
co

n=J

2562.

~ (3n-2) (X-3)1I ~ (n+l)!2/J + 1

2550.

L
n=1

nn (x + 3)n.

2563. L(-I)"
n=0

t2n

+1) Y

(x-3~
II

-t

1.

Sec. 2]

Functional Series

309

Determine the circle of convergence: 2564.

L inzra.
n=o ao
,..,

ex

ao

2566. ,...,

n=l ao

(z-2i)"

n3 n

2565. ~ (1
2568. 2569.

+ ni) zn. 2567. ~ 2n n=o n=o (1 +2i) + (1 + 2i) (3 + 2i) z + ... + + (1 + 2i) (3 +2i) . .. (2n + 1 + 2i) zn -I- . Z Z2 1 + l - i + (I-i) (1-2i) -t-

~ Z211

2570.

n=o

L Cn~2~ir z",

ao

+ (I -

z"
i) (1 - 2i) (I - ni)

2571. Proceedi ng from the definition of uniform convergence, prove that the series 1 +x+x2 + ... +xn + ... does not converge uniformly in the interval (-1, 1), but converges uniformly on any subinterval within this interval.
Solution. Using the fortnula for the sum of a geometric progression, we get, for I x I < 1, xn + 1 Rn' (t) -X'I+I + xfJ + 2 + --, -I-x Within the interval (-1, 1) let us take a 5ublnterval {-I +a, I-a), \\'here a is an arbitrarily small positive number. In this subinterval I x I ~ I-a, Il-x'~a and, consequently, (l-a)n+1 I R,. (x) I c=; .
u

To prove the uniform convergence of the given series over the subinterval (-l+a, I-a], it (TIust be shown that for any >0 it is possible to choose an N dependent only on e such that for any II > N we ",'ill have the ine quality I R n (x) 1< e for all x of the subinterval under consideration. (I_a)n+l Taking any e>O, let us require that < e; whence (l-a)1J+ I<ea, a
(n

that is, n + I > h~~l(~~) [since In (I-a) < 0) and In (ea) . In (ea) . n>ln(l_a)-l. Thus, puttIng N In(l-a)-I, we are convinced that when n > N, I Rn (x) I is indeed less than e for all x of the subinterval [-I a, I-a] and the uniform convergence of the given series on any sub interval within the Interval (-I, 1) is thus proved. As for the entire interval (-I, I), it contains points that are arbitrarily . . xn + J closetox=l.andsince hm Rn (x)=lIm -1 -=ClO, no nlatteJ how large n IS.. %-+1 x .... J -x

+ 1) In (I-a) < In (ea),

310

Series

[Ch.8

points x will be found for which Rn (x) is grea ter than any arbi trarily large number Hence, it is impossible to choose an N such that for n > N we would have the inequality I R" (x) 1< 8 at all points of the interval (-1, 1), and this means that the convergence of the series in the interval (-1, 1) is not uniform.

2572. Using the definition of uniform convergence, prove that: a) the series

1 +Ti -t- 21+


x2

x2

.. -1- 11T+ ..
(_l)"-J x ::n

xn

converges uniformly in any finite interval; b) the series


X4
,,\6

T-2"+3- -tconverges
(-1, 1);

+ ...

uniformly

throughout
1 1

the
1

interval of convergence

c) the series
1 ++3x+

.. -t;;X+ ...

converges uniformly in the interval (1 -I-i), (0) \vhere B is any positive nUlnber; d) the series
(X 2 _X 4 )

+ (x<l-x )-t-(X6 _X8) + ... + (X21J_X211+2) +_ ...


8

converges not only within the interval (-1, I), but at the extremities of this interval, however the convergence of the series in (- 1, 1) is nonuniform. Prove the uniform convergence of the functional serIes in the indicated intervals:
2573.

l: ::
n=l
00

CD

on the interval [-I, I].

2574. ~~ sin nx ~ 2n
1l=1
00

over the entire number scale.

2575. ~ (_l)n-J

;nn on the interval


xn

lO,

11.

Applying termwise differentiation and integration, find the .sums of the series: 2576.

x+ 2 +3"+ +n T
x2
x3
Xl
XS

x2

x3

...
n

2577. x- 2

x + 3- .. + (-l)n-l n + ...
x:. n I

2578.

x+3"-rs-l- +2n-l-t- ...

Sec. 8]
Xl

Taylor's Series
x5 x 2n J

311

2579. x- 3 + 5 - .. +(_l)n-J 2n-l 2580. 1+2x-l-3r + ... (n-t-l)x n + ... 2581. 1-3x2 -t- 5x 4 - +(_l)n-J (2n-l)x 2n - 2 + ... 2582. 12-t-23x+34x2 +n(n+ l)x"-I+ ... Find the sums of the series:

+ ..

+ + ...
X 4n _ 3

2583. -+ 2-13+ x x X
XS

11 .. + x n+ ..

x9

2584. x+ 5

-1- 9
1
~

+ + 4n-3 +
1 1 (_l)n-J
J

2585*. 1- 3 . 3 + 5.32-7.3:1+ ... +(2n-l)3 1l 2586. 2+22


1

+ ...

+2"3+ .. +2'-z+ ..

2n-l

Sec. 3. Taylor's Series


to. Expanding a function in a power series. If a function f (x) can beexpanded, in son1e neighbourhood I x-a I < R of the point a, in a series of po\vers of x-a, then this series (called Taylor's series) is of the form
1(.t)-f(a)+1 (a) (x-a)+2t(x-a) ++-n,-(x-a)

._,

r (a)

,(nl (a).

+...

(1)

\Vhen a=-O the Taylor scri~s is also called a Maclaurin's series. Equation (1) holds If \vhen I x-a I < R the rellzainder ternz (or sin1ply remainder) of the Taylor series

RrI (x) = f (x) as n --+ 00. To rva I uate the r C111 ain der,

f (a)
0 ne

L 1 kl(a) (x-a)k1
tl (k)

-+

k=J

can n1 a k e

USC 0 f

the forin uIa

Rn (x) =

(x

(~t-l )1

a)"+J

+ J) [a

+ 0 (x-a),

\vhere 0

<0<1

(2)

(Lagran!e's lo"n). Exanlple t. Expand the function 1(x) = cosh x in a series of powers of x. Solution. We find the derivatives of the given function 1(x) = cosh x. I' (x) =-= SInh x, f" (x)Ul =:: ccsh x, {'" (x) = sinh x, ... ; generally. f(n) (x) = cosh X, if n is even, etnd , ) (x) == sinh x, if n IS odd. Putting a= 0, we get 1(0) = 1, 1'(0)=-=0,1"(0)=-1,1"'(0):.=0, ... ; generally, [<")(0)=1, if n is even, and I(n) (0) =- 0 if n is odd. Whence, from (1), we have:

coshx=1

+21+41+ + (2n)! +...

x2

x4

x2n

(3)

To determine the interval of convergence of the series (3) we apply the: d' Alembert test. We have
n ~ 00 (2n
11111

. I

x +2

2n

+2)1 : (211 )t

,\ 2n

= ltm

n -+ etJ (2n

+1) (211 +2)

312

Serie~

[Ch. 8

for any x. Hence, the series converges in the interval - 00 remainder term, in accord with formula (2), has the form:

< x<

00. The

Rn (x) = (n + 1)! cosh e x, if n is odd, and


..\n+1

xn + 1

Rn (x) =
Since 0>

(n

+ 1)!

sInh ex, if n is even.


le~x_e-~xl

e > 1, it follows that e~x+e-6x I cosh e x 1= 2 ~ e' x I,


Ixl n + 1

I sinh e x 1=

<; el x I,
~,.

and therefore I R n (x) J.,;;;; (n + 1)1 e' x I. A series with the gen~r81 term nl converges for any x (this is made immediately evident with the help of d' AleInbcrt's test); therefore, in accord with the necessary condition for convergence,

. I xl n + 1 11m - - - = 0
lZ -+> 00

(n

+1)!

'

and consequently lim Rn (x) = 0 for any x. This signifies tha t the sum of the
n-+oo sert~s {3) for any x is indeed equal to cosh x. 2~. Techniques employed for expanding in

power series.

Making use of the principal expansions


I.

e =1

+fi+2f+'" + iiT+'"
x
2

x2

xn

(-00

< x<

00), (-00

. x ~ I x5 n II. slnx=n-31+5f- ... +(-I)

x2n + I
(2n+l)I+'"
x
2n

<X<

00),

11 I. cos X == 1 -

21 + 4f - ... + (_I)n (2n)1 + ...


m(m-l) +m TI x + 21 x + .
2

x"

(- 00 < x < 00),


(-I <x<I)*).
(-I<x~I),

IV. (1+>mt-l x -

... +m(m-I).~.!{I1l-n+l)xn+ ...


V
~2

In(l+x)::,:::x-~+3- ... +(-l)n-ln+'"

Xl

xn

-and also the formula for the sunl of a geometric progregsion, it is possible, in many cases, simply to obtain the expansion of a given function in a power series, without having to investigate the remainder term. It is sOllletim('s advisable to make use of termwise differentiation or integration when expanding a function in a series. When expanding rational functions in power 'S~ries it is advisable to decompose these functions into partial fractions.
*) On the boundaries of the interva1 of convergence (i. e., when x = - 1 and x = 1) the expansion 1\9 behaves as follows: for m;:;: 0 it converges abso lutely on both boundaries; for 0 > m > -1 it diverges when x == - 1 and ~onditlonally converges when x = 1; for m E;;-1 it diverges on both boundaries.

Sec. 8]

Tuy/or's Series

313

Example 2. Expand in powers of x *) the function 3

t(X)=(I_x) (1 +2x)'
Solution. Decomposing the function into partial fractions. we will have 1 2

!(X)=I-x+l+2x'
Since
_1_= 1+x+x 2 -l- ... I-x

==,~ xn
~

r:IJ

(4)

n=o

and
I

~2X = 1-2x + (2X)2_ =- L


n=o

tT-

n n (-l) 2 x n ,

(5)

it follo\vs that we finally get


f(x) =- ~ x n +2 ~ (_l)n 2 n x n = ~ [I
Il=O

+ (_1)n 2 n + 1 ] x n .

(6)

n=o

n=o

The geometric progressions (4) and (5) converge, respectively, when I x I < 1 1 1 and I x I < "2 ; hence, formula (6) holds for I x I < 2' i. e., when

-"2<x<"2'
30 Taylor's series for a function of two variables. Expanding a function of two variables f (x, y) into a Taylor's sertes in the neighbourhood of a point (a. b) has the fornl

f (x, y) =f (a,
+(y-b)

b)

+ -it [(x-a) ~ + (y-b) ~] f (a, b) + ~1 [(x-a)


f (a,
b)
IS

tx+

+... + ~ [(x-a) ~+ (y-bl ~r f(a. b) +...


then called a ,Haclaurtn's sertes.
ax (x-a)
y=b
Her~

(7)

If a = b = 0, the Taylor series notation is as follows:

the

a [ (x-a) ax + (y a [

b) ify

a]

I (a, b)
2

at (x, y)
x=a
a
2

+ at (;~

)
y=b

(y-lJ);

x=a

iJJ (x~a)ax+(y-b)ay

f(a,b)=

1(x,

ax"

y)

(x-ar~ -I-

x=a
IJ=b

(x-a)(y-b)
y=b

+ a2f(x, y)
ay!
IJ=b

(y-b)2 and so forth.

x=a

x=a

*) Here and henceforward we mean "in positive integral powers".

~14

Series

[Ch.8

The expansion (7) occurs if the remainder term of the series


Rn(x, y)=f(x, y)- { {(a,b)+ n
8S

1 [ (x-a)ax+(y-b)ay iJ a]k {(a, b) } -+0 L kI


k=1

n --+

00.

The remainder term may be represented in the form


1 [

Rn(x, Y)+(n+l)! (x-a)ax+(y-b)ay

a] n+1 f(x,Y)
x=a+~ (x-a) y=b+'J (y-b)

,v here 0 < e < 1.

Expand the indicated functions in positive integral powers of x, find the intervals of convergence of the resulting series and iJlvestigate the behaviour of their remainders: 2587. aX (a> 0). 2589. cos (x -t- a). . ( 1(; ) 2590. sin! x. 588 2 Sin x +4 2591*. In (2+x). Making use of the principal expansions I-V and a geometric progression, write the expansion, in powers of x, of the follo\ving functions, and indicate the intervals of convergence of the series: 2x-3 2598 cos 2 x. 2592. (x-l)2.
2593. x2-4x + 3 2594. xe- 2X 2595. e
X2

3x-5

2599. si n 3x -t- x cos 3x. 2600. 2601.


x 9+x 2
1

4-;\2

2596. sinh x.

2602. In 11 +x .
-x

2597. cos2x. 2603.ln(l+x-2x 2 ). Applying differentiation, expand the following functions in po\\'ers of x, and indicate the intervals in which these expansions occur: 2604. (l+x)ln(l+x). 2606. arcsinx. 2605. arctanx 2607.ln(x+VI+x 2 ). Applying various techniques, expand the given functions in po\vers of x and indicate the intervals in which these expansions occur: 2 2608. sin2 xcos 2 x. 2612 x - 3x + 1 2 x x -5x+6 2609. (I + x) e- 2613. cosh' x. 2610. (I + eX)I. 1 2614. 4 -4 2611. x. -x

Vs +

Sec 3]

Taylor's Series

315

2615. In (Xl
x

2616.

+ 3x + 2). SSi:X dx .
x
o

2618.

r
x
o

n (I ~X) dx

2617. ~e-x2dx.

2619.

SVI-x". dx
o

Write the first three nonzero terms of the expansion of the following functions in powers of x: 2620. tan x. 2623. sec x. 2621. tanhx. 2624. lncosx. 2622. el.O~X. 2625. eX sin x. 2626*. Sho\v that for computing the length ot an ellipse it is possible to make use of the approxinlate formula
s

~ 2na ( I - ~)

where e is the eccentricity and 2a is the major axis of the ell ipse. 2627. A heavy string hangs, under its o\vn \veight, in a catenary line y == a cosh ~, \vhere a =.!i and H is the horizontal c.l q tension of the string, \vhilc q is the \veight of unit length. Sho\v that for small x, to the order of x\ it 111ay be taken that the string hangs in a parabola y=a 1- 2~. a 2628. Expand the function x 3 -2x 2 -5x-2 in a series of po\vers of x -I 4. 2629. f(x}===5x 3 -4x 2 -3x+2. Expand f(x+h) in a series of powers of h 2630. Expand In x in a series of po\vers of x-I. 2631. Expand J.. in a series of po\vers of x-I. x 2632. Expand -; in a series of po\vers of x + 1. x
2633. Expand 2634. 2635. 2636. 2637. Expand

x-i. 2638. Expand cos x in a series of po\vers of x- T. 2639*. Expand lnx in a series of powers of :+;.
Expand cosx in a series of powers of
2

Expand eX in a series of pqwers of x + 2. Expand Vi in a series of powers of x-4.

X2+~X+2 in a X2+~X+7 in a

series of powers of x series of powers of

+4. x +2.

316

Series

[ell. 8

2640. Expand .. r" in a series of powers of 1 +" , 1+% x 2641. What is the magnitude of the error if we put approximately
e~ 2
1 1 1 + 2f+3f+41?

2642. To what degree of accuracy will we calculate the number ~, if we make use of the series arc tanx=xXl

+5-.'

XS

by taking the sum of its first five terms when X= I? 2643*. Calculate the number ~ to three decimals by expanding the function arc sin x in a series of powers of x (see Example 2606). 2644. How many terms do we have to take of the series

cosx= 1- 21 + ... ,
in order to calculate cos 18 to three decimal places? 2645. How many terms do we have to take of the series slnx=x-3f+ ... , to calculate sin 15 to four decimal places? 2646. How many terms of the series
e
x
Xl

x2

x x =1+ 11 '-21+ ...

have to be taken to find the number e to four .decimal places? 2647. How many terms of the series
In (1 +x) =x-2"+ ... '
x2

do we have to take to calculate In 2 to two decimals? to 3 decimals? 2648. Calculate to two decimals by expanding the function V 8 -l- x in a series of powers of x. 2649. Find out the origin of the approximate formula r Va' +x~a+;a (a>O), evaluate it by means of V 23 , putting a = 5, and estimate the error. 2650. Calculate 19 to three decimals.

vr
V

Sec. 3]

Taglor's Series

317

2651. For what values of x does the approximate formula


cosx~l-

x2

yield an error not exceeding 0.01? O.OOI? O.OOOI? 2652. For what values of x does the approximate formula sin x~x yield an error that does not exceed 0.01? 0.001?
1/2

2653. Evaluate

S-x-dot to
1 1

Sinx

I s. four decllna

2654. Evaluate ~ e- x2 dx to four decimals.


o

2655. Evaluate ~ Vxcosxdx to three decimals.


o

x . I s. 2656. Evaluate .J sin yX dx to three declma


o
1/4

r
1

2657. Evaluate ~
o
lID

VI +x

dx to four decimals.

2658. Evaluate ~ Vi eX dx to three decimals.


o

2659. Expand the function cos (x - y) in a series of powers of x and y, find the region of convergence of the resulting series

and investigate the remainder. WI ite the expansions, in powers of x and y, of the following functions and indicate the regions of convergence of the series: 2660. sinxsiny. 2663*. In(l-x-Y1-xy). 2 266 t. sin (x + gl). ~664"'. arc tan ; + y
2662*. l-x+ It
l+x-y

-xy

2665.

\vers of hand k.

f (x,

y)

= ax" -1- 2bxy + cy". Expand f (x + h,

Y -f k) in po-

x == 1 + h, !I = 2 + k. 2667. Expand the function eX +Y in powers of x-2 and y + 2. 2668. Expand the function sin (x y) in powers of x and

2666. f (x, y) = x' -2y' -1- 3xy. Find the increment Qf this function when passing from the values x= 1, y=2 to the values

1(;

Y-2

318

Series

[Ch.8

Write the first three or four terlTIS of a power-series expansion in x and y of the functions: 2669. eX cos y.
2670. (1 +X)I+Y.

Sec. 4. Fourier Series


to. Dirichlet's theorem. We say that a function f (x) satisfies the Dirichlet conditions in an interval (a, b) if, in this interval, the function 1) is uniformly bounded; that is I f (x) I ~ M when a < x < b, where M

is constant; 2) has no more than a finite number of points of discontinuity and all of them are of the first kind [Le., at each discontinuity ; the function f (x) has a finite limit on the left f (~-O) = linl f (~-e) and a finite limit on the right

f (;+0)= lim f (;+e) (e > O)J;


8
~o

8~O

3) has no more than a finite number of points of strict extrenlum. Dirichlet's theorem asserts that a function f (x), which in the interval (-xt, n) satisfies the Dirichlet conditions at any point x of this interval at which f (x) is continuous, may be expanded in a trigonometric Fourter sertes:

f (x)

= ~ +a

cos x + b1 sin x + a 2 cos 2x -1- b2 sin 2x + ... + an cos nx +

--1- bn si n nx -1- . , ( 1)
where the Fourier coefficlents an and bn are calculated frorn the formulas
I an=n-

S I S . (n=l, 2, ... ). f(x)cosnxdx(n=O, 1,2, .. ); bn=n f(x)slnnxdx


-~

-n

If x is a poinf of discontinu ity, belonging to the interval (-1t, Jl), of a function f (x), then the SUITI of the Fourier series S (x) is equal to the anthl11Ctical mean of the left and right limits of the function:
I S (x) =2" [f (x-a)

+ f (x + 0)). + 0) + f (1t-O)].
f (x) is even li. e., f (- x)
=

At the end-points of the interval


S (-n)

x = - xt and x = xt,

=S

(xt) =2" [f (-xt

f (x)], then in formula

2. Incomplete Fourier series. If a function (l)

bn=O (n= 1, 2, . )
Clnd
n

an =

~S t (x) cos nx dx
o

(n =

0, I, 2, .. ).

Sec. 4)

Fourier Series

319

If a function f(x) is odd [i.e., {(-x)=-{(x)], then an=O (n=O, 1,2 . ) and n
bn =

5f
o

(x) sin nx dx (n

= I,

2, .. ).

A function specified in an interval (0, xt) may, at our discretion, be continued in the interval (- n, 0) either as an even or an odd function; hence, it may be expanded in the interval (0, xt) In an incomplete Fourier series of sines or of cosines of nlliltir-le arcs. 3. Fourier series of a period 21. If a function {(x) satisfies the Dirichlet conditions in SOlTIe Interval (-I, 1) of length 21, then at the discontinui ties of the function belonging to this interval the following expansion holds:
{(x) ~-2+al cos
a0 xtx

T + b1 sin T +a 2 cosl +b 2 sin -l-+'

xtx

2nx

. 2nx

nxtx . nrtx .. +ancos-l-+bnsln - l

+... ,

where

1 an = T
1 bn=y

5{
-I

(x)

nnx dx (n = 0, I, 2, . ), cos 1-

5f
-I

(2)

. nxtx 1 2 , ... ). (x) SUl- dx (n=,


l

X=

At the points of discontinuity of the function f (x) and at the end-points l of the interval, the SUlll of the Founer series is defined in a manner
similar to that which \ve ha've in the expansion in th~ interval (-xt, n). In the case of an expansion of the functton f (x) in a Fourier series in an arbitrary Interval (a, a+2l) of length 2l, the limits of integration in fornlulas (2) should be replaced respectively by a and a-1-2l

Expand the follo\ving functions in a Fourier series in the interval (- re, n), deterrnine the surTI of the series at the points of discontinuity and at the end-points of the interval (x = - Jt, X == n), construct the graph of the function itself and of the sum of the corresponding series [outside the interval (- re, n) as well]: CI when -n<x~O, 2671. f(x)= { c2 \vhen O<x<1t. Consider the special case \vhen c, = --1, c2 = 1. ax when -- n < x ~ 0, 2672. f (x) = { bx when 0 ~ x < n. Consider the special cases: a) a = b = 1; b) a = - 1 b = 1; c) a=O, b=l; d) a=l, b=O. 2673. f (x) = x 2 2676. f (x) = cos ax. 2674. f (x) = eax. 2677. f (x) = si nh ax. 2675. f (x) = sin ax. 2678. f (x) = cosh ax. 2679. Expand the function f (x) = rr 2 x in a Fourier series in the interval (0, 2n).
t

320

Series

(Ch. 8

2680. Expand the function f (x) in sines of multiple arcs in the interval (0, 31). Use the expansion obtained to sum the number series:
a) 1- 3 c) 1-5
1 1 1 +5 1

=:

+ ... ;

b) 1+5-7-n+13+17- ... ;

1 1 1 +"7-n+ 13- ...

Take the functions indicated below and expand them, in the interval (0, Tt), into incomplete Fourier series: a) of sines of multiple arcs, b) of cosines of multiple arcs. Sketch graphs of the functions and graphs of the sums of the corresponding sel ies in their domains of definition. 2681. f (x) = x. Find the sum of the following series by means of the expansion obtained:
1 -t- 32 + 52 +
2682. f (x) = Xl. Find the sums of the follo\ving number series by means of the expansion obtained:

1) 1 + 22
2683. 2684.

+ 3 + ;
2

2) 1-

2Z'- 32 .
n

1 42

-r .

f (x) = eax . f (~) = f

l 0 when i o=;;;;;x<n.

1 when 0 < x < ~

2685. f (x) = J

x when 0<x~2'

l n-x when ~ <x<n.


functions, in the

Expand the following sines of multiple arcs:


2686. 2687. 2688.

interval (0, n), in

f (x) = f (x) = f (x) =

X {

when 0 < x 0=;;;;; ~


n

o when 2 < x < n.

x (n-x).

sin

Expand the following functions, in the interval (0, n), in cosines of multiple arcs: 2689. f (x) = { 1 when 0 < x ~ h, o \vhen h < x< 31.

Sec. 41

Fourier Series

~21

2690.

f (x) = {

I _..!- when 0 < x ~ 2h,


02h

when 2h<x<n.
sin x.
COS

2691. 2692.

f (x) = x
f (x) =
{

x when 0 < x ~
n

i- '

-cosx when 2"<x<n.

2693. Using the expansions of the functions x and Xl in the interval (0, n) in cosines of nlultlple arcs (see Problems 2681 and 2682), prove the equality

2694**. Prove that if the function f (x)

is even and we have

1 ( ;- + x ) = -I ( ~ - x ).

then Its fourier series in the interval

(-n, n) represents an expansion in CaStrIeS of odd lTIultiple arcs,

and if the function f (x) is odd and f ( ~ -I= f ( ~ - x ) I then In the interval (-n, n) it is expanded in sines of odd multiple arcs. Expand the rollo\ving functions in l:ourier series in the Indicated intervals: 2695. f (x) = I x I (- 1 < x < I). 2696. f (x) = 2x (0 < x < 1). 2697. f(x)=e x (-l<x<l). 2698. f(x)=lO--x (5<x<15). Expand the rollo\vin:{ functIons, in the indicated intervals, in incoInplete Fourier series: a) In sInes of ~llultlple ales, and b) in cosines of multiple arcs: 2699. f (x) = 1 (0 < x < 1). 2700. f (x) = x (0 < x < I). 2701. f(x)=x 3 (0< x<2n).

x)

x when O<X~ 1, 2-x when I <x< 2. 2703. Expand the following function in cosines of multiple arcs 10 the interval (~. 3):
2702. f(x)= {

f (x) = I
11-1900

when 2" < x :e;;; 2, \ 3-x when 2<x<3.

Chapter IX

D1FFERENTIAL EQUATIONS

Sec. 1. Verifying Solutions. Forming Differential Equations of Families of


Curves. Initial Conditions
1. Basic concepts. An equation of the type F(x, y, y' . ... , y)(n)==o,
(I)

where y=y (x) is the sought-for functi l1 n, is called a diUerential equatton of order n. The function y=(p(x), which converts equation (1) into an identity, is called the solution of the equation, while th~ ~raph of this function IS called an integral curve. If the solution is represented irnplicitly, (D (.-t, y) .:=0, then it is usually called an lntegral Example t. Check that the function y = sin x is a solutton of the equation y"+y=O. Solution. \Ve have: y'=cosx, y"=-sinx and, consequently, y" -t y == - sin x sin x === 0. The Integral (1) (x, y, C t , . . . , Cn) === 0 (2)

of the differential equation (1), which contains n independrnt arbitrary constants Ct , , Cn and is equivalent (in the given region) to equation (I), IS called the general tntegral of this equation (in the respective region). By ass\~n ing definite values to the constants Cit ... , CIl in (2), we get fJarLlcullr
tntegrals.

Conversely, if we have a family of curVf\S (2) and elirninate the raranl eters CIt ' Cn from the system of equations
(1)=0,
d<D dx=O,
dn<D ... , dxn=O,

we, generally speaking, get a differpntial equation of type (I) whose general integral in the corresponding region is the relation (2). Example 2. Find the differential equation of the family of parabolas y:=. Ct (X-C Z)2. (3) Solution. Differentiating equation (3) twice, we get: y'=2C.(x-C z) and y"=2C t
(4)

Eliminating the parameters Ct and C2 from equations (3) and (4), we obtain the desired differential equation 2yy" =y,l.

Sec 1)

t'ertfyinR Solultons

323

It IS easy to verify that th~ function (3) converts this equation into an identi ty. 2. Initial conditions. If for the desired particular solution y ==Y (x) of a differential eq uatto n y(n) = f (x, y, y', . , y(n-J)) (5) the inltlal conditions
y (x o) -= Yo,
y' (x o) = y~, , y(n - I ) (x o) = y~n -I)

are given and we know the general solution of equation (5) y=cp(x, Cit ... , Cn)' then the arbitrary constants CI , [roln the system of equations
,

Cn are determined (if this is possible)

Yo -= cp (x o, CI , , Cn), y~ == cp~ (x o, Cit ... , Cn),

Example 3. Find the curve of the family

Y= C.e x + C2e- 2X ,
for which y (0) = 1, y' (0)
Solution. We have:
=:: -

(6)

2.
y' = Cle x -2C 2e- 2X

Putting x=O in \\'hence and, hence,

fOflnula~ (6)

and (7), we obtain


-2~CI-2C2'

(7)

1=C1 +C 2 ,

Determine \\J"hether the indicated functions are solutions of the given differential equations: 2704. xy'=2y, y=5x 2 2705 Y '2 =X 2 +y, 2 t Y=x.
2706. (x l- y) dx -t- x dy -= 0, Y = 2x 2707. y"+y=O, y=3sinx-4cosx. 2708. :;: + Ct)2 X = 0, X = C. cos rot + C2 sin cot. 2709. y"-2y'+y=O; a) y=xe", b) lJ=x 2 e". 27 to. y" - (AI +- A2 ) y' + A1 A2 Y = 0, !I = CleA,x -~ C 2e 'A x. Show that for the given differential equations the indicated relations are integra Is: 27\11. (x-2y)y'=2x-y, xl-xy ~ y2=::C".
J

CI

Xi

11*

324

DtUerential Equations

lCh. 9

Y= In (xy). Form differential equations of the given families of curves (C, C I , C a , C, are arbitrary constants): 2714. y=Cx. 2721. In":'= 1 +ay 2715. y = ex!. (a !! t ) 2 2C IS a parame er . 2716. Y = x. 272 ( )2 2 2. y-yo = px 2717. xl+y'=C 1 x (Yo' P are parameters). 27 18. y= Ce. 2723. -C 2X-t-C e-x ...a C 2 2) Y - Ie 2 2719. JI. = (x - y ' . 2724. Y = C\ cos 2x+ C 1 sin 2x. II 2720. y 2 -t.. .!.=2+Ce- 2 . 2725. y=(CI+C~x)ex+Ca
I

2712. (x-1/+1)y'=1, y=x-l-Ce'. 2713. (xy-x)y" + xy'2 + yy' -2y' = 0,

xy-plane.

x 2726. Form the~ differential equation of all straight lines in the

vertical axis in the xy-plan~. 2728. Form the differential equation of all circles in the xy-plane. For the given families of curves find the lines that sati5fy the given initial conditions: 2729. X 2 _ y2 = C, Y (0) = 5. 2730. y=(C I -l-C 2x)e 2X , y(O)=O, y'(O)=l. 2731. y=C l sin(x-C 2 ), y(n)=l, y'(n)=O. 2732. y = Gle- x + C2 ex + C aelx; y (0) = 0, y' (0) = 1, y" (0) = - 2.

2727. Form the differential equation of all parabolas with

Sec. 2. First-Order Differential Equations


1. Types of first-order differential equations. A differential equation of the first order in an unknown function y, solved for the derivative y', is of the form y'=f(x, y), (1) where f (x, y) is the given function. In certain cases it is convenient to consider the variable x as the sought-for function, and to write (1) in the form x'=g(x, y), (1 ') 1 where g (x, y) = f (x, y).
Taking into account that y' = :~ and x' =:;. the differential equations (1) and (1') may be written in the synlmetric form p (x, y) dx+ Q (x, y) dy= 0, (2) where P (x, g) and Q (x, y) are known functions. By solutions to (2) we mean functions of the form y=cp (x) or x='\I' (g) ahat satisfy this equation. The general integral of equations (1) and (1 '), or

Sec. 21

First-Order DiOerential Equations


<D (x, y, C) =0.

325

equa Hon (2), is of the form where C is an arbitrary constant. 20 Direction field. The set of directions
tana=f(x, y)

is called a direction field of the differential equation (I) and is ordinarily depicted by means of short lines or arrows inclined at an angle Q. Curves f (x. y) =-= k, at the points of which the inclination of the field has a constant value, equal to k. are called isoclines. By constructin~ the isoclines and direction field, it is possibl~, in the simplest cases. to give a

rough sketch of the field of int~gral curves, regardin~ the latter as curves which at each point have the given direction of the field. Exalnple 1. USing the method of isoclines, construct the field of integral curves of the equation
y' :=2X.

Solution. By constructing the isoclines x = k (straight lines) and the direction field. we obtain approximately the field of integral curves (Fig. 105). The family of parabolas is the ~eneral solution. Using the method of isoclines. make approximate constructions of fields of integral curves for the indicated differential equations:

y=-+C 2

Xl

2733. 2734. 2735. 2736.

y' = - x. y' = - ~ . y y' = 1 f- yl.


y' =x+y .

2737. y'=xl+y

x-y

326

Dlfferenttal

Equations

[Ch. 9

U {a < x < A, b < y < B} and in this region has a bounded derivative /~ (x, y), then through each point (xo, Yo) that belongs to U there passes one and only one integral curve y=ep (x) of the equation (1) (ep (xo)=Yo]'

3. Cauchy's theorem. If a function

f (x,

y)

is continuous in some region

4. Euler's broken-line method. For an approximate construction of the integral curve of equation (1) passing through a given point Mo (xo, Yo)' we replace the curve by a broken I ine wi th vertices M i (xi, Yi), where
Xi+l=Xi+L\ Xi, Yi+l=Yi+L\Yi'

AXi=h (one step of the process), AYi = hI (xi, Yi) (i = 0, I, 2, .. ).

Example 2. Using Euler's method for the equation

find y (I), if y (0) = I (h We construct the table:

= 0.1).

,
I

xi

Yi

L\

lJ;= 20

xiYi

0
I 2

3
4

0 0.1 0.2 0.3 0.5


0.6

5
.6

o4
0.7 0.3 0.9 1.0

1 1 1.005 1 015 1.030 1.051


1.077

o 005 0.010 o 015 o 021

o 026

9 10

1.109 1.148 1.194 1.248

o 032

0.039 o 046 0.054

Thus, y (1) = 1.248. For the sake of comparison, the exact value is
1

Using Euler's method, find the particular solutions to the given differential equations for the indicated values of x: 2738. y'=y, y(O)=l; find y(l) (h=O.l). 2739. y'=x-t-y, y(l)=l; find y(2), (h=O.l). 2740. Y'=-l~X' y(O)=2; find y(1) (h=O.l).
2741. y'=y_2x, y(O)=l; find y(l) (h=O.2).
II

Sec. 3)

DiOerential Equations wlth Variables Separable

321

Sec. 3. First-Order Differential Equations Orthogonal Trajectories


~eparable

with

Variables

Separable.

I". First-order eql.lations with variables separab Ie. An eq ua tion wi th variables is a first-order equation of the type
y' =

t (x) g (y)

( 1)
(I')

or

X (x) Y (y) dx+X 1 (x) Y. (y) dy=O


Dividin~

both sides of equation (I) by g (y) and multiplying by dx, we get

dll) = f (x) dx Whence, by integrating, we get the general integral of equag(y hon (I) in the fonTI

5gdy
1

(y)

=Sf(X)dX+C

(2)

Similarly, divining both sides of (\quclhon (I') by XI (x) Y (y) and integrating, we get the general integral of (1') in thp form
X (r) dx + SY (Y) dy = C (x) Y (y) S,X.
I

(2')

If for some value y=Yo \ve have R (Yo) =0, then the function Y=~/o is also (as is directly evident) a solution of equation (I) Similarly, the straight lincs x -=ll and y -== b will be the intet!ral curves of equation (1'), if a and b 3re, respectively, the roots of the equations XI (x) =0 and Y (y) =0, by the lcft Sldc'i of whlch we had to divide the inItial equation. Example 1. Solve the equation
Y == ,

~ .

11

(3)

In particular. find the solution that sathfies the initial conditions


y(l) =2

Solution. Equation (3) may be written in the tonn

dx=-x
Whence, separating variables, we have
dy =_ dx y x

dy

and, consequently, lnIYI=-lnlxl+lnC" where the arbitrary constant In C. is taken in logarithmic form. After taking antilogaritluns we get the general solution
(4)

where C = CII When dlvldlng by y we could )Olie the solution y=O, but the latter is contalned in the formula (4) for C=O.

328

DifJerential Equations

[Ch. 9

Uti1izln~ the ~fven Initial conditions, we get C = 2; and, hence, the de sired particular solutIon is 2 y=-. x

2 Certain di tfertntial equationc; that reduce to equations with variables separable. Differential equation~ of the fonn y'=f(ax+by+c) (b#O)

reduce to equations of the form (I) by means of the substitution u=ax+bu+ e, where U IS the new sough t-for functIon 3 Ortho~onal traJectoric:s ar~ curves that intersect the hnes of the given family <1>(x,y,a)=O ta IS a parametpr) at a right angle. If F(x,Y,y')=O is the diflerentIal equation of the family, then F (X. y, -

~,) =0
ellips~s

Is the differential equaHon of the orthogonal trajectories.


Exalnple 2. Find the ortho.Jonat trajectories of the family of x2 + 2// 2 =- a2 Solution Difff'rentiatlng the equation (5), we find the tion of the famll y x -f- 2yy' -= O.

(5)

dlder~ntJal

equa-

Fig. 106

Whl\nce. repla("n~ y' by - y"


orthogonal traJeclones

we get

the differential

equation ef the

x_ 2y =O or u,=2y. 1/ ., x Integrating, we have y = ex! (lalnlly 01 parabolas) (Fig. 106).

Sec. 31

DiOerential Equations with Variables Separable

329

4. Forming difl'erential equations. \\'hen forming differential equations In geometrical problems, we can frquently make use of the geometrical meaning of the deriva tl ve a s the tangent of ~n angle fornled by the tangent hne to the curve in the pos'tive x-direction. In olany cases this makes it I=ossible straightway to cstabltsh a rplationship hptween the ordinate y of the desired curve, its abscls~a x, and the tanllent of the angle of the tangent line y', that is to say, to obtain the diffe.ent\al equation. In other Instances ~see Problems 2783, 2890, 2895), u~e is made of the geometrical significance of the dpfinite integral as the area of a curvihnear trapezold or the length of an arc. In thi~ cac;e, by hypothesls we have a simple lntegral equation (since the desired function is under the '\ign of the Integral); however, we can readily pass to a differential equatlon by dlfterentiatIng both ~ides. Example 3. Find a curve passing through the pOint (3,2) for which the segnlent of any tangent line contained between the coordinatE:' axes is divid~d in half at the pOlnt of tangency. Solution. Let M (x,y) be the mid-pOint of the tangent line AB. which by hypothesis is the point of tangency (the pOlnts A and B are points of lnter')ection of the tangent line with the y- and x-axes). It is given that OA =2y and OB = 2x. The slope of the tangent to the curve at M (x, y) is dy OA !I (jX=- VB=--X This is the differential equation of the sought-for curve. Transforming, we gel

d\+~=o
x y
and, conscquentl y, lnx+lny -=lnCorxy==C. Utilizing the initial condition, \ve determlne C =32 ==6. Hence, the desired curve is the hyperbola >.y =.:. 6.

Solve the differential equations: 2742. tan xsin 2 ydx-t-cos 2 xcotydy=O. 2743. xy' -- !I = yS 2744. xyy' =:-: 1_x2 2745. y-xy' = a (I -t-x 2 y'). 2746.3 p x tan ydx-f-(1-e X )scc 2 ydy=O. 2747. y' tan x =!J. Find the particular solutions of equations that satisfy the indicated initial conditions: 2748. (I + eX) y y' = eX; !J =- I \vhen x = O. 2749. (xy2.t-x}dx-4-{x 2 y-y)d!J=O; y=l \vhen x=O. 2750. y'sin x=ylny; y:.=l when

x=;.

Solve the differential equations by changing the variables:


2751. 2752. 2753. 2754. y'=(x +y)2. [I = (8x -~- 2y + 1)2 (2x + 3y- I) dx -t (4x + 6y- 5) dy = O. (2x-y) dx+ (4x-2y -1- 3) ely = O.

330

Dzt!erential Equations

[Ch.9

In Examples 2755 and 2756, pass to polar coordinates:


2755. y' = 2756. (X 2+y2)dx-xydy=O. 2757*. Find a curve whose segment of the tangent is equal to the distance of the point of tangency from the origin. 2758. Find the curve whose segment of the normal at any point of a curve lying between the coordinate axes is divided in two at this point. 2759. Find a curve whose subtangent is of constant length a. 2760. Find a curve which has a subtangent twice the abscissa of the point of tangency. 2761 *. Find a curve whose abscissa of the centre of gravity of an area bounded by the coordinate axes, by this curve and the ordinate of any of its points is equal to 3/4 the abscissa of this point. 2762. Find the equation of a curve that pa5ses through the point (3,1), for which the segment of the tangent between the point of tangency and the x-axis is divided in half at the point of intersection with the y-axis. 2763. Find the equation of a curve which passes through the point (2,0), if the segment of the tangent to the curve between the point of tangency and the y-axis is of constant length 2. Find the orthogonal trajectories of the given families of curves (a is a parameter), construct the families and their orthogonal trajectories. 2766. xy=a. 2764. x 2 + y2 = a 2 2765. y2 = ax. 2767. (x-a)! r- y2 = a2

~-x y

Sec. 4. First-Order Homogeneous Differential Equations


1. Homogeneous equations. A differential equation p (x, y) dx+ Q (x, y) dy=O
(1)

is called homogeneous, if P (x, y) and Q (x, y) are homogeneous functions of the same degree. Equation (I) may be reduced to the form

y'=f

(~):

and by means of the substitution y=.-:XU, where u is a new unknown function, it is transformed to an equation WIth variables separable. We can also apply the substitution x == yll. Example 1. Find the general solution to the equation

Sec. 4)

First-Order Homogeneous Differential Equations

331

Solution. Put y=ux; then u+xu'=eu+u or

dx e -ud u=-.
x Integrating, we get u=-ln InE..., whence x y=-xlnln-. x 2. Equations that reduce to homogeneous equations.

If
(2)

and

6=1::~:Io,

then,puttinginto equation (2) x=u+a,


~

the conc;tants a and

y=v+~. where are found from the following system of equations,

ala+bl~+cl=O, a2a+b2~+c2~O,

we get a homogeneous ditTerenttal equation in the variables u and u. If 6=0, then, puttIng in (2) a1x-t-b 1y=u, we get an equation with variables separable.

Integrate the ditTerential equations: 2768. y' =- JL -1. 2770. (x- y) y dx-x1dy = O.
x

2769. y':= _

y x+ x

2771. For the equation (Xl +y2) dx-2xydy=0 find the family of integral curves, and also indicate the curves that pass through the points (4,0) and (1,1), respectively. 2772. y dx i- (2 Vxy-x) dy = O. 2773. xdy-ydx=Vx 2 -f-y 2 dx. 2774. (4x 2 + 3xy + y2) dx + (4 y l + 3xy + x 2) dy = O. 2775. Find the particular solution of the equation (x 2 -3!1)dx+ -t- 2xyriy = 0, provided that y = 1 when x = 2. Solve the equations: 2776. (2x-y -+ 4)dy+(x-2y+ 5)dx=O.
, 1-3x-3y 2778' x+2y+ 1 2777. Y == l+x+y Y =2x+4y+3 2779. Find the equation of a curve that passe~ through the point (1,0) and has the property that the segment ~ut off b) the tangent line on the y-axis is equal to the radius vector of the point of tangency. 2780**. What shape should the reflector of a search light have so that the rays from a point ~ource of light are rel1ected as a parallel beam?

332

DitJerential Equations

(Ch. 9

2781. Find the equation of a curve whose 5ubtangent is equal to the arithmetic mean of the coordinates of the point of tangency. 2782. Find the equation of a curve for which the segment cut off on the y-axis by the normal at any point of the curve is equal to the distance of this point from the origin. 2783*. Find the equation of a curve for which the area conta ined between the x-axis, the curve and two ord inates, one of which is a constant and the other a variable, is equal to the ratio "of the cube of the variable ordinate to the appropriate abscissa. 2784. Find a curve for which the segment on the y-axis cut off by any tangent Iine is equal to the abscissa of the point of tangency.
Sec. 5. First-Order Linear Differential Equations. Bernoulli's Equation
1. Linear equations. A differential equation of the form y' + p (x). y Q (x)
=.::;

( I)

of

de~ree

one in y and y' is called linear. If a function Q (x) =:; 0, then equation (1) takes the form y' + p (x) y = 0

(2)

and is called a homogeneoLls linear differential equation. In this case, the variables may be separated, and we get the general solution of (2) in the form
y=Ce.

-J

P(x) dx

(3)

To solve the inhomogeneous linear equation (1), we apply a method that is called variation of parameters, which consIsts in first finding the ~eneral solution of the respective hornogeneous linear equation, that is, retationship (3). Then, assumin~ here that C is a function of x, we seek the solution of the inhomo~eneous equation (1) in the form of (3). To do this, we put into (1) y and y' which are found from (3), and then from the differential equation thu5. obtainerl we determine the function C (x). We thus get the general solution of the inhomogeneous equation (1) in the form
y=C (x).e Example l. Solve the equation y' = tan xy+cos x. Solution. The corresponding homogeneous equation is y' - tanxy:=:O. Solving 1t we get: 1 . Y ::-c.-ccsx

-f P

(x) dx

(4)

Sec.

5J

Bernoulli's Equation'

333

Considering C as a function of x, and differentiating, we nnd:

=_1_ de cos x dx

+ cos 2 X

sin x

.c

Putting y and y' into (4). we get: I de sin x C de - - . -+--.C=tanx.--+cosx or -=cos 2 xt cos x dx cos 2 X cos X 'dx whence
C (x) =

cos 2 xdx= } x+ { sin 2x+C\.

Hence, the general soJution of equation (4) has the form

y= ( hon

~ X+{ sin 2t+C\ ) CO~X


y==uv,
(3)

In solving the linear fquation (1) \ve can also make use of the substitu ..

where u and v are functions of x. Then equation (I) will have the form

lu' -t- P
If we req U Ire fha t

(x) u 1v -f-

v' u -= Q (x).

(6)

u'

-1- P (x) u == 0,

(7)

then fronl (7) we find ll, and fronl (6) we find v; hence, from (5) we find y. 2). Bernoulli's equation. A first-order equation of the form y' + P (.()!J -= Q (x) y7.,

where a ~ 0 and a '# 1, is called Berflolllll'~ equation It is reduced to a linear equatioll by nleans of tlll substitution z == yJ-'1. I t is also lossib1e to apply dire('tly the substitution y-::=.uv, or the Iuethod of variation of paranleters. Example 2. Solve the equation
l

y'==-y+x x

lIrf . tl.

Solution. This is Bernoulli's equation. Pulting \ve


~et

U'I1+I1'U=~

UI1 +x IW

or

v (U'-

/I) +v'u=x iii1.

(8)

To detennine the function u we require that the relation

u,_i u=Q
x

he fulfilled, whence we have Putting this expression into {8), we get

V'.,,4=xYvX".

334

DiOerential Equations

lCh.9

whence we find v:

11=( ; lnx+c y.
y=x4 (

and, consequently, the general solution is obtained in the fonn

~ lnx+C

r.

Find the general integrals of the equations: 2785. dy _J!.. = x.


dx x

2786. ~dY

+ 2y = x

x' .

2787*. (1 -+ y2) dx = tV 1 y2 sin y-xy) dYe 2788. y 2 dx- (2xy 3) dy = O.

Find the particular solutions that satisfy the indicated conditions: 2789. xy' + y-e" = 0; y = b when x === a.
2790. y ' - -x Yl z-l-x=O; y=O when x~o. 2791. y'-ytanx=_l- ; y=O when x=O.
cos x

Find the general solutions of the equations:


2792. 2793.
dc!J!.
X

+.JL = x

xy2.

2xY~-!I +x=O.

2794. y dx x1y) dy=O. 2795. 3xdy=-=y(1 +xsinx-3y 3 sinx)dx. 2796. Given three particular solutions y, Yl' Y2 of a linear

+ (x- ;

equation. Prove that the expression Y2-Y rernains unchanged for Y-Yl any x. What is the geometrical significance of this result? 2797. Find the curves for wh ich the area of a triangle formed by the x-axis, a tangent line and the radius vector of the point of tangency is constant. 2798. Find the equation of a curve, a segment of which, cui off on the x-axis by a tangent line, is equal to the square of the ordinate of the point of tangency. 2799. ,Find the equation of a curve, a segment of which, cut off on the y-axis by a tangent line, is equal to the subnormal. 2800. Find the equation of a curve, a segment of which, cut otT on the y-axis by a tangent line, is proportional to the square of the ordinate of the point of tangency.

Sec, 6]

Exact DitJerential Equations. Integrating Factor

335

2801. Find the equation of the curve for which the segment of the tangent is equal to the distance of the point of intersection of this tangent with the x-axis from the point M (O,a).

Sec. 6. Exact Differential Equations. Integrating Factor


1. Exact differential equations. If for the differential equation P (x, y) dx+ Q (x, y) dy=O
(1)

the equality ~: = ~~ is fulfilled, then equation (1) may be written in the

form dU (x, y) = 0 and is then called an exact dtUerential equation. The general integral of equation (1) is U (x, y) = C. The function U (x. y) is determined by the technique given in Ch. VI, Sec. 8, or froln the formula
x
II

U=
~sce

~p
Xo

(x, y) dx

+ ~ Q (xo y) dy
Yo

a (6(2 y -t- 4y S)
:=

Chi VI I t Sec, 9). Example I. Find the general integr al of the differential equation (3x 2 + 6xy 2) dx + (6x 2 y + 4y') dy = 0, xy . a(3x 2+ So 1Ut Ion. Th IS IS an exac t d'ff 1 eren t' la I equa tlon, since 0 6 2) == y

ox

= 12xy and, hence, the equation is of the form dU = O.

Here, whence

au - = ax
u= ~
(3.\1

3x!

+ 6xy

and

- = 6x"y + 4y' ay'

au

+6xyl) dx+ cp (y) =x l

+ 3X1y 2+ cp (y).

(by Y hypothesis); from this we get q>'(U)=4tj' and q>(f)=Y"+C. We finally get U (r, y) =XS + 3X 2y 2 + y4+ CClt consequently, X' + 3x y2 + y4=C is the sought-for

Differentiating U with respect to y, we find

~aU = 6x!y -t <p' (y) = 6x!y + 4y'

general inte~ral of the equation. 2. Integrating factor. I r the left si de of equation (1) is not a total (exact) differential and the conditions of the Cauchy theorem are fulfilled, then there exists a function ,.... =,.... (x, y) (integrating factor) such that
)1

(P dx+ Q dy) =dU.

(2)

Whence it Is found that the function ,.... satisfies the equation


ay (JLP)=ax (flQ)

The integra ting factor ,.... Is readily found In two cases: 1 aQ ) 1) Q ay - ax = F (x), then,.., = JL (x);

(OP 1 (OP OQ) 2) -p ay - ox = F

(Y), then ~t = Jl (y).

336

Dif1erential Equations
Example 2. Solve the equation
a

(CIl. 9

(2XY+X2y+~)dx+tx l +y2)dll=O.
t

Solution. Here P = 2xy + x 2y + l/3 Q = Xl + y2


and

Q oy - ox
. Stnce

(OP OQ) = 2x +x! + y2-2 x!+ y2


o(J..tP) iJ(~Q) ay=--axor

= 1. hence. J..t= J..t (x).


iJQ

J..t oy = J..t ax

oP

+Qd,....
dx
t

it follows that

dJL=.! (ap _aQ)dX=dX ,.... Q iJy ax


2xy + x 2y + ~
3

and ln~ .. =x.

JL=r.

Multiplying the equation by IJ. = eX. we obtain


eX (

) dx-{-e" (Xl + yl) dy -===0

which is an exact differential equation. Integrating it, we get the general integral

Find the general integrals of the equations:


2802 (x+ y) dx + (x -1- 2y) dy = O. 2803. (Xl + y2 2x) dx -{- 2xy dy = o. y - y2) dy = O. 2804. (Xl -:- 3xy 2 + 2) dx - (3x 2

2805. xdx-ydy=
y' y4

x dy-y dx

. \ 2+ Y2 2806. 2xdx+ y2 -3x dy-=O.


2

2807. Find the particular integral of the equation

(x+e~) dx+ef (1-~) dy=O.


\vhich satisfies the initial condition y (0) = 2. Solve the equations that admit of an integrating factor of the form ~ = 11 (x) or I..t = I..t (y): 2808. (x+ y2) dx-2xy dy = O. 2809. y(l -~-xy)dx-xdy=O. 2810. Y dx + (y'-In x)dy = O. x
2811. (x cos y - y sin y) dy + (x sin y -~ y cos y) dx = O.

~ec.

7)

Fir~t-Orde,

D;Oerential Equation, /lot Solved for Derivative

337

Sec. 7. First-Order Differential Equations not Solved for the Derivative


to. First-order dlft'erential equations of higher powers. If an equation
F
(x, y, y') =0,

(1)

,\ hich for example is of degree two in y', the.l by solving (1) for II' we get

two equations:
y'=f,(x,y), y'= f2(X, y). (2) Thus, generally spt'aking, through each pOint M" (xo, Yo) of some region of a plane there pass two integral curves. The general integral of equation (1) then, generally speaking, h'1s the fOrln

<I> (x, y, C) === cD. (x, y, C) cD 2 (x, y, C) =0,

(3)

\vhere (D. and cI>2 are the general inte~rals of equations (2). Besides, there may be a slnRular lllte{!ral for equilt10n ~1). Geometrically, a singular integral is the cnvelope of a fal1lily of curves (3) and Inay be obtained by elinlinating C frolll the s~'steln of equations
<D(x,y,C)=O,

<D~(X,y,C)=O

(4)

or by eliminating p=y' fronl the system of equatIons

F (x, 11, p) =-.=- 0,

F~ (.r, !J,

p)

= o.

(5)

We note thflt the curves defined by the 1quations (4) or (5) are not alwavs solutions of equation (I); thert'fore, in each casc, a check is necessary. Exalnple 1. FJ nd the general and sln~ular integrals of the equation
XII'2- r 2xll'-Y~0.

Solution. Solving for y' we have t\\o hOlllogeneous cquations:


y'

=-

+ 1/ 1+ ~.

y' = -

1-

1t

lit'fined in the region


x(x+y)

>0.

the general integrals of which are

or

(2x+y-C)-2

yx

+xy=0,

(2x+ y-C) +2
(x 2 +xy):::0

Yt

+xy=O.

Multiplying. we get the general integral of the given equation

(2x+ y-C)2-4
or

(y-C)2=4Cx (a family of parabolas). DifTerentiatin~ th~ general integral \\'ith respecf to C and eliminating C, \ve find the singul ar in tegral

y+x=O.
(It ll1ay be verified that y

+x = 0 is

the solution of this equation.)

338

Differential Equations

(Ch.9

It is also possible to find the singular integral by differentiating xp2+2xp-y-=O with respect to p and eliminating p. 2. Solving a differenti al equation by introducing a parameter. If a firstorder differential equation is of the form x= cp (y, y'). then the variables y a'ld x may be determined from the system of equations
l=aq>+aq>dp p ay a dy' x = q> (y, p).

\\'here p=y' plays the part of a parameter. Similarly, if y='I' (x, y'), then x and yare deternlined from the system of equations a", a"'dp p= ax + ap dx' y='I' (x, p). Example 2. Find the general and singular integrals of the equation
y=y'2_ xy'

+x 2' +x 2
2

Solution. Making the substitution y' = p, we rewrite the equation in the form
y=p2- xp

Diffelentiatiog with respect to x and considering p a function of x, we have


p=- 2pdp _p_x dp +x dx dx

or ~~(2P-X)-(2P-X). or~=l. Integrating we get p=x+C. Substituting into the original equation. we have the general solution
x2 x2 y=(X+C)2- x (x+C)+2 or Y="2+CX+C2.
Differentiating the general solution with respect to C and eliminating C, we
J

obtain the singular solution: g=X (It may be verified that g=T is the 4 solut ion of the given equation.) If we equate to zero the factor 2p -x. which was cancelled out, we get p= ~ and, putting p into the given equation, we get g=~. which is the same singu lar sol ution.

Find the general and singular integrals of the equations: (In Problems 2812 and 2813 construct the field of integral curves.) 2y, 1 0 28 t 2 y ,2 -x U ;- = 2813.4y,Z-9x=O.
I

Sec 8]

The Lagrange and Clairaut Equations

339

2814. yy,2_(xy+ l)y' tx=O. 2815. yy,2 -2xy' + y = o. 2816. Find the integral curves of the equation y'!-t-y"=1 that pass through the point M (0, ;). Introducing the paranleter y' =p, solve the equations: 2820.4y==x,,+y,2. 2817. x=siny'+-lny'. 2 + '2 2818 Y=Y ,2 p )', 2821 eX-~ 2819. y=y'" +2Iny'. - 2y'
Sec. 8. The Lagrange and Clairaut Equaticns
to. Lagrange's equation. An equation of the forrn y = xfP (p) -1- 'I' (p),
(1)

\vhere p==y' is called Lagrange's equatIon Equation (1) is reduced to a linear equation in x by differentiation a nd taking into con'Sideration that dy -= p dx: p dx == cr (p) dx -1- [xcp' (p) + '1" (p)] dp. (2)
If p ~ cp (p), then from (1) and (2) we get the general solution in parametric form:
x= Cf (p)

+ g (p),

y --= [Ct (p)

+ g (p)] q) (p) + 'I-~ (p),

where p is a parameter and f (p) .. g (p) are certain known functions. Besides, there [nay be a Singular solution that is found in the l1~ual \vay. 2. Clairaut's equation. If In equation (1) p =-.: ql (p), then we get Cla;raut's equ.atlon y::= xp -1- 'P (p). Its general solution IS of the fornl y;-: Cx -t- 'I' (C) (a farnl1y of ~tralght lines). There IS also a parttcular solutiO'} (envelope) that results by clilninating the parameter p fronl the systenl of equ~t1011~
x==- - 'P' (p), { y==PX+~lP)

Example. Solve the equation


y

--2y'x+,.
y

Solution.

Puttin~

y' 7-= P we have y = 2p\


p dx == 2p dx

ing dy by p dx, we get

*;
P

(3)

different; llting and replac-

+ 2x dp- ~

or

dx 2 1 dp=-px+ pl
~quation,

Solving this lineal

we will have

x= 2 (In p+C).

I p

340

DiOerential Equations

(Ch. 9

Hence, the general integral will be


X=

f ;2
l

(In p+C).

y=2px+

To find the singular integral, \ve form the system 1 1 y=.2px+-, 0==2x- 2 p p In the usual way. Whence

x= 2p z'
and,
con~equently,

Y=p

y=

2 V2x.

Putting y into (3) we are convinced that the function obtained Is not a solution and, therefore, equation (3) does not have a singular integral.

Solve the Lagrange equations: 2822. y =..!- x (y' + 2824. Y == (1


2823 2 -'

y-y + J' y. Find the general and singular integrals of the Clairaut equations and construct the field of integral curves: ,2 ' 2826 . y=xy +y . 2827. y=xy' +y'. 2828. Y = xy' + l/rl 1---(y-')-2.
2829. Y = xy'

1/-1'2-

P,) . Y

+ y') x + y".
I, , 2 Y (2x-}- y).

2825 Y

""

=-

~- J, . y

2830. Firld the curve for which the area of a triangle formed by a tangent at any point and by the coordinate axe\) is constant. 2831. Find the curve it the distance of a given point to any tangent to this curve is constant. 2832. Find the curve for which the segment of any of its tangents lying between the coordinate axes has constant length l~

Sec. 9. Miscellaneous Exercises on First-Order Differential Equation')


2833. Determine the types of diflerential equations and indicate methods for their solution: e) xy' -t- y = sin y; a) (x + y) y' =-= x arc tan 1!.. ; x f) (y_xy')2 = y'd; b) (x- y) y' = y2; g) Y == xeY '; c) y' = 2xy + Xl; h) (y' -2xy) Vy=x l ; d) y' = 2xy -t- y';

Sec. 9]

Miscellaneous Exercises on

Fir.~tOrder

DiOerential Equations

341

i) y' = (x +_ y)2; j) X cos y' + y sin y' =-= 1; k) (x 2-xy) y' = y4;

1) (x 2+ 2xy 8) dx -1+ (y2 + 3X2y2) dy = 0; m) (x ' -3xy) dx + (x 2 + 3)dy=O; n) (xy' + In x) dx = y2dy,

Solve the equations:


2834. a)

(x-ycosf)dx+xcos~dy-=O; b) x In ~ dy-ydx == O. y

2835. xdx =
2836. 2837. 2838. 2839. 2840.

(x; _yl) dy.


.+

(2 xy 2 - y) dx x dy =- O. xy' .~. Y =-= xy2 In x. y = xy' -l- y' In y'. Y == xy' -1- V -ay'. x 2 (y + I) dx+ (xS_I) (y-l) dy== O. 2841. (1 -t- y2) (e 2.-.; dx-e Y dy) -(I -1- y) dy =-= O. 2 2842. y'_y2x 2 1 =1. 2845. (l-x )y'+xy=a.
A ,

2843. yeY = (yS -t 2xeY ) y'. 2846. xy - x+ I-x = O. 2844. y'+y cos x == sin x cos x. 2847. y' (x cos y -i- a sin 2y) = I. 2848. (x 2 y-x 2 -l-y--l)dx -t-(xy+2x-3y-6)dy=O.

2849. y' = ( 1 +~;:lr 2850. xy3 dx (x 2 y 2) dye ' 3.\.2 2851 .y=-= .\ I\- Y -tl 7=';

2852. 2dx+ ... /!... dy- ...

V!I

11dx = O. V x
286 t. eY dx + (xeY - 2y) dll = O. 2862 =2 ' _ ,2 Y, :y -j ~y 2863. Y ="A(l-+-Iny-Inx).

2853. y' = Y -I- tan ~ .

2854. yy' + y' = cos x. 2855. xdY-I ydx=-~y!dx.


2856. y' (x -J-- sin y)

VI

== 1.

2864. (2e X + y4) dy, (y + 2

2857. Y t}E = - p -J- p2 dy 2858. x 3dx-(x 4 t- y3).1,y=-=.O.


2859. x y'2_l-3xYY'-l2860. x dx+ y d~
1

X - ye dx = O.

-r X 2+y2
+

+2y 2=O.

2865. y =2 \x+y-I 2866. xy(xyl-t-l)dy-dx=

)2

=0.

xdy--ydx
11
z

=.

2867. a (xy' 2y) = xyy'. 2 2868. xdy-ydx=y dx.

342 2869. (x'-l )3/2 dy 2870. tan

DiOerential Equations

lCh.9

dy x dx -y = a.

+ (x + 3xy l/ Xl 3

1) dx = O.
2 )

2871. Va 2 x 2 dy --I- (x+ y- Val 2872. xyy'8_(X 2 +y2)y'+xy=O.

+x

dx= O.

2873. y = xy' y ~ 2874. (3x 2 2xy_ y 2) dx+ (x 2-2xy- 3y 2) dy =-- o. 2875. 2yp :: = 3p.,. 4y B.

+ .

Find solutions to the equations for the indicated initial conditions: 2876. y,=y+l; y=O for x= 1. x 2877. eX-Yy' = 1; y= 1 for x= 1. 2878. cot xy' + y = 2; y == 2 for x == O. 2879. eY (y' + 1) = 1; y == 0 for x == 0. I 2880. y' +y=cosx~ Y=2 for x===O.
2881. y'-2y=-x 2 ; Y=4 for x==O. 2882. y' +y=2x; !J=-l for x==o. 2883. xy' =y; a) y= 1 for x= 1; b) y=O for x=O. 2884. 2xy'=y: a) y=l for x=--= 1; b) y=O for x==O. 2885. 2xyy'+X I _ y 2=O; a) y=O for x=O; b)y==l forx=O; c) y==O for x=l. 2886. Find the curve passing through the point (0, I), for which the subtangent is equal to the sum of the cooldinates of
)

the point 'of tangency. 2887. Find a curve if we know that the sum of the segnlent~ cut off on the coordinate axes by a tangent to it is constant and equal to 2a. 2888. The sum of the lengths of the normal and subnorlna\ is equal to unity. Find the equation of the curve if it is kno\vn that the curve passes through the coordinate origin. 2889*. Find a curve whose angle formed by a tangent and the radius vector of the point of tangency is constant. 2890. Find a curve knowing that the area contained between the coordinate axes, this curve and the ordinate of any point on it is equal to the cube of the ordinate. 2891. Find a curve knowing that the area of a sector bounded by the polar axis, by this curve and by the radius vector of any point of it is proportional to the cube of this radius vector. 2892. Find a curve, the segment of which, cut orr by the tangent on the x-axis, is equal to the length of the tangent.

Sec. 91

Miscellaneous ExercIses on First-Order Differential Equations

343

2893. Find the c~rve, of which the segment of the tangent contained between the coordinate axes is divided into half by the parabola y2 =- 2x. 2894. Find the curve whose normal at any point of it is equal to the distance of this point from the origin. 2895*. The area bounded by a curve, the coordinate axes, and the ordinate of sOlne point of the curve is equal to the length of the corresponding arc of the curve. Find the equation of this curve if it is known that the latter passes through the point (0, 1). 2896. Find the curve for which the area of a triangle formed by the x-axis, a tangent, and the radius vector of the point of 2 tan~ency is constant and equal to a 2897. Find the curve if we know that the mid-point of the segnlent cut off on the x-axis by a tangent and a normal to the curve is a constant point (a, 0).
When forming first-order differential equations, particularly in ph\sical problelns. it is frequently advisable to apply the so-called method of differentials, which con~ists in the fact that approxirnate relationships between infinltesinlal i"crenlent~ of the desired quantities (these relationships are accurate to infinitesinlals of higher order) are replaced by the corresponding relationships between their dlfftr~nttals. This doe,; not affect the result. Problenl. A tani< contains 100 litre,; of an aqueous solution containing 10 kg of salt. Water IS rntcnng the tank at the rate of 3 htres per minute, and the mixture is f1owin~ out at 2 Idres per Jllinute. The concentration is maintained unlfonn by stirring. I-fo\v tTIuch salt \vill the tank contain at the end of one hOll r? Solution. The concentration c of a substance is the quantity of it in unit volume. If the concentration is uniforlTI, then the Quantity of substance in voltllTIe V is eV. Let the quantity of salt in the tank at the end of t minutes be x kg. The quantity of solution in the tank at that instant \vill be 100 t litres, and. conscquent\ y. the t'Oncentralion c = 100 + t kg per Ii Ire. Duril1~ tinle dt, 2dt litres of the solution 110\\'S out of the tank (the solution contains 2c dt kg of salt). Therrfore, a change of dx in the quantity of salt in the tank is given by the relationship 2x -dx=2c dt = 100 + t dt. This is the sought-for differential equation. Separating variables and integrating, we obtain In x= - 21n (100+ t) In C or C
x

x = ( 100

+ t)

I .

The constant C is found fronl the fact that \\ hpn t = 0, \ -= 10. tha tis, C= 100.000. At thr expiration of one hour, the tank will contain
X=

lOf~~~ ~ 3.9

kilograms or salt.

344

DiOerential Equations

(Ch. 9

2898*. Prove that for a heavy liquid rotating about a vertical axis the free surface has the form of a paraboloid of revolution. 2899*. Find the relationship between the air pressure and the altitude if it is known that the pressure is 1 kgf on 1 cm 2 at sea level and 0.92 kgf on 1 em! at an altitude of 500 metres. 2900*. According to Hooke's law an elastic band of length 1 increases in length klF(k=const) due to a tensile force F. By how much will the band increase in length due to its weight W if the band is suspended at one end? (The initial lenglh of the band is I.) 2901. Solve the same problem for a weight P suspended fr0l11 the end of the band. When solving Problems 2902 and 2903. make use of Newton's law, by which the rate of cooling of a body is proportional to the difference of temperatures of the body and the ambient 1l1edium. 2902. Find the relationship between the temperature T and the time t if a body, heated to To degrees, is brought into a room at constant temperature (a degrees). 2903. During what tirne will a body heated to 100 cool ofT to 30 0 if the temperature of the room is 20 and during the first 20 minutes the body cooled to 60? 2904. The retarding action of friction on a disk rotating in a liquid is proportional to the angular velocity of rotation_ Find the relationship between the angular velocity and tillle if it ic; known that the disk began rotating at 100 rpm and after one minute was rotating at 60 rpm. 2905*. The rate of disintegration of radium is proportional to the quantity of radium present. Radium disin1egrates by one half in 1600 years. Find the percentage of radium that has disintegrated after 100 years. 2906*. The rate of outflow of water from an aperture at a vertical distance h from the free surface is defined by the formula
0

v=c2gh,
where c ~ 0.6 and g is the acceleration of gravity. During what period of time will the water filling a hemispherical boiler of diameter 2 metres Aow out of it through a circular opening of radIus 0.1 m in the bottom. 2907*. The quantity of light absorbed in passing through a thin layer of water is proportional to the quantity of incident light ann to the thickness of the layer. If one half of the original quantity of light is absorbed in passing through a three-metrethick layer of water, what part of this quantity will reach a depth of 30 metres?

Sec. 101

IJigher.Order DttJerentlal Equations

345

2908*. The air resistance to a body falling with a parachute is proportional to the square of the rate of fall. Find the limiting velocity of descent. 2909*. The bottom of a tank with a capacity of 300 litres is covered with a nlixture of salt and some insoluble suhstance. Assunling that the rate at which the salt dissolves is proportional to lhe ditference between the concentration at the given time 8nd the concentration of a saturated solution (I kg of salt per 3 litres of water) and that the given quantity of pure water dissolves 1}3 kg of salt in 1 minute, find the quantity of salt in solution at the explraiion of one hour. 2910*. lhe electromotive force e in a circuit with current i, resistance R and self-induction L is made up of the voltage drop

Ri and the electromottve force of self-i nduction L :;. Determi ne the current i at tilne t if e ==- E sin rot (E and (J) are constants) and i = 0 when t = O.
Sec. 10. Higher-Order Differential Equations
to. The case of direct integration. If
y(/.)

= f (.\),

then

Y=~dx~ .,. ~f(.\)dX+CIXII-I+C1Xn-I+ ... +Cn'

'----..--" 1 'me's
2. Cac;es of reduction of order. 1) If a dJ1Yerential equation does not contaJn y cxpl1cltly, for instanc~,
F(x,

i/,

y")=O,

then, assuming y' = p, \\'e get an equation ot an order one unit lower:
F (x, p, p'):=O.

Example 1. Find the particular solut;on of the equation


xy" +y' +x=O,

that satisfies the conditions


y=O, y' =0 \vhen x=O.

Solution. Putting y' =p, we have !I' = p', whence


xp'+p+x=O.

Solving the latter equation as a linear equation In the function p.


we get

346

DtUerenltal Equations

[Ch 9

From the fact that y'=p=O when x=O, we have O=C.-O, Le., C1=O. Hence, x p=-2

or
dy x dx=-2'

whence. integrating once again,

w~

obtain
x:!

y=-4"t- Ct
Putting y=-O when x= 0, we find C 2 -= O. Hence. the desired solu lion is 1 X2 Y = _-4 .
2)

particular

If a differential equation does not contain x explicitly, for instance,


F(y, y'. y")=O

then, putting y' =p, y" =p dd P , \ve get an equation of all order one unit
y

lower:

F(y, p, p~~)=O.
Example 2. Find the particulnr solution of the pquation
yy"_y'2=y4

provided that y= 1, y'=O when x=O. Solution. Put y'=p, then if=pddP and our equation becomes
I

IJP : : -p"=

~/.

We have obtained an equation of the Bernoulli type in p (y is considered the argument). Solving it, we find
p=

VCI +y2.

From the fact that y' =p=O when y= 1, we have C1 = -1. lienee, p=
or

y V y2-1

:~= y Vy"-1.
Integrating, we have
arc cos -

t}

x = C2_
y=s~cx.

I Putting y=l and x=O, \ve obtain C2 =O, whence -=cosx or y

Sec. 10]

Higher-Order DiUeTential Equations

347

Solve the following equations:


2911. y" =
2912.

!-. x

2920. yy" = yly'


2921. yy" - y' (1 2922. y" = 2923. (x

+ y,l.
+ y') =
O.

y"= -2~S'

2913. y"

= 1 _ y,2.
==

2914. xy"t y'


2915. 2916. 2917. 2918. 2919.

O.

+ 1) y" -

x, .

(x -t- 2)y' + x+ + 2 = O.

yy"=y,2. 2924. xy"=y' In y'. x yy,,+y,z=O. 1 ,,2 " ' (ll-x 2 )y"+-y,2+ I ==O. 2925 y +-.r y =xy. y' (1 + y'2) = ay". 2926. xy'" + y" = 1 + x. x1y" -t-xy' = 1. 2927. y",Z + y"Z = 1. Find the particular solutions for the indicated initial conditions: 2928. (1 +x 2)y"-2xy'=O; y=O, y'=3 for x=O. 2929. 1 + y,Z = 2yy"; !I = 1, y' = 1 for x = 1. 2930. yy" + y,J = y,:i; y =:= 1, y' = 1 for x = O. 2931. xy"=y'; y=O, y'=O for x=O. Find the general integrals of the following equations: 2932. yy' = V y2 + y,Z y" _ y' y".
2933. yy" = y,Z -t- y' yZ y,2. 2934. y'" _ yy" = y2 y '. 2935. yy" y,2 _ y,ol In y =

V +

o.

Fi nd sol utions that satisfy the indicated conditions:


2936. y"y' = 1; y

= I, y' = 1

for x =

-} .

2937. yy"-t-y'2=I; y=I, y'= I for x=O. 2938. xy"=

VI

.~ y,Z; y=O for x= 1; y= I for x=e J


1

2939. y"(1 +lnx)-+-x y'=2+lnx;y=2' y'=l tor x=l.


2940. y" =

~ ( 1 + 1n ~'); y =

i ' y' = 1 for x = 1.

2941. y"_y'2+ y'(y_I)=O; y=2, y'=2 for x=O. 2942. 3y'y"=y+y,i_~1; y=-2, y'=O for x=O. 2943. y2+ y '2_2yy"=O; y=l, y'=1 for x=O. 2944. yy,+y,J+yy"=O; y=l for x=O and y=O for x=-l.

348

DiOerelltial Equations

[Ch.9

2945. 2y' + (y,2 -6x). y" = 0; y = 0, y' = 2 for x = 2. 2946. g'y. yg" -y'. = 0; y= 1, y' = 2 for x= O.

2947. 2yy"-3y,2=4y l; y=l, y'=O for x=O. 2948. 2yy" -t- yl_y,a = 0; y = 1, y' = 1 for x = O. ",. 1, 1 f 1 2949

= Y - y; Y = - 4"' y

2 or x = .

2950. Y"+;2eJ'2 Y'-2yy,2=O; y=l, y'=e for

xO-=-;e'

2951. 1 -t- yy" -1- y,Z = 0; y = 0, y' = I for x == I. 2952. (1 -t- yy') y" = (1 y,2) y'; y == 1, y' = 1 for x = o. 2953. (x+l)y"+xy,2==y'; y=-2, y'=4 forx=l.

2954. y' = xy,,2 -t y,,2. 2955. y' =xy" y" _y"z. 2956. y",1 = 4y".

Solve the equations:

2957. yy'y":=: y,3 -t- y"l. Choose the integral curve passing through the poi nt (0, 0) and tangent, at it, to the straight line y + x = O. 2958. Find the curves of constant radius of curvature. 2959. Find a curve whose radius of curvature is proportional to the cube of the norma 1. 2960. Find a curve whose radius of CUIvature is equal to the normal. 2961. Find a curve whose radius of curvature is double the normal. 2962. Pind the curves whose projection of the radius of curvature on the y-axis is a constant. 2963. Find the equation of the cable of a suspension bridge on the assumption that the load is distributed uniformly along the projection of the cable on a horizontal straight line. The weight of the cable is neglected. 2964*. Find the position of equilibrium of a flexible nontensite thread, the ends of which are attached at two points and which has a constant load q (including the weight of the thread) per unit length. 2965*. A heavy body with no initial velocity is sliding along an inclined plane. Find the law of motion if the angle of inclination is a, and the coefficient of friction is Jo'.
(Hint. The frictional force is J1N, where N is the force of reaction of the plane.)

2966*. We may consider that the air resistance in free fall is proportional to the square of the velocity. Find the law of

motion if the initial velocity is zero..

Sec. 111

Linear Differential Equations

349

2967*. A motor-boat weighing 300 kgf is in rectilinear motion with initial velocity 66 mjsec. The resistance of the water is proportional to the velocity and is 10 kgf at 1 metre/sec. How long will it be before the velocity becomes 8 m/sec?

Sec. 11. Linear Differential Equations


1. Homogeneous equations. The functions Yl=q>l (x), Y2=q>2(X), . . . . , Y = q>n (x) are called linearly dependent if there are constants C., CI , , Cn not alj equal to zero, such that C.y. + C1Y2 + ... + CnYn e= 0; otherwise, these functions arc called linearly t11dependent. The general solution of a homogeneous linear differential equation
y(1I)+Pl (x) y(fJ-1)

+ ... -1- P n (x)y=O


IS

(I)

with continuous coefficients P,.(x) (i=l, 2, ... , rz)


y = C1Y. + C~Y2 + . + CnYn,

of the fornl

where Y., YI' .. , lin are linearl y independent sol utions of equa tion (I) (fundamental system of SO/Ii/tons). 2. Inhomogeneous equations. The general solution of an inholnogeneolLs ltnear different ial eq ua tion y(n) PI (.v) y(II-I)+ + Pn (x) Y =f (x) (2)

\\,ith continuous coefficients P,. (x) and the right side

f (x) has the fonn

Y=Yo+Y,

where Yo is the general solution of the corresponding homogeneous ~qua lion (I) and Y is a particular solution of th~ given inhomogeneous equation (2). If the fundam~ntal system of soluHons 'il, y", ... , Yn of the homogeneous equation (I) is kno\vn, then the general solution of the corresponding inhornogeneous equation (2) Inay be found from the formula

y= C. (x) Y. +C 1 (x)

Y.+.. +C n (x) Yn'

where the functions C;(x) (i=l, 2, .. ,11) are determined from the follo\vjog system of equations:

C~ (x) Y. -t- C; (x) Y2


C; (x)

Y; + C~ (x) y~

+ +

+ C;l (x) Y. =

0,

+C~ (\)y~==O,

c~ (x) y~t1-2)

+C; (x) y~n-I) +

+C~ (x) y~n-I) = 0,


+ C~ (x) y~'-.) = f (x)

1 ~
J

(3)

C: (x) y~n-.) + C~ (x) y~n-.) +


(the method of uariation of parameters). Example. Solve the equation

xy"+y'=xl

(4)

350

DiOerential Equations

[Ch.9

Solution. Solving the homogeneous equation xy"+y'=O, we get y=C 1 In x+C 1

(5)

Hence, It may be taken that Yl=lnx andY2=1 and the solution of equation (4) may be sought In the form y =C 1 (x) In x + C2 (x). Forming the system (3) and taking into account that the reduced fonn of the equation (4) is y" + y' =x, we obtain x

Whence
X' x' x' CJ(x)=a+A and C2(x)=-31nx+g+B

and, consequentl y,
x' Y=g+A Inx+B,

where A and B are arbitrary constants.

2968. Test the following systems of functions for linear relationships: a) x, x + 1; e) x, x!, Xl ; b) xs, - 2x 2 ; f) e", e2 " , el x ; c) 0, 1, x; g) sin~, cos x, 1; d) x, x + 1, x -+. 2; h) sin 2 x, cos! X, 1.

2969. Form a linear homogeneous differential equation, knowing its fundalnental system of equations: a) Y1 = sin x, Y2 = cos x; b) Y1= eX, Yt = xe x ; 2 c) YI=X, Yt=x , x d) Yl=e , Yt=exsinx, Ya=e"co5x. 2970. Knowing the fundamental system of solutions of a linear homogeneous differential equation Y1=x, Yt=x!, y.=x' , find its particular solution y that satisfies the initial conditions
YI.~=l=O) y'I.~=l=-I)

Y"IX:l=2.

Sec. 12]

Linear DiOerential Equations with Constant Coefficients

351

2971*. Solve the equation


y" + ! y' )'

+y =

0,

. .ts par t leu . I ar so It si n x . k nowlng I u Ion U, = -x2972. Solve the equation x! (In x-I) y" -xy' -t- Y = 0,

knowing its particular solution Y = X. I By the method of variation of parameters, solve the follo\ving inhomogeneous linear equations. 2973. x1y" -xy' = 3x'. 2974*. X2Y"1-Xy'-y=x2.
2975. y'" -t-y'::.=secx. Sec. 12. Linear Differential Equations of with Constant Coefficients
S~cond

Order

1. Homogeneous equations. A second-order llncar equation \\'ith constant coefficients p and q WI thou t the right si de IS of th~ fOrIn y" + py' +qy =0 (I)

If k. and 11 2 are roots of the characteristic equation cr (k) === k' + pk +q ==- 0,
then the general solution of
til ree \va ys:
~quation

(2)

(1) is \\ritten in one of the following

1) y -.::- Cle".>" + C~ek.x if k , and k 2 are real nnd k,:l= k~; 2) y -= e'l.X (C. + C 2 x) if k, == k 2; 3) y -=e?'x eC , cns PX+C 2 SlIl ~.\) if k. =a-t-~t and k2-=-a-~1 ((3:f:. 0).

2. Inhonlogeneous eljuations. The general solution of a linear neous differential equation y"+py' -+qy=f(x)
01 ay be wntten in the form of a sunl:

lllhoI110~e

(3)

Y=Yo+ Y,

where Yo is the general solution of the corresponding equation (I) without rl~ht side and deternlined from fOrJnulas (1) to (:3), and Y is a particular solution of the given equation (3). The function Y may be found by the method of undetermined roefficl~llts in the following simple cases: l. f (x) =e ux PII (x), where Pn (x) is a polynomial of degree n. If a is not a root of the characteristic equation (2), that is, <p (a) ::1= O. then we put Y =e'J. x Qn (x) \\hcre Q,. (x) is a polynomial of degree n with undetermined coefficients. 1f a IS a root of the characteristic equation l2), that is. <p (a) =0. then Y=reuxQn(x), "here r is the multiplicity of the root a(r=l or r=2). 2. f (x)=eGX (P n (x) cos bx+ Q", (x) sin bx].

352

DifJerential Equation.s

(Ch.9

If q> (a bi) :F 0, then we put


Y = eax [SN (x) cos bx -1- TN (x) sin bx),

where SN (X) and T f:I (x) are pol ynomials of degree N-max {n, m}. But if q> (a btl = 0, then y = xr eax [S N (x) cos bx + TN (x) sin bx],

where, is the multiplicity of the roots a


r= 1).

bl (for second-order equations,

is

us~d

In the general case, the method of variation of parameters (see Sec. 11) to solve equation (3). Exall1ple t. Find the ~eneral solution of the equation 2y"-y'-y=4xe2". Solution. The characteristic equation 2k 2 - k - l =0 has roots III =- I and
The general solution of the corresponding homogeneous equation
x

k.= - ; .

(first type) is 1/ 0 = Cle" +C 2e z. 1 he right side of the given equation is f {x) = =4xe2X =tOXp"(x). l-Ience, Y=e 2X (Ax+8), since n=1 and 1-=.0. DiOeren tiating Y tWice and pu tting the derivatives into the given equa tion, we obtain:
2e 2X (4Ax+ 48
2X

+ 4A)-e

2X

(2Ax+ 28

+ A)_e

2X

(Ax-j- B)

= 4xe~x.

Can,cplling out e and equating the coefficients of identical powers of x and the absolute ternlS on the left and right of the equality, we helve 5A =4 and 4 28 7A+58=O, whence A=S and 8=-25.
Thus. Ye f '>'
( :

x- ~~) and the general solution of the given equa tion


Y= CIe X

IS

+C e-7 (+ e~.,x ( S-x-2"5. 4 28)


2

Example 2. Find the general solution of the equation y"-2y' +u=xe"". Solution. The charactenstlc equation k 2 -2k+] ==0 has a double root k = 1 The ri~hl side of th(:l equation l~ of the {orin f (x) =xex ; her~, a = I and n=-l. The particular solution is Y =x 2e" (Ax+ B), since a cOIncides \1Jith the doubl~ root k=-l and, consequently, r=2. Diflerentlating Y h\ice, substItuting into the equation, and equating the coerticil.>nts. we obtam A =

i.

B =0. Hence, the general solution of the given

equation will be \\rltten in the form


y=(C I +Cax) eX

+~

x'e".

Example 3. Find the Jl:en~ral solution of thE:' equa tion y" + I/=X sin x. Solution. The characteristic equation k 2 + l=-O has roots III = i and k 2 = - I . The general solution ot the corresponding honlogeneous equation will Isee 3, where a=O and p = 1J be

Yo=C. cos x+ C.. sin x.

The right side is of the form

I {xJ =eIJx IP n (x) cos bt + Qm (x) sin bxJ.

Sec. /2]

Linear DiUerential Equations with Constant Coefficients

353

where a =0, b = 1, P n (x) =0, QIn (x) =x. To this side there corresponds the particular solution Y, Y =x [(Ax+ B) cos x+ (Cx+ D) sin xl (here, N= I, a=O, b== 1, r= I). Differentiating twice and substituting into the equation, we equate the cOE'fficlcnts of bolh sides in cos X, x cos x, sin x, and x S1n x. We then get four equations2A+2D==O,4C=O, -2B+2C=O, -4A=1, [rom which we deterx2 x . 1 1 mine A = -4' 8=0, C=O, D=4 Therefore, Y= --:rcosx+-:rsinx. The general solution is
Xl x y= C. cos x+C 2 sin x-"4 ros x+T sin x.

30 The princi pie of superposition of sol utions. If the righf si de of equation (3) is the sum of several funct:ons

I (x) -=11 (x) +'2 (x) + ... + In (x)


and Yi (i = 1, 2, 3, ... , n) are the corresponding solutions of the equatIons y"+py',-qY=--.fi(x) (i=I, 2, , n), then the sum y=Yl+ Y 2+ ... Y n

is t he so1uti 0 n

0f

equa t ion (3).

Find the general solutions of the equations: + 6y = O. 2982. g" + 2y' + Y= o. 2977. y"-9y=O. 2983. y"-4y' +2y=0.. 2978. y" - y' == O. 2984. y" + ky = O. 2979. y" -I- Y == O. 2985. Y = y" + y'. 2980. y" - 2y' + 2y = O. y'-y 2981. y"t 4y' + 13y=O. 2986. 7 =3. Fi nd the particular sol utions that satisfy the indicated conditions: 2987. y" -5y' -J- 4y = 0; Y= 5, y' = 8 for x = 0 2988. y" + 3y' +- 2y = 0; Y = 1, y' = -1 for x = o. 2989. y" + 4y = 0; y = 0, y' = 2 for x = O. 2990. y" + 2y' = 0; y = 1, y' = 0 for x = 0
2976. y" -5y'

2991. y"=Y2; y=a, y'=O for x=O.


a

2992. y"+3y'=O~ y=O for x=O and y=O for x=3. 2993. y" n 2 y=O; y=O for x=O and y=O for x= 1. 2994. Indicate the type of particular solutions for the given

i Ilhomogeneous eq uat ions:

a) y"_4y=x 2 e2X ;

b) y"+9y=cos2x;
12-1900

354

DiUerential Equations

[Ch.9

c) y"-4y' -t-4y= sin 2x+elx ; d) y" + 2y' 2y= eX sin x; e) y" -5y' + 6y = (xl-t- I) e" + xelX~ f) y" - 2y' + 5y = xe" cos 2x-x1 e" sin 2x.

Find the general solutions of the equations: 2995. y" - 4y' + 4y = Xl . 2996. y" -y' + y = xlt 6. 2997. y" + 2y' + y = e2X 2998. y" -By' + 7y= 14. 2999. y" - y =-=: eX. 3000. y" y = cos x. 3001. y" + y' -2y = 8 sin 2x. 3002. y" + y' -6y = xe2X 3003. y"-2y' -t-y= sinx+sinhx. . 3004. y" + y' = sin 2 x. 3005. y" - 2y' -+ 5y = eX cos 2x. 3006. Find the solution of the equation y" + 4y= sin x that satisfies the conditions y = I, y' = 1 for x = O. Solve the equations: 3007. ~:~+<illx=ASinpt. Consider the cases: 1) P=/=<ilj

2) P =

0>.

3008. 3009. 3010. 3011. 3012. 3013. 3014. 3015. 3016.


3017.

y"-7y'+12y=-e4 ". y"-2y' =x 2 -1.

y"-2y'+y=2e x . y"-2y'=e 2X +5. y"-2y' -8y= eX -8 cos 2x. y" ~t y' = 5x + 2e x


y"-y'===2x-1-3e x . y" 2y' y= eX +e- x . y"-2y' IOy= sin 3x +e x

+ + y" -4y' + 4y =
+

2e IX

+ ~.

3018. y"-3y' =x+cosx.


3019. Find the solution to the equation y"-2y'=e1x +xl - l

that satisfies the conditions y=-lr. y'=l for x=O. Solve the equations: 3020. y"-y=2xsinx. 3021. y"_4y=e2X sin 2x. 3022. y" + 4y = 2 sin 2x-3 cos 2x+ 1. 3023. y" - 2y' + 2y = 4e x sin x. 3024. yIP = xe" -t- y. 3025. y" + 9y= 2x sin x+ xelX

Sec. 12)

Linear DiDerential Equations with Constant

Coefficient~

355

3026. y"-2y'-3y=x(1 +e'X ). 3027. y"-2y' =3x+2xe". 3028. y"-4y' +4y=xet ". 3029. y" + 2y' -3y = 2xe-" + (x -t 1) eX. 3030. y" + y = 2x cos x cos 2x. 3031. y" -2y = 2xe x (cos x-- sin x).

Applying the method of variation of paranleters, solve the following equations: 3036. y" + y = _1_. 3032. y" + y = tan x. cosx
3033. y" + y = cot x. 3034. y" -2y' 3035. y" 3037. yn + y = _1_.

+y= ex. x

1 snx

3038. a) y"-y=tanhx.
b) g" -2y = 4x 2 exJ

+ 2y' + y ="7.

e-"

3039. Two identical loads are suspended from the end of a spring. Find the equation of nlotion that will be p~rfornled by one of these loads if the other falls.
Solution. Let the increase in the length of the spring under the action of one load in a state of rest be a and the Jnass of the load, In. Denote by x the coordinate of the load reckoned vertically from the position of equilIbrium in the case of a single load. Then
d2 x m dt 2 =mg-k (x+a),

mg d~ where, obviously, k=a- and, consequently, dt 2


tion is d\':
x=CtCOS

V:

g =-a x.

The

~enerat

~olu-

t+Ctsln

V~ ~

t.

The initial condItIOns Yield x=a

and dt =0 when t =0; whence C, =a and C,=O; and so


X=QCOS

V:

t.

3040*. The force stretching a spring is proportional, to the increase in its length and is equal to 1 l<gf when the length increases by 1 cm. A load weighing 2 l<gf is suspended fron) the spring. f"ind the period of oscillatory motion of the load if it is pulled downwards slightly and then released. 3041*. A load weighing P=4 kgf is suspended from a spring a.nd increases the length of the spring by 1 em. Fi,nd the law of motion of the load if the upper end of the spring performs a vertical harmonic oscillation y=2sin30tcm and if at the initial instant the load was at rest (resistance of the medium is neglected). i12

356

DiOerent fat E quat ions

[Ch. 9

3042. A material point of mass m is attracted by each of two centres with a force proportional to the distance (the constant of proportionality is k). Find the law of motion of the point knowing that the distance between the centres is 2b, at the initial instant the point was located on the line connecting the centres (at a distance c from its midpoint) and had a velocity of zero. 3043. A chain of length 6 metres is sliding from a support without friction. If the motion begins when 1 m of the chain is hanging from the support, how long will it take for the entire chain to slide down? 3044*. A long narrow tube is revolving with constant angular velocity ro about a vertical axis perpendicular to it. A ball inside the tube is slidin5 along it without friction. Find the law of nlotion of the ball relative to the tube, considering that a) at the initial instant the ball was at a distance a from the axis of rotation; the initial velocity of the baH was zero; b) at the initial instant the ball was located on the axis of rotation and had an initial velocity vO
Sec. 13. linear Differential Equations of Order Higher than Two with Constant Coefficients
to. Homogeneous equations. The fundamental systelTI of solutions Yl' Y., .. , Un of a homogeneous linear equation with constant coefficients
y(n)+a1y(n-I)+ .. +an_ty' +any =0
equation (2)

(1)

is constructed on the basis of the character of the roots of the characteristic

Namely, t) if k is a real root of the equation (2) of multiplicity m, then to this root there correspond m linearly independent solutions of equation (1):
Yl =ekx , YI = xek" , ... , Y m =xm-'ekx ;
2) if a ~l Is a pair of complex roots of equation (2) of multiplicity nl, then to the latter there correspond 2m linearly independent solutions of equation (I):

1I.=e'l.xcospx. y.=e'l.Xslnpx,

Ya=xe'l.Xcos~x, Yf,,=xe'l.Xsin~x, ... ... Yam_l=x m - 1e'l.x cos ~x, Y2m=X m - 1e'l.x sin ~x.

2. Inbomogeneous equations. A particular solution of the inhomogeneou equation y(n) -t a1ycn -1) + ... + all - 1y' + anY = f (x) (3)

II sought on the basis of rules 20 and 3 of Sec. 12.

Sec. 141

Euler's Equations

357

3049. y"'-3y"+3y'-y=O. 3050. ylV -f-4y==O. 3051. ylV t- 8y"+ 16y=O. 3052. yIV+y'=O. 3060. 3058. y'V -2y" + y = O. 3061. -6054. ylV _ally = O. 3062. 3055. ylV -6lJ" + 9." = O. 3063. ':1 3064. 3056. ylV + aly" = O. 3065. 3057. ylv+2y"'+t/'=O. 3066.

Find 3045. 3046. 3047. 3048.

the general solutions of the equations: y"'-13y"+12y'=O. 3058. ylv+2y""+y=O. y'" _y' = O. 3059. y(n) -+!!..} yCn-I)-t_ y'" +y=O. ylV -2y" = O. -J- n (n -1) en-I)
)2

Y +~1 y' +y=O.

...

yIV_2y"'+y"=e x ylV -2y'" + yIP = Xl. y'" -y = xl-I. ylV -t- y'" = cos 4x. y'" +- yIP = x" + I + 3xe-. y'" + y" y' + y = xex' y"'+y'=tanxsecx. 3067. Find the particular solution of the equation y'" +2y" +2y' +y=x

that satisfies the initial conditions y(O)=y'(O)=y"(O)==O.

Sec. 14. Eul er's Equations


A linear equation of the form (ax + b)n y \ll) 1- AI (ax -t- b)n-l y (n-1l + ...

+ An

-1

(ax

+ b) Y + AnY -= f (x),

(1)

where a, b. AI' ... , An_I' An are constants, IS called EIller's equation. Let us Introduce a nc\v Independent vanable t, putting ax -1- b -=e t . Then 91 = ae- t r!:!!. y" ==a!e-~t 1_

' Y

dt'

(ddIe! d!/) dt'


)

II' _

:s -:st (d " ~ d 11 dl l -ae dl:J- dt l -l- 2 dt


3 2
l

an d so for th

and Euler's equation is transfornled into a linear equation with constant coefHclcll ts. fxarnple 1. ~olve the equation x"y" + xy' y = 1. Solution. Putting x==-e t , we get

dy= _tdl/ dx edt'

dll/= -2t(d!Y_ d l/\

dx"

\ dt"

dt ) .

Consequently, the given equatio 1 takes on the fonn


d!y

dt,,+y=l,

whence

or

y=C I cos t +C" sin t + 1


g= C1 cos (In x)

+ C. sin (In x) + 1.

358

DiOerential Equations
For the homogeneous Euler equation xny(n) + A1xn-1y,n-u+ ... + An_1xy' + Any=O

[Ch. 9

(2)
(3)

the solution may be sought in the form Y=~.

Putting into (2) y, y', ... , y(n) found from (3), we get a characteristic equation from which we can find the exponent k. If k is a real root of the characteristic equation of multiplicity m, then to it correspond m linearly independent solutions k Yl =xk. Y 1 = /l.ln x, Ys =x (In X)I, Ym =xk (In x)m-l. If a ~i is a pair of complex roots of nlultipllcity m, then to it there correspond 2m linearly independent solutions Yl =x~ cos (~ In x), YI = xt% sin (~ In x), Ys =xt% In x cos (~ In x), Ya= x fJ ln xsln (~ In x), ... , Y2m-l = x fJ (1n x)m-l cos (~ In x). Y2m = x fJ (In x)m-l sin (~ In x).
Example 2. Solve the equation
x 2y -3xy' +4y=O.
rt

Solution. We put

y=xk.

y'=kX k -

y"=k(k-l)xk -

Substituting into the given equation, after cancelling out xk we get the characteristic equa tion
Solving it we find

Hence, the general solution will be Y = Ct x2 + Ct,x 2 In x.

Solve the equations:


3068. 3069. 3070. 3071. 3072.
X dx l
2
2

d2 y

+ 3x dy dX + y =

O.

x y"-xy'-3y=O. x 2 y" -f- xy' +. 4y = O. xSy'" -3x' y" 6xy' -6y = O. (3x --1- 2) y" 7y' = O.

3073. y" = 2~.


x

Y' Y 3074. y"+-+"!=O. x x 3075. xy"-4xy' +6y=x. 3076. (1 +X)I y"-3(1 +x)y' +4y=(1 +x).

Sec. lt51

Systems of DiUerential Equat ions

359

3077. Find the particular solution of the equation x 2 y" -xy' + y= 2x that satisfies the initial conditions y= 0. y' = 1 when x = 1.
Sec. 15. Systems of Differential Equations
Method of elimination. To find the solution, for instance, of a normal systenl of two first-order differential equations, that is, of a system of the fonn dy dz (1) dx=f(x,y,z), dx=g(x, y, z). solved for the derivatives of the desired functions, we differentiate one of them with respect to x. We have, for example,
d2g

dX 2 =ax+ayf+

of

af

az g.

af

(2)

Determining z from the first equation of the system (1) and substituting the value found,

z=q> ( x, y,

:~)

(3)

into equation (2), we get a second-order equation with one unknown function 1/. Solving it, we find Y = '" (x, CI' C2 ), (4) where C I and C2 are arbitrary constants. Substituting function (4) into formula (3), we determine the function z without new integrations. The set of fornlulas (3) and (4), where y is replaced by 'i'. yields the general solution 01 the system (1). Example. Solve the system

:~ +2y +4z= 1 +4x,


{ dz
3 2 dx+ y - z ="2 X

Solution. We differentiat-e the first equation with respect to x:

dly +2~+4 dZ=4. dx 2 dx dx


From the first equation we determine

Z=} ( 1 + 4x- :~ -2Y )

and then

frOln th esecond we
and

WIll

dz 3 2 1 3 1 dy havedx=2x+x+4-2Y-4dt(.

. Putting

:~

into the equation obtained after differentiation, we arrive at a secord2

order equation in one unknown y:

dx'

dy

+ dx dy -6y=- 6x

-4x+3.

360
Solving it we find:

DifJerential Equations

(Ch.9

and then
1 ( 1 +4x---2y dY ) = -C e2X +-!e C _ 3X _ _ 1 X2 z=1
4 dx
4

We can do likewise in the case of a system wi th a larger number of equations.

Sol ve the syste ms:


3078.

~=z.
{

dz dx=-Y.

3085. { dz

:~ +3y+4z

=2x,

dx-Y-z=x,

3079.

dz { dx -J- Y + 3z = o.

:~=Y+5z,

y=O. z=O when x=O.


3086.

3080.

~=-3Y-z.
dz { dx =

~; -4x-y + 36t =0,


d~ +2x--y +2e'=O,
y2

y-- z.

x=O, Y= 1 when t =0.


3087.

( ~7=Y' 3081. t dt =
dl/
Z,

:=T'
=

dy

dx=2 Y .

l :;=x.
dt =Y dy
dz

* dx dy dz. 30 88 a) X 1+-3 2 1y = -y 2z ' xy 2=2


b) ~= dy =dz.

dx

+z,

x-y y-z

x+y dy

z'

c) ~

3082.

dt=x+z,

z-x

=.Ex-y'

t dt=x+y.
3083.

isolate the integral curve passing through the point (I, 1, -2).
d!J+ = 1 d" Z , 3089. { dz 2 dx + x2 Y = In x.
d2

:~=y+z,
dz

dx=x+y+z.

3084.

{ dx -4y- 2z = cos x.
dz

~ + 2y + z =

sin x,

3090.

d~ +2y+4z = eX,
y - 3z = -x.

2 { d z dx 2 -

Sec. 16]

Integration of DiOereniial Equations by Power Series

361

3091**. A shell leaves a gun with initial velocity V o at an angle a to the horizon. Find the equation of motion if we take the air resistance as proportional to the velocity. 3092*. A materia I point is attracted by a centre 0 with a force proportional to the distanc~. The motion begins from point A at a distance a from the centre with initial velocity V o perpendicular to OA. Find the trajectory.

Sec. 16. Integration of Differential Equations by Means of Power Series


If it is not possible to integrate a differential equation with the help of elementary functions, then in some cases Its solution may be sought in the form of a power series:
co

y = ~ Cn (x-xo)n.
n=o

(1)

The undetermined coefficients cn (n = 1, 2, ... ) are found by putting the series (I) into the equation and equating the coefficients of identical powers of the binomial X-X o on the left-hand and right-hand sides of the resulting equation. We can also seek the solution of the equation
y'-:=f(x, y); y(xo)=--=Yo
(2)

in the form of the Taylor's series


y(x) =

L. y(n~~xo) (x-xo)n,
n=o

(3)

where y (x o) = Yo, y' (x o) = f (x o' Yo) and the subsequent dprivatives y(nl (x o) (n =- 2. 3, ... ) are successively found by difT(r~ntiating e'luation (2) and by putting Xo in place of x Example 1. Find the solution of the equation
y"-xy=O,

If y=Yo, y' =y; for x:=.:O. Solution. We put

whence, differentiating, we get

y" = 2.lc.

+3.2c.x +... + n (n-I) cn x n - + (n + 1) ncn + Xn - + + (n + 2) (n + I) en + 2xn +...


2 1 1

Substituting y and y" into the given equation, we arrive at the identity

[21c. + 32cax + ...

+ n (n-I) cnx n - + (n + I) nCn +lX n - 1 + + (n +2) (n + I) cn + 2x n + ... ]-x (CO+c1x + ... +cnxn + ... ] ==:0.
I

Collecting together, on the left of this equation, the terms with identical powers of x and equating to zero ihe coefiiclents of these powers, we Will

362

DiOerent ial E quat ions

[Ch.9

have
cJ=O;

32c.-co=O, c'=3.2;
C,

Co

4.3c4 -cJ =O, c4 =4.3'


and so forth.

CJ

5 .4c,-c.=O.

= ;'14

Generally,
cak

2.3.5.6 ..... (3k-l) 3k'


C ak+2=O

3.467 ... 3k (3k + lr (k= 1,2,3, ... ).


cak+1 =
)

Consequently f
X' xci xk y=co ( 1+ 2.3 + 2.3.5.6 +... + 2.3.56 ..... (3k-l) 3k +...
X4

+
)

+C

x7

x,kq. 1

1 (

x+3.4+3.4.6.7+-J- 3.4.6.7 .... 3k(3k+l)+ ,

(4)

where Co = Yo and C1 = Applying d'Alembert's test, it is readily seen that series (4) converges for - 00 < x < + 00. Example 2. Find the solution of the equation y' = x y; Yo = Y (0) = 1. Solution. We put " ", , -Yo 2 Yo a Y=Yo+Y ox+2f x + 31 x + ...

Y;.

We have yo=l, y~=O+l= 1. Differentiating equation y'=x+y, we succes. fiIn d y" = 1+y, ' Yo= " 1+ 1=2, Y" =y, ' " Yo ,,, = 2,etc. Consequent 1y, sJvely

y= 1 +x+2f x

2 a +31 x + ...

For the examp'le at hand, this solution may be written in final form as y=1+x+2(eX -l-x) or y=2eX-l-x. The procedure is sim ilar for differential equations of higher orders. Testing the resulting series for convergence is, generally speaking, COIDplicated and is not obligatory when solving the problems of this section.

With the help of power series, find the solutions of the equations for the indicated initial conditions. In Examples 3097, 3098, 3099, 3101, test the solutions obtained for convergence. 3093. y'==y+x2 ; y=-2 for x=O. 3094. y'=2y+x-l; y=yo for x=l.
3095. y'

=+
yl
I

Xl;

=;

(or

=O.

3096. y'=X _ y 2; y=O for x=O. 3097. (l-x)y' = 1 +x-y; y=O for x=O.

Sec. 17)

Problems on Fourier's Method

363

3098*. xy" + y = 0; Y = 0, y' = 1 for x = o. 3099. y"+xy=O; y=l, y'=O for x=O. 3100*. y,,+J:..Y'-t-y=O; y~l, y'=O for x=O. x
3101 *. y"
2

3102.

+ 2- y' + y = 0; y = 1, y' = 0 for x = O. dx dx dt + x cos t = 0; x = a; dt = 0 for t = O.


X
2

Sec. 17. Problems on Fouriefs Method


To find the solutions of a linear homogeneous partial differential equation by Fourier's method, first seek the particular solutions of this special-type eq uation, each of which represents the product of functions that are dependent on one argument only. In the simplest case, there is an infinite set of such solutions un (n= l, 2, ... ), which are linearly independent among themselves in any finite number and which satisfy the given boundary conditions. The desired solution u is represented in the form of a series arranged according to these particular solutions:
(I)
n=l

The coefficients
condit ions.

en

which reInain undetermined are found from the initial

Problem. A transversal displacement u =U (x, t) of the points of a string with abscissa x satisfiE:'s, at time t, the eqnation

att. =a
where at.

au
2

:2

au ax
2

'

(2)

To (To is the tensile force and Q is the linear density of the Q string). Find the form of the string at ti tne t if its ends x =0 and x= I are

u
O~--_-..._---~--.-

r'--'

Fig. 107

fixed and af the initial instant, t = 0, the string had the form of a parabola
U

= ~: x

(i-x) (Fig. 107) and its points had zero velocity.

Solution. It is required to find the solution u = u (x, t) of equation (2) that satisfies the boundary conditions u (0. t) =0. u (I, t) =0 (3)

364 and the initial conditions

Differential E quat ions


4h u (x, 0) = j2 x (I-x), ,

(Ch.9

Ut

(x, 0) =0.

(4)

We seek the nonzero solutions of eauation (2) of the special form U = X (x) T (t). Putting this expression into equation (2) and separating the variables, we get T" (t) X" (x) (5) aIr (t)= X (x) Since the variables x and tare indeprndent. equation (5) is possible only when the ~eneral quantity of relation (5) is constant Denoting this constant by - AI. we find two ordinary dIfferential equations: T" (t) + (aA)2. T (t) = 0 and X" (x) + ).,IX (x) =0. Solving these equations, we get T (t) = A cos aAt + B sin a).,t. X (x) = C cos ).,x + D sin ).,X. where A, B. C, D are arbitrary constants. Let us determine the constants. From condition (3) we have X (0) = 0 and X (I) = 0; hence, C = 0 and sin Al = 0 (since D cannot be equal to zero at the same tinle as C is zero).
For this reason, Ak =

~n ,

where k is an mteger. It will readily be seen that

we do not lose generality by taking for k only positive values (k = 1. 2, 3...). To every value Ak there corresponds a particular solution kan . kan ) . knx uk= ( Ak cos-/- t + Bk sin -1- t sin -llhat satisfies the boundary conditions (3). We construct the series
CJ)

~ ( kant . kant) . knx U= ~ Ak cos -1-+Bk s1n -1- Sill -,k=J

whose sum obviously satisfies equ3 tion (2) and the boundary conditions (3). We choose the constants Ali ann Bk so that the sum of the series should satisfy the initial conditions (4). Since
CJ)

au ~ kart ( at = ~ -/- k=J

Ak

. kant
Sill - , -

k~nt) knx + Bk cos -1sin -l-

it follows that. by putting t = 0, we obtain


CJ)

~ . k:rrx 4h (x, O ) = ~ A k sin -1- == j2 x (I-x)

and

k=J

au (x,
iJt

CJ)

0)

~ kan . krrx ~ -l- B k sin -lk==J

c:::iO.

Sec. 17]

Problems on Fourier's Method

365

Hence, to determine the coefficients All and Bk it is necessary to expand in


a Fourier series, in sines on1 y, the function u (x, 0) = f unc t Ion

;2'; x

(I-x)

and

the

au (x, at

0)

==.

From familiar formulas (Ch. VII I, Sec. 4,3) we have

r4h knx 32h , dX=n1k A~=T J lI x (l-x)sinI

'

if k is odd, and All = 0 if k is even:


kart 2 -L- Bk=T

SOSin-,-dx=O, B,,=O.
I

knx

The sought-for solution will be


_32h~
00

u-

n3

cos----l . (2n+ l)nx (2n -1- 1)1 sin I

(2n

-t- I) ant

n=o

3103*. At the initial instant t = 0, a string, attached at its ends, x = 0 and x = I, had the form of the sine curve u = A sin 'IT/x, and the points of it had zero velocity. Find the lorrn of the string at time t. 3104*. At the initial time t = 0, the points of a straight string 0 < x < I receive a velocity ~~' = 1. Find the form of the strin~ at titne t if 1he ends of the string x=O and x=l are fixed (see Problem 3103). 3105*. A string of length 1 = 100 em and attached at its ends. x=O and x=l, is pulled out to a distance h=2 cm at point x == 50 cnl at the initial time, and is then released without any impulse. De,ernline the shape of the string at any time t. 3106*. In longitudinal vibrations of a thin homogeneous and rectilinear rod, whose axis coincides with the x-axis, the displacenlent u = U (x, t) of a cross-section of the rod with abscissa x satisfies, at time t, the equation
iJ2

dt2 =
l

au ax
2

where a =.!!(E is Young's modulus and Q is the density of the Q rod). Determine the longiturlinal vibrations of an elastic horizontal rod of length 1 = 100 cm fixed at the end x = 0 and pulled back at the end x = 100 by Iii = 1 CIU, and then released without Impulse.

366

DiDerential Equations

[Ch. 9

3107*. For a rectilinear homogeneous rod whose axis coincides with the x-axis, the temperature u = u (x, t) in a cross-section with abscissa x at time t, in the absence of sources of heat, satisfies the equation of heat conduction

au

at =a

ax! ,

au
2

where a is a constant. Determine the tenlperature distribution for any tinle t in a rod of length 100 cm if we know the initial tenlperature distribution
U

(x t 0) = 0.01 x (IOO-x).

Chapter X

APPROXIMATE CALCULATIONS

Sec. 1. Operations on Approximate Numbers


to. Absolute error. The ab~olute error of an approximate number a which replaces the exact number A is the absolute value of the difference bet\\'een them. The number ti, which satisfies the inequality
IA-al~L\,

(1)

is called the limIting absolute error. The exact nUlnber A is located within the litnits a-&~A~a+& or, more briefly, A=a 6. 2. Relative error. By the relative error of an approximate nUlnber a replacing an exact number A (A > 0) we understand the ratio of the absolute error of the number a to the exact nurnber A. The number ~, which satisfies the ineq uality I A-at ~ (2) A 1Iilrl::a, is called the limiting relative error of the approximate number a. Since in actual practice A =::::a, we often take the number 6=~ for the lilniting
a

relative error. 3. Number of correct decimals. We say that a positive approximate number a written in the fornl of a decimal expansion has n correct decimal places in a narrow sense if the absolute error of this number does not exceed one half unit of the nth decinlal place. In this case, when n > I we can take, for the limIting relative error, the number

6=2~(I~r-1
where k is the first significant digit of the number a. And conversely, if it 1 ( 1 ' n-l is known that 6E;;; 2 (k + I) 10 ) . then the number a has n correct deci mal places in the narrow meaning of the word. In particular, the number a

definitely has n correct decimals in the narrow meaning if f> E;;; ~ n . If the absolute error of an approximate nUlnber a does not exceed a unit of the last decimal place (such, for example, are numbers resulting from measurements made to a definite accuracy). then it is said that all decimal places of this approximate number are correct in a broad sense. If there is a larger number of significant digits in the approxinlate number, the latter (if it is the final result of calculations) is ordinarily rounded off so that all the remaining digits are correct in the narrow or broad sense.

(/0)

368

Approximate Calculations

[Ch. 10

Hencetorth, we shall assume that all digits in the initial data are correct (if not otherwise stated) in the narrow sense. The results of intermediate calculations may contain one or two reserve digits. We note that the exanlples of this sectiCJn are, as a rule, the results of final calculataons, and for this reason the answers to them are given as approximate numbers with only correct drcimals. 4. Addition and subtraction of approximate numbers. The limiting absolute error ot an algebraic sum of sevE'ral numbers is equal to the sum of the limiting absolute errors of these numbers. Therefore, in order to have, in the sum of a ~mal1 number of approximate numbers (all decimal places of which are correct), only correct digits (at least in the broad sense), all summands should be put into the form of that summand which has the smallest number of decimal places, and in each summand a reserve digit should be retained. Then add the resulting numbers as exact numbers, and round off the sum by one decimal place If we have to add approximate numbers that have not been rounded off, they should be rounrled off and one or two reserve digits ~houl d be retained. Then be guided by the foregoing rule of addition while retaining the appropriate extra digits in the sunl up to the end of the calculations. Example 1. 215.21 +14.182+21.4==215.2l1)+14.1(8)+21 4=250.8. The relative error of a sunl of poc:;itive terms bes between the least and greatest relative errors of these terms. The relative error of a d,n'erence is not amenable to simple counting. Particularly unfavourable in this sense is the ditference of two close nunlbers. Example 2. In subtracting the a~proximate numbers 6 135 and 6.131 to four correct decimal places, we get the dIfference 004. The limiting relative

~ 0.00\ ~ 0/'01 I error is B 0.004 =4=0.25. Hence, not one of the decimals of the difference is correct. Therefore, it is always advisable to avoid subtracting close approximate numbers and to trttnsform the given expression, if need be, so that this undesir~!-Ie operation is omitted. 5. Multiplication and division of approximate numbers. The limiting relative error of a product and a quotient of approximate numhers is equal 10 the sum of the linliting relative errors of these numbers Proceeding froln this and applying the rule for the number of correct decilTaals (3), we retain In the answer only a definite nunlber of decimals Example 3. The product of the approximate numbers 25.34.12 = 104.236. Assuming that all dEcimals of the factors are correct, we find that the limiting relative error of the product is

6=2.20.01

I 01 ~0.003. + 4.2.

Whence the number of correct decimals of the product is three and the result, if it is final, should be written as follows: 25.34 12 = 104, or more correctly, ~5 34.12= 104 2 (J.3. 6. Powers and roots of approximate numbers. The limitIng relative error of the mth power of an approxi mate number a is equal to the m-fold linliting relative error of this number The limiting relative error of the mth root of an approximate number a is the 7. Calculating the error of the result of various operations on approxi-

..!.th m

part of the limiting relative error of the number a.

mate numbers. If lill., ... flail are the limiting absolute errors of the appro-

Sec. I)
xirnate numbers

Operations on Approximate Numbers


all _

369

an' then the limiting absolute error ~S of the result

S === f (at~ . . . an)


may be evaluated approximately from the formula

~S = I:~ I~al + +I :!n IM n


The limiting relative error S is then equal to

6S =

~; I = I :1.1 f;; + .+I:!n It'tr =


+ . + Idln'l dan tian

= dln'l da L\a l l

Example 4. Evaluate S = In (10.3+ V4_4); the approximate numbers 10.3 and 4.4 are correct to one decimal place. Solution. Let us first compute the limiting absolute error L\S in the
general form: 8=ln (a+ vb),

~S= a+ ~

b(~a +~ ;bb)'

We

have

~a=M~ ;0; V4."4 =


is then equal to

2.0976... ; we leave 2.1, since the relative error of

the approximate number 4.4 is equal to

~} ~= ;0;

the absolute error

~ 2 8~) = ~;

we can be sure of the first decimal place. Hence,

1 ( 1 1 1) 1 ( I) 13 L\S=IO.3+2.1 20+2--20.2.1 =12.4.20 l+U =2604=::::0.005. Thus, hvo decimal places will be correct. Now let liS do the calculations with one reserve decinlal: log (10.3 + y44) ~ log 12 4 -== 1.093, In (10 3+ yU)~ 1.0932.303 = 2.517. And we ~('t the answer: 2 52 8. Establishing admissible errors of approximate numbers for a given error in the result of operations on them. Arplying the formulas of 7 for the quantities L\S or f,S given us and conSIdering all particular differentials

Ia~ I~ak
ab~olute

or the quantities a~ ~,ai equal, we calculate the admissible errors L\a" ... , L\a n , " of the approximate numbers at ... an' that enter into the operations (the principle of equal effects). It should be pointed out that sOlnetimes when calculating the adlnissible errore;; of the argul11ents of a function it is not advantageous to use the principle of equal effects. since the latter nlay make delllands that are practically unfulfilable In these cases it is advisable to nlake a reasonable redistribution of errors (if thi~ is possible) so that the overall total error does not exceed a specified quantity. Thus, strictly speaking, the problem thus posed is indeterm inate. Example 5. The volume of a "cylindrical segment", that is,''a ~olid cut off a circular cylinder by a plane passing through the diameter of the base (equal to 2R) at an angle a to the base, is computed from thp formula

I I

V=

; R'

tan a. To what degree of accuracy should we measure the radius

370

Approximate Calculations

[Ch. 10

R:::::: 60 cm and the angle of inclination a so that the volume of the cylindrical segment is found to an accuracy up to 1%? Solution. If l\V, l\R and L\a are the limiting absol ute errors of the quantities V, R and a, then the limiting relative error of the volume V that we are calculating is 3L\R 2L\a 1 6=R+sin2a~100

We assume

3L\R

1 2L\a 1 ~ 200 and sin 2a. ~ 200 Whence


R 60 cm l\R E;;; 600 ~ 600 = 1 mIn;
A sin 2a 1 d 9' ua ~ 400 <400 ra lan~

Thus, we ensure the desired accuracy In the ans\ver to I % if we measure the radius to 1 mm and the angle of inclination a to 9'.

3108. Measurements yielded the following approximate numbers that are correct in the broad meaning to the nunlber of decimal places indicated: a) 1207'14"; b) 38.5 cm; c) 62.215 kg. Compute their absolute and relative errors. 3109. Compute the absolute and relative errors of the following approximate numbers which are correct in the narrow sense to the decimal places indicated: a) 241.7; b) 0.035; c) 3.14. 3110. Determine the number of correct (in the narrow sense) decimals and write the approxinlate numbers: a) 48.361 for an accuracy of 1%; b) 14.9360 for. an accuracy of 1 ?1l; c) 592.8 for an accuracy of 2%. 3111. Add the approximate numbers, which are correct to the indicated decimals: a) 25.386 + 0.49 +3.10 + 0.5; b) 1.2.102 + 41.72 + 0.09; c) 38.1 +2.0 +3.124. 3112. Subtract the approximate numbers. which are correct to the indicated decimals: a) 148.1-63.871; b) 29.72-11.25; c) 34.22-34.21. 3113*. Find the difference of the areas of two squares whose measured sides are 15.28 cm and 15.22 cm (accurate to 0.05 mm). 3114. Find the product of the approximate numbers, which are correct to the indicated decimals: a) 3.49 8.6; b) 25.1 1.743; c) 0.02 16.5. Indicate the possi ble limits of the results.

Sec. 1]

Operations on Approximate Numbers

371

3115. The sides of a rectangle are 4.02 and 4.96 m (accurate to 1 cm). Compute the area of the rectangle. 3116. Find the quotient of the approximate numbers, which are correct to the indicated decimals:

a) 5.684 : 5.032; b) 0.144 : 1.2; c) 216:4. 3117. The legs of a right triangle are 12.10 cm and 25.21 cm (accurate to 0.01 cm). Compute the tangent of the angle opposite

the first leg. 3118. Compute the indicated powers of the approximate numbers (the bases are correct to the indicated decimals):
a) 0.4158 2 ; b) 65.2 1 ; c) 1.5 2 3119. The side of a square is 45.3 cm (accurate to I mm). Find the area. 3120. Compute the values of the roots (the radicands are correct to the indicated decimals):
a) Y2.715; b) V65.2; c) J,/81.1.

3121. The radii of the bases and the generatrix of a truncated cone are R=23.64 cmO.OI cm; r-=17.31 clnO.OI cm; 1= = 10.21 em + 0.01 cm; 1t == 3.14. Use these data to compute the total surface of the truncated cone. Evaluate the absolute and relative errors of the result. 3122. The hypotenuse of a right triangle is 15.4 cm + 0.1 cm; one of the legs is 6.8 cm 0.1 cm. To \\yhat degree of accuracy can we determine the second leg and the adjacent acute angle? Find their values. 3123. Calculate the specific \veight of aluminiulTI if an aluminium cylinder of diameter 2 cm and altitude II em weigh5 93.4 gmt The relative error in measuring the lengths is 0.01, while the relative error in weighing is 0.001. 3124. Compute the current if the electromotive force is equal to 221 volts 1 volt and the resistance is 809 ohms I ohm. 3125. The period of oscillation of a pendulum of length l is equal to T

T=21t

V-

where g is the acceleration of gravity. To what degree of accuracy do we have to measure the length of the pendulum, whose period is close to 2 sec, in order to obtain its oscillation period with a relative error of 0.5%? How accurate must the numbers 1t and g be taken? 3126. It is required to nleasure, to within 1%, the lateral surface of a truncated cone whose bas~ radii are 2 m and 1 m, and the generatrix is 5 m (approximately). To what degree of

872

Approximate Calculations

[Ch, 10

accuracy do we have to measure the radii and the generatrix and to how many decimal places do we have to take the number n? 3127. To determine Young's tnodulus for the bending of a rod of rectangular cross-section we use the formula
I

=4 d3bs'
where I is the rod length, band d are the basis and altitude of the cross-section of the rod, s is the sag, and P the load. To wha t degree of accuracy do we have to measure the length land the sag s so that the error E should not exceed 5.50A) , provided that the load P is known to 0.1 ~u, and the quantities d and b are known to an accuracy of 1%, I ~ 50 em, S ~ 2.5 cm?

liP

Sec. 2. Interpolation qf Functions


to. Newton's interpolation formula. Let Xo, Xl' , Xn be the tabular values of an argument, the difference of which h=~xj (L\Xj=Xj+l-xi; i=O,I, .. , n - 1) is constan t (table Inter val) and YOt Yl' ., YtI art' the correspon din~ values of the function y Then the valup of the function y for an intermediate value of the argument x is approximately given by Newton's interpolation formula

Y=Yo+q.,1yo+ q (q2~ I) ,12yO + ... + q (q-I). n~q-n + I) ,1nyo

(1)

x-Xo where q = -hand ~Yo = YI- Yo, L\2yo = L\y.- !lyo,

are successIve fini te

dU'erences of the furction y. \\ hen x=Xj (t=O, 1, , n), the ro1ynomial (1) takes on, accorciingly, the tabular values Yi (l =0, 1, . . , n). As particular cases of l'\ewtc,n's fOrlllula we obtain: for n= 1, linear Interpolation; for n ~ 2, qupdratlc Interpolation. To simplify the use of Newton's fonnula, it IS advisable first to set up a table of finite differences. If Y=f (x) is a polynomial of degree n, then
L\nYi = const and
~n+IYi=-O

and, hence, formula (1) i~ exact In tile general case, if f (x) ha') a continuous derivative f (n+J) (x\ on the interval la, b), which includes the points x o, Xl' , x n and x, then the error of formula (1) is

R ()n
X -

Y- k..
;=0

"n....

q(q-l) ... (q-i+l) If

~'Yo_
(2)

whE're ~ is some intermediate value between xi (I =0, 1, ... , n) and x. For practlcal use, the {ollowing approximate formula is more convenient:

Sec. 2)

Interpolation 0/ Functions

373

If the number n may be any number, then it is best (0 choose it so that the difference ~n+ lyO::::: 0 w,thin the "mits of the given accuracy; in other words. the difterences 6.'l yo should be constant to wi thin the given places of decimals Example 1. Find sIn 26"15' using the tabular data sin 260 =0,43837.
sin 27 ~ 0.45399, sin 28 -:- 0.46947.

Solution. We set up the table


X,

I
1

Y,

I I
AI/,

A'II.

o 1
2

27(1 28

26

0 42837 0 45399 0 46947

I 1562 I --14 1548

Here, 11=60', q= 60' 4 . Appl ying formula (1) and uSing the first horizontal line of the table, we have
sin 2615' =0.43837+

261fi'-26

f
31

.!- ('!--l
~,

0.01562+ 4

' . (-0.00014)=0.44229.

Let us evaluate the error R2 USing formula (2) and taking into account that if y = Sill x, then I y,nl I ~ 1, we will have:

IR21~

f(f- I )({-2)(3t)3 =12d5rl.333~410. 7 1


I
_I

180

Thus, all the decimals of sin 2015' are corr~ct. Using Newton's formula, it is alsc pOSSible, from a ~iven intermediate value of the function y. to find the correspolldlng value of the argument x (inverse interpolation). To do this. first deterrnlne the corresponding value q by the method of successIve approx,nlatlon, putting
q'O) =Y-Yo tiyo

and

(I) ( ( I ) 1) A2 q(l+l)==q(O)_L:L-=-. -lh._ 21 I!J.Yo

(I

= 0, 1. 2. . .. ).

Here, for q we take the common value (to the given accuracyl) of two successive approximations qC1n) '='lcm+l). Whence x=xo+q.h. Exanlple 2. USln!! the table

I
2 2 2 4

II=-

,lnl1

4.457 5.466

1.009
1 229

0.220

2 6

6.695

approximate the root of the equation Sinh x = 5.

'874

Approximate Calculations
Solution. Taking Yo = 4.457, we have

(Ch 10

(I) _

q -q

5-4.457 0.543 -0 538. 1.009 =1.009- , (0) + q(O) (l_q(O A2 yO =0 538 + 0.538.0.462 0.220 =
(0)_

~Yo'

= 0.538 +0 .027 = 0.565;

1.009

qIZ)=0.538+ 0.565;0.435.

~:~:=0.538+0.027=0.565.

We can thus take x= 2.2+0.5650.2 =2.2 +0.113=2.313.


2. Lagrange's interpolation formula. In the general case, a polynomial of

degree n, which for x=xi takes on given values Yi (i =0, I, ... , n), is given by the Lagrange interpolation formula
JJ

(X-Xl~ (X-X z\ . .. \X-Xn ) ) Yo X-Xo~ ~X-X2)) .(~-Xn) )YI (X O -x1 (XO-x 1 Xo-X n X1-X O XI -X 2 X1 -Xn (x-x o) (X-Xl)" .(X-Xk_l) (X-Xk+l)" . (X-X n ) (Xk-Xo) (Xk-Xl)" (Xk- Xk-I)(Xk- Xk+l)' .. (Xk-Xn) Yk (X-X o) (X-Xl)" .(X-Xn - 1) ... (Xn-X o) (Xn-X J ) (Xn-X n - l )

+ ...

Yn

3128. Given a ta hie of the values of x and y:


x

I I

2
3
10

3
15

I
I

4
12

5
9

Set up a

tabl~ of the finite differences of the function y. 3129. Set up a table of differences of the function y=x s _ -5x 2 +x,-l for the values x= 1, 3, 5, 7, 9, II. Make sure that all the finite differences of order 3 are equal. 3130*. Utilizing the constancy of fourth-order differences, set up a ta ble of differences of the function y = x 4 - IOx l + 2x -t- 3x for integral values of x lying in the range I ~ x ~ 10.

3131. Given the table

log 1 = 0.000, log 2 = 0.301,


log 3 = 0.477,

log 4 = 0.602,
log 5 = 0.699.

Use 1inear interpolation to compute the numbers: log 1.7, log 2.5,

log3.i, and log 4.6.

Sec. 2]

Interpolation of Functions

375

3132. Given the table

sin 10=0.1736, sin 11=0.1908, sin 12 = 0.2079,

sin 13=0.2250, sin 14=0.2419,

sin 15 = 0.2588.

Fill in the table by computing (with Newton's formula. for n = 2) the values of the sine every half degree. 3133. Form Newton's interpolation polynomial for a function represented by the table

40

85

3134*. Form Newton's interpolation polynomial for a function' represented by the table
x
y

I
x

2 3

I I
4

8 50

10 83

I 11 I 27

I
2

Find y for x=5.5. For what x will y=20? 3135. A function is given by the table
-2 I

I I

25

-8

1-151 -23

Form Lagrange's interpolation polynomial and find the value of y for x=O. 3136. Experiment has yielded the contraction of a spring (x 111IU) as a function of the load (P kg) carried by the spring:

I I 10 P I 49 I 105
x
5

15 172

20 253

25 352

30 473

35

40 793

619

Find the load that yields a contraction of the spring by 14 rom..

376

Approximate Calculations

[Ch. 10

3137. Given a table of the quantities x and y


x
y

131415
-3 I 25 1 129

381

\A>mpute the values of y for x = 0.5 and for x = 2: a) by means of linear interpolation; b) by Lagrange's formula.
Sec. 3. Computing the Real Roots of Equations
10 Establishing initial approximations of roots. The approximation of the roots of a given equation (1 ) f (x) =0 consists of two stages: 1) separating the roots, that is. establishing the intervals (as small as possible) within which lies one and only one root of equation (1); 2) computing the roots to a given degree of accuracy If a function I (..\) is defined and continuous on an interval [a. b] and f(a).' (b) < 0. then on [a. b) thrre is at least one root ~ of equaticn (1). This root will definitely be the only one if I' (x) > 0 or f' (x) < 0 when a <x < b. In approximating the root; it is advisable to use millimetre paper and construct a graph of the function y = I (x). The abscissas of the r-oints of intersection of the graph with the x-axis are the roo~s of the equation f (x) =0. It is sometimes conveni(:'nt to replace the given equation with an equivalent equation q> (x):.-:: 1f (~). Then the roots of the equation are found as the abscis~as of f,oints of intersection of the graohs l/ = {fJ (x) and y == (x). 2. The rule of proportionate parts (chord method). If on an interval (a. b] there i~ a unique root ; of the rql1ation f (>.) = O. where the function f (x) is continuous on [a. b), then by repla( ing the curve y = f (x) by a chord passing through the points [a, f (a)l and [b. I (b)), we obtain the first approximation of the root
C1 = Q -

I (b)-I (a) (b-a).

f (a)

(2)

To obtain a second approximation c2 , we apply formula (2) to that one of the intervals (a, cll or [Cl' b) at the ends of which th(:' function f (x) has values of oppcsHe sign. The succeeding approximations are constructed in the same manner. The sequence of numbers en (n = I, 2, ..) converges to the root ~. that is,

c2 ,

Generally sreaking, we should continue to calculate the approximations CIt until the decitnals retained in the araswer cease to change (in accord with the specified de~ree of accuracy!); for intermediate calculations, take one or two reserve decimals Thi~ is a general remark. If the function f (.\) has a nonzero continuous derivative f' (x) on the interval [a. b], then to evaluate the absolute error of the approximate root

Sec. 3]

Computing the Real Roots of Equations

377

en' we can make use of the formula P:-c 1~lf(cn)1


where J..t =
~
n
a~.t<.b

J..t'

min

f I' (x)

3. Newton's method (method of tangents). If f' (x) :1= 0 and f" (x) :A 0 for a ~x ~ b, where I (a) f (b) < 0, f (a) (a) > 0, then the successive approximations X n (n = 0, 1, 2, ... ) to the root ~ of an equation f (x) =0 are computed froln the formulas n f (x 1 2, .... ) (3) xo=a, xn=x - -!' ( - t) ) ( rz.=,

r'

Xn -

Under the given assumptions, the sequence tonic and lim xn=~.
n-+e:tJ

X n (n

= 1,

2, ... ) is nl0no-

To eval uate the errors we can use the formula

where f.1 = min

a~x~b

I f'

(x)

I.

For practical purposes it is 1110re convenient to use the simpler formulas xo=a, x,,=xll _ t -af(x n - t ) ~n=l, 2, ... ), (3')
where a =

t ~a)'

which yield the same accuracy as formulas (3).

If f (b) /" (b) > 0, then in fornlula'; (3) and (3') we should put xo=b. 4. Iterative Inethod. Let the given equation be reduced to the form
x==q>(x), (4)

where' <P' (x) I ~ , < 1 (r is constant) for a ~ x ~ b. Proceeding from the initial value xo' which belongs .to the interval [a, bJ, we butld a sequence of numbers Xl' X 2 , according to the following law:
X1=q> (x o)' x 2 =q> (Xl)' . , xu=q> (X"_l)' (5)

If a <. X n ~ b (n = 1, 2, ... ), then the 1itnit ;= lim x n


n
~
IX)

sive ap proximat ions to the root ;. The evaluation of the absolute error of the nth approximation to xn is given by the formula

is the only root of equation (4) on the interval [a, b); that is, xn are succes-

Therefore, if xn and x,,+t coincIde to within e, then the limiting absolute error for xn will be - 8 1 In order to transform equation f (x) =0 to (4), we replace the latter with an equivalent equation x=x-Af (x), where the number A:. 0 is chosen so that the function d!! [X-AI (x = I-Af' (x)
.t

-r

.378

Approximate Calculations

[Ch. 10

should be small in absolute value in the neighbourhood of the point Xo (for example, we can put I-At' (x o) =0]. Example 1. Reduce the equation 2x-ln x-4 =0 to the form (4) for the initial approximation to the root Xo = 2.5.. Solution. Here, f(x}=2x-lnx-4; f'(X)=2-1.. We write the equivax lent equation X=X-A (2x-ln x-4) and take 0.5 as one of the suitable val ues of A; this number is close to the root of the equation

,II-A ( 2
I

=0, that is, close to 1..!-6:::::: 0.6. x x-2.5 . The initial equation is reduced to the form x=x-0.5 (2x-lnx-4) x=2+2"lnx.
I

_..!-) I _

or

Example 2. Compute, to two decimal places, the roof; of the preceeding equation that lies bet\veen 2 and 3. Computing the root by thJ; iterative method. We make use of the result of Example 1, putting Xo = 2.5. We carry out the calculations using formulas (5) with one reserve decimal.
xt

=2+2" In 2.5::::::2.458,

I x2 =2 +"2 In 2.458:::::: 2.450,

xa=2+ In 2.450:::::: 2.448, 2


x4 =2 +2 In 2.448 ~ 2.448.
I

And so ~ ~ 2 45 (we can stop here since the third decimal place has 'become fixed) Let us nQw \evaluate the error. Here, 1 lp(X)=2+{lnx and lp'(X)=2 x. Considering that all approximations to xn lie in the interval [2.4, 2.5], we get
f=max I lp' (x)

'=2.~.4=0.21.

Hence, the limiting absolute error in the approximation to x. is, by virtue of the remark made above,

~= 1~~~1
Thus, the exact root
~

0.0012::::: 0.001.

of the equation lies within the limits 2 447

< ~ < 2.449;

we can take '~2.45, and all the decimals of this apprOXimate number will be correct in the narrow sense.

Sec. 8)

Computing the Real Roots of Equations

379'

Calculating the root by Newton's method. Here,


f(x)=2x-lnx-4,

f' (X)=2-~,

f (3) 1" (3) > O. Hence, the conditions


We take
Q=

On the interval 2 ~ x ~ 3 we have:

l' (x) > 0 and


()f

3 for

Xo =3

r (xl> 0;

are fulfilled.

f (2) f (3) < 0;

1 ( 2- 3

)-1 =0.6.

We carry out the calculations using formulas (3') with two reserve decimals: x 1 =3-0.6(2.3-ln3-4)=24592; XI =2.4592-0.6 (22 4592-ln 2 4592 -4) = 2 4481; x, = 2. 4481-0.6 (2 2. 4481-ln 2.4481-4) = 2. 4477; x 4 =2.4477 -0.6 (2 2 4477 - In 2.4477 -4) =2 4475. At this stage we stop the calculations, since _the third decimal place does not chan~e any more. The answer is: the root. ;=2.45. We olnit the evaluation of the error. 5. The case of a system of two equations. Let it be required to calculate the real roots of a system of two equations in two unknowns (to a glven degree of accuracy): f (x, y) =0, (6) { q> (x, y) =0, and let there be an initial approxinlation to one of the solutions (~, l)) of this system x = X o' Y = Yo' This initial approximation may be obtained, for example, graphically. by plotting (in the sallle Cartesian Loordinate systenl) the curves f (x, y) =0 and q> (XI Y) ::::::0 and by determining the coordinates of the points of intersection of these curves. a) Newton's method. Let us suppose that the functional determinant
1= a(I, q a (x, y)

does not vanish near the initial approximation x=xo' y=Yo' Then by Newton's method the first approxitnate solution to the system (6) has the form XI = X o + uo' Yl = Yo ~o' where ao' ~o are the solu tion of the syst~ln of two. linear equa lions I (x o' Yo) aof~ (xo Yo) ~of~ (x o, Yo) = O.

+ + q> (x o, Yo) +o.oq>: (x o, Yo) + ~oq>~ (x o, Yo) = O.


X 2 =X 1

The second approximation is obtained in the very same way:

+U

l ,

Y2=Yl

+ Pl'
YI) =0,

where a p

~I

are the solution of the systeln of linear equations

f (Xl' {

YI) +alf~ (Xl' YI)

+ P.f~ (Xl'

q> (x I' !II)

UI

cp~

(x I' Y I)

+ ~ Icp;, (Xl'

YI) = O.

Similarly \ve obtain the third and succeeding approximations.

380

Approximate Calculat tons

(Ch

10

b) Iterative method. We can also apply the Iterative method to solving the system o( equations (6), by transforming this 'iystem to an equivalent one
x= F (x, y), { y=cD (x, y) (7)

and assuming that

IF~(x.y)I+I<D~(x. y)l~r<l; IF~(x.

Y)f+I<D:J(x.

y)l~r< 1

(8)

in some two-dimensional neighbourhood U of the initial approximation (xo' Yo), which neighbourhood also contains the exact solution (s, 11) of the system. The sequence of approximations (x n , Yn ) (n === I. 2, ... ), WhlCh converges to the solution 0' the system (7) or, what IS the sanle thing, to the solution .of (6), is constructed according to the following law:
XI = F (x o, Yo), YI = cD (x o, 110)' xz=F (XI' YI)' Yz=<1) (Xl' YI)' X a = F (x z, Y2)' Ya-= cD (x 2' Y2)'

If all (x n, Un) belong to U, then lhn xn ==~,


n
-+
o7J

lim y" = 1').


-+
\XJ

The following technique IS advIsed for transforming the system of equal tions (6) to (7) with condition (8) observed. \Ve consider the system of equations at (x. Y) + ~q> (x, y) -= 0,
{

vt (x,

y)

+ 6q> (x,

y)

= 0,

which is equivalent to (6) provided that


x= X

I~: ~

:;l:

o.

Rewrite it in the form

+ at (x, y) + ~(P (x, y) ==: F (x, y), y == Y + vI (x, y) -+- (~tp (x, y).;:; <}) (x, y).
1

Choose the parameters a, ~, y, ~ such that the partial derivatives of the functions F (~, y) -:nd <D (x, lJ) will Ul equal or close to zero In the Initial arprOXlIllatlun; In other words, we find u, ~, y, ~ as approxImate solutions o the systeln of equations
1

+ at: (xo'
at~
(x o'
(x o'

vf~
1

+- ~cp~ (xo, Yo) + ~q>~1 (x o, Yo) + ~q>~ (.to'


Yo)

Yo)

= 0,

Yo) = 0, lJo)

+ yt~1 (xo, Yo) + ~CPII (x o'

= 0, 110) = O.

Condition (8) will h(' ohserved in such a choice of parameters a, p, y, fJ on the ilSSUlllption that the partial derivatives of the functions t (x, y) and q> (.\, !J) do not vary very rapidly in the neighbourhood of the initial approximation (x o, !lo>' Example 3. Reduce to the form (7) the sYliitem of equations
x2+y2_1=O, { x3 -li=0

.given the initial approximation to the root xo=0.8. Yo =0.55.

Sec. 3]

Computing the Real Roots of Equations

381
y;

Solution. Here,
"" (x o, Yo) = 1.1;

. Write down the system (that is equivalent to the initial one)


{ a (x
2

I (x, y) = X2 + y!-l, q> (x, y) =xl q>~ (xo' Yo) = 1.92, q>~ (x o' Yo) = - 1.
V(x2+y2-1)+~(xl_y)=O

I~ (x o, Yo) = 1.6,

+ y2_1) -t- ~ (x

_y)

=0,

(I a, ~ 1#= 0)
y, l>
y).

in the form
x=x+a(xl+yl-l)+~

y = y +V (x 2 + y2_1)

+ ~ (x

(xl_y),
l -

For suitable numerical values of a, system of equations

~,

y and l> choose the solution of the

{
l. e., we put a

1+1.6a+l.92~=0, 1.1a-~=0, 1.6y -t- 1. 92~ = 0, l+l.ly-~==O;

~-O.3, ~ ::::::-0.3, V:::::::-0.5, 6 ~ 0.4. Then the system of equations x=x-O.3(x2+y2-1)-0.3(xl-y), { y == y -0.5 (x 2 + y2_1) + 0 4 (xl_y),

which is equivalent to the initial system, has the fornl (7); and in a sufficiently sInall neIghbourhood of the pOInt (x o, Yo) condition (8) will be fulfilled.

Isolate the real roots of the equations by trial and error, and by means of the rule of proportional parts compute thenl to two decinlal places. 3138. x 3 - x -I 1===0.
3139. x4-~-05x-I.55=0. 3140. x l -4x --I == O.

Proceeding fronl the graphically found initial approximations, use Ne\vton's rnethod to compute the real roots of the equations to two decimal places: 3141. x l -2x-5=O. 3143.2 x ==4x.
3142.2x-Inx-4=0.

3144.logx=-. x

Utilizing the graphically found initial approximations, use the iterative method to C01l1pute the real roots of the equations to two decirna I places: 3145. xl -5x--t 0.1 =0. 3147. x'-x-2=O. 3146. 4x == cos x. Find graphically the initial approximations and compute the real roots of the equations and systerns to two decinlals: 3148. x'-3x+ 1 =0. 3151. xlnx-14=0. 3149. x l -2x 2 + 3x-5 = O. 3152. Xl + 3x-0.5 = O. 3150. x 4 +Xl - 2x-2 = O. 3153. 4x-7sin x = o.

382

Approximate Calculations

[Ch. 10
2

3154. XX +2x-6 = O. 3155. eX + e- 1x -4 = O.


3156.
l y2

{x + -1=O, xl_y=O.

3157. { x +y-4=O, y-Iogx-l =0.

3158. Compute to three decimals the smallest positive root of the equation tan x = x. 3159. Compute the roots of the equation x tanh x = 1 to four decimal places.
Sec. 4. Numerical Integration of Functions
1. Trapezoidal formula. For the approximate evaluation of the integral
b

) f (x) dx
a

[I (x) is a function continuous on

[a, b]] we divide the interval of integratiol1


11

[a, b] into n equal parts and choose the interval of calculations h = b-a .

Let xi=xo+ih (xo=a, xn=b, i=O, 1, 2, ... , n) be the abscissas of the parti tion points, and let Yi = f (xi) be the corresponding values of the integrand Y = f (x). Then the trapezoidal formula yields
b

Sf(x)dx=::h (Yo~Yn+YI+Y.+"'+Yn_l)
a

(1)

with an absolute error of

h2 Rn ~ 12 (b-a).M 1 ,

where M2=maxlr(x)1 when a<ax~b. To attain' the specified accuracy e when evaluating the integral, the interval h is found from the inequality
12e h 2 ~(b-a) M 2
2

t )

That is, h must be of the order of Ye. The value of h obtained is rounded off to the smaller value so that b-a
-h-=n

should be an integer; this is what gives us the number of partitions n. Having established hand n from (1), we compute the integral by taking the values of the integrand with one or two reserve deciJnal places. 2. Simpson's formula (parabolic formula). If n is an even number, then in the notation of 1 Simpson's formula

S f(x)dx=::~ ((Yo+Yn)+4(YI +Y.+ ... +Yn-I)+


G

+2(Y2+Y,+ . +Yn_2)

(3)

Sec. 4)

Numerical Integration

01

Functions

383

holds with an absolute error of

h4 Rn ~ 180 (b-a) M"

(4)

where M 4 =max I t lV (x) I when a~x<b. To ensure the specified accuracy e when evaluating the integral, the interval of calculations h is determined from the inequality
180 (b -a) M 4 <; 8.

hC

(5)

That is, the interval h is of the order

ve. The number

h is rounded off

to the smaller value so that n = b-a is an even integer. n Remark. Since, generally speaking, it is difficult to determine the interval h and the nunlber n associated with it from the inequalities (2) and (5), in practical work h is determined in the form of a rough estimate. Then, after the result is ohtained, the number n is doubled; that is, h is halved. If the new result coincides with the earlier one to the number of decimal places that we retain, then the calculations are stopped, otherwise the procedure is repeated, etc. For an approximate calculation of the absolute error R of Simpson's quadrature formula (3), use can also be made of the Runge princIple, according to which

where 11 and ~ are the result~ of calculations from formula (3) with interval h and II = 2h, respectivel y.

3160. Under the action of a variable force F directed along the x-axis, a material point is made to move along the x-axis froln x = 0 to x = 4. Approximate the work A of a force F if a table is given of the values of its tTIodulus F:
x
1

0.0

0.5 11.0 11.5

1 2 .0

1 2.5

3.0

I
I

3.5 6.75

4.0 10.00

F 11.50 10. 75 1 0.50 I 0. 75 11.50 1 2 .75 1 4.50

Carry out the calculations by the trapezoidal formula and by the Simpson formula.
I

3161. Approximate

~ (3x l -4x)dx by the trapezoidal formula


o

putting n = 10. Evaluate this integral exactly and find the absolute and relative errors of the result. Establish the upper limit L\ of absolute error in calculating [or n = 10, utilizing the error formula given in the text.

384

Approximate Calculations

[Ch. 10

3162. Using the Simpson formula, calculate

5:~Xl
o

to fOUf

decimal places, taking n = 10. Establish the upper limit d of absolute error, using the error formula given in the text. Calculate the following definite integrals to two decimals:
3163. 3164. 3165.

5l+x dx
0

3168. 3169. 3170.

5SI~X dx.
0

51+x dx
0 I
0

5SI: x dx.
0

51+x, dx
I
1

5co;x dx.
2
I

n
2

3166. ~ xlogxdx. 3167.

3171.

5l+x xd x.
cos
0
1
0

5IO~x dy.
I

3172. ~ e- X2 dx.

5 ~XX2 by applying the substitution x = +. Verifl the calculations


U)

3173. Evaluate to two decimal places the improper integral

by applying

Simps~~'s
b

formula to the integral


2

51 ~\2'
I

where b

is chosen liO that

5 l~XI<{. 10-

3174. A plane figure bounded by a half-wave of the sine curve y = sin x and the x-axis is in rotation about the x-axis. Using the Sitnpson formula, calculate the volume ot the sol id of rotation to two decilnal places. 3175*. Using Simpson's formula, calculate to two decimal . x2 y2 places the length ot an arc of the ellIpse T+ (0.6222)2= 1 situated in the first quadrant.
Sec. 5. Numerical Integration of Ordinary Differential Equations
to. A method of su(cessive approximation (Picard's nlcthod). Let there be given a first-order differential equation
y'=/(x, y)
subject to the initial condition y=Yo when x=x o.
(})

Sec. 5]

Numerical Integration of Ordinary DiOerential Equations

385

The solution y (x) of (1), which satisfies the given initial condition, can, generally speaking, be represented in the form
y (x) = lim Yi (x)
1-+
r:D

(2)

where the suCCeStHVe approximations Yi (x) are determined from the fonn ulas

Yo (x) = Yo,
x YI (x)

=Yo+
(i =

~f
Xo

(x, YI_I (x)) dx

0, 1, 2, ... ).

If the right side

f (x,

y) is defined and continuous in the neighbo~lrhood

R{lx-xoJ~a, IY-Yo'~b}
and sa t isfies, in this neighbourhood, the Lipschitz cond itlon

(L. is constant), then the process of successive approximation (2) definitely converges in the interval
Ix-xol~h,

where h =

m~n ( a, ~)
_

and M = m;x I f (x, y) I. And the error here is


n

Rn-,y(x)-Yn(x)I~/v'L

I X-X o In+l
(n+I)1

'

If

I x-xo ' ~ h.
The method of successive approximation (Picard's method) is also applicable, with slight nl0difications, to normal systems of differential equations. Differential equations of higher orders rnay be written in the fonn of systems of differential equations. 2. The Runge-Kutta method. Let it be required, on a given Interval X o ~ x ~ X, to find the solution Y (x) of (1) to a specified degree of accuracy 8. To do this, we choose the interval of calculations h=X-xo by dividing the interval [x o, Xl into n equal parts so that h4 xi are determined from the formula

< 8.

The partition points

xl=Xo+ih

(t

=0, 1, 2, ... , n).

By the Runge-Kutta method, the corresponding values Yi=Y (x;) of the desired function are successively computed fron] the formulas
Yi+1 =Yi+ ~Yi.

~Y'-.!. +2k(i)+2k(t)+k(i '-6 (k(i) 1 I ..,

13-1900

38.::.6

.-,.;A:..:.!:..p!:....pr:....:o:.....x--.im_at_e_C_a_lc_u_la_t_io_n_s

[ C_"'h_._10

where

(3)

To check the correct choice of the interval h it is advisable to verify the quantity

e_
-

k(i)-k(t)

2 a k~t)-k~t)

The fracti'Jn should amount to a few hundredths, otherwise h has to be reduced. The Runge-Kutta method is accurate to the order of hI. A rough estiIn ate of the error of the Runge-Kutta method on the given interval [x o, Xl Illay be obtained by proceeding from the Runge principle:

R = I Y2rn--Ym 15

'

where n = 2rn, Y2tn and are the results of calculations using the schenle (3) \vith interval h and interval 2h. The Runge-Kutta method is also applicable for solving SYStClllS of dlffe~ rential equations y'=f(x, y, z), z'-=cp(x, y, z) (4) with given initial conditions Y==Yo, z=zo when x==x o. 3. Milne's method. To solve (1) by the Mtlne nzethod, subject to the initial condi\ions Y=Yo when x=xo, we in SOlne way find the successIve values YI = Y (XI)' Y2 = Y (x 2 ), Y, == Y (x a) of the desired function y (x) [for instance, one can expand the solution Y (X) in a series (Ch. IX, Sec. 17) or find these values by the lTlethod of successi ve approximation, or by using the Runge-Kutta Inethod, and so forth]. The approximations and for the following values of Yi (i =4, 5, ... , n) are successively found frool the formulas

Ym

Yi

Yi

Yl=Yi-.+ ~h (2/; __ -/; __ +21;_.), }


&::

Yi=Yi-2+

3" (Ii +4Ii-l + Ii-I)'


To check we calculate the quantity

h _

(5)

"'here fi= f (xi, Yi) and

fl= f (xi, Yl).


1

ei=29 YI-Yi

1-

=:7

(6)

Sec ..5]

Numerical Integration of Ordinary Differential Equations

387

If 8i does not exceed the unit of the last decimal 10- m retained in the answer for Y (x). then for Yi we take and calculate the next value Yi+t, repeating the pr8cess. But If 8i > 10-tn, then one has to start from the beg1nning and reduce the interval of calculations. The Illagnitude of the initial interval is detennined approXinlately fronl the inequality hoi < lo-m. For the case of a solution of the system (4), the Milne formulas are written separately for the functions y (x) and z (x). The order of calculations remains the same. Example 1. Given a differential equation y' =y-x \vith the initial condition y (0)= 1.5. Calculate to two decinlal places the value of the solution of this equation when the ar~ument is x -= 1.5. Carry out the calculations by a combined Runge-Klltta and Milne method. Solution. We choose the initial Interval It frOll) the condl tion hoi < 0.01. To avoid involved writing, let us take h-=0.25. Then the entire interval of integration frool x=O to x= 1.5 is divided into six equal parts of length 0.25 by means of points xi (i == 0, 1, 2, 3, 4, 5, 6); we denote by Yi and the corresponding values of the solution y and the derivative y'. We calculate the first three values of y (not counting the initial one) by the Runge-Kutta ruethod [frotH fonnulas (3)]; the remaining three values - Y4' Ys, Y6 - we calculate by the Milne I11ethod [from formulas (5)] The value of Ye will obviously be the answer to the problem. We carry out the calculations with two reserve decilnals according to a definite schenle consisting of h\'o sequential Tables I and 2. At the end of Table 2 we obtain the answer. Calculating the valueYl' Here, f(x, y)=-X+IJ, xo==-O, Uo=1.5

!Ii

y;

h = 0.25. !J.Yo =

~ (k~O) + 2k~O) -I- 2k~O) + k~O =


=

(0.3750+20.3906+20.3926+0.4106) =0.3920;

k~O)

== f (x o,

Yo) h ==

(- 0 -1- 1.5000)0.25 -= 0.3730;


k(O)

k~O)=f ( xo+~.
k~O)=f ( x o

Yo+-+) h=(-0.125+ 1.5000+-0.1875) 0.25= 0.3906;


k(O) )

+2 '

Yo++

h=(-O 125+1.5000+0.1953)0.25=0.3926;

k~O) == f (xo-t-/z,

Yo+ k~O h = (- 0.25

+ 1.5000 + 0.3926) 0.25 =

0.4106;

approximate number are guaranteed). Let us check:

YI =Yo+ ~Yo== 1.5000+0.3920== 1.8920 (the first three decilnals In thts

8=

k~O)_k~o) = 1 0 .3750-0.39061 "'='156=0.13.

k~O) _k~O)

10.3906-0.3926 1

20

By this criterion, the interval h that we chose was rather rough. Sim ilarly we calculate the values Y2 and Y. The results are tabulated in Table 1.
13*

388

ApproxImate Calculations

[Ch. 10

Table 1. Calculating

.vI' YJ' .vI


I

f(x, y)=-x+y;

by the Runge-Kutta Method. h=0.25

Value of i

xl

YI

Yi~

k(i)
J

t (Xi+: '
Y'+T

== f (xi,

Yi)

kIll)

k(i)

1
2

0 0.25 0.50 o 75

1.5000 1.8920 2.3243 2.8084

1.5000 1 .6420 1 .8243 2.0584

0.3750 0.4105 0.4561 0.5146

1.5625 1.7223 1.9273 2.1907

0.3906 0.4306 0.4818 0.5477

Value of i

t( Xi+i '
Yi+T
k~i) )
1 .5703 1.7323 1 .9402 2.2073

k(i)

f (xi h, Yt-j- k~i

k(i)
4

~Yi

Yl+1

0
]

2
3

0.3926 0.4331 0.4850 0.5518

1.6426 1.8251 2.0593 2.3602

0.4106 0.4562 0.5148 0.5900

0.3920 0.4323 0.4841 0.5506

1 .89-20 2.3243 2.8084 3.3590

Calculating the value of .P4. We have: f (x, y) = - x+ y, h=O.25, x4 = 1; Yo= 1.5000, YI = 1.8920, Y2 = 2.3243, Y. = 2.8084;
I

y~ = 1.5000,

Y; = 1.6420,

y~ = 1.8243,

y; = 2.0584.

Applying formulas (5), we find


Y4=Yo " + 4h 3 (2Y -Ya + 2y.) =
1

= 1.5000 + 4.0 25 (21.6420-1.8243

if, = f (x it) = 4,

h -/ " Y'=Y2 +"3 (Y4 +4y.

==

+ 3.3588 = 2.3588; + Y = 2.3243+ 0.25 3 (2.3588+4.2.0584 + 1.8243)=3.3590;


I )

+2.2.0584) = 3.3588;

s,

I~Y: 1_,3.3588-3.3590 , _ 0.0002 7 10-' < 1 0 001. 29 29 - 29:::::' 1"'.'

hence, there is no need to reconsider the Interval of calculations.

Sec. 5]

Numerical Integration of Ordinary DiOerential Equations

389

We obtain Y4 = ~ = 3.3590 (in this approximate nu'mber the first three decimals are guaranteed). Similarly we calculate the values of y, and Yo' The results are given in Table 2. Thus, we finally have

y (1.5) = 4.74.
4. Adams' method. To solve (1) by the Adams method on the basis of the initial data y (x o) = Yo we in some way find the following three values of the desired function y (x):

(these three values may b~ obtained. for instance. by expanding y (x) in a power senes (Ch IX, Sec. 16). or they may be found by the method of suc cessive approximation (1). or by applying the Runge-Kutta nlcthod (2) and so forth]. With the help of the nurnbers xO' x.. x 2 Xa and Yo. Yl' Y2. Ya we calculate qo' qt. q2' qat where
qo = IzY~ -= hf (x o Yo). q2 = h Y~ === hf (XI' Y2)' qt

= hy; = hf (XI'

YI)'

qa = hy::= hI (x a Yale

We thrn form a diagonal table of the finite differences of q:

li.y=
x

1/ =f (x,

y)

"

;; l/n+ I -lIn

q=y'h

aq=Qn+l-q",

li. 2 q= =li.qU+I-li.q,l

=li. 2q,.+ .-li.'Jq"


~3ql)

li.1q=-

Xo

110

~!lo

XI

IY\ I
II/I I
1

~YI ~Y2
~Ya ~Y4
~y,

x2 1 Y2!
XI

x4

Y41

XII Y.

I qo f (XI' I q, f I q" f a, I qa t (X Y4) I q4 f (XI' UI) I q.


f (X Ot Yo)
YI)
(x 2 Y2)

~qo

t:tlqo
/).2

~ql
~q2
~qa

QI

,\'q.
~3q2

~2q2

(X

Ya)

&2qa

4t

tiq4

XII Y.I

390

Approximate Calcu lat ions

[Ch. 10)

The Adam5 method consists in continuing the diagonal table of differences with the aid of the Adanzs formula
u y ,,--qn+2 uqn_l
A_I A

5 + 12

A2

3 qn-2+"8

AI

qn-I'

(7)

Thus, utilizing the numbers qa, 6,q2' t1 2qh &a qo situated diagonally in Ihe difference table, we calculate, by means of formula (7) and putting n::.:.- 3
in It, L\YI=ql+-}

L\ql+~ L\lql + ~

L\'qo. After finding L\YI. we calculate

Y4= Ya + t1Ya And when we know X4 and U,a, we calculate q4= hf (x 4 , Y4)'

introduce Y4' &Ya and q4 into the difference table and then fill into it the finite differences 6,qa' 6,"q2' !J.3 q1J which are situated (together with q4) along a new diagonal parallel to the fIrst one. Then, utilizing the nunlbers of the new diagonal, we use formula (8) (putting n=4 in it) to calculate ~Y4' Y5 and q, and obtain the next diagonal: qa, t1q., ~Iqa, 6,aq2' Using this diagonal we calculate the value of y, of the desired solution y (x), and so forth. The Adams formula (7) for calculating fiy proceeds from the assumption that the third finite differences 6,3 q are constant. Accordingly, the quanti tv h of the initial interval of calculations is determined frol11 the inequality h 4 < lo-m [if we wish to obtain the value of y (x) to an accuracy of
lO-m].

In this sense the AdalTIS formula (7) is equivalent to the fonnulas of Milne (5) and Runge-Kutta (3). Evaluation of the error for the Adams nlethod is complicated and for practical purposes is useless, since in the general case it yields results with considerable excess. In actual practice, we follo\v the course of the third finite differences, choosing the interval h so stnall that the adjacent differences ~3qi and 6,3 qi + 1 differ by not more than one or t\\70 units of the ~iven decil11al place (not counting reserve desimals). To increase the accuracy of the result, Adams' fOrlnula may be extended by terms containin~ fourth and higher differences of q, in which case there is an increase in the number of first values of the function y that are needed when we first fill in the table. We shall not here give the Adams fonnula for higher aceuracy. Example 2. Using the combined Runge-Kutta and Adanls nlethod, calcu late to two decinlal places (when x = 1.5) the value of the solution of the differential equation y' = y-x wi th the Initial condi tion y (0) == 1.5 (see Exanl pIe I). Solution. We use the values Yl' Y2' 1JI that we obtained in the solution of Example 1. Their calculation i~ given in Table 1. We calculate the suhsequent values Y4' Ys, y, by the Adams method (see Tables 3 and 4). The answer to the problerTI is U, = 4.74. For solving system (4), the Adams formula (7) and the calculation scheme shown in Table 3 are applied separately for both functions Y (x) and z (x).

Find three successive approximations to the solutions of the differential equations and systems indicated below. 3176. y'=XI_t-yl; y(O)=O. 3177. y'=x+y+z, z'=y-z; y(O)=l, z(O)=-2. 3178. y" = - y; Y (0) = 0, y' (0) = 1.

f (x,

Table 2. Calculating J.'4' Y5' )'6 by the Milne Method. y) == - x h; h = 0.25. (ItalIcised fIgures are Input data)

:
Value ot I

X,

IJI.

U: =/

(Xi'

Y.>

Y,

,-I

U.=!

(x,.

y,)

= y,

1.

Yi

Y: =/

(X,.

Y,)

Reconsider interval of calcula t 10 ns, following Indications of formula (6).

o
I

500 /1. 1 1.5000


25

IIIJIIIIIIIIIII 1111111111111111111/11 111111l1li111/1 111111/111111/1111111111111/1111111111111/11111//111/1//1/11//1///11/1111/1111/111/11111

10, 11.8921 1.6420 /.512.32431


1.8243

1111111111l1li1111111111111111111111111111111111111111111/11111111111111//111111111111 I/JIIIIIIIIIIIIIIIII

111/11111111111[1111111111/11111111

/111111111111111111111111/111111111111111111111111111111111111111111111111111111111111111111111111111111 11/11111111111111/11111111111111111

i a /.75/2.80841 2.0584 11/1/1/111111111111111111[1111111111111111111111111 1111111111111111111 I 1111111111111111111111111111111/11111111111111111111111111111111111 i - /1.00 111111111111111111111111111111111111 3.3588 I 2.3588 I 3.3590 I ~71O-5 13.3590 I 2.3590 IDo not reconsider
4

5 6

11.251111111111111 111/1/1/111111111111111 3.9947 1 2. 7447 j 3.9950 j 11.50 IIIIIIIIIIIIIIIIIIIIIIJIIIIIIIIIIIII 4.7402


I

~ 10- 5
5

13.9950 I 2. 7450

I Do not reconsi,der

3.2402 1 4.7406 j ~ 1.4.10Answer:

j~ll
I
y (1.5) =4.74

I Do not reconsider

Table 3. Basic Table for Calculatlni Y4' .vI'

.ve by the Adams Method. h = 0.25 (Italicised figures are input data)
f (x,
y)

=-

+ y;

....
0
Q.I

,
t,

Yi

6.Yl

=f

Yi= (xl,

qi=

Y;h

6.q,

li,2q,

/i:lq,

gil

>-

01

"Il

111111111111111111 t .5000 I 0.3750 I 0 0355 o.25 f t .8920 111111111111111111 1.6420 0.4105 0.0456 210 501 2.3243 111111111111111111 t .8243 I 0.4561 0.0585 3/0.751 2.8084 0.5504 2.0584 0.5146 I 0.0751 411 001 3.3588 0.6356 1 2.3588 0.5897 0.0964 511 251 3.9944 0.7450 I 2.7444 0.6861
0

I t .5000

I 0.0101 I 0 0028
I 0.0129 I 0 0037 I 0.0166 10.0047

I I

I I

I 0.0213

I
I

I
I

Table 4 Auxi liary Table for Calculating by the Adams Method t1y,=qi+ ; l\qi_l+

t21\2Qi -.+ ~ l\'qi-I


.!.. 6. 2q'_2
.!. 6. 3q. S &- 9

Value of

q,

!.2

6.q t-1

12

dY,

0.5146

0.0293 0.0376 0.0482

0.0054 0.0069 0.0089

0.0011 0.0014 0.0018

0.5504 0.6356 0.7450

4 5

o 5897
0.6861

ISec. 6]

Approximating Fourier Coefficients

393

Putting the interval h = 0.2, use the Runge-Kutta method to calculate approximately the solutions of the given differential equations and systems for the indicated intervals: 3179. y'=y-x; y(0)=1.5 (O~x~I). 3180. y' = JL_ y 2; y (I) = 1 (I ~ x ~ 2). x
3181. y'=z+l, z'=y-x, y(O)=l, z(O)=1 (O~x~l).

Applying a combined Runge-Kutta and Milne method or Runge-Kutta and Adams method, calculate to two decimal places the solutions to the differential equations and systems indicated below for the indicated values of the argument; 3182. y' = x + y; y == 1 when x == O. Compute y when x = 0.5. 3183. y' = x 2 -1- y; y = 1 when x = O. Compute y when x = 1. 3184. y' =2y-3; Y-== 1 when x=O. Compute y when x=0.5. 3185. = -x +2y-+ z, z' = x -t- 2y + 3z; y = 2, Z = - 2 when x = o. Conlpute y and z when x = 0.5. 3186. = - 3y-z, z' =y-z; y=2, z=-I \vhen x=O. Compute y and z when x == 0.5. 3187. y"=2-y; y=2, y'= - I when x=O. Cornpute y when x = I. 3188. Y3Y"-i-1=0; y=l, y'=O when x=l. Compute y when x = 1.5.

{y'
{y'

3189.

~2t~+i-coS2t=0; x=O, x'-'l when t=O.

Find x (rt) and x' (n).


Sec. 6. Approximating Fourier Coefficients
Twelve-ordinate scheme. Let Yn =f (x n ) (n = 0, 1, ... , 12) be the values of the function y = f (x) at equidistant poi nts x n = of the interval lU,2n),

:It;

and Yo - Yn We set up the tables:

I
Sums (~)

YO Yl Y2 Ya Y4 Ys Y, Yn YIO Y, Ya Y7
Uo U1

Differences (l\)

u2

Ua U. Us U,

VI V 2 Va V. Us

Sums

Uo Us U 2

ua
Sums

Us Us U.

Dillerences

Differences

394

Approximate Calculations

[Ch

10

may be determined approximately from the formulas:


000 = 50 +S 1+ 52 + 5.. 60 1 = to +0.866t l -l- 0.5t 2 6a 2 =50 -5.+0.5 (51-51)' 6a l = t o-t 2

The Fourier coefficients an' bn (n = 0. I. 2. 3) of the function y = f (x)


6b l = 0.50'1 + 0.8660 2 6b 2 = 0.866 ('t 't 6b a =0'1- 0 a.

1+ 2 ).

+ Ga.
(1)

ya 1 1 where 0.866 = -2- =:::: 1- 10 - 30


We have
/(x)

~ a;

+L
n=J

(an cos nx + bn sin nx).

Other schemes are also used. Calculations are simplified by the use of patterns. Example. Find the Fourier polynomial for the function y= f (x) (0~x~2n) represented by the table
Yo
38

UI
38 12

Y.
4
14

Solution. We set up the tables:

y 138 38 12
32
v

4 14 48 - 24 - 27 - 23 4 20 - 13 - 19 28 41 27 20 - 20 7 - 20 33

18 18

u 138 70 20 -

6
38 70

u _ 18 _ 19 _ 13 I
b

v
0
't

20 56

51 89

I 27 6 4 28 41 I-21-37 33 45 28
= 3.8;

From formulas (1) we have

ao=- 9.7; a l = 24.9; a2 = 10. 3; a l bl =13.9; b2 =-8.4; b.=0.8.

Co n sequen t I y f (x) ~ 4.8+ (24.9 cos x + 13.9 sin x) + (10.3 cos2x-8.4 sin 2x) + + (3.8 cos 3x

+0.8 sin 3x).

Using the 12-ordinate scheme, find the Fourier polynomials for the following functions defined in the interval (O,2n) by the

Sec. 6]

Approximating Fourier Coefficients

395

tables of their values that correspond to the equid istant val ues of the argument. , 3190. Yo =-7200 Ya==4300 Y6=7400 yg=7600 y, = 300 Yol == 0 Y" = - 2250 Y10 = 4500 Y2 = 700 Ys = - 5200 Ys = 3850 Yl l = 250 3191. Yo=O Ya=9.72 Ys=7.42 Y e =5.60 YI -== 6.68 Yti = 8.97 Y7 == 6.81 y,o = 4.88 Y2 == 9.68 Ys == 8.18 Ya= 6.22 YII = 3.67 3192. Yo-=::2.714 Ya=I.273 Ys=O.370 Ye =-0.357 YI = 3.042 Y4 = 0.788 Y7 = 0.540 YIO = - 0.437
Y2=2.134 Ys =0.495 Y8=O.191 YII-==0.767 3193. Using the 12-ordinate scheme, evaluate the first several

Fourier coefficients for the follo\ving functions:


a)
1 f (x) = 2n
2

(Xl - 3nx 2 -t- 2n 2 x)

(0 ~ x ~ 2n), (0 ~ x ~ 2n).

b)

f (x) =

n 2 (x-n)2

ANSWERS

Chapter I
~

1. Solution. Since a=(a-b)+b, then lal<;la-bl+lbl. Whence ,a-bl~ la I-I bl and I a-b 1=1 b-a I~ I b I-I a I. Hence, I a-b I ~ I a I-I b I Besides, la-bl=la+(-b)I<;lal+l-bl=lal+lbl. 3. a) -2<x<4; b) x < - 3, x > 1; c) -1 < x < 0; d) x > O. 4. -24; -6; 0; 0; 0; 6. 5. I;

I .. r - 4 ; , I+x!;
7
I

lxI-I, l+x2 ; I/f l+x!.6.11;


13 1

..r - -

.. ~

11

"2;

5 I 0.7. f(x)=-3"x+"3.
-I<:x<+oo;

8. f(x)="6x -6"x+l. 9. 0.4. 10.

2(x+fxl). 11. a)

b)-oo <x<+oo. 12. (-00,-2), (-2, 2), (2, +00).13. a) -00 <x~- 2 , V2~x<+00; b) x=O, Ixl~ V2. 14. -1<:x~2. Solution. It should be 2+X-X2~0, or x 2-x-2<;0; that is, (x+l)(x-2)~0. Whence either x+l~O, x-2~0, i. e., -1~x~2 or x+l~O, x-2~0,i.e.,x~-1, x~2, but this is impossible. Thus, -1~x~2. 15. -2<x~0. 16.-oo<x~-I, O~x~l. 17. -2<x<2. 18. -1<x<l, 2<x<+oo. 1 n 19. -3~x~1. 20. l<;x~IOO. 21. k11~x~krc+2(k=0, I, 2, ... ). 22. <p (x) = 2x4-5x 2 -10,

'1' (x) =

- 3x'

+ 6x. 23. a) Even, b) odd,

c) even, d) odd,

e) odd.24. Hin!. Utilize the identity 28. a) Periodic,

f (x) = {If (x) +f (-x)] +ilf (x)-f (-x)].


periodic

T=j n, b) periodic, T= 2~, c) periodic, T ='1:, d)


y=~x, c
if O<;x<:c; y=b if

T='Jt, e) nonperiodic. 27.

c<x~a; S=2~X2 C

if O.,.;;x"';;C; S=bx-~ if C < x";;;a. 28. m=q1x when 0 .,;;;x.,.;; 1\; m= =Q1 l l+q!(x-l l ) when 11 < XE;;I I +l!; m=Qll1+q212+Q.(x-l1-l2) ,vhen 11+1 2 < X~ll +1 2+l.=I. 29. cp [ (x)]==2 2X ; 'I' [q> (x)]=2 X2 30. x. 3t. (X+2)2.
37.

-2";

11

0;

11

38. a) y=O when x=-I, y>O when x>-I, y<O

when x<-I;b) y=O when x=-l and x=2,y>Owhen -1<x<2, y<O when -00 <x<-l and 2<x<+00; c) y>O when -oo<x<-1-oo; d) y=O when x=O, x=- Y3and x= Y3, y>O when - V3 < x < 0 and V3<x< + 00, y<O when - oo<x<- V3 and O<x< V3; e) y=O when x= 1, 1 y>Owhen-oo<x<-l and l<x<+ 00, y<O whenO<x<1 39. a) x=2 (y-3)

(-co<y<+oo);

b)

x=l"Y+l

and

x=- VU+1 l-l~y<+oo);

Answers
e)

397

x=

=i

I-ys V-

(-oo<y<+oo);
40.
,

d)

x=2IOY (-oo<y<+oo); e) x=

tan y (

-~ <y<~).
41. a)

x=y when -co<yo;;;;;O; x="y when

O<y<+oo.

y=u

10

u=2x-5; b) y=2u , u=cosx; c) y=logu,

u=tanv, v=~; d) y=arc sinu, u=311 , v=-x l 42. a) y=sln1x; b) y=


1; c) y=-i-when -co<x<O and y=x when OQ<+co. 46. Hint. See Appen dix VI, Fig. 1. 51. Hint. Completing the square in the quadratic trinomial we will have y=yo+a(x-x o)2 where xo=-b,'2a and Yo={4ac-b 2)/4a. Whence the desired graph is a parabola y=ax' displaced along the x-axis by Xo and along the y-axis by yo. 53. Hint. See . Appendix VI, Fig. 2. 58. Hint. See Appendix VI, Fig. 3. 61. Hint.
-00

=arctan logx; e) U=2(x!-I) if Ixl<;l, and y=O 43. a) y=-cosx 2 , n';;lxl~ 21t; b) y=log(10-10X ),

if

Ixl> I.

<x<

The graph is a hyperbola y= m , shifted along the x-axis by x o and along

the y-axis by

Yo' 62. Hint. Taking the integral part, we have y=i -

(x+ ;) (Cf. 61).65. Hint. See Appendix VI, Fig. 4. 67.Hlnt. See Appendix VI,
Fig. 5. 71. Hint. See Appendix VI, Fig. 6. 72. Hint. See Appendix VI, Fig. 7. 73. Hint. See Appendix VI, Fig. 8. 75. Hint. See Appendix VI, Fig. 19 78. Hint. See Appendix VI, Fig. 23. 80. Hint. See Appendix VI, Fig. 9. 81. Hint. See Appendix VI, Fig. 9. 82. Hint. See Appendix VI, Fig. 10 83. Hint. See Appendix VI, Fig. 10. 84. Hint. See Appendix VI, Fig 11. 85. Hint. See Appendix VI, Fig. 11. 87. Hint. The period of the function is T = 2njn. 89. Hint. The desired graph is the sine curve Y = 5 sin 2x with amplitude 5 and period n displaced rightwards along the x-axis by the quantity
1 { .90. Hint. Putting a=A cos q> and b=-A sin q>, we will have y=A sin (x-q where A= al+b l and q>=aretan( - : ) - In our case, A=lO, q>=O.927. 92. Hint. cos l x =} (l +cos 2x). 93. Hint. The desired graph is the sum of the graphs

Yl = x and Y2 = sin x. 94. Hint. The desired graph is the product of the graphs Yl = x and y" = sin x. 99. Hint. The function is even For x>O we determine the points at which 1) y=O; 2) y=l; and 3) y=-1. When x~ -1-00, y --+ 1. 101. Hi n t. See Appendix VI, Fig. 14. 102. Hi nt. See Append 1X VI, Fig. 15. 103. Hint. See Appendix VI, Fig. 17. 104. Hint. See Appendix VI, Fig. 17. 105. Hint. See Appendix VI, Fig. 18. 107. Hint. See Appendix VI, Fig. 18. 118. Hint. See Appendix VI, Fig. 12. 119. Hint. See Appendix VI, Fig. 12. 120. Hint. See Appendix VI, Fig. 13. 121. Hint. See Appendix VI, Fig. 13. 132. Hint. See Appendix VI, Fig. 30. 133.Hint. See Appendix VI. Fig. 32. 134. Hint. See Appendix VI, Fig. 31. 138. Hint. See Appendix VI. Fig. 33. 139. Hint. See Appendix VI, Fig. 28. 140. Hint. See Appendix VI. Fig. 25. 141. Hint.

398
Fornl a table of values=

Answers

o
x y
0
0

I -1
.. I
-1

-2
-8
4

-3
27

8
4

27
9

.. I

1I

Constructing the points (x, y) obtained, we get the desired curve (see Appendix VI, Fig. 7). (Here, the parameter t cannot be laid off geometrically!) 142. See Appendix VI, Fig. 19. 143. See Appendix VI, Fig. 27. 144. See Appendix VI, Fig. 29. 145. See Appendix VI, Fig. 22 150. See AppendIx VI, Fig. 28. 151. Hint. Solving the equation for y, we get y= Y25 - x!. It is now easy to construct the desired curve froln the points. 153. See Appendix VI, Fig. 21. 156. See Appendix VI, Fig. 27. I t is sufficient to construct the points (x, y) corresponding to the abscissas x==O,

i, a. 157.

Hint.

Solving the equation for x, we have x=..:. 10 log y-y <*). Whence we get the points (x, y) of the sought-for curve, assigning to the ordinate!, arbl trary values (y> 0) and calculating the abscissa x froln the formula <*) Bear in nlind that log y ~ - 00 as y ~ O. 159. Hint. Passing to polar coordinates r=Vx 2+y2 and tan rp=1!. , we will have r=e'~ (see Appendix VI, Fig 32)
x

160. Hint. Passing to polar coordinates x=r cos <p, and y=r sin cp, we wIll

have r =
162.

cos q>

3~in ~c~s all' SlIl <p

(see Appendix VI, Fig. 32)

161.

F = 32

+ I,

Be

Ymax=15 when x=5. 163. n 1 whenx=T' 164. a) x1 =2' x 2 =2; b) x=O.68; c) x1 =1.37, x2 =10; d) x=0.40; e) X= 1.50; f) x=O.86. 165. a) x 1 =2, Yl =5; x 2 =5, Y2=2; b) x 1 =-3, YJ.=-2; x 2 =-2, Y2=-3; xa==2,Ya~3;x .. =3, y.. =2; c) Xl ==2,
y=O.6x(10-x);
Yl=2; x2~3.1, Y2~-2.5; d) xl~-3.6, x,:::::2.9,
YI~-3.1;

ab . ab slnx ; Ymax=2 Y=2

Ya::::: 1.8; x4 =:.:::3.4, Y4~-1.6;

Y2=-

~2.
a)

166. n>

:8'
3

e) x'=T'
b) n> 10; c) 168.
0

:rt

Y-2 Y'=T;

x2~-2.7,

x2

Y2:::=29; 5rc

= 4"'

a)

n~4;

n~32.
e
(8
~

167. n

>~
a) 0 02;

-1 =N.

N= 9; b) N=99; c) N=999.

~=5

<

1).

b) 0 002; c) 0.0002. 169. a) log x

x> X (N);
171.

c) \ f (x) ,

>

when

< -N when I x \ > X (N).


1

<x<

(N); b) 2x

>
1

170. a) 0; b) 1; c) 2; d)

N when 7
Hint.

30 .

2".

172. 1. 173.

-2'

174. 1. 175. 3. 176. 1. 177.

4".

178.

3.

Use the formula 12 + 22 + n2 =6"' n (n -1-1) (2n + 1). 179. O. 180. O. 18t. 1. 182. O. 183. 00. 184. O. 185. 72. 186. 2. 187. 2. 188. 00. 189. O. 190. 1. 191. O. 192.
00.

+...

193. -2. 194.

00.

195.

1 2"

196.

a-I

3a l

197. 3x. 198. -1. 199."2 .

Answers
200. 3. 201. 208. 2

399

202.

"9'
2

1 3 203. - 56' 204. 12. 205. 2'

206. 213.

-3. -2'
5

207.

1.

Vx 209. 3 V x
~
2'
2 n' 3
~

t I l
210.

-"3'

211. O. 212.

2"'

214.

"2'

215. O. 216. a)

sin 2; b) O. 217. 3. 218. { . 219.

222. cos a. 223. - sin a. 224. 11. 225. cos x. 226. 228. 235. 243.

;2'
e2

. 220. n. 221. 227. a) 0;

~.
b) I.

2 n'

229. 236.

230. O. 231. 237.

1 Y3'
238. 246.
1t.

232. 239.

2 1 (n _m 2 ). 233.2"

234.

l.

3
0

-"4'
245.

4"'
247.

240. 1. 241. 1. 242. 248.

4'.
e-&.

244.

n'

O.

e- 1
x-+o

e- 1

249.

250. eX.

251. e. 252. a) 1. Solution. lim (cos x) x

::=

lim [1-(I-cos x)) x


x-.o

~i:

0(

1 - 2sin -i)x = ~i~ l(1 - 2 sln i)


2 2
2

---x2sin2 _
2

1 ]_2~in2f .\
lim

=e
Since Iitn ( - - X'-~o x

x-+o

2~fn:!.!..) I
x

2sin

=-2lim
x--+o

[(Sin i)2 X2] - - -. =-21


2"
x 4x
Solution.
11m ( - 251:'

x -+0

x Iitn-=O, it follows
4

] 1 that lim (cos x) x = eO == 1. b) .. r- ' x~o r e

As

in

the

preceding
Si11
2

.!case (see a ),
I1m (cos x )X2
I

X-.O

= ex-+o
2

f)

. S'Ince l'1m

(_ 2 2i)
X

x~o

= _ 2 lim
x->o

r l(Sin ~ )2 X~2] = _ .:- .


X

4x

It

follows that lim (cos X)i> =e


x-+o

~
Hint.
1n a

';'e'

2
253. In 2. 254. 10 log e. 255. 1. 256. 1. 257.

-~.

2.')8.

I.

Put eX -1 =a, where a ~ O. 259. In a. Hint. Utilize the identity a=e


260. In a
Hint. Put

l.. = n
"2 .
1

a, where a-+-O (see Exalnple 2.59)

261.

a-b.

262. I. 263. a) 1; b)

264. a) -1; b) 1. 265. a) -1; b) 1. 266. a) 1; b) O.

267. a) 0; b) 1. 268. a) -1; b) 1. 269. a) -1; b) 1. 270. a) -00; b) +00.

400 271. Solution. If x kn (k =0.

Answer~

1, 2... ).

then cos! x
n

<1

but if x=kn, then cos1x=1 and y=1. 272. y=x when O-c;x< I;

and Y =0; 1

Y=2

when x==l; y=O when x> I 273. Y=lxl 274. Y=-T when x<O; y=O when x=O;
1< x <

Y=T

n;

when

x> O.
XI

275.

y= 1 when
X.-+ 00.

O<;x~

1; y=x when 279.


2n;R.

+ 00.

61 276. 450' 277.

-'-1i;

278.

n.

e 1 Yen + 1 2 4 l' AC l ab 280 e-l' 281. 13".282. n . 8. n~ n=3' 285. 2 286 . k =1. e 2 -1 I x'+ 1 b=O; the straight line y = x is the asyrnptote of the curve y = x2 + 1 .
287.

Q\nl = Qo ( 1

+~

r.

where k is the proportionali ty factor (law of

compound interest); Qt = Qoekt 288. I x I> c) I x , > 1000. 289.


b) Ix-11<0.005;

'x-

..!.., e
0

a) \ x \ > 10; b)
1;

\ x I> 100;

1\<

\vhen

<e<

a)
N=~;

I x- 1 I < 0.05;
a)
~=0.1;

c) Ix-l \<0.0005 290. \x-2\<

b) 6=0.01; c) 6 = 0.001. 291. a) Second. b) third. }.


c) 3.

~93
3.

a) 1; b) { ; c)

i;

i.

292. a) 1; b) 2;

d) 2; e) 3. 295. No

296. 15. 297. -1. 298. -1.

29S..
2)

300.

a)

yfO=Y9+1=3
1.03 (1.0309);

VI +{;
3)

1.03 (I 0296);

b)
d)

0.985 (0.9849); c) 3.167 (3.1623) 10.954(10.954). 4) 301.

Hint.

1) 0.98(09804); 5) 1.12 (1.125); Hint.

0.0095 (0.00952);

3.875 (3.8730);

6) 0 72 (0.7480); 7) 0.043 (0.04139). 303, a) 2; b) 4; c)

~;

d) -}. 307.

If

x>O, then when lL\xl<x we have \Yx+L\x-Yxl= =1L\XI/(Yx+L\x+yx)~IL\xI/Yx.309. Hint. Take advantage of the
inrquality

I cos (x+ L\x) -cos x I ~ I L\x I 310. a) x i= ~

+ kn.

where Ii is an

integer; b) x :. kn;, where k is an integer 311. Hint. Take advantage of the inequality Ilx+L\xl-,y.l'~\~x\ 313. A=4. 314. 1(0)=1. 315. No
316. a) /(O)=n; b) /(0)=;; c) /(0)=2; d) {(0)=2; e) {(O)=O; f) {(O)=l.

317. X= 2 is a discontinuity of the second kind. 318. x=-1 is a removable discontinuity. 319. x=-2 is a discontinuity of the second kind; x=2 is a removable discontinuity 320. x=o is a discontinuity of the first kind. 321. a) x==O is a discontinuity of the second kind; b) x=O is a removable discontinuity. 322. x=O i~ a relnovable discontinuity, x= kn; (k= 1, 2, ... ) are infinite discontinuities
323.

x= 2n;k ~

(k = 0,

1, 2, ... )

are

infinite

discontinuities.

324. x= kn (k='(), 1, 2, ... ) are infinite discontinuities. 325. x=O is a discontinuity of the first kind. 326. x= - I is a removable discontinuity; %= 1 is a point of dislontiiluity of the first kind. 327. x=-l is a discon-

Answers

401

tinuity of the second kind. 328. x= 0 is a removable discontinuity. 329. x= 1 is a discontinuity of the first kind. 330. x = 3 is a discontinuity of the first kind. 332. x= 1 is a discontinuity of the first kind. 333. The function is continuous. 334. a) x=O is a discontinuity of the first kind; b) the function is continuous; c) x= k'Jt (k is integral) are discontinuities of the first kind. 335. a) x=/tt (It is integral) are discontinuities of the first kind; b) x=k (k ~ 0 is integral) are points of discontinuity of the first kind. 337. No, since the function y = E (x) is discontinuous at x= 1. 338. 1.53. 339. Hint. Show that when Xo is sufficiently large, we have P (-x o) P (xo) < o.

Chapter II
341. a) 3; b) 0.21; c) 2h+h 2 342. a) 0.1; b) -3; c) 344. a) 624; 1560; b) 0.01; 100; c) -1; 0.000011. 345. a) a~x; b) 3x 2 Ax a. 3 2 3 (2. 2x L\x + (~X)I . 2x + L\x . 3 (A 2

Va+h-Va.
2x(2~~
L\x

x ox)

+ (L\x)

x L\x+ L\x),

c)

Xl

(x+ L\X)2'

x2(x+ AX)2 '

x+L\xd)

Yx;

1 _ ; Yx+L\x+Yx

2x(2\x_ 1); e)

-1);

f) In 348.

~X). 346. a) -1; b) 0.1; c) -h; O. 347. 21. 15 cm/sec. 349. 7.5. 350. f (x + L\x) - f (x) . 351. f' (x) = Jim f (x + L\x) - f (x)
Lin (1+
L\x
\x~o

X~i\X;

L\x

352. a) L\q>; b) dq> = lim L\q>, where q> Is the angle of turn at time t.
l\t
dt \t-+o L\t
353. a) L\T ; b) dT = lim L\t dt 6/-'+0

~ T, where T is the
~t

temperature

at

time

t.

354. dQ ;;:::: lim ~Q, \vhere Q is the quantity of substance at


dt

355.
c)

a)
~x -+0

~m;
L\x

\t -'0 L\t

time t.

b) litn L\m
6x-+oL\x

356.

a)

_.-!.-::::::: 6

0.16; b)

-2~1 ~-O 238;


Solution.

~l =:::: -0.249;
L\x

y~=! = - 0.25.

357.

sec' x.

y' = lim tan (x + ~x) - tan x = lim


X Urn 1 ~x ....o cos x cos (x

+ ~x)

=
~
12

sin ~ = lim sin L\x X \x -+0 L\x cos x cos (x ~x) \x-+o L\x _1_ = sec 2 x. 358. a) 3x!; b) _ c) _1_; cos 2 X x' 2 t"

.!.;

d)

--=..!- .
sin 2 x

359.

Solution.

l' (8) = lim f (8 -t- L\x) -f (8)


I1x-.o

= lin1

AX~o

. V 8-l-~x-V8 == 11m
L\x

Ax-+o ~x

lV (8 + ~X)2+ V

8+~x-8

~x

_ (8 + L\x) 8+ VB!]

= lim 1 = .!.. . 360. f' (0) = - 8, f' (1) ==: 0, ~x-+o V(8+ L\x)I+2 V8+L\x +4 12 I' (2)=0. 361. x,=O, x2 =3. 2Hint. For the given function the equation I' (x) = f (x) has the form 3x = Xl. 362. 30m/sec. 363. 1, 2. 364. - 1. -1 365. f' (xo) = - ! . 366. -1, 2, tan q> .= 3. Hint. Use the results of Exa Inp Ie 3
Xo and Problem 365. 367. Solution. a)

I' (O) ==

~x-+o

linl

V (L\X)2
Ax

1
~x.-+0V Ax
11111 J

_.:::

OO~

402
b) /,(1):: 11m
Ax-+o

Answers

V1+&x-l= lim
&x
Ax-.+-o

V (Ax)e
&x

=00;

c)

f'-

(2k

+ 1 n)=
2

=
=

lim
Ax-+-o

I
1

cos(2k+ 1 n+~x)l . 2 = lim I sin &x


&x
Ax~-o
C

I =_ I;
3

i' + (2k

+ 1 n) =
2

lim (sin&xl =1. 36k. 5x -12x!+2. 369.


Ax-+-+
0

&x

-.!.-+2x-2x l
5

370. 2ax+b.

371 -15x! - . 372 rna t m -

a
I

375. 2x

373 .. 6ax 374 n . . -2" rr Q2+b 2 x 3 8 ~ 2 8 4b -fix l -3x-. 376. 3"x l . Hint. y=XIX I =x l 377. 3x1 x1

+ b (m+n ) tTn+n-1 .

2a - 3x ----:v-. V Xi

38t. 385.
389

Y (

-2x 2 -6x+ 25 1-4x 380. (x 2 -5x+ 5)2 Xl (2x-l)2 1 4-2 y-)I . 382. 5 cos x-3 sin x. 383. - 2 - . 384. z 1l sin 2x (sin x-cos X)2

378.

be-ad (e+dx)!

379.

tlsin t.

386.

y'=O.

387.

cotx-+.
S111

388.

arcsinx+

x I-xl
5

xarc t anx.

390 .

8 x (7) xe x-I- . 391.

x 392 ex -xa. x-2 393 . 5x"-x xe. -ex- .

394. eX (cos x-sin x). 395. xle X. 396. eX (arc sin x+ y_l__) .397. x (2Inx-l) . 1 -x! In 2 x 2 2 In x 2 21n x l . 398. 3x Inx. 399. -+-2-2. 400. -110--.401. slnhx+xcoshx. x x x xn x 2xcoshx-x2 sinhx - t h2 404 -3(xlnx+sinhxcoshx) 402. cosh 2 x 403. an x . . x In 2 x-sinh! x .

405. 407.
411.

-2x

1-~

406. 408.

. '1r 1 arc sin I1 x + .. r 1


rl-~

x-yX2="farccoshx 2 2 -1

x Yx

12ab

+ 18b y.
2

412.

415.

V (a +

bx

2
bX I )2

1+2xarctanhx 410. ~(ax+b)2 (l_x2)2 e c x 2 -1 -x 2 16x (3 + 2X )3. 413. - - . 414. V (2x-l)8 l-x 2

fl+~

. x. arc sin

416.

Vr Va2 -1.
x2

418.

l-tan 2 x+tanol x cos2 x

419.

- ~. . 420.2-15 cosl x sin x. 42t. -16cos ~ . Hint. x=sin-It + 2 sin 2 x cot x sin a 2t +cos-2t. 422. sin x 423. sin1x 424. ;COsx+2sinx . (1-3cosx)a cos4 x 2 15sin x-l0 cosx 2 cos x + 3 si n x 1 425. 3 S1l1 X co~ X 426. 2 V I-Xl 'V I + arc sin x 1 3 (arc sin x)! - 1 427. Y 428. 2 (1 -t- x 2) arc tan x y I-x! (1 + x 2) (arc tan X)2

Answers
429.
eX +xex 1 x +x 2

403

Vxe

430.

2e V 3 (2e
433.

2x In 2
X -

2x

-. + 1)2 + -x

5 In4 x .

432. 434.

(2x-5) X

a X cos(x'-5x+ 1 ) - - - X2 COS 2

~
x

-a sin (ax + ~).


437.

sin (2t

+ q.

435.

_2COS

sin' x

436.

-1

x cos 2x2 sin 3xl


--1 ..r
f

438.

Solution.

sin 2

.:..

..r
f

1 (2x)' 1-(2x)2

= ..r 2 f 1 -4x 2
x2

. 439.

X ,

- 2 ..r 440.
~:'-l

x-x 2

- 1 441. . 2

1+x

442. 1-1 f . 443. _10xe- 444. _2tS- X2 In 5.

+x

445. -2x 1- 7

2xl0 2x (1 +x In 10).
449.

446. 450. 453.

sin 2t -t- 2f t cos 2t In 2. 447. -2x l-x 2


1 451.
2

..r
f

-ex
1- e2X

448.
(eX

cot x log e.

----X

21n x

1 x In x

452.
454.

(1 + 1n

+ x) X (1 + x 2 ) arc tan x

1-x 2-4 -_--.J (eX +5 sin x-4arc sin x) Y l-x 2 2x fIn x + I

+ S cos x)

..r--

1 )

455. Solution. y' = (sin! 5x)' cos ;

-I- sin! 5x 2 cos ; ( - sin


456. 4x+3 (x- 2)8
1

i ){ = 15 sin! 5x cos 5x cos! i - f sin! 5x cos i sin i .


x .459. (1_x 2 )5
7

+ sin! 5x (cos! ~ )' =

3 sin!5x cos 5x 5 cos! ~

+ 2 (f . rx +X )

+
.

457.

x t4x-6 . 458.
(x- 3)5

x2

V 2t
x5

x-I
2 -

460. - : ;2r - - . 461. .. r f (a -I ~ 2)3 f (1

x2

+x

)'

462.

(1 +
465.

VYx X)3 463.


468.

V--a-2 (1 +x)
a-3x 2 Ya-x

2x -t- 1

464.
466. 469.

--:t-r====:::::l:::;::====::::::::.:::=:: . V(x-l)3 (x --1- 2)5

2ubrnnx"- 1 (a 1- bA 'l )m-l 467. (a_bx 'l )Tn+J 3x 2 + 2 (a -1- b + c) x + ab + be + ac 2 Y(x+a) (x+b) (x+c)

(x+ 2)'
470.

1+2

Yy

471. 2 (7t .-!- 4) 475. 1

Vat

4 cos 2x _(U-~) sin 2x . 483. O. 479. 3 cos x cos 2x. 480. tan x. 481. - - 482. ~ .. rr======:::;;:=== 2 2

sin 4 x cos 4 x .

6Yy V(y+ y)2 + 2. 472. ..r y-a .473 ' ..r X1 474 S1na X COS 2 X. f t2ay-y2)' f e +1 477. x cos Xl. 478. 3t 2 sin 2t'. 476. 10 tan 5x sec! Sx.
2 f a sin

484.

sin'" x 1 arc sin x (2 Jrc cos x-arc sin x) .. r 485. 2 , I-xi

487. xarccosx(1-x 2 ) /2
49t.

yf=X2 .

2 ..r x , 2x 2 -1

x + ~ cos

486.

-1 +xl

488.

Ya -

1 bx l

.489." (" ~ .490. 2 V a x

a -x.
2

-x. 2x - X l

492. arc sin

yx.

493.

.r
,

5 1-2Sxl arc sin 5x

404

Answers
1 Y l-ln

494.

491.

~na

497. 4x ' /6 x

-x cos x 2 501. 2m p (2ma mx + b)p-Iamx In a. 502. efJt (a cos pt-p sin Pt). 503. ea.x sin px.
504. e- x COSI3x. 505. xn-1a- XI (n-2x 2 In a). 506. 507.
511.

V 1

498. I ~nl ~

1-2xcosa+xl
499.
a

498. 500.

1 5+4sillx
sin 2xe,fn x
2

Y eax.
~

y tan x (I

3cot -In 3 ( \). x sin I

508.

2ax+b . 509. ax2 + bx + c

Y a 1+
l

Xl

+ YCOs"'X In a). y810. I+ x

V'

-2 I x-I 2x+11 Hint. - - . 513. - tan - - . 514. 2 x Ina x x2 x x -x-2 2 3x - 16x + 19 I y=5In(x-2)-3In(x+n.515. (x-I}(x-2)(x-3)' 516. sln'xcosx' . r2- -6x 15a In 2 (ax + b) 2 517. r x - a 518. (3 _ 2xl ) 1 n (3 _ 2x ) ' 519. ax + b

I 2ax +x2

. 512.

520. 524. 527. 529. 53t. 535

Y
X

+a

.
2

521.

mx + n x 2 -a 2

'

522.
3 1-9x 2

Y2 sin In x.

523.

sin' x

yf:t:Xi.

525. x + 1 . 526. xl - I

[ 2arc sin IX In 2

+ 2 (1- arc cos 3x)}.

(3:~: ~:-ln + slnllax )


x (1
1

3 cos bx
. 532.
t 2)

+ In 2 x)
2

1 -1+,. x

x{I+ln x) 536. arc sin 3 x /.


(I _
/2

x! x +x2 -2
C

a cos ax cos bX; b sin ax sin bx. 528. cos bx 530. Y 1 + In x -I1- x 2 arc sin x x x

I+2' x .
SUt

2 x 2 - 3x 5 3 3 . . .F 534. - cosx r sin x xoi_1 1R 537 6' sIn h2 2xcos h 2x. 538. eax (acoslpx+

1 1-1 n 2 x

+ ~ sinh ~x)~
I

539.

6 tanh 2 2x (I-tanh! 2x)

540. 2 coth 2x. 541.

1 -1 2 542... r 543. - - . 544. . 545. 546. x arc tanh x 2 x r 1n x-I cos2x sin x l-x 2 547. x arc sinh x. 548. a) y' = I when x> 0; y' = - I when x < 0; y' (0) does t exist; b) , (2 I 49 ' I 550 f'() { - I when x ~ 0. no y -= x. 5 Y x = _e- x when x > O.

--,.-~'~-. l' a +- Xl
ol

552.

ya "2+3'
I

553. 611.554.
,

a)

" , f- {O)=-I, f+ (0)=1; b) f- (0)


0; d)

=x .

=(i ,

, -2, + (0) = c) f _ (0) = I,

a;

f + (0) =

f _ (0) = f + (0) = 0, e) f _ (0)

"

and

x-3 f + (0) do not exist. 555. I-x. 556. 2+ -4-' 557. -I. 558.0 561. Solution. We have y'=e- x (I-x). Since

e-x=JL.
x

it follows that
2

y,=.JL (I-x)
x

or xy' =- y (I - x) 566. (I + 2x) (I + 3x) + 2 (1 + x) (I 567. _ (x + 2) (5.\2 + 19x + 20) 568. (x + I)" (x + 3)5

+ 3x) + 3 (x + 1) (1 + 2x).
2

x -4x + 2 Y x(x-I) (x-2)'

Answers
3x2 +5 3(x' +I)
3 (x-I)

40&

569. 571.

V---xr
2

(x-2)9(x 2 "':"7x+l)
570.
(x-I)(x-2)(x-3)V(x-I)I(X-3)c'
2

xl+I'
8

I~XI + X-.~4
(x

+ 2)

(x

+ 3)

If 572. xX (l + In x), 573. xx' +1(1 + 2 In x).

574.

Vx

Y'X-2.. 1 ~n x ,575. x I ( 1+

In x ). 576. xXX xX( ~ + In x+ In'x ). (cos x)&tn x (cos x In cos x - sin x tan x).
580.
(arctanx)X

577. 579.

rio X

(Sl~ x + cos x Inx).

578.

(1+~r[ln(I+;)+I~X]'
, xy 2 ' 2-cos x : c) xy
2

x[tnaretanx+(l+xltaretanx]'
b)

= ---:f.
1

10

t -1 -2t 582. 2" t. 583. t + 1 . 584. 1_/2.

581. 3 2

a)

x~=3(I~X');
b

t (2 - t I)
585. 1-2tl
'

+ 5e!

586. 3

Vi'

587. t (t l + I)

t +1

588.

tan t, 589.

' = ~ -11 when t < 0, 93 590. - -b t an t . 591. - tan 3t. 592. Yx a w hen t > 0 . 5
594. tan /. 596. 1. 597. tity. 601. -5 602.
2
00.

-2e at

x(3x+2y) -. x2+2y .605. x 607 2y 2 _ l_y3 608 10 606. x 3(x!-y2)+2xy-l+3 xy l+4 y' 10-3~(jsy y cos! Y y 1- x2- y2 2 ' 609. -1. 610. 1 -x cos2 612. (x+y). 613. y = y 611. -x I + x2+~ Y I I ~ x+y x 616. =eY-I=x+y-I' 614. f+e 615. XYy x-y

_Vu

--2- 603. -"2. 604 . ay

b
2x

599. No. 600. Yes, since the equality is an Iden-

x2

yY
L

617. cy + x V~ cx-YX2 +y2

. 618.

x In y -y .!L.. 620. a) 0; b) -.!..; c) O. ylnx-xx 2 e


~

622. 45;

2 arc tan 2 ~ 63 26'. 623. 45. 624. arc tan -

36 21'. 625. (0, 20); (1, 15);

(-2, -12).626. (1, -3).627. y=x2 -x+ 1. 628.

-1 629. ( "8' 1 -16 I) k=lT

631. y-5=0; x+2=0. 632. x-I =0; y=O. 633. a) y=2x; Y=

-2- x;

b) x-2y-l=0; 2x+y-2=0; c) 6x+2y-n=0; 2x-6y+3:rt=-=O; d) y=-=x-I; y=l-x; e) 2x+y-3=0; x-2y+l=0 for the point (I, I); 2x-y+3=0; x+2y-l=O for the point (-1, 1). 634. 7x-IOy+6=O, n2 IOx+7y-34=0. 635. y=O; (n+4)x+(n-4)y--4-=0. 636. 5x+6y-

Y"2

6x-5y+21 =0. 637. x+y-2=0. 638. At the point (1,0): I-x at the point (2,0): y= -x+2; y=x-2; at t he pOln . t y=2x-2; y=-2-; -13=0,
(3, 0):

y=2x-6;

3-x y= -2-.

639.

14x-13y+ 12=0;

13x+ 14y-41 =0..-

406

Answers

640. Hint. The equation of the tangent is 22..

crosses the x-axis at the point A (2x o' 0) and the y-axis at B (0, 2yo)' Finding the midpoint of AB, we get the point (x o' Yo)' 643. 40 36'. 644. The parabolas are tangent at the point (0, 0) and intersect at an angle

Xo

+ 2Y = 1. Yo

Hence, the tangent

arctan
648.

~ ~88' at
652. 653.
2 ;

the

point

(1, I).

647.

8 t =8,,=2; t=n=2Y2.

1~2'

T=2asin~tan~; N=2asin~; 8t=2asin2~tan~:


arctanT'
2 ;

Sn=asint.

654.

2+2cp.

31

655.

St==431 2a;

Sn=a;

YI +431 n=a Yl +431 tan ~l=2n. 656. St=a; Sn= (Po -; ; t=Va +Q:; n=~Va2+Q~; tanll=-q>o' 657. 3ctn/sec: 0; -9cln/sec 658. 15 em/sec. 659. - %- mjsec. 660. The equation of the trajectory isy=x tan ut=2na
2

- - - x2 2v:cos 2 a
1/ V

The

range

is

v :sin 2a

The

velocity,
V o sin

v:-2v ogtsina+g 2fl; the slope of the velocity vector is vocosa Hint. To detennine the trajectory, elinlinate the parailleter t from the given systcnl. The range is the abscissa of the point A (Fig. 17). The projections
of velocity on the axes are

a-gt

V (~~r + ('~r ;
66l.

~.

and

~
the

The magm tude of the velocity is

the velocity vector is directed along the tangent to the with velocity
0.4 62. ({, {) .

trajectory

Diminishes

663. The diagonal Increases at a rate of -- 3.8 cln/sec, the area, at a rate of 40 cm 2 /sec 664. The surface area increases at a rate of 0 211 In 2 jsec,

the volume, at a rate 01 0.0511: mlJsec. 665. ~ em/sec 666. The mass of the rod is 360 g, the density at M is 5x gjcrn, the density at A is 0, the density at B is 60 gJcIn. 667. 56x6 + 210x4 668. ex2 (4xl -1- 2). 669. 2 cos 2x 2 670. 2(1-x ) 671. -x 672. 2 arc tan x+~ . 3 (I +X2)1 Y(a l +x2 )3 I -j-x2 2 2x arc slnx 674 1 t x 679 y'" -_ 6. 8 f'" (3 673. --+ 'i/" - COSal - . , 6 O. ) ==4 3'20 1 - x 2 ( I - x2 ) 2 a a
681.

YV=(X~41)5'

682. yVI= -Msil12x

684.0; 1;2; 2. 685. The velocity

is v=5; 4 997; 4.7. The acceleration, a=O; -0.006; -0.6. 686. The law of motion of the point M 1 is x =a cos wt; the velocity at time t is -aw sin wt; the acceleration at time t is -aU>2 cos rot. Initial velocity, 0; initial acceleration: -am2 ; velocity when x=o is -aro; acceleration when x=o is O. The maximum absolute value of velocity is au>; the InaximUln absolute value of acceleration is aw2. 687. y(n) =nlan. 688. a) n! (l_x)-(tz+l), 1 L b) (_I)II+1 .: . ~~~3). 689. a) sin (x+ n ~ ; b) 2" cos (2X +n ~ ) ;
2nx
J

Answers
c)
g)

407
)
(-I)"+I.n !. (I +X)n-t 1 ,
f)

(_3)n -'x.
e

d)

(_l)n-l (n-I)!.

(1 +x)n'

2n!

(l-x)'Z+I'

' ['2 +( -l)~J. h) (-l)n-J(n-l)!a" 690 ) x x 2n - I 51 n x n 2' (ax b)" . a x e ne .\ 1 b) 2"-'e- zx 2 (_I)" x Z 2n (-1)"-'x+ n (n 2- ) (-I )"-Z ] : c) (I-Xl) X

cos

(X + n;) -2nx cos ( x+


2n x
2

(n-/) n) -n (n-I) cos ( x+ (n -;2) n) ;


f or
693. a)
4 n~

d)

(-l)n-l.l.2... (2n-3) ('-(2 -I)]' ) (-I)n6 (n-4)! 2n + I X n ,e _ xn _ I


y(h)

691

(O):::-(n-I)!
c)

692.

a) 9t'; b) 2t 2 +2; c)

-VI-t

aSln

-:-~t;

b) 3
X (I

I. t . t: a cos sin

-I t ; d) t -: It' 694. a) 0; b) 2ea t. 695. a) (I 4a sin4 2" a sin


696

+t I) X

+3t

).,

b)

699

703.
707.

2t 700 4e (2sin t - cos t) 701 _ 6 at (I + 3t t 2) 702 "t'" sint (sint+cost)s e . . m d1x _ -1" (x). d1x 3 [f" (X)]2_f' (x)f"'(x) p2 b4 dy2 - [I' (X)I ' d yl = [I' (X)]5 705. - it 706. - a 2 y s 2y! + 2 d 2Y y d2x l i i I 1 -----ys-.708. dx2-=(I-y)l; dy 2=Y2 709. 256 710. 16.
2 cot t

t l+t (l _t)I

(cos t + sin t)1

-2e-

t 697

(d!Y) 1 dx 2 t=o =

71 t. a) x=1 718. 721. 725.

3~ b) -

3a 2 x

yS; . 712. L\y = 0.009001 ~ dy -== 0.009. 713. d (I-Xl) = I when


714. dS=2x!:J.x, L\S=2xL\X+(L\X)2.
n

1 and L\x=-a. No.


719.
1[

717. For x=O.

dY=-f2~

-0.0436.

720. 723.

dY=2700~0.00037.

dY=-45 ~ 0.0698.

a dx --z-+ x a

2.

dx dx .)2 724. ..r--= -x r a 2 -XZ -x 2 -2dx I + cos cp 726. - 2xe dx. 727. In x dx. 728. -1- 2 . 729. . 2 d<p. -x sin q>
722. ---;;;+T x

-mdx

(I

730.

l'tdt IOx+8y -1+e2t. 732.- 7x+5y dx. 733.


12

-ye Ydx
x

-Ldx. 734 ~+Ydx. x-y x-y


4+0.025~O

y2- xe
735.

rrdx.737. a) 0.485; b) 0.965; c) 1.2; d) -0.045; e)

81.

738. 565 cnl'. 739. Y5~2.25; Y17:::::::4.13; Y70~8.38; Y640==:::253. 740. VlO:::::=2.16; V70~4.13; V200:::::::5.85. 741. a) 5; b) 1.1; c) 0.93~ - (dX)2 -x (dX)2 d) 0.9. 742. 1.0019. 743. 0.57. 744. 2.03. 748. a/ 749. '{ . (I-xl) :I (l-x 2 ) 2 d 2 Inx+-x---,xr 2 cos x sin X) (x). 750. -Slnx 75t. 2lnxX-3 (dx)!. 752. _ e-xx

X (x -

ox +
'"

6 d)1 753 x. .

)(

384 (2 _ (dX)4 x)s 754.

3 2n sin (2x + 5

+2 nn)

(dx) 11 ..

408

Answers

755. excosa sin (x sin a

+na) (dx)n.

758. No. The point x=


763. (2, 4).

is a discontinuity of the function. 762. 6=0. 14 n I 765. a) S=g; b) ;=4. 768.1nx=(x-I)-"2(x-I)2+


t:

757.

No, since

f'

(2)

does not exist.

-1)1 ,w here + 2 (x 3!;'

~=

+e(X -I),
.

0<

e <. 1
Xl
5

769 Xl SlOX=X-3f
6

+ 4f x sin .

'='1'

x x 1vhere ~1=eJx, 0<8.<1; slnx=x-3f+51 -6fsin~2' where ~2=e2X, x 2 Xl xn - t x n :. ()<O,,<1. 770. eX==I-1-x+2f+3f+ ... +(n_I)I+ii1e", where ;=ex,

o < e < 1.
0<6 < I.

772. Error: a) -16

X'

(1

+~)

5/
2

b) -

5 81 (I

Xl

773. The error is less than


I

~=~.

+~)

8( ;
3

In both cases ~ = 8x;


Solution. We have

775.

v
\ve

a x) -+ -x = ( 1+CI-X

I
t

X ) I . 1-a

Expanding both factors in powers of x,


I

X) I ::::: I +-I -x get: ( 1 + a 2 a

-1

Xl

8 a2

- .
,

1- -X ) a

::::: 1+-2 a

1 x

3 x +-8 a
2 2

Multipl) In:,:. we will have: .. / a+x:::::: 1 + ~

a-x

+ 2a ~.
2

Then, expanding e a in

ro\vcr~

get the same po 1ynomial ea ::::: I of ~,we a


779. I 780. 3.
789. 1.

x + 2a x +a

2 2

777.

1 -"3.

778 00
786. \.

781.

2"

782. 5. 791. a.
792.

783. 00.
00

784. O. 785.

2"

n!

2 788. - .
11:

790: O.

for n

>

I; a for n = 1;
800. ea. 801. 1. 810. Hint.

o for
802

n < 1. 793. O.
1

795."5. 796.

12

797. -1.

799. 1.

803.

I I 804. - . 805. - . 806.

where S=~ (a-sin a) is the exact expression for the area bh 2 3 of the seglnent (R is the radius of the corresponding circle). Find lim
u -~
0 _

-i-,

807. 1.

808. 1.

Chapter III
811. (-00, -2), increases; (-2, 00), decreases. 812. (-00, 2), decrease~; (2, 00), increases. 813. (- 00, 00), increases. 814. ( - 00, 0) and (2, 00), increases; (0, 2), decr~ases 815. (- 00, 2) and (2, 00), decreases. 816. (- 00, I). increas~s; (1, 00), decreases. 817. (- 00, -2), (-2, 8) and (8, 00), decreases. 818. (0, 1), decreases; (1, (0), increases. 819. (-00, -I) and (1, 00), in

creases; (-1, 1), decreases .crea!>es; (

820. (- co, co), increases

821. (0, -}-).


00 ,2),

de-

~,

(0). increases. 822. (-2, 0), increases. 823. ( -

decreases;

Answers
(2. 00). increases.

409 .

824. (-00. a) and (a. 00), decreases: 825. (-00,0) and


827. Ymax =

(0, I), decreases; (I, <Xl), increases

when

x=~.

828. No

extremum. 830. Ymin =0 when ~==O; Ymln=O when x = 12; Ymax = 1296 when x = 6. 83t.Ymin~-O.76 when x~0.23; Ymax=O when X:=I;Ymin~-0.05when x:::=::: 1.43. No extremum when x = 2. 832. No extremum. 833. Ymax =-2 when x = 0; Ymin = 2 when x = 2
=-3

834. Ymax = 16 when x = 3.2. 835. Ymax =


lr-

lrr 3 when

x= - Va; Ymin=3, 3 when x=}ra 836. Ymax= r 2


839. Ymin=-2

,r-

when x=O

837. Ymax=-ya \vhen x=-2Y3; Ymin=

838. Ymtn-=O when x= I; Ymax=1 when x==O

va when x==2V[ r3
1

3 \vhen

x-= ( k-

i) n;

Ymax=~ V3"

when

x=

(k +{ n)
when

(k=O, I, 2, .. ).

840. Ym3x-=5 when x=12 kn;

Ymax=5coS~ when
Ymln=1

X=12(k{)n; Ymin= x=6(2k+l)n


(k-=-=O,

=-5cos';whenx=12(k})n;

1 1 ... ).841. Ymin=O when x=O. 842. Ymin = when x= e . 4 1 when x=l 843. Ymax == e 2 when x = '2 844. Ymin == 1 wh~n e ; Ymin = 0 4 1 x=O 845. Ymin = - - when x=-l. 846. Ymln=O when x-=O; !lm.lx ~e2 e when x=2 847. Ymin=e when x= 1. 848. No extremunl. 849. Srl1al1esl 1 value is m=- -} for x=-I; greatest value. M =."2 when x == 1. 850. nz = 0 1. 2,

-e

when x=O nnd x= 10; M =5 for x=5.


M==l

851.

1 m=2"

when x=(2k

n + I) T;

for

x=2 (k=-=-O. 1.

krt

2... ).

852. m-==O when x=l; A1=J1 \vhrn

x=-I.

853. nl=-l \vhen x=-I; M=27 when x=3. 854. a) fn----6 when x == I; M ~ 236 when x = 5; b) nt = -1579 when x = -10; M = 3745 \vhen 856. p=-2. q::..::.4. 861. Each of the terms must be equal to a 2

x=12.

862. The rectangle must be a square with side

f.

863. Isosceles. 864. The

side adjoining the wall must be twice the other side cut-out square must be equal to base. 867. That whose

i-.

865. The side of tllt:

866. The altitude must be half the

altitude

is equal to the diameter of the base


radius of its base

868. Altitude of the cylinder,

~~;

vi,

where

1<

is the radius of the given sphere.


where

869. Altitude of the cylinder. R Y2" R is the radius of the given sphere. 870. Altitude of the cone, ~ F

410

Answers
871. Altitude of the cone,

where R is the radius of the given sphere. where R is the radius of the given sphere. cone

~ R,

{r,

872. Radius of the base of the

where r is the radius of the base of the given cylinder. 873. That

whose altitude is twice the dianleter of the sphere. 874. cp = 1, that is, the crosssection of the channel is a semicircle. 875. The central angle of the sector is 2n

Vf.

876. The altitude of the cylindrical part must be zero; that

is, the vessel should be in the shape of a hemisphere. 877. = X 878. 2 +2Y = 1. 879. The sides of the rectangle are a V2' and b y~ where
Xo Yo a and b are the respective selniaxes of the elli pse.

h (I: -d: ):.


y~).
r -

880. The coordinates of

the vertices of the rectangle which lie on the parabola (; a; 2 88t.

( ~3'
1

~)-

882. The angle is equal to the greatest of the numbers


h
883.

.arc cos -k
885.
Pmin =

and

arc tan -d .
b) X=

.4M

=aV

Vii
p

a)

d x=y= Vi";

V3;

y=d

-.

/2 V 3"'

+ Vi

884.

Y2
;

886. X=

-. /2aQ V-q

V2a'lQ. 887. VMm. Hint. For a completely elastic impact of two spheres, the velocity imparted to the stationary sphere of (nass m l after inlpact with a sphere of mass m 2 moving with velocity [) is equal to
nZI
1 2,+n v

N,\ve take the closest integer which is a divisor of N). Since the internal resistance

n1 2

. 888. n = ... / N R (If this number is not an integer or is not a divbor of


r

of the battery

is

n;,
2

the physical meaning of the solution obtained is as

follows: the internal resistance of the battery must be as close as possi ble to the 2 external resistance. 889. Y="3 h. 891. ( - 00, 2), concave do\vn; (2, 00), concave up; M (2, 12), point of inflection. 892. ( - 00, 00), concave up. 893. (- 00. -3), concave down, (-3, 00), concave up; no points of inflection. 894. l - 00, -6) and (0, 6), concave up; (-6, 0) and (6, 00), concave down; points of inflection M1
Ml
00,

(-6, -{), 0 (0, 0), (6, {). 895. (- V3') and (0, v3"), concave up; (- va. 0) and (va, 00), concave down; floints of inflection M v3: 0) and 0 (0, 0). 896. (4k + I) i ' (4k+3) i), concave up; (4k +3) i, (4k+5) i) ,concave down (k=O,
I, I

I, 2, ... ); points of inflection, (2ft + I) 0)' 897. (2kn, (2k + I) n), concave up; 2k-l):rt, 2k1t).concave down (k=O, 1, 2, ... ); the abscis-

i,

~as

of the points of inflection are equal to x= kn.

898.

(0, V"ii). concave

Answers
,iown; (

41 ]

~ea'

00),

concave up; M

(~ ii' -

:e.)

is a' point of inflection.

899. ( - 00, 0), concave up; (0, 00), concave down; 0 (0, 0) is a point of inflection. 900. (-00, -3) and (-I, 00), concave up; (-3, -I), concave

down; points of inflection are M1 (-3,


y == o.

902. x == I, x ~ 3, Y == 0 903. 904. Y == x. 905. y == - -", left, y==x, ri~ht. 906. y==-I, left, y== I, right 907. x= I, y==-x, left, y==x, right 908. y==-2, left, y-=2x-2, rIght. 909. y=-=2 910. x-::O. y==l, left, y==O, right. 911. x=O, Y==l. 912. y~O. 913. x-=-l. 914. y=x-n, left; y=x+n, right. 915. y=a. 916. Ymax==O when x==O; Ymin ==-4 when x==2; point of inflection, M 1 (I, -2). 917. Ymax= 1 \vhen

~) and x == 2, y == l.

M 2 (-I, :). 901. x=2,

x=

Y3;

Ylhin=O

when

x=O;

points of

inflection

M I , . ( I,

~)

918. Ymax=4 when x=-I; YmlO=O \\'hen x=l, point of inflection, M 1 (0,2). 919. Ymax=8 when x=-2, Ymin==O \vhen x==2; point of inflection, M (0,4). 920. Ymin=-l \vhen x-:-::O; points of inflection M 1 2( Y5, 0) and

- 125 . 921. Ymax ==- -2 \vhen x =-= 0; Ymin = 2 \\'hen x -:= 2; asymptotes, x ==-= 1, y =- x-I. 922. Points of inflection M I, 2 ( 1. =f 2); aSylnptote x==O. 923. YmJx=-=-4 when x-=-l; Ymln~4 \vhcn x==l; as)lnptote. x===O. 924. Ynain = 3 when x= 1; pOint of Inflection, 1\1 (-' 2, 0); aSyl1lptote,
M a,4 (

I,

64 )

V-

x=O. 925. lIlhax={ when

>.=0,

points of II1flectlOn,

MI , .

(l' -{-);

asytnptote, y = 0 926. Ymax --- -2 \vhen x-=-- 0; asynlptotes, x --:: 2 and Y::= o. 927. Ymin-=-=-J when x:=.:--l; YmJx= 1 \vhenx= 1; pOlnt~ of inflection, 0(0.0) and M I ,2

( 2Y3,
0 (0,

~2; aSy1l1ptote, y=:::O


M

J-r 3 )

928. !lmax=1

\\'hen x--:4;
929. POint

point of inflection, of
inflecti~l1,

(5, f);

aSyl1lptotes, x = 2 and y =--= 931.

O.

0); aSy111ptotes, x-=- 2 and y -= O.


x --= 0, x.:=: 4 and Y == 0

930. Ym.1X -== -

27 T6

\\Then x =--=

"3;

nSYlll ptotes,

YmJx = -4 \vhen

x=---l; Ymin==4 \\hen x== I; aSylnptotes, x==o and y=3t 932. A (0, 2) and R (4, 2) are end-points; Ymax=2Y"2" \vhen x=2 933. A (-8, -4) and B (8, 4) are end-points. POint of inflection, 0 (0, 0). 934. End-polllt,

V V + ~;).
6

A (-3, 0); Ymin-=-2 \vhen x=-2. 935. End-points, A(-V-~ 0), 0(0,0) and B(V~ 0); Ymax= 2\vhen x=-I; point of inflection, M(3+2Y3,
1
936.

points of lI1f1ection, M I , 2 ( 1, 0). 937. Points of inflection, M 1 (0, I) and M 2 (1, 0); asyrnptote, y=-x. 938. Ymax=O when x=-I; Ymin=-I (when x=O) 939. YmJx~2 when x=O; points of inflection, M I 2 (1, asymptote, y=-=O. 940. Ymin=-4 when x=-4; Ymax-=4 when x=4; point of inflection, 0(0,0); asymptote, y==O. 941. Ymin= 4 when x=2, Ymln= 4 when x=4; Ymax==2 when x=3. 942. Ymin=2 when x=O; aSylnptote, x= 2.
x=O,

Ymax = 1 when

V2);

V-

V-

943. Asymptotes, x=2 and y=O.

944.

Ymin-=

Y3 Vi

when x=Y3;

412

Answers

YmdX=and

Y3 V2

when x=-3; points of inflection, M)

M. (3,

~);

asymptotes, X= l

945. Ymln=

V2

(-3, -

~),

0(0,0)

when x=6; point point

of Inflection, M (12, of IIlflection,

VI2100 );asy m ptote,X=2 946.Ymax=! when x= I; e


M. ( -a,
inflection,

M (2, ~); asymptote, y=O.


and of

947. Points of inflection, 948. Ymax=e2 when Y=O.

MI (-3a,
x=4;

l~aa)

); asymptote, Y=?

points

M I , . ( 82[2 , el ); asymptote,

M9. Ymax=2 when

x=O; points of inflection, MI , . (l.

~).
M
(

950. Ymax=l
2

when x=I; !/rnin=O when x=O. 951. Ymax=O.74 when x=e =::::.739: point of inflection, M (e Mi :::::: 14.39, 0.70); asymptotes, x=O and Y =0. 952. Ymln = - 4e
a
2

when

x=

a... yi"' pOInt of Inflection,

a ye;' -

4e2

3a

953. Ymin = y",O

when

x = e;

point of inflection, M Yrnax="T=::::'O.54


4

(e 2 , e~);
\vhen

asym ptote,
1

x=

1;

when

x ...... O.

954.

x=s--1:::::::-O.86;

Ymin=O when x=O: point of inflection,

(+-1~-0.63; ~~0.37):

Y -; 0 as x ...... -1 0 (limiting end-point). 955. Yrnln = 1 when x = y-2'; points of inflection, M I , 2 ( 1.89, 1.33); asymptotes, x=- I. 956. Asymptote, y=O. 957. Asymptotes, y=O (when x -+ (0) and y=- x (as x ~ - (0).

958.

I- ~ , 0]. 959. Periodic function with k . rn \vhen x=4 n+2kn; Ymax= Y 2 when x=4+ 2kn
interval points of inflection, perIod 211:. n Yrnln = Mk (

Asymptotes, x=_!, x=O, Y= I; the function is not defined on the e period 2n. Ymin = -

Y2
... ); with when and

(k=O,
Periodic

I,

2,

~ n + 1m,
3 when

0).

960.

function

3 4

.rY

5 x ="'3 n + 2kn;
points of

Yrnax =

3 1r '4 Y 3
0)

X=

3+ 2kn (k=O,

I, 2, ... );

inflection,

Mk(kn,

N k (arc cos ( - {- )
'On

+ 2kn, ~ YIS).

961. Periodic function with period 2n. Ymln=-2


I

the Interval [-no nl, Ymax=~ whenx=:: x= n; Ymln =0 when x=O; points of inflection, M 1

when

M a, 4 ( 2 ~O, -0.95).

(0.57, 0.13) and 962. Odd periodic function with' period 21t. On inter-

-val [0. 2nJ, Ym3ll= 1 when

x=O: Ymln=0.71, when x=i: Ymax=l when

Answers
x

413

=2;

Ymln=-I when x=1t; Ymax=-O.71 when x=4"1t;Ymin=-1 when

x =

n;

Ymax= 1 when
M a (2.36, 963.

x=2n;
0);

points

of

inflection, -0.86);

M 1 (0.36, M s (4.35, Ymln = I,

0.86);
-0.86); when ... );

M 2 (1.21, 0.86);

M 4 (3.51,

M. (5.50, 0).
~

Periodic function with period 2'1:.

x=4+2k~; Ymax=--2-whenx=-"41t+2kn

VI

~2
2,

(k=O,

asymptotes, x= inflection, M k

965.

Even 3r 3

per iodie function

~ V2) (k=O, -2( 4+k~,


with r 3

~ n + kn

964. Periodic function with period n; points of

3 I, 2, ... );asymptotes,x=4n+k~.
period 2n On the interval [0,
.1t

Ymax =

4 I ,r- when x = arc cos ,r- ; Ymax = 0 when x::.:: ~; Ymin = -

x=arcCOs ( -

~3> Ymln=O

3r 3

4 .. r- when

when x=O; points of inflection, M 1 n-arcsin

(; ,

0) ;
Even

M. (arcsin

~2, 4~7);

M.(

~2, _ 4~f).

966.

periOdIC function with period 2~. On the interval [0, ~] Ymax = I \vhen 2 x==O; Ymax= lrwhen x=arc cos - ,r- ; Ymin= ,rwhen 3r6 r6 3r 6

( I)

x=arc cos

:6" ;
l

Ymin = - 1 when x=n; points of inflection, M 1

,0);

M. (arc cos arc cos ( 967. Odd function. Points of inflection, M k (k~, k~) lk =0, I, 2, ... ). 968. Even function. End-points, Al 2 (2 83, -1 57) Ymax == I 57 when x=O (cusp); points of inflection, M 1 ~ ( 1.54, -0.34). 969. Odd function. Litnlting points of graph (-I, ' - 00) and (I, 00). Point of inflection,

Y~. ~ y~);

M.(

V~),

-{ y~}

0(0, 0); asymptotes, x=

I.

970. Odd function. Ymax =

~ -I + ?kn when

=~

+kn;

Mk(k~, 2k~);

n+ 1 +2kn when X= ~ n+kn; points of inflection, 2k+1 asymptotes, x= - 2 - 31 (k=O, I, 2, ... ). 971. Even
Ymln=

function. Ymin=O when x=O; asymptotes, fJ=~

~ x-I (as x -i""-oo) and

y =2""x-1 (as x-++oo). 972. Ymin=O when x=O (node); asymptote, y=l.
973. Ymln = 1

+~
of

when x= I; Ymax= 3: -I

when

x=-I;

point of

inflection (centre of symmetry) (0, ~); asymptotes, y = x 2~ (left) and Y = x (right). 974. Odd function. Ymln=I.285 when %=1; Ymax=I.856 when

x=-I;

point

inflection,

(0,

~);

asymptotes, Y=

~ +n

(when

x-.-oo) and Y=2

(as x-++oo). 975. Asymptotes, x=O and y=x-ln2.

414

Answers

976. Ymll1=1.32 when x=I; asymptote, x=O. period


2n.
Ymln=

when x=

i- n + 2kn ;

977. Periodic function with


Ymax=e when x=

lrr5-1 (k=O, I, 2, ... ); points of inflection, M k ( ;lTCSin---+2k31, e 2

}IS-I')
2

+2kn

Y5-1 YS+l\ and N k ( -arcsin-2 -+(2k+l)n, 978. End-points, A(O, I) and B (I, 4.81). Point of inflection, M (0.28, 1.74). 979. Points of innection~ M (0.5, 1.59); asymptotes, y=0.21 (as x -+- 00) and y=4.81 (as x ~ -1- 00). 980. The domain of definition of the function is the set of intervals (2k31, 2k31 + 31), where k = 0, 1, 2, ... Periodic function wit h period 231. 31 Ymax=O when x=2"+ 2kn (k==O, I, 2, ... ); asymptotes, x=k31.

e')'

981.

The

domain

of

definition

is the set of intervals [( 2k - { ) n,

( 2k +
X=

-4- )

n], where k is an integer. Periodic function with period 2n.

Points of inflection, M k (2kn, 0) (k ==0, I, 2, ... ); asynlptotes, 31 T+2k31. 982. Domain of definition, x> 0; monotonic increasing function;
(k

asymptote, x=O.

983.

Domain of definition, rt

I x-2kn 1< ;
Ynllll = I when

== 0, I, 2, ... ). Periodic function with period 2rt


-+

x==2kn (k==O, I, 2, ... ); asymptotes, x==T+kn.

y = 1.57; Y -i""-1.57 as x

(linliting

984. 985.
1

Asymptote, End-points, when


O)~

end-point). 986. Ymln=(

AI, .(1.3I, 157); Ymln=O when x=O.

f)"e ::::::0.69
A (+0,

x == e
Ymax =

0.37; y

-+

I as

-+

987. Limiting end-point,

e e =::::: 1.44 when x == e ~ 2.72; asymptote, Y.::= I; point of inflection, M I (0.58.0.12) and M2 t4 ~5, I 40). 988. xmin == -I \vhen t == 1 (y == 3); Ymtn = - t when t = -I (x = 3) 989. To obtain the graph it is sufficient to vary t from to 2rt. xmin = - a when t = 3t (y == 0); xmax ==a when t = 0 (y = 0); Ymin -:: - a (cusp) when

t=+T (x=O);
when 1=4' 990.
31

331

Ymax==+a (cusp) when


531 731

t==2

~.

(x==O); pOints of inflection

4' T' T
I

3Jt

( a a) x= 2y"2" , Y= V2 .
I

xmin=--

when t = - I (y=-e); Ymax=- when t=1 (x=e); points of e

inflection when 1 = I.e., (

Y2,

Le., ( -

:::~,

V 2eva )

and when 1 =

V 2.

)12' Y -2 eV -" eva);

asymptotes, x=o and y=O. 991. Xmin= I and Ymln = 1

when t=O (cusp); asymptote, y=2x when t

-++OJ.

992. Ymin==O "'hen t=O.

Answers
a dx, '993. ds=y
~osa=

415

y. cos a=-; Sin a-= - - .

V aI_xi

Va 4 ---.c2 x2

;sina-==-

bx

Ya 2 - c2 x!

a a -x ..r - ,where c=-ra"-b 2 995. ds=


p V p2+ . y"

994

r d s=-

ya

_C X
2 2

2 2

dx;

1 =Y

V p2+y 2 dx;

cosa=

V p2+ y 2

y ;. sma=

996. ds=

V'

a -dx,
x
a

cosa==

~VXa;

sina-==-

VI.
~

997.

ds=cosh~dx;
a

cosa=_l_; cosh ~

sin a'-" tanh : . 998. ds = 2a sin

dt; cos a =sin

~;

sin a= cos

. 999. ds =

== 3a sin t cos t dt; cos a = -cos t; sin (J,:= sin t. 1000. ds == a 1 cp2 dq>; cos ~ 1 a .. r--2 1 a = 1001.ds= 2 f 1 dcp; cos ~= 1002. ds == - - dcp; 1 -1- cp2 q> 1 <p2 cos l .!

V +

+q>

V-_. +

Sill

~ = cos

-t

2
1004.

1003.
sin

ds = a cos

~ dcp;
.

sin

~ = cos ~.
a
2

ds-=

~,

y 1 -1-

(In a)2 dcp;

p=
1

V 1 + (In a)2

1006. K---=36. 1010.

K -= a

:'2

1007. K-=3 at

Y2

1005. ds =-= - d<p; sin p= cos 2q>. , a b 6 1008. KA=b 2 ; K B =(i2' 1009. K=13 V13'

both

vertices.

101 I.

1012. ( t015.

_ 1n 2 2'

Y2 )
2

. 1013. R -

_I

i '3 )
RIc.,st = =

and

(:'

-3).

(1 -1- 9X 4)3/2/

6x

. 1014. R -

_ (b 4x2

+ a4y!)3/2 ab
4 4

+ 1)21 1016. R = I 2 3 R =-:: (y2 4y


1

aSHl

. 2t 1 .

1017.

R == I at I.

1018.

R=
(2,2).

= I r V!+FI . 1019. R = ~ a cos ~


t023. ( _1; a, 1: a ). 1024.

/.

1020.

(X-3)2+

(y- ~

I p I.
1025.

1022.

~.

(X+2)2+

+ (y -3)2 = 8.
2 4

1026. py2 =
\vhere

~ (X -

.!-.
p)' (scmicubical parabola). 1027. (aX)
I

+ (bl")

a ==

C 3 ,

c2 ::::: a2 _b 2

Chapter IV
In the answers of this section the arbitrary additive constant C is olnit.. 5 x. ted for the sake of brevity. 103t. 7 a2x7 1032. 2x 3 +4x 2 +3x. 1033. T+

(a

+3b) Xl + abx 2

1l-1

1034. a x + 2
2

abx4

-1- 7

b2x"

1035.

3 '

2x '1m=::-

2px. 1036.

nx 11 n-l'

416 / V nx.
4

Answers
9 4
I

1037.

1038. alx-Sa
l

X I

+7a I
1041.

9 -

X I

-3.

Xl

1009.

2x 5 Yx
l

+ x.

3x 3x 1040. --13- 7

Vx

Vx - 63V-x.

2x 2m
4m

+1

yX 4xm + n

2m + 2n + 1 +
1
arc arc tan sin

Yx

2x 2n Yx
4n

..r..r1042. 2a r ax- 4ax + 4x r ax- 2xl


1044.
2

1 In ...rf 10 x+
Sin

\x- V VIOl .
10

5 r ax

2x' ../"_

1043.

Y7
1046.

+1 x 7".
x Y
2

1045. In (x

+ .rr 4+ Xl).

1047. arc

~-In(x+ yX1 +2). 1048*. a) tan x-x. Hint. Put tan1x=

=sec 2 x-l; b) x-tanhx. Hint. Put tanh 2 x=1

-x; b) x- coth x. 1050.

In(;e~ I'

1051. a In

I I
aC x
dx+

cos

1 h I . 1049.a)-cotxx

Solution.

Sa a x dx =
;;~~ =

=-a5~(a-X)=-alnla-xl+alnc==alnl-C-I. a-x a-x


2 =1+2x+l'

1052. x+lnI2x+ll

Solution. Dividing the numerator by the denominator, we get


Whence
1053.

5 + =5 5
2x 3 2x+l dx 3 11

=x+ ln I2x+ll
a
ba-a~

-2x+41nI3+2xl.
x2

2 dx d (2x + 1) 2x+l=x+ 2x+l = x a 1054. 7]-bllnla+bxl.


1057.

1055. aX+~lnlax+~I. 1056.2"+x+21nlx- 1 1.


-~ 1n

2+2>:+

Xl

I x +31 1058.
t060.~
\

x x' 4"+3" +x 2 +2x +31n I x-II.


4

x-a =
dx

b?

In 1 x+ll+ x

1 + 1

Hint.

5(x+l)2= x dx S(x(x+l)2 + 1) - 1 dx=


2 '1r (
a

1059. a2 x+2ab 1n! x-al-

1062. -3ij'" (a-bx). Sx+I-J (x+l)2' 1061. -2b f l-y. X1 1 t063. V Xl + 1. Solution. S .:xdx =2 S + I) = y' Xl + l. 1064. 2 yx + . +1 +1
2

dx

... r -

Xl

+ lnlx -2

t065.

1 -./3 ..rarc tan x JI -5'


f

1 Ya-b 1067. II"" In - .. f"' a+b+x Y 2 f a"-b 2 r a+b-x a-b


1069.
1071.

Ir

15

1066.

I.

1 .rIn 4 ,,14

Ix Y Y7-2 V Y21
x
7 +2 2 arc
tan

t068.

x- .rr2

- (

~2
2

+
1

~I

In I a2 -x ll )

1070.

x1072.

x 1r-' r 2

In (x 2

+4) +arctan ~
...

Y2 1n (2

.. r.. ~ r 2x+ r 7+8x l ).

1 ... is Vsarc sin x JI 7'

1 5 ~r3 __ 1073. 3"ln l 3x -21- 2 f6 1n " V3+

Ix

Y21 Y2 1074.

Y35 arc tan

/S V 7"-

Answers
-{-In (5xl +7). 1075.

417

i Y5x
1

+ I+

;5
f

1n (xY5+

Y5x

+ 1).107&.

Yx
1

-4 +

+ 31n I x+
1079.

wr-r x2 -41 .

1077.

1 2 In lxi - 5 1.

1078. 1081.

I 41n (2x 2 +3).

2U In (a1x' + b2 ) + a

ax 1 Xl arc tan b 1080. 2" arc sin (ii".


Y(arc sm x)'.

3" arC tan x'.


4

( arc tan 1084.


2 1086.

1082.
1085.

} In I x' + Y x6 - 1 I 1083.
1 -8 In (1

-i

+ 4x 2)
1088.
1

(arc tan 2(,'

1087. -

e-"'''.

--2 ea. 1091. l i b bX--x -2x. 1092.


1093. - 2
X 2 +1.

-~

3-1- Ya 3X + n a~ n a n a In a W/ r aX
1

-:31~4 4 2 - ' ' ' . (aX bX)

J!

r-------::::----

In (x+ YI
2X

+ X2)~

1089. e1 +e- 1.l090. ; eli +2x2 2

1094.

1 X2 2 In 7 7.

1095.

-e x 1099.

1096.
x
4

2 5 V-X li15

1097. Inle"-II. 1098.


- 3 I~ 2 In (2" + 3)
t 102.

-,~
hX

Y(a-be")'

~(ea +1)3.

1100.

i-

Hint. 2" ~ 3 """

1 In 11 2b 1 +e_e- hX

( 1- 2}: arc sin et

3)'

l 1101. In a arctan (a").


-

1103.
I

1104.

1 b

cos (a + bx) . -In lOX

'105.

..rx y 2 ~1l) Y"2


1109.

1106. x-

2a cos 2ax.

1107. 2 sin r x.
Stll!

.r-

1108.

xcos (log x)
S1 n 2( +-4-

"2 - - 4 hint

Sitl 2~.

. Hint. Put

1 x ="2 (I-cos 2~). 1110.

. S Hint. ee

. 09 In 11

co t ax lIlt. -tan(ax+b). 1112. ----x. a a


1

ttt3. alnltanil. 1114.


1116.

~lnltan(?;+i)l.
I I "2 cos (l-x ).

ItIS.
1118.

Inltan

-2 tan (x).

1117.

x-

I V-2 cot x

axtwr- I.
b

r 2-

_Y2Inltanx~I.I119. -lnlcosxl.

1120. Inlsinxl. 1121. (a-b)X -2InlcosVXI.1I24.


1126

xlnlslna~bl. 1122.5Inlsin~l
I

1123.

~Inx
Ct S

1128.

l 1125. i t X I sin (x + I). n' an x I. 1 1


4 4 a sin ax
5

asin . I "2
1130.

a
X

1 27 SI1)4 6x 1 ~.

1129.

- -3 In (3 +cos 3x).
32 11.. 3 t an 4 3 x "4

- -2

I wr-f

2x.

1131.

2 .r - 9" r (1 +3 cos x)

1133 . " 3 2 .r-t - ,r an x.

1134.
t4-1900

3CO~ 'x. 1135.;

(tan3x+cos\x). 1136.

{(lnltan~I+2sll1ax)-

418
1

Answers
2

1137. 3Q1nlb-acot3xl. 1138. 5cosh5x-5s1nh5x.1139-2+4s1nh2~.


tt40. In tanh

a .

I i I
Y5'
x.

tt41.

2 arc tan e~.

1142.

In 1tanh x I. tl43. In cosh x.

5 tt44. In I sinh x I 1145. - 12 xarc tan


1148.

V (5-x
1
-X2

)'.

1 In I ~-4x + 1 . 1147. 4 Y5 1 X 1146. 4"


...

-"2 e
2 ).

1149.

13 V 2" arc tan

(r 11 a) '2 x

x +x-2In/x+1I1151. -}._. 2 ,. eX 1 1 1152. Inlx+cosxl. 1153. 3 (Inlsec3x+tan3xl+~3). 1154. --1-. sin x nx

1._ 1n (X
3

Y3+ (2+3x

1150.

'a

1155.

lnl tanx+ Ytan l x-21. 1156.

1157. 1161. x
1165.
l

a SI n x 1158. l"il""a.

V\X +
a

Y2

arc tan (x Y2">- 4 (2X:+ I)

1)2 1 1 --2- 1159. 2 arcsin (x 2 ). 1160. a-tanax-x.

sin x . tan x x n ) -, 1162.arcsln-1163.alntan 2a+T 2 2 2 --

I (

I.1164."4 3 V (1+lnx). "


1167.

-21n I cos Yx-II.


2

1166.

{-In tan

I i /.

x + 1n (14+ x ) +arctanx. 1168. -lnlsinx+cosxl 1169. Y21n tan 2 (2


- 2x 1172. e

eare tan %+

I-

Y2 cos
sin 2X

';'2'
1173.

1170.

x+ ~21n / :~ ~~ /. 1171. In 1x 1+2 arc tan x. 5 . X Y3 . r--2 ya arcstn--+ y 4-3x. 1174. x-In(l+e X).
2

1175 r 1 a;ctanx,la-+bb.1176.1n(eX+Ye2X-2).1177.-.!.lnltanaxl. f a2 -b 2 a a

1178. -2n cos

( 2nt)

r+q>o
~

1179.

1 1n /2 + In x 2-1nx 1180.

( I
f

arc cos 2

1181. - e- tan %. 1182.

arc sin

(-7i) .
2

1183. -2cot2x. 1184. (arc

st

r
I.

X)2

- Y 1 -Xl.
1187.
dx

1185. In (sec x -I-

Y sec

x -I- I). 1186.

~2 arc tan (
1188.

t;:;) .

Hint.

Ssin! x +dx2 cos ---cos x =S1 +dx


2 2

4 ,. 5

~r- In ~5 -I- 'lill 2x

5-sin2x

Jt

==.\ta~~:2'
n

j-Vlln(x-I-Y1-I-X'))"
1/;i ~

1189.

i-

sinh (x'-I- 3).

1 3tanh % 1190 . 3 1191. a)

arc cos -

Y2

x when x >

"lry

2;

Answers

419
e)

c)8~(5r-3)';
1192.

d)

~ V(x+1)1-2VX+'1;

In(sinx+Vl+sin'x).

[(2Xi;5)12 - 5

(2~lt5)tt].
1198.
/'

1193. 2

(V; - ; +2 yX~2Inll+Vii)
1196.

1194. Inl

~2X+T-1 1.
y

1195. 2arctan Vex-I.

Inx-ln2Inllnx+
1199. j.(COS'X-5)X

2x+I+1

+21n21. 1197.
X

(arc~tlx)l.

}(tr-2)

VeX + I.
l

yeGs x. 1200. In

II+Vx
1

+1

Hint. Put x=-t . 1201.

-i Vl-x'+
'1r-x2 a2
f

1 +2 arc sinx.

1202.

x2 .. r - 4 .. r - -'3 y2--x 2 -:r y2-x 2

1203.

-a arc cos ~. 1204. arc cos 1.., if x x x


Put

>

0, and arc cos ( 1206.

-!.) if x <0 *) x

Hint.

x=-}.
2 y

1205. J"X'i+1-1n 11
z

+ ~XZ+TI.

V~ . Note. The

substitution x =!. may be used in place of the trigonometric substitution.


1207.
1 .. rx .. r2- +"2 arc sin x. 1208. 2 arc sin r x. 1210. "2 y x -a2 -+al '1r-I +2" In 'x+ y X 2 _ a2 1. 1211. x In x-x. 1212. x arc tan x-'2ln (l-t- x2 ).

x ..r - -2
1 _x

1213.

x arc sin x+
x+1 -ex-.

YI

Xl.

1214.

sin x-x cos x.


1218.
e~x

1215.

x Sl; 3x + cn~ 3x .

1216.

1217. -

x In 2 + 1

2 xlf1i2

2f (9x

-6x + 2). Solution. In place

of repeated integration by parts we can use the following method of undetermined coefficients:

~ re1xdx=(Ar+Bx+C)e'X
or, after differentiation,

x2elX = (Ax 2 + Bx + C) 3eIX + (2Ax + 8)e1x


Cancelling out elX and equating the coefficients of identical powers of x, we get:

1 = 3A; 0 = 38 + 2A; 0 = 3C + 8,

1 8=-2 ; C=2j. 2 whence A="3; In the general form, Pn(x)eaxdx=-. ax = Q (x) e , where P n (x) is the given polynomial of degree nand Qn (x) is a p01 ynomial of degree n with undetermined coefficients 1219. _e- JC (x 2 +5).
x

Hint.

See

Problem

1218*.

1220.

-3e

a (x'+9x 2 +54x+ 162). Hint. Se~

*) Henceforward, in similar cases we shall some'times give an answer tha' is good for only a part of the domain of the integrand.

14*

420
Problem 12IS*. 1221.

Answers
_ x c~ 2x

+ SillS2x.

1222.

2x

+ I~X+ II sin 2x+

+ 2X: 5 cos 2x

Hlnl. It is also advisable to apply the method of undeter

mined coefficients in the form

~ Pn (x) cos px dx = Qn (x) cos px + Rn (x) sin px,


where Pn (x) is the given polynomial of degree n, and Qn (x) and Rn (x) are pol ynomials of degree n with undetermined coefficients (see Problem 1218*). Xl x' In x 1 2 1223. 3 In x- 9" . 1224. x In x- 2x In x 2x. 1225. -- 2x l - - 4x 2

1226. 2

V -x In x-4

..rx +I X x 1 r x. 1227. -2- arc tan x-"2. 1228. 2 arc sin x-"4 X
2 2

X arcslnx+ ~ VI-xl. 1229. x In (x+ VI +x2 ) _ VI +xl 1230. -x cot x+

+ In I sin x I
1233
1

1231.

-~ + 1 It!-.I sin x n an 2

1232.
2

eX (sin x--cns x)

ax (sin x +cos x In 3)

+ (In 3)1
Xl Xl

1234 eax (a sin b"t-b cos bx)


.

-cos (1nx)). 1236. --2-(x1 +l). 1237. 2e


x 2 -1

e-

al

+b
y-

1 35

2 sin

!- [ .

(I
3

X2

x (Vx-l).

+3x ) Inx- g +2"-3x. 1239. -2- 1n I+x- x . 1240. --x---x--X-.


Xl x I I I + x2 1241. [In(lnx)-l).lnx. 1242. aarctan3x-T8+162In(9x2+1).1243.-2-X

I-x

1238. 3-X2+ In 2 x 21n x 2

(x

nx

)_

1 x(arctanx)2-xarctanx+"21n(l+x2).
. xarcslOx-2x.1245. arc sin x +In
x

r X arc sin .. f

. x +2 ..r f x

1247.

x -1246. -2 yrI-xx 1 I-xl x x tan 2 x+ In I cos 2x ,_ ~ 1248 e2 4 2' . 2


X

I + V I
S

1244.

.. r - x (arc sin X)2+ 2 r l-x 2 X

249 +xcos(2Inx)+2xSin(21nx) 1. 2 10 x l . x dx 1250. - 2 (Xl + 1) + 2 arc fan x. Solution. Putting u = x and dv = (x 2 -t- 1)2 '

COS2X-2Sin2x X( 5

1)

1 x dx x we get du=dx and v= 2(x2 +1)' Whence (x 2+1)2 2(x 2 +1) dx x I I x 2(xl+l) 2(xl+l)+2arctanx+C. 1251. 2a2 iarctan(i+

+S
X
f

(1

+
X

+ xl~al)'
..r l- -

Hint.

Utilize the identity 1....


x tJ'

;1

[(xl+al)-x l ).

1252.

1-

at a -x'+2

arc sin

Solution. Put
II

dU=-V

andv=x;wehave Yal Xldx=XVa2-xl-S,::::~2dX= aI-xi Va 2 -x2 l l . , ygl_XI_s(al-xl>-alctx;::x Vaa-x,I-S va -x dx+ a2S dx .

XdX

U= f

.. r - -

ai-xI and dv=dx; whence

Yal-xi

Val-x'

Answers
Consequently, 2

421

SVa2-x2dx=x a 2_-x2+a2arcsin :
HInt. See Hint. See Problem 1252*. 1255.

1253.i VA

+x2+
2

+~ +~
X In

In

I x+

VA +x" I.
Vll
3

Problem 1252*. 1254.

arc sin

xarc tan V3 1259.

Ix+ 2x-7

2 . 1257.

i. I

i2
1

arc tan

x;

-i V9-x +
I. 1256.
~

~X
X

arctan

6x-1

VIT'

1258.

In (r-7x+ I. +

7 V3

2 1n (x 2-4x+5)+4 arctan (x-2).

1260. x-2'ln (x 2+

9 2x+3 + 3x + 4) + y"7 arc tan "7 .


1262. 1265. 1267.
1268. In 1271. 1273.

1261. x + 3 In (x l -6x + 10) + 8 arc tan (x -3).

/-2

arc sin ~ 1263. arcsin(2x-I). 1264. In x+ 3 Vx 2-4x+5. 1266.

I f

V Xl +

px+q

-2 Vl-x-x2 -9arc sin

W.

I.

I V \.
1+

V5x l -2x+ I + 5
l-x 2 x

~'5

In ( x V5-

~5 +

V5x -2x+ I)

1269.-arcslD 2-:-:,. 1270. arcsin Xy 5 (I-x)


1272.
~

2--V-

-arCS1tl
2x-1 -4-

. r-- I . f x-x2+'8arcS10(2x-l)o

1 + ' I

x+l.r y x~+2x+5+2In(x+l+, x +2x+5).


1274. 1276. -

2 Ir,2

(x> V2).

2x+ 1 .r - 4 - y 2-x-x 2 +
1 3-~1flX .. r- arc tan y 3 J

9 2x+l +arc S111 - 8 \i

1275.

IIX - 3 1 -4 In - l - I .
x)

V-
11.

1277. In ( 1279.
1281.

+{ + VI +eX +e2 X

1278. -Inl cosx + 2+ Vcos l x+ 4CI,SX +

V'1-4 In x-Inl x- 2 arc sin 2 -:;~ x .

, 5

1280. 12

0-

~b In

x +-1- b I. x a

x+3In/x-31-3In)x-21
1

1282
161

..!.-lnl(X-l)\~+3)~1

,X + 2)4

1283. In

(x-l)4
\x

+ :3)7

(X-4)51

1284. 5x

+ In

(X-4)

2.
I

.1285I~X+ln IX~II.

(x-I)

1286.

~x+~lnl(2X_I/~~X+1)81
___ 9__ I
1289. 8

1287.
_

;-(X~2)i-X~2.
1292. x++ln
2

2(x-a) 2(x+I)O 49(x-5) 1 1290. -2(X 2 _ 3X+2)2' 1291. %+Inl


1288.

.,,-ml

27 ~ In 1 ~-5 49 (X+2)+\i4J x+2

1;::1-

-i

arc tan x.

1293. Sl2 1n I X-31-2~ In I x- 1 1 +

6~ In lx + 4x +5) + I~O X

422
2 1294. t xarcan(x+).
X In xl-x

Answers
1 6 In x-x+ .(x + 1)1 1

+ .;_ r3 arc tan ~r-l r~ .


1296.

1295.

.~_ X 4r2

x! + x

Y2 + 1 Y2 x Y"2 JI- 2+ 1 +-4- arc tan I-xl'


1297. 1299.

1 x2 + X + 1 I Tin xl-x+ 1 + 2 Y3 X
1298.

xarctan

x'-I ' XYa

2(1+x2)+

arc tan x 2

2(x2+2x+2)+

2x-l

+arctan(x+I).

x+2 5 2x+1 Inlx+II+3(X2+x+l)+3 Y3arctan ya-

3x-17 I 15 In (x'+x+ 1).1300. 2 (x2- 4x +5) + 2 1n (x 2 -4x+5) + 2" arc tan(x-2). _x 2 + x I I I 1301. 4 (x+ 1) (x2+ 1) +"2 ln I x+ II-Tin (Xl + 1)+4 arctan x.

- 2

3 x 3 15x5 +40x s +33x -Sarctanx- 4 (X 4 _1)+16 In x+l 1303. 48(I+x2 )' + . 15 x-I +48arctanx. 1304. x x2_2x+2+21n(x2-2x+2)+3arctan(x-l).
1302. t305.

IX-II

+ 11) 1306. I ~ I 12X4 + 1- Y Y51' 1307. - -4 In I x +x4-II--;y=-ln 2 r 5 2x + I + 5 +2Inl;_~I. 1308. ~ (2InIXlx~II-~I-XI~I)'


21 (81nl xl +81-1n I xl
4

2' ln lx4- 112(

13

x-

4)3

3 +-4 + x-

1309.

xii +

+In x-I.
1311.

X- 2 1

1310.

1 In lx7+11. In)xl- 7

Hint.
1

Put

1=(x7 +1)-x7
I

1 5 1 In I x I-SIn) x + 1 1+ 5 (xs + I)' 1312.


g (x-I).
1315.

3" arc tan (x+ 1)-6 arc tanx

X -2- 1313.

x+1

I 4 (x-I)!

I 7 (X-I)7' 1314.

-arc tan x.

2Yx=I[(X-;I)"+3(X

X[2 V(ax+b)S-5b

V(ax+ b)Zl

1317.

2 arc tan

l. s Y x+l.
I)2+ x
3

- 5x 5 + 3x 3 -X--

1316.
1318.
X5 _

l~alX
6

lrh/-) 6 x V "/x+2r x-6In(l+ 11 x. 1319. "7 x- 6"/;5 VS V +2Vx-3 Vx-6vx-31n II + Vx 1+ 6 arc tan vx.

-3

Vr+

V"X -

1320.

In

I (JrXT~_1)2 1-.;x+2+
x+ I

r 3

arc tan 2

Y7

+ I . 1321. 2

rX-2 V'2x

xarc tan

... /x 2. 1322 .

lrVX~-I 1 -2 arc tan r I-x. 1323. 2 (x-2) +2 In I x


1 Z2+ Z + 1 2 22+ 1 2z 3 1n (Z_V+Y3arctan Y3 +zl-l'

+, x -II 1324.

lr-~-

z=

where

x 1 x-I 1325.

V2x+3 2.x+3 .r 1 - - - x - ' 1326. ---8- r x2 -x+l+ In(2x-l+


16

Answers
+2
'If , f x

423

7 8+4x'+3x -x+l). 132. IS

4 ..

r-l-x. - I 13 8 ( S 5 3 1 2. 16 x -:24 x +"6 x

s) X

..r 5 .. r (1 3 ) ..r 3 1 X y 1 + x2 -16 1n (x+ y 1 + x2 ). 1329. 4x4 + 8x! r x2 - 1- 8 arc sln-X


1330. 2(x+l)2 r x +2x-"2arcSlnx+l 1331. - 4 - y x -x+l+ S lnx
X (2x-I+2

.. r-z--

2x-l .. r

19

1 1 -b- x' 1 Yx'-x+I). 1332. -2 -;r-----==-. 1333. -4 In y 1+2x2


(2x! -

Vx - + 1 + 1 V--x- +1-1
4
,. 4 3

- -} arc tan

va arc tan 73 2z+ 1 + -S


1337. - 2

V x- + 1.
I

1334.

1) Y 1 + Xl 1335. 3x s

TO In

(z - 1)' z~+z+ 1

where

Z ==

VlXi +.

1336.

1 4 + 3x - 8" x (2 -t- xa) 2/,

(x"

+ 1)1.

2 1338. sin x- ; sin' x. 1339. -cosx+3" cos s x -

-Seas x.

..

1340 stn' x sln l x 1 x 1 1 " 2-:rcOS -3---S- 1 3 4 -;rcos

1342. -'2'--

s,ft' X

1. 3x sin 2x sin 4x 2 sin' x - 21n I Sln x I 1343. T - -4- + ~ x sin 4x x sin 4x sin a 2x 5 1 1 1344. 8-32. 1345. 16-64+----:ta 1346. T6x+i2sin6x+64 sin 12x+ 1 cotl x 2 1 144 sin a 6x. 1347. -cot x - -3- . 1348. tan x "3 tans x + "'5 tanS x. cot a x cotS x tanS x 1 1349. - - 3 - - -S- 1350. tan x + - 3 - - 2 cot 2x. 1351. "2 tan' x+

+
-I

3 In I tan

3 x - 4 tan 1 x 1352. ~ 1 + 21n tan 2 xl . 1353. -2Y2 X x 1- 2 tan2 4

cos "2

X [In tan 1355. 1357. -

I i I+ I i + ~ ) I]
In tan (
sin 4x 16 cos4 4x

3 sin 4x 32 cos2 4x

3 32 1n tan

I (2x +""4 I.
1t )

1354. 4

s~::: - : ~:~I: +f In
1356. 1359.
1

I i I
tan tan! 3"
x

5" tan Sx -x.

cot 2 X

2-- 1n I sin x I

x x x +tanla-3tan"3+31n cos "3


1362. -

I I+x.
COSiO

1358. - 3" cot' x+cot x +x. 1360.


Xl sin 2\:1 4--8-

coP x 1361. - - 3 - .

' x. 1363. 2 Vtan x. 1364. 2 ~2>< ..r1 .. rZI Xln + l f 2+1_ .. r _ arc tan~. where z=Vtanx. 1365. _C 0 s8x+ Z2_ Z Y2+1 r 2 z2-1 16 cos 2x sin 25x sin 5x 3 5x x 3 x +-4- 1366. ----00- +10 1367. 5 sin "6 +3 sin"6. 1368. 2 tt")s"3-

~ V cos x + ~

x -l~

V cos

_ .!. 2 cos x.

1369 sin 2ax


4a

+ x cos 2b 2

1370 t cns <p

sin ~2mt + cp) 1371 ~ + 4uJ.. 2 ..

424
5x
7x 1

Answers
I 1 I

+2J+28 1372. 24cos6x-I6cos4x-acos2x. 1373. 4 1n


tan
1374.

Si '1

S1 f1

tan~-2
2 x
.

';'2 I
In

+2

In tan (

i+i) \.

1375.

x-tan ~.

1376.

-x+tanx+secx.

tan ~-5 1377.

tan2"-3

.13;8. arctan (I+tan put 3 sin x

~).

1379.

HX-~lnI2sinx+

+ 3cf's x Solution. We + P(2 ~1I) x + 3cos X)' Whence


a=T3'
X

12

~=-13
x
';ll

We

2a-3p ==3, 3a+ 2P=2 and, consequently, 3 Slf1 X + 2 Cf'S x 12 5 have 2s11lX+3cosxdx=I3Jdx-13X

+ 2 cos x ==a (2 sin x + 3cos x) +

x+ c ~ " 1381. } arc tan ( ta~

S2

(2

~i11

+ 3J I~"~ v)' dx= -13 12 5 2 sinx+3cos x I. x -13 In 1 X)

1380. -In Icosx-sinx,.

Hint. Divide the numerator and denominator of the

fraction by ces2 x 1382.

~15 arctan ( V~~n x ).

Hint. See Problem 1381.

. 1383.

1 In 12 tan x + 3- vG . v-. I Hint. 3 2 tan x + 3 + V IJ

See Problem 1381.

1 In X 1384. -5

y X tan t an x

51

See Problem 1381. 1385. -2(1 Hint.

1 2 -cos x )2 1386. In(1 +5i11 x).


x

1387.

2 r 2

I .r- In

V2+ V - c;to 2x 2 - Sill 2x


1 lr- arc tan r2

\388.
2 .,t

1 5-')in x -4 In -1- - .

S111

1389.

..r- arc tan


the

2 r 3

X --.,-- -

2tan!--1 2

3tan~-1
2r2
Hint.

Use

identity
2

r3

1
(2-Std

1
= 2-S111

tan~

x) (J-sln x)

3-S1n

x 1390.

-x+ 21n
tan

2+1

. Hint.

, I-sin x +c 0 s x 2 cosh' x Use the IdentIty I + Sltl x - cos x ==-1+ 1 Sl'l x-cos x . 1391. -a--coshx. 1392. ~ + SIO~ 2x + sin: 4x 1393. sin~4 x 1394. _ + sin: 4x . 2 2 x 1 tanh! x 1395. In tanh 2" +coshx. 1396. -2coth2x. 1397. In (cosh X)--2-

cotha x 1398. x-coth x - -- . 1399. arctan(tanhx). 1400. y-garctan 3

(3

x tanh 2 +2)

Y5

or

2 ("~)l V5 arc tan eX , 5

sInh x sinh 2x x .. 1401.--- - - 4 - - 2' Hint. Use the Identity 2 1402.

-SU1

. h - I

x-cos

== lsinh x + cosh x).

.;_In V2 2
,

cosh x

+ Vcosh 2%).

Answers
1403. -21405.

425

x+l lr
y

. x+l x ..r - ..r - 3-2x-x"+2arcsln -2- 1404. 2 ' 2 +x 2 In(~+ r 2+x 2 ) .. x .. r - -2 9 . . r - -2 x-I '1r l "2 f 9+x -2"ln(x+ f 9+x ), 1406. -2-" x -2x+2+

+} In (x-I +
2t+ I 1408. - 4 -

Y x 2-2x+ 2) 1407. Yx 2-4-2In I x+ y;t=4l .. r - 1 I . r-X-3 .. F ,. Xl +x-Sln 2x+ 1+2 y xl+x I 1409. -2- ,. x 2 -6x-7l

- 81n I x-3+ Vx 2- 6x-71. 1410. i4 (2x + I) (8x 2 +8x + 17) Vx l + x+ 1+

Yx +x+I). 1 x Y2 1413. r_arctan V 1414.


,. 2
l
2

27 +128 In (2x+I+2

14tt.

.. IX-2 V x-I

x-I 1412. 4Vx2-2x+5'


2

1- x 2 X (x4-2x -I- 5x -51: + { ) 1416. X cos 3x Stfl 3x X Cf'S X SHI x 1417. - - 6 - + 1 8 + - 2 - - - 2 eX 1419. "2

1 VI + x +x Y .r-In.r 2 y 2 y 1 + x2 -x 2

Y21

1415. e T X

2X

ir (XI

~2 sin 6x +

i- cos 6X-~ sin 6x )


+cosx )-Slnx. . ]

(2 Sl1l2t 5

e 1418."8 (2-sin2x-cos2x).

2X

+l:( S

2~

4 ~ln 4X+Cf 1420 eX ( (. 1 7 ' "2 x sin x

S4t)

x I I .. F 1421. -"2+31nle"-I/+lfln(e"+2) 1422. x-ln(2+e"+2,. e2x +x+I).

1423.

~ rXl In ~ +; + In (I-x 2) +x2 ] -Xln(x+Vl+x 2 )+2x. 1425.


1426.
sin x

1424. x In 2 (x+ VI +x l )-2 VI +x'X

(X 9) 5x+6 2"-100, arccos (5x-2)-1QO X


2

X V20x-25x l -3.
X

cosh x-cns x sinh x


2
2
2

1427. I

2 (n -

I) at

[(XI+~2)II-1 + (2n- 3) I 11_1] ; 1 = 2~2 (x ~ a + ~ arctan ~ ); I a = 4~a X 3 xJ cos x sinn X n -1 . X (3XI + 5a X [ 2a 2 (Xl + a2)2 + 2a arc tan a 1428. In = n + -n- 1n-" ' 3x cos 3 sin 2x I sin x 4 . 1 8c I &=8 4 -1-6-; ,=- cos x 5 -15cosxslnx-ISosx.
l
2 )

-1

X""l

'

1429.1n=(n_l)Cosn-lx+n_l/n-l;

sin

n-2

sin x 1 1'=2cos1x+-2 1n tan

I (
-x

2+"4
10

1t )

1&= - 3 '+-3 tanx. cos x

~nx

1430. In=-xne-" +n1n_t;

110 =-e
_1_

(x

+ 10x'+
Y7
~) +

+ IO9x + ... + 1098


8

... 2x+ 109.. 1).

1431. V14 arc tan

V2(x-l)

1432. In V xl -2x+2-4 arc tan (x-I). 1433. (x 2 1)1 +i In (x1 + x+

1 I ... i - r 1 1 +"2arctan(2x+I). 1434. 5 1n V x"+5 1435. 21n x+2 -x+2-x+3 ~

IX+31

1436.

~(lnl':x~:II-x~I)'

1437.

~ (XI~2+~aretan;2)

426

Answers

1 ( 2x -L x 1 1438.4" I-xl ,In x - I

I + I)

2 2x-I + 3 .r_arctan .r- ,. :3 y 3

1 x- 2 1 2x - 1 1439. 6(xl-x+I)I+6xl-x+I+ x(3+2 Yi) 1 4 I 1440. ..r 1441. - - 3 .. F---2 I 1-2 y x x x y x x

1442. In(x+{+YXI+X+l). 1443.


1445. (2X-I. 4. 1-2x+ 1 1448. x (xl-I'
.. r

Y2X-:
1448.
1451.

V'(2x)'. 1444.

-V~+l'
1

-2 (V5-x-I)I-4In (1 + V5-x).

1477.1nlx+Jlx -1
X

--I z
1450

-2

1 ... tl- x l! Y l+xl

arc sin 1/-2.

.X arc sin

1 , .2(x+l)

. x

~-1

,.x~+1

1 -8 In

1(4-

'

1449.
21

2x

X+4.

Hint. xl+4x ="4 -X- x+4'


1453.

1(1

1)
(x

1 .. r- X 8,3

1452.

2
(x
3

x .. r-2y x - 9-

- i1n I x+ y' XZ -91.


1454.

In

~ (8x-l) (x-4x' +~ arc sin (8x-l).


1455.
2

_(xt l ) V.l.I+2X+2-fln(x+I+Yx'+2x+2).
y'(x
2

2x + 1 + 2 (Xl + X + 1

/.

+ 2x + 2)

2 ,-

2x -t 2

3 I x

1)'
I

1457.

1 + -6 In (z +z + 1) .r - 'Xln(xl + y 1+x
4 ).

VI-X a - 1 1 1 1458. --3 In I z -11 I-x' + 1 1 2z + 1 1 + x' 5 .. r- arc tan where z= 1459. -2 X f 3 3 x 3x Sill 2x sin 4x 1460. 8+-4-+32. 1461. Inl tanxl-cotlx-3 In

IV

1456.

y~

v- ,
3

-"4 cot 4 x.
1464.

1462.

C0S 5t" 3 - -20 SiU4 5x - 40 sin 2 5x + 40 In

-cotx 3 cos 5x

2 YtcotX)3

.1488. {sin 2x.


X

1487. tanl (
x

i + ~ ) +21n Icos ( ~ + ~ ) I
..r- arc tan
,. 10

Itan"2 5x I
f

1463.

5 I2lcos! x- 6) tans x
1465.
-3-

5 -V cos x.
2

+ -5- 1468. - ~3x

tanS x

arc tan
1

4 ~an

2Y_ .
3

1469.

(2 ,rtanx) .
10

1470. arc tan (2 tan x+ 1).

1471.

"2 In I tanx + sec x 1- 2


tan

cosec x. 1472.

(3 xarc tan

(tan

~\ I V3)- Y2 x

'xarctan (
X In (sin ax

V"2

i)

I 1473. Inltanx+2+ytan l x+4tanx+ll 1474'-a x


2

+ .r ,. a l +5111

ax).

1475."3 x tan 3x +9 In I cos 3x I

1476.

4"-

Xl

x sin 2x cos 2x elx ~-4---~- 1477. Tl 2x - 1). 1478

1 a

eX3

1479.

3' -In

xl

Vl-x-

_-

Answers
- } In I x-I 1- ;~- ;~1481

427
Xl

~.

1480.

VI +

arc tan x-In (x.+

VI + xl).

1 3x 1 5x I x I I I -sln---sin--- sln- 1482. -I t . 1483. n l+cotxl-cotx. 2 2 3 2 10 2 + anx sinh 2 x .. r - I 1481. - 2 - . 1485. -2cosh y I-x. 1486. Slncosh2x. 1487. -xcothx+ 1 x I I eX -3 +lnlsinhxl. 1488. 2eX-4+4InleX-21. 1489. arctan-- . 2 2 4 4 I I + 2x 10- 2X X ln 1490.7" +I)1- 3 Vtex+l)I. 1491. In4 1-2x 1492. -2InIO X

V(e

x i_ ) . X ( x2 _ 1 + _ +_ In 10 21n2 10 1494.lnj

1493.

y 1+ x 2

_x_l_arctanx.
X

1495. 1497.
I

eX + I + 1 2 1 4 (x.arCSlnJ..+x + 2 Vxl _ 1). 3 X

'1r-x Vex + I - 1 . r e +1+ln V--

x (coslnx+sinlnx). 1496. 2

-iJ.
1501.

+ ~COS5X- ~ sin 5x ). 1498. ~ [(X -2) arc tan (2x + 3) + ~ In (2x 1499. {Vx-xl+(x- ;)arCSin V"X. 1500. x 1x 1 2
Chapter V

I ( -x2 cos5x+ 2 xstn5x+3xcos5x+

+ 6x +

5)~

T2 210 -1 1502. voT -g -2" . 1503. 3. 1504. lii""2. 1505. 156. Hint. Divide the interval from x= I to x=5 on the x-axis into subintervals so that the abscissas of the points of division should form a geob-a.

nletric progression: xn=-I, Hint. See Problem 1505.

Xl

=xoq,

X 2=

r(Jq2, . "

Sil1a+Sin2a+ ...

+Shlna=~ [cos~-cos(n+~)aJ.
2S1n

1507.

I-cos x.

X n =x(lqn. 1506. In!!... a Hint. Utilize the tormula

1508. 1)

"2

~=

- __1_. 2) dl __l_ 5 I _ lr--. 2 -X4 -Xl In a ' db - In b 1 09. n x. 1510. y 1 +.t . 1511. xe -e . cos x I I 3 1512. .r- + 2 cos 2 1513. x = nn (n= 1,2,3, ... ). 1514. In 2. 1515. - -8 2 y x x x
1516.

eX -e- x = 2 sinh x. 1517. sin x. 1518. } . Solution. The sum s" =

may be regarded as the Integral sum of the function f(x)=x on the interval [0,1]. Therefore, lim sn=
I

1 2 I (I 2 n- I) =2+"2+ + n-I -2-=- + -+ n n n nn n ... + -n

-+<Xl

xdx=-2 S o

I 1519. In 2. Solution, The sum sn = --I

=*(_1-1 +_1_2 I+nI+n-

-+,,, +~)
I+n-

n+

I I + --2 +, .. + -= n+ n+n

may be regarded as the integral sum of

428 the function

Answers

f (x) = I ~ x

on the interval [0,1] where the division poi nts have


1

the form
1520. p

Xk=I+~ n
1521.
7 2

(k=l, 2, , n). Therefore,

n~ct>

lim

Sn=51 dX == In2. +x
o

+1 .

"3.

1522.
9

3 = 33 "3 .
1

100

1523.

4" .

1524.

"3.

16

1525. -

1526.

In 3' . 1527. In 8 1528. 35 15- 321n 3. 1529. arc tan 3-arc tan 2= 1 4 n 1 n 11: = arc tan 7 1530. In 3". 1531. 16 1532. 1- Y3 1533. "4. 1534. '2.
1536.

. 1 l+YS 1535. 3" In - - 2 -


1540. O. 1541. 9

8+ '4.

1537.

1538. In 2.

1539. I-cos 1.

~3 + ~

1542. arc tan

e- T'
~+~

1543. sinh 1= ;

(e- ~ ).
1550.
1554.

1544. tanh (In 3)-tanh (In 2)=

. 1545. -

sinh 211:. 1546. 2. 1547. Di 1549. Diverges.

verges. 1548. I I p' if p< I; diverges, if p:;;;:: l.

.
31.

1551. Diverges. 1552. 1. 1553. _1- ' if p>l; diverges, if p- 1

1555.

;5'

p~ 1.

1556. Diverges.

1557. Diverges.
2

1558. I; 2

1559. Diverges. 1565. 3


2n

1 . 1 1560. In a' 1561. DIverges. 1562. 7i

n 1 1563. 8"' 1564. 3"

1 ln3 +T

Y3

1566. Diverges 1567. Converges 1568. Diverges 1569. Converges. 1570. Converges. 1571. Converges. 1572. Diverges 157J. Converges. 1574. Hint. B (p, q)= I/J 1 ~ f(x) dx+ ~ f (x) dx, where f (x) =x P-' (l-x)q-,; since lim f (x) x'-P= 1

and lim (I-x) -q f (x) = I, both integrals converge when I-p< 1 and I-q< I.
% -+ 1

II

x~o

CIJ

that is, when p>O and q>O. 1575. Hint.

(p) =

~ f (x)
o

dx

+ ~ f (x) dx,
I

wherE

I (x) =

xp-1e- X. The first integral converges when p>O, the second when p i~

n
2

arbitrary. 1576. No. 1577.2

Y2 Sy'tdt.
I

1578.

S YI~Sl11lt' 1579. S dt,


n
8

In a

In

1580.

' (arc tan 5 I +t


QI)

t)

dt. 1581.
1t 1/y 5

X=

(b-a) t +a. 1582.4-21n3. 1583. 8- 2

V3 31 ,

1584.2-2

1t

41

1585.

1586.

1589. 4-n. 1590.

1 ~ 7+2Y7 1 1t Sin 112. 1,,91. In - - 9 - 1592. 2+4

n 1 - - y 1 + a2

n 1587.1-- . 4

1588.

.. r- n y 3-- . 3 na2 1593. 8

Answer"
1594.

42f)

2' 1599. 2 -

31

1.

1600. l.

e2 +3 t601. -8-

1 1602."2 (e1r

+ll.

1603. 1.

1604. a2 ~ b 1605. a2 ~ b2 ' 1606. Solution. r (p I) = xPe- X dx. Applying " o the fornlula of integration by parts, we put x P = U, e- x dx = dv. Whence du =px P - 1 dx, v = -e- X and

tIJ

r(p+I)=[-xPe-xI:'+p ~ xP-'e-xdx=pr(p)
o

(*)

tf P is a natural number, then, applying formula account that


~

ll

p times and taking into

r(l)=~ e-xdx=l,
we get:
o

(p+ 1)=p1

... (2k-I)~ 'f =2k b 1607 12k = 1.35 2.4.6 ... 2k 2' 1 n is an even nurn er; I tk + 1 = 2 . 4 . 6 . 2q ., 2k l ' b 1.::i.5 ... (2k+l),ln= + lsano dd numer

128 6331 10 =-= :~ 15; 110 = 512 .


(p-l)!(q-l)! 1 (m+l n + l \ . " 1608. (p -t-q-l)! . 1609. 2 B - 2 ' -2-)' Hint. Put sin x=t. 1610. a) Plus; b) minus; c) plus Hint. Sketch the ~raph of the integrand for values of the argument on the interval of integration 1611. a) First; b) second; 1 1 3 . 1 c) first. 1612. 3' 1613. a. 1614'"2 . 1615. 8"' 1616. 2 arc Sill "3
1617.

2<I<V5. 1618. 9<1<7' 1619. i3 n <1<7 11 .

1620.

O<I<a2.

n2

Hint. The integrand increases monotonically. 1621. 1624. 1. 1625.

~ </< ~.

1623. s=

.
~

Hint. Take account of the sign of the function. 1626. 4

1627. 2. 1628. In 2. 1629. m2 1n 3. 1630.


1634. lOa' 1635.4.1636.
1639. ab [2 r 3-1n (2+
2 32

nat. 1631. 12. 1632. ~ p2. 1633. 4 ~


I

3.

1637.

2-3.

11

1638 .

1 2 =2(coshl-I) e+e-

..r-

..r3 f 3)]. 1640. 8" nal . Hint. See Appendix VI, Fig. 27.

1641. 2al e- ' . 1642. : a l 1643. 15n. 1644. See Appendix VI, Fig. 23. 1647. al

In 3. 1645. 1. 1646. 3MI Hint. HInt. See Appendix VI. Fig.

(2+ ~ ).
a-

1648. 2n+'3 and 6n-

a 1649. 3

16

4 V3 32 4 3 n - - and "3 n + -- . 1650 t nab.

va
3

24.

430

Answers

1651. &tal. 1652. It (hi + 2ab) . 1653. 6ltal . 1654.

al . Hint. For the loop, the parameter t varies within the limits 0 E:; t ~+ 00 See Appendix VI, Fig. 22. ltal. Hint. See Appendix VI, Fig. 28. 1656. &t'al . HInt. See Appen1655. l l dix VI, Fig. 30. 1657. It: . 1658. al. 1659. It: . HInt. See Appendix VI,

33 60 9 14 -8 F Ig. 16 2"lt. 1661. 3


1664. n f 2.
1666. ~r2

V2 a.
I

1662. (I-el)"'o' 1

1tp2

663

.a

1(31

'3+-2~r-

V3)

Hint.

Pass to polar coordinates.

1665. 27 (10 flO-I).

-a2 Hint. Utilize the formula cosh 2 a-sinh 2 a. = 1. 1667. V2 + In (I + 2). 1668. VI +el - Y2 + In (VI+ei-I)(

Vh

1669. 1+

~
a

In

~.

V2 +

I) .

1670. In(e + Vel-I). 1671. In(2+ V3") 1672. {(el+I).

e2b -1 sinh b 1 2 3 1675. InW-l+ a - b =ln-.-- 1676. -2 aT. b e sin h a l Hint. See Appendix VI, Fig. 29. 1677. 4 (a:;-b ) 1678. I6a. 1679. na YI +4n l +
1673. aln- .1674. 2a

lrf

+; In (21t+ VI +4ltl) . 1680.8a. 1681. 2a [Y2 + In (Y2 + I)]. 1682.


2

aVl+m 1 4 + In 3+V5 2 1683. m 1684. 2 (4 + In 3). 1685. 30' 1686. :r 31ab -1 88 3 31 2 31 4 31 1687. aJ31 4 (eI + 4 -e). 16. 8 168 9. vx=T. 1690. v'=7
2

na s

v:

1691.

31 16rra a 32 a 4 a 3 v X =2"' vv=231. 1692. -5-' 1693. 15 31a . 1694. "3 np . 1695.

1696.

n;1 (\5-16.ln 2). 1697.


c)

2ltla. 1698.
1702

lt~IH.

10 31

1699. :: lthla. 1701. a) 5nla';


8 31a. 3

a. b ) 6n aa,

na (9 31I 16) 6 .

32 31aa. 105
nabh -3-'

1703 1707.

1704 1708.

4 na a 2i

1705.
1709.

h ( Ab +a B ) 3" AB + 2 +ab. I 2nalh.

1706.

16 1710. :ra. 171t.nal Vpq. 1712. nabh

(hI'. 4 I+). 1713. ltabc.


3

128 105 a.

8 2 3Ul o. '3

1714. 8;lVI7'-I]; 17t6.

~6nal[5V5-8].

t7t5.

2n[V2+ln(V2+I)].

It (V5 - y'2) +It In 2 (V2 + I) 1717. n [V2 + In (I + Y2)J. V5+1 na 2 31a 2 12 n 1718. T (el+e- I + 4)=2 (2+ sinh 2). 1719. nal. 1720. (e-l) (e 2 +e+4).

1721. 4n!ab Hint. Here, y== b Va2 -xl Taking the plus si~n, we get the external surface of a torus; taking the minus sign, we get the internal sur2 face of a torus. 1722. 1) 231b l + 2nab arc sin e; 2) 2na l 1(b In 11 e, where

8=

Vaa
l

-e

64M 32 2 (eccentricity of ellipse). 1723. a) -3-; b) 1631' a2 ; c) 31a

Answers
1724.

43t

1~8nal.

1725. 2na l (2-Y2). 1726.


1728.

1~8nal.

1727.
1729.

MX=:

Yal+bl ;

a .. ~ M Y ="2 r a 2 +b 2

ab 2 M a =2;

a2b Mb=y.

a' M x =M Y ="6;'

a 3 2 x=Y=a' 1730. Mx=M Y =5 a2 ; x=y=Sa. 1731. 2na 2 1732. x=O~ a 2 + sinh 2 - a sin a 4 - 4a y 4 sinh 1 1733. x =-a- ; y=O. 1734. x=1ta; Y=3 a . 1735. x=3xt:
4b y -=3xt'

1736.

x=Y=20' 1737. X=1ta; Y=r;a.

1738.

(a ) 0,0, 2" .

Solu-

tion. Divide the hemisphere into elementary spherical slices of area da by horizontal planes. We have do = 2na dz, where dz is the al titude of a slice ..
a

2n

~ az dz

Whence z =

;Jla

=;.

Due to symmetry, x=y=O. 1739. At a dis--

tance of : altitude from the vertex of the cone. Solution. Partition the

cone into elements by planes parallel to the base. The mass of an elenlen tary layer (slice) is dmi=y1tQ2dz, where y is the density. z is the distance
M

of

the
h

cutting
Zl

plane

from

the vertex

of the cone, Q = ~ z. Whence

S;:

dz

x-== Y=O.

z = I0 - xtr 2h 3

= : h.

1740. (0; 0;

+:
n

a ). Solution. Due to symmetry,

To determine 2' we partition the hemisphere into elementary layers (slices) by planes parallel to the horizontal plane. The mas~ of such an elementary layer dm = ynr 2 dz, where y is the denc;i ty, 2 is the distance of the cutting plane from the base of the hemisphere, r = Ya 2 _z 2 is the
a

~ (al o

Zl) Z dz

radius of a cross-section. We have: z =


1
4

2 -na 8 3

3 =aa.
1

1741. I=na 8
1

1742. la=aab' ; I b =3 a 'b. 1743. J=T5hb'. 1744. /a=4"nab 3 ; Ib=T1ta'b.

1745. I =

in (R:-R:).

Solution. We partition the ring into elementary

concentric circles.

The

mass of each such

R2
the moment of inertia J =2nS r l dr=

element dm = y2Jtf dr

and

~ n (R:-R:); (y=I). 1746. J = I~ nR~Hv.

Solution. We partition the cone into elementary cylindrical tubes para lie!

R1

to the axis of the cone. The volume of each such elementary tube is dV = 2xtrh dr, where r is the radius of the tube (the distance to the axis of
the cone), h = H ( 1- ; ) is the aHi tude of the tube; then the moment 01

432
R

Answers

Inertia I=y

S2nH (1- ~ ) r'dr= yrr~.H where


o

y is the density of the

'Cone. 1147. 1= ; MaJ. Solution. We partition the sphere into elementary

cylindrical tubes, the axis of which is the given diameter. An elementary


volume dV=2nrhdr. where, is the radius of a tube. h=2a
a

"1/ Y

1-

':

is its altitude. Then the moment of inertia I =4rray

r -(1- a J o

,2
2

rldr =~ rra5y, 15

where y is the density of the sphere. and since the mass M =

~ naly.

it fol.

22 -2 2 2 lowsthatJ=r;Ma 1748. V==2Jtab; S=4n 211b. 1749. a) x=Y=5 a ; --9 -4r b) x=y= lOP. 1750. a) x=O, Y:=3 It Hint. The coordinate axes are cho-

sen so that the x-axis coincides with the diameter and the origin is the
centre of the circle; b)

x= ~
f.;,

Solution. The volume of the solid - a double

cone obtained from rotatin f1 a trIangle about it~ bClc;;e, is equal to V -==-!.. '!'b1z 2
3

'

where b is the base. Iz is the altitude of the tnan(!le. By the GuldIn theorem, the same volulne V-=2Jtx 2bf1. whl're x is the dIstance of of
1752. 1755.
-1 .
tiL'

centre

gravity

trom
c~

the

base.
1753.
x==

\\ hence

=3

It

175t.
1

~ In (1 +~). 2g 11= ~ In (II a bl);

h= ~ x

=_! V 1754. S =- 10 nz. n o [btl-Ia-bt.) In a~bIJ. 1756. A=


"111 wt;
(I)

v av

=]y R2H!
1157.

Hint. The elementary force (force or gravity) is equal to the

weight of water In the voluIne of a layer cf thlcknpss dx, that IS, dF = = ynR2 dx, where y is the weight of unit voluml' of \vater. Hence, the elementary work of a force dA =-=. ynR~ (H -x) dx, wht're x is the water level.

A=l~ yRJHJ.

1758.

A-~'Jl4Y RTM::::::079JOI=079IO'

kgm.

1159. A=ynR'H. 1760. A= mgh ; Aoo=mgR. Solution. The force acting h

l+ R

on a mass m is equal to F= k of the earth. Since for


f=

m~ , '

where r is the distance from the centre


R+h

R we have F=mg, it follows that kM =gR 2 The

$ought.for work will have the form A = k d, = kmM - R h) = mgh R . c : : : - --. When h=J we have ArrJ=mgR. 1761. 1.8.104 ergs. Solution. h

~~

(~

1+1[

Answers

433

The force of interaction of charges is F= eO~1 dynes. Consequently. the work x


%.

performed
=eoel

in

moving charge el

from

point

XI

to XI Is A =e'l

5c;:=
x)

ergs. 1762. A = 800 n In 2 kgm. Solution. For an XI 2 Isothennal process, prJ =Pov o' The work perfornled in the expansion of a gas from volume
Vo

(...!.-- 1-) = 1.810 x


to volume

VI

is A =

rp dv = Pov o In V VI 1763. J o
Va

VI

A:::::: 15,000 kgtn.

Solution. For an adiabatic process, the Poisson law pull = Pov~, where
k ::::::: lA,

hold.
4

true.

Hence

A=

Pov. ' Pov~ dv ~ k-I Sv


k
Vo

[I _

(V

o ) k-

VI

J] _

1764. A =3" nJ.LPa. Solution. If a is the radius of the base of a shaft, then

the pressure on uni t area of the support p

==.!!.". M

The frictional force of a

ri ng of width dr, at a distance , from the centre, is rd,. The work pera formed by frictional forces on a ring in one complete revolution is
4nJ.LP X dA = 43lJ.LP i2 ,2d,. Therefore, the com pi ete work A = (i2

2JJ.:

S,2 dr = 33tJ..tPa.. 4
a
o

1765.

i- MR oo
2

Solution. The kinetic energy of a particle of the disk


do is an element of area,' is the distance of it
Q

dK=T=-2dO, where
{ronl the axis of

mo 2

Qr 2oo 2

rotation,

is the surface density


R

Q=

:rr.RI'

Thus.,

dK =2JtR2 r2da . Whence K=R2


1767.

Moo 2

Moo25
o

MR2CJ)2 r 1 dr=--4-

3 1766. K=20,,><MR2W 2.

K=~ R 2oo 2 =2.31O' kgm. Hint. The amount of work required is equaf
. bh P=T.
2

to the reserve of kinettcenergy. 1768.

(a +2b) h 2 1769. P=--6--~11.3.10' T

1770. P =abVJlh. 1771. P = llR;H (the vertical component is directed upwards).


1773. 99.8 cal. 1774. M -= -2-- gf cnl. 1775. a (a I) (k JS the a a C . gravitatIonal constant). 1776. npa 8....1 . Solution. Q = v2nrdr = 21tp -4.... 1 (a 2_,2) rdr== 1772. 633

gm

hb"p

kMm.

5
0

S
0

1tp =2J.Ll

laT - T r
B ,2 C

npa 0 = 8l11' 1777. Q=

Sv a dY:Z::3 2 ab P iLl
l

21,

Hint. Draw thex-axfs

15-1900

434

Answers

along the large lower side of the rectangle, and the y-axis, perpendicular to
it in the middle. 1778. Solution.

s= ~
v.

5
V2

dv; on the other hand,

:~ =

a,

whence dt =1. do, and consequently, the acceleration time is t =


a

ra J
til

t',

dv

=5.

1779.

Mx =- S~ (x-t)dt+ ~ x=x
o

~
2

[xt-

~]:+~ x= ~x (1-+)

1780. Mx = -

5(x-t) kt dt +Ax= ~ W-x


o

),

1781. Q =0.12 T RI: cal.

Hint.

Use the J6ule-Lenz law.

Chapter VI
1782. 1784.

2 X2)X. V=3(y2_

1783.

2 ~r 2 S="3(x+y) r 4z -l-3(x-y)2.
y2_x2 2xy ' X2_ y 2 2xy ' y2_ x 2 2xy '

f ( -2' 3 =-3;

f(l, -1)==-2.

1785.

-2--2

2xy x -y

1786. f(x, x2 )=I+x-r. 1787. z = - R I2. 1788.f(x)=

R4

Hint. Represent the given function in tile form replace

I(

~)=

V(; r+

VI-I-x 2 x
I and

by x. Then

1789.

f(x,

y)=x

-;

xy. Solution.

Designate

x-Y=v. , u2 -uv
2

x=-2-' Y=-2-; f(u, v)=-2-

u-t;v

u-v

u+v

u-v

-2-+ (U-V)I -2- =

x+y=u,

=-'-2- It remains to name the arguments u and v, x and y. 1790.

f (u)=

Vx - I =

=u +2u; z=x-I+ VY.


u;

y I + y2 = 1 f ( t ), i. e., f (y) = y 1 + y2. Then f ( ~ ) = 1 +( ~ rand Z=X 1 + (~r = JI x"+y2. 1792. a) Single circle with centre at origin, including _ the circle (x + y2 ~ 1); b) bisector of quadrantal angle y = x; c) half-

Hint. In the identity x=l+f(VX" -I) put then x =;= (u + 1)2 and, hence, f (u) = u 2 + 2u. t 791. f (y) = = VI +y2; z= VX 2+y2 Solution. When x= I we have the identity

plane loeated above the straight line x+y=O (x+y > 0); d) strip contained bet~een the straight lines y= I, including these lines (-I ~y~I); e) a square formed by the segments of the straight lines x= 1 and y= I, including its sides (-I ~x~ I, -1 <aU ~ I); f) part of the plane adjoining the x-axis and contained between the straight lines y:= x, including these lines and excludi,ng the coordinate origin (- x ~ y E:; x when x > 0, x <; y ~-x when x<O); g) two strips x~2. -2~y~2 and x~-2, -2~y~2; h) the ring contained between the circles x2 +y2=a 2 and x2+y2=2a2, including the boundaries; i) strips 2nn ~ x~ (2n + I) rt, Y ~ 0 and (2n+ I) n ~ x ~ (2n + 2) 1, y.-; 0, where n is an integer; j) th~t part of the plane located above the

Answers

435

par~bola Y = - x2 (x2 + Y > 0); .k) the entire xy-plane; I) the entire xy-plane, with the exception of the coordinate origin; m) that part of th~ plane located above the parabola y2=X and to the right of the y-axis, including the pOtnts of the y-axis and excluding the points of the parabola (x ~ 0, y > Y x); n) the entire place except points of the straight lines X= 1 ~nd y =0; 0) the family of concentric circles 21tk~x2+y2~n(2k+l) (k=O, 1, 2, ... ). 1793. a) First octant (including boundary); b) First, Third, Sixth and Eighth octants (excludIng the boundary); c) a cube bounded by the planes X:c:: I, y =~ 1 and z== 1, including its faces; d) a sphere of radius 1 with centre at the origin, including its surface 1794. a) a plane; the level lines are straight lines parallel to the straight line x+y=O; b) a paraboloid of revolution; the level lines are concentric circles with centre at the origin; c) a hyperbolic paraboloid; the level lines are equilateral hyperbolas; d) second-order cone; the level lines are equilateral hyperbolas; e) a parabolic cylinder, the gcneratrices of which are parallel to the straight linex+y:+-I=O; the level lines are parallel lines; f) the lateral surface of a quadrangular pyramid; the level lines are the outlines of squares; g) level lines are parap olas y =- Cx2 ; h) the level lines are parabolas y = C I) the level Hnes are the circles C(t 2 +y2)=-2x. 1795. a) Parabolasy=C-x2 (C>0); b) hyper2 bolas xy = C (I C I ~ I); c) circles x + y2 = C2; d) straight lines y = ax + C; c) straight lines y-=CX (x =1= 0). 1796. a) Planes parallel to the plane x -t- Y -1- z --= 0; b) concentric sphere~ \vith centre at origin; c) for u > 0, one-sheet hyperboloids of revolution about the z-axis; for u < 0, two-sheet hyperbololds of revolution about the same axis; both families of surfaces are divided by the cone X 2+y2_ Z2=0 (u=O). 1797. a) 0; b) O;c) 2~ d) e'l; e) linlit does not exist; f) linlit does not exist. Hint. In Item(b) pass to polar coordinates In ltcnls ~e) and ef), consider the variation of x and y along the straight lines y == kx and show that the given expression may tend to different lilnits, dependin~ on the choice of k. 1798. Continuous. 1799. a) Discontinuity at x=O, y==O; b) all points of the straight line x==y (line of discontinuity); c) line of discontinuity IS the circle X 2 +y2= 1; d) the tines of discontinuity are the coordinate axes.

Yx;

1800 Hint.

Puttin~

Y = YI

== const,

\ve

~et the function <rl (x) =

2XYI x2 +

y: ,

which

is continuous everywhere, since for Yt :P 0 the denolninator x 2 + Y~ :P 0, and when YI ~ 0, (PI (x) === O. Sinl Harly, when x=x 1 = const, the function , CP2 (y) = 2x !J is everywhere continuous. Froln the set of variables x, Y, the -t- y2 function z is discontinuous at the point (0, 0) since there is no lim z. Indeed,

x:

x~o

passing to polar coordinates (x= , cos cp,y = , sin q>l, we ~et z == sin 2'Pt \vhence it is eviden t that if x ---+ 0 and y -+ 0 in such nlanner that q> = const (0 ~ q> ~ 231), then z -+ sin 2({). Since these linliting values of the function z depend on the direction of cp, it follows that z does not have a lilnit as x -+ 0 and y -. o. dz 2 az 2 az 2y az 2x 1801. dx=-=3(x -ay), ay=3(y -ax). 1802. ax=(x+y)2' ay=-(X+y)2
1803.

l/-+O

az=_
ax

x2'

ay

az=~
x

1804
xy

az_ x ax- X
x2

1805. ~= y2 ax (x 2 -1_ y2)3!2 '


y

ay

az

= Yx2+ y2 (x + Y x 2+ y2)

1807. ox =

az

(x 2 + y 2)3!:. .

y 2' 1806 dz _ . ax 2_

az__ y ay- Yx2_y2

+ y2'

ay = x 2+y2 1808. ax =yx

az

Y x2 + y2'
az

iJz _ ayY -

436
iJz
..:3

Answers

=xY lnx.

1809.

ox

~=_1!..,
Xl

sin

.1L

"'"cos

1L oZ::z.!..e
X

sin

ay

JL xcoslL.
X

1810.

iJx

~~

I - 2y xy'l - JY I (xl_yl)'

Y2x

at!:::a x.~~.
,. Y

OZ

yx l 2x l - 2y I y I (Xl_yl)'
X

1811.
Y

ox = Yi
uz

az

X +a

cot

yi'

~% = _
uy

OU iJu _ ' 1 '.1813. ox=yzxYlnz, iJy=xzxYlnz, oz==xyzxy I. 1814. fx (2,1)=2' ., , 1 z 1 '1/(2, 1)=0. 1815. Ix (1,2,0)= 1, ' , (1.2.0)=2' f (I, 2,0)=2 .

2y , Y iJu

X ~ra_ cot

1812. iJU -== yz (xy)Z-I. iJau ==xz(xy)Z-I.

~u = (xy)Ztn(xy).

1820. -

+y
tan

1 1 lnx+siny-'2. 1828. I) tana=4. tan~=oo. tanY=T; 2) tana~oo, tan

x 3/ 1821. (x 2 +yl + Z2) , 1

r. 1826.

y z=arc tan -+<1' (x).


x

1827. z=-2

x2

p=4.

Y=4

1 oS 1 iJS 1 1829. iJa =2" h, iJb =2 h, iJh =2 (a+b).

as

1830. Hint.

Check to see that the function is equal to zero over the entire x-axis and
the entire y-axIs, and take advantage of the definition of partial derivatives. Be convinced that f~(O. O)=f~(O, 0)=0. 1831. L\f=4L\x+L\y+2L\x2 , +2L\xl\Y+~XI~y; df=4dx+dy; a) t1f-df=8; b) L\f-df=0.062. 1833. dz=3{x 2 -y)dx+3(y2_ X )d y. 1834. dz=2xy 3dx+3x 2y 2dy. 1835. dz= (.l.2 1/2)2 (xy 2 dx-x 2y dy). 1836. dz= sin 2xdx-sin 2ydy. 1837. dz =y 2xY- 1dx+

+x)/(l+ylnx)dy. 1838.dz=
t8oW. dz=O.

2
x
2+

1841. dz= ~ ( dyX S111

(xdx+ydy). 1839.df=-+1 (dX-!..dY)J. x y Y

J!.
x

dx ) .

1842.

df (1, 1) = dx -

2dy.

1843. du = yz dx +lx dy + xy dz.


1845. du= (x y +:
X ( xy

+ : ) dz].

r-1[ (

1844. du

y+

~) zdx+

(1- ~2) xzdy + (xY+f) In ~<


y
Cl11 3

Y Xl +1y2 +Z2 (x dx + y dy + z dz).


dZ). 1847. df (3. 4. 5)--.:

1846. du

= X2y:~ Z4 (y dx +x dy- 2:
1850.

= 25l5dz-3dx-4dy).

1848. dl =0.062 cm; L\l =0.065 cm. 1849. 75

(rela-

tive to inner dimensions).

em. Hint. Put the dilTerential of lke area

g 1854. n a -:; ~l g f 19 e' (t 1n t - I) du .= t In 2 t 1857. i[j=

of the sector equal to zero and find the differential of the radius froIn that. 1851. a) 1.00; b) 4.998, c) 0.273. 1853. Accurate to 4 lnetres (more exactly,
1855. da=..!.. (dy cos a-dx sin a).

4.25 m).

1856.

d~t =
t+

t x ( x ) Yu eot yy' 6- 2y 2

1858. dt = 2t In t tan

du

2 ~(ttl+ l)tant+(t +1)lnt 1859 ~=-=O 1860 ~- (. )COSX( t--;T t cos2 t dt dx - S1n x cos x co x

Answers

437

az y . dz 1862 az . a.<>V-I. dz -slnx 1n sin x). 1861. a-=-~+ -,. a-=U~ , d-= x x Y2' d-=-l x +x x x az, , az , Y] Y =x [ q>'(x)lnx+-X .1863. ax=2xfu(u,v)+yrYfv(u, V); ay z:-2Yf u (U,v)+
+xrYf~(u.v). 1864 ::=0, :; = (x+ ~ ) f' (x y + ~). + f~ (x, Y, z) ("'~ (x, y) + "'~ (x,
1875. 20 V5-2
1880. 13. 1881.

;;=1.

1867.
y) <p' (X.

:~=Y(1-:2)f'(XY+~); :~=f~(X. y. z)+<p' (x)f~ (x, y. z)+


1865.
1873. The

perimeter increases at
1+2ta+3t~

a rate of 2 m/sec. the area increases at a rate of 70 rnB/sec. 1874.

Y2 km/hr.
C0S

1876.

_9~3.

1877. 1. 1878.

~2

1 + t 2 + te

. 1879. _

~3-.

68

+ C0S ~ + C(l5 V 3 .
2 1

1882. a) (2, 0),


1886.61

. . b) (0, 0), and (1. 1),

c) (7, 2, 1). 1884. 91- 3j. 1885.

1 4" (51-3j).

+ 3j + 2k. 1887. I grad u 1=6;


3 YIO.

cosa=3", cos~=-3' cosY="3' 1888. cos<p=

1889. tan<p:::::8.944;
abcxy

cp === 8337'.

1891

(b x2 +a!y2)'/.a' QXay- (b 2 x 2 +a 2 y 2)'/2' ay"=2 abex" 1892 a z _ 2 (y-x 2 ) . ~_ _ 2t . a"z 1_ - (b!x2 + a'y2)'/2 . ax 2- (x 2+ y)2 ' ax ay (Xl + y)2' a y2(x" + y)2 a2u a2 a!z XI/ alz a2, x2 u 1893. a-a = 3/ 1894. a-a-- = O. 1895. -a 2 = - a - 1896. :i2=-a2= x y {2xy + y2) I X Y x, v X y u _ alu _~__ _ a"u_. d2 ~~ ~-J '3-1 1-1 ""-0 Z2 - 0 , a x vy A -0 y vZ i~ -aZ a x -I. 1897. ax a y az ::z:a~yx y. Z
2

dz ax 2

abcL/2

az
2

_ _

az
2

,2-

1898.

a'z ax oy2=

-A 2

y cos (xy) -2x sin (xy).

1899.

f" x

(0, 0)

m (m -

1);

f:y(O, O)==mn; 1;/1/(0, O):=n(n-l).

1902. Hint.

USIng the rules of differen-

tiation ~nd the definItion of a partial derivative, verify that f~ (x, y) ~ x 2 - ll2 4, 211 2 ] , = y [ - 2 - " I -f-- ( I ! 2 (when,\2 + y2 :;Ie 0), f x (0, 0) -= 0 and, consequently, ~+y x+y) that for x =-= 0 and for any y, (0, y) -== - y. Whence (0, y):z: -1; in par'1 ticular, (0. 0) = -1. Sirnilarl y, we find that f;'x \0, 0) ~ 1. , " " iJ2 z 2 "

':'1

f:

1:

1903.

-a:z=2I u (u, V)+4Xlfuu a2 z , , ,

(u, v)+4xYf uv (u, v) +y I vl,(u, v);

ax ay = 'v (u, v) + 4xyf uu (u, v)

" + 2 (Xl + y2) f" uv (u, v) + xYfC'v (u.

v);

a , al/z :=21
2
2

u (u. v)

+ 4y f" uu (u,
l

v)

+ 4xYl" uv (u,
,,,

v) +x f vv (u, v).

2 "

1904.

a!u O~I

== fxx + 2f~l<PX +I Zl (<Px) 2 + fz<px,

"

",,,

438
a"z 1905. ax" '"

Answers

= fuu (q>x) " + 2fuv<l'x"'x + fvv ("'x) + fu<l'xx + 'v"'xx; o"z " " " " " I'" ,,, ,,, ax ay = Iuu CPxq>y+ fuv (<I'x'l'y +'I'x<l'y) + Ivv'l'x'i'u + fuq>xy + fv'i'xy;
I " ,,, I"

"""

ay 2 =fuu (<I'y)" + 2fuv<py'l'y+fvv('I'y)"+fuq>yy+fv'l'yy


II ,,,

iJlz

",

""'"

1914.

X [(y dx +

u(x, y)=<p(x)+'I'(y). X dy)" + 2dx dy].

1915. u(x, y)=xq> (y)+'I' (y). 1916. d2z=e XY X 1917. d"u = 2 (x dy dz + y dz dx + 2 dx dy).

1918. d2 z=4cp" (t) (x dx+y dy)1

+2cp' (t)(dx' +dyl).

1919. dz= ( ;

fY x

X (Yln~dx+xln!-dY);
y

ey

d 2 z= (!..)XY [(ylln" ex+ y )dx2+ \ y y x

+2 (eYf~ +eXf~+xe2Yf:u+e x + y (1 +xy) f;tV+ye2Xf~v) dx dy + + (xeYf~ +x1e"Yf: u +2xeX+Yf~v+ e"Xf;,v) dy". 1922. d'z = eX (cos y dx 3 - 3 sin y dx" dy - 3 cos y dx dy" -l- sin y dya). 1923. d'z = - y cos x dx 3 -3sinxdx2 dy-3cosydxdy"+xsinydy3. 1924. df(l, 2)=0; d"f(l, 2) = = 6dx" + 2dx dy + 4. 5 dy 2. 1925. d"f (0, 0, 0) = 'ldx" + 4dy" + 6dz 2 - 4dx dy + ya X + 8dxdz+4dy dz. 1926. xy +C. 1927. x' Y--3 +sin x+C. 1928. x-t- y 1 x x +In(x+y)+C. 1929. -2 In(x"+y2)+2arc tan-+C. 1930. - + c.

+2 (x y In ex In ey ~+ In":") dx dy + (X"ln"!...-~) d y2 ] . y ey y y 1920. d"z = a2f~u (u, v) dx" + 2abf;'v (u, v) dx dy + b2f:v (u, v) dy". 1921. d z = (yeXf~ + e2Yf:u + 2yex +Y f;w + y2e!Xf~v) dx" +
2

--+

1931.

Yx"+y"+C. 1932. a= -1, b= -1, z= ~-+Y2+C. 1933. X2+y2+ Z2_tx y + xy +xz + yz C. 1934. x3+ 2xy 2+ 3xz + y!-yz-2z + C. 1935. x"yz-3xy2Z -t+ 4x"y" 2x+ y+3lz +C. 1936. ~+ 1L+~+ C. 1937. Yx 2+y2 -t- Z" + C y z x 1938. A= -I. Hint. Write the condition of the total differential for the xy

expression Xdx+Ydy.

1939. f;=f;.

1940. u=Sf(Z)dZ+C.
a

1941.

~=

---a" y'

bZx. d"y b4 day 3b 8x . . . -d .=-2'3; -d 3 = - 4 5 1942. The equation definlngy IS the x ay x ay


1943. ddY = 1 Y"

equation of a pair of straight lines.

:;=0
1946.
1948.

Yy)I'
x

1945.

'!1t. =
dx

ax-y
XJJ

+ ay ;
-z"

d"y (a + 1) (Xl + y") dx"= (ax-y)3


2

(:t=1=3

-xy

In''~l'

1944. ddY =--L. ; x y- 1

or

-I;

(~:.t=1=8
dx

or

-8.

1947.

c!lt. =

JL ;
x

d"y 2y dxl=XZ .

nx

~=x"-yz;

=xsiny-cosz cos x-y sin z

~=6Y"-3xz-2. 1949. az=zsinx-cosy. az 3 (xy -2") ax cos x- y sin z' 2 1950. oZ=_I; Oz I 1951. az=_c x; ~=_c:21J. ox ay =2'. ax a2 z ay b?z ,.

ay

au

-==.,-

Answers
1953.

439
dx =
dz

2v d 2 -(I+y)2 y.

1965.

1971.

a)

440
1983. =

Answers

3x+4y+12z-169=O. 1985. x+4y+6z=21 1986. x+y+z= (2 1987 At the points (1, 1. 0), the tangent planes are parallel to the xz-plane; at the points (0, 0, 0) and (2, 0, 0), to the yz-plane. There are no points on the surface at which the tangent plane is parallel to

y a2 + b2 +
O

the xy-plane. 1991.

n . 1994. Projection on the xy-plane: 3

{Z:=+O! 1-0 x y -xy- - .

Projection on the yz-plane: {

;:::Z2_

I =0

Projection on the xz-plane:

IY=O 3x 2 \ 4+ z2 - 1 =0.

Hint. The line of tangency of the surface with the cylin-

der projecting this surface on SOIne plane is a locus at which the tangent plane to the given surface is perpendIcular to the planf of the proJcclion 1996. f (x + h, y+ k) =ax 2+ 2bxy + cy2+ 2 (ax + by) h + 2 (b..: +cy) k -t-- ah 2 + + 2bhk + ck 2 1997. f (x, y) = l-(x + 2)2+ 2 (x + 2) (y-l) + 3 (y_l)2. 1998. ~f(x,y)=2h+k+h2+2hk+h2k. 1999. f(x, y, z)=(x-l)!-t-(y-l)2+ + (z-I)I+2(x-I)(y-I)-(y-I)(z-I). 2000. f(x+h, y+k, z+l)= = f (x, y, z) + 2 (h (x - y - z) + k 2 (y - x - z) + I (z - x - y)] + t (h. k, 1). 3"(2 y _ y' X +y2 x4+6x 2y2+ y" 2001. y+xy+ 3! .2002.1-4! 2003. I+(y-I)+ 2

,-+

+tx-I)(y- I).

2004.

1+[(.t -I)

(y

I)]

+ [(x-I)
_

-t-

(x-I)+ (y+I)]'
31

2005

a) arctan I+a __ ~+..!- (u+P)_.!(U2 RI). 1-~ -- 4 2 4 -p,


[(3m 2
4m)

2!

(y+I)]2

b) "' / (I +a)m

+ (3n

V2

-4n) ~2]o 2006. a) 1.0081; b) 0.902. Hint. Apply Taylor's formula for

+ (l + ~)n :::::: I +! (ma +n~) +! 2 4 32

2 _

2mna~ +

the functions: a) f (x, y) = In the neighhourhood of the point (I, I); b) f (x, y) = yX in the neighbourhood of the point (2,1). 2007. z = 1 2 (x - 1) - (y-l)-B (X_I)I+ 10 (x-I) (y-l)-3 (y-I)! 2008. zmln == 0 when x= 1, y=O 2009. No ex\remum. 2010. zmin ==- 1 when x= 1, y==O. 2011. zmax= 108 whenx=3,y=2.2012.z min=-8 when x= Y2~'y=and when x= ,r..1. ab =- r 2, y= f 2. There IS no extremum for x=y=O. 2013.2 max ==---.;r-at

Yx- V;

+ ...

yi
b

3 r 3

the FOlots x=Y3' Y=Y3 and x=-Y3' Y=-Y3; 2ml n = - 3 . a b a b at the pOints x=Y3t y=-Y3 and x=-Yat Y=YS2014'Zmax==1 when

ab

Y3

(2 = } ) at points of the circle x + y = I. 2016. zmax= Y3 whenx= I, y =-1.


2 2

x=y=O.

2015.

2 m in=0

when x=y=O;

nonrigorous n18XilllUIlJ

2017.

Umin=-a

4 2 1

when

x=-3"' Y=-3 %=1.


t

2018. u min=4 when

y= I, z= I. 2019. The equation defines two functions, of which one has a maximum (zmax = 8) when x = 1, y = -2; the other has a minimum (zmn=-2Jwhenx=-l,y=-2, at points of the circle (x-I)2+(y+2J 2 _.:25, each of these funcbons has a boundary ext-emum (z =3). Hint. The functions mentioned in the answer are explicitly defined by the equalities

x= ~ .

Answers

441

the boundary of the circle (x-l)2 + (y + 2l =: 23, at the points of which both functions assume the value z =3. This value is the least for the first function and is the greatest for the second. 2020. One of the functions defined by the 2) for x =-= - 1, Y = 2. the other has a equation has a rnaxim UIn (<'max mininlum (zmin == 1) for x = - 1, Y == 2, both functions have a boundary extremUlTI
-4":: -

z=3 23-(x-l)2-(y+2)2 and consequently exist only inside and on

at the points of the curve 4x l -4y2-12x + 16y-33 = O. 2021. zmax = for 1 x==Y=2' 2022. 2 max =S for x=l, y=2; Zmin-':-:-S for x=-l, y=-2
2023. zmin = 13 for x = 13' Y ==

36

18

I3'

12

2024. zmJx =

2 t-

V2-

for

x == "8

7n

+ ktt.

2 - V2 3n Sn Y= 8" kn, zmin ~ - - 2 - for x =8" + kn, Y= krt. 2025. lJ m1l1 = =-9 f~r x==-I, y==2, z=-2, umax =9 for x=l, y=-2, z=2. 2026. llm.lx --= a for x == a, y = z = 0; umin =.: c for x == y = 0 z -=-= c. 2027. umax=2.42.63 for x=2. y=4, 2=6. 2028. umax=44/27 at the P011ltS 4) 4 3' 3' 3" ; (4 3"' 3' 3" : 3' 3' 3 ; umtn -=4 at the potnts (2, 2, 1) (2, I, 2) (I, 2, 2). 2030. a) Greatest value z=3 for x=O, y = I; 2 b) smallest value z=2 for x= 1, y=O. 2031. a) Greatest value z=---;r for 3 r 3 ,;2 (T 2 .. /2 x= V 3' Y= 3: smal1t'st value z=- 3 for x= V 3' 9n

8+

( 4 4 7)

(7

4)

Y3-

~-

y~;

b) greatest value z = 1 for x =

I, !/--= 0; smallest

value
n .

z=- 1 for x=O, Y= I. 2032. Greatest value z=--2- for x=Y=3 (Internal maximulll); slTIallest value z=o for x===y-~O (boundary rTIlnlmum). 2033. Greatest value Z= 13 for x==2. y ==-1 (boundary maXllnUnl)~ snlallest value 1.:.=-2 for x-=,y== 1 (internal minimum) and for x=O, y=-1 ,boun1 nary minirn urn). 2034. eu be. 2035. 2V, 2V, 2" 2V. 2036. I sosceles

V3

V - V- V= Va-. V;; V; Va. 2039. M ( triangle. 2037. Cube. 2038. a


X=::

. . 3 3 p 2040. Sides of the tnangle are -4 p, -4 p, and -2 . 2041.

In x

111 1

+ nl 2x 2 + rnat a , + 111 + lila


2

-+. +) .

11= nl.lh+ m 2lJ2+ n1 aYa.


In.

+ nIl + nla

2042. ~+1L-t-~=3. 2043. The dimensions of the

abc

parallelepi ped are

.~a_.
y

,2):--, r 3

,~_,
r 3

where a, b, and c are the semil=

axes of the ellipsoid. 2044.


y'=

x=y=2~+ V 2V.

~.

2045. x=

Vf '

:2

2046. Major axis, 2a=6. minor axis. 2b=2. Hint. The square of

the distance of the point (x, y) of the ellipse from its centre (coordinate origin) IS equal to x 2 + y2. The problem reduces to finding the extrenlum of the function X 2 +y2 provided 5x 2 +8xy+5y 2=9. 2047. The radius of the base ot the cylinder

442

Answers

is

V
the

2+

;5

the altitude R

2-

;5'
(

where R is the radius of the

spkere. 2048. The channel must connect the point of the parabola with point of

(~, ~)

th~

SlD p v2 4 the ray passes frem one medium into the other, must lie between AI and B l ;

204ft. 11 Y2730. 2050.

s~n~=~.

straight

line

~,

-:

Hint. Obviously,

t~e

1; its length is !.-~2=


point M, at which

p. The duration of motion cos a cos p of the ray is __ a_+ ~. The problem reduces to finding the minimum v 1 cos a 2 cos p of the functien f(a, ~)= __ a_+~providedthat a tan a+b tan~:=:c. 0lCOS a v 2 cos P
2051.

AM =_a_, 8M =~, AIM =a tan a, 8 1 M =b tan

a=~.

2052. 11 :1.:/.=

~1: ~. : ~.'

Hint. Find the minimum of the

function f(/ 1 , 12 , 1.)=/~Rl'+/:R2+ I:R. provided that 11 +/ 2 +1.=7- 1. 2053. The isolated point (0, 0). 2054. Cusp of second kind (0, 0). 2055. Taenode (0, 0). 2056. Isolated point (0, 0). 2057. Node (0, 0). 2058. Cusp of first kind (0, 0). 2059. Node (0,0).2060. Node (0, 0). 2061. Origin is isolated point if a > b; it is a cusp of the first kind if a = b, and a node if a < b. 2062. If among the quantities a, b, and c, none are equal, then the curve does not have any singular points. If a === b < c, then A (a, 0) IS an isolated point; if a < b == c, then B (b, 0) IS a node; if a === b = c, then A (a, 0) is a cusp 2 of the first kind. 2063. Y= x. 2064. y2=:.2px. 2065. y-=- R. 2066. x / a +

+ y'/.=l'!a.

2067.

XY=~S.

2068. A pair of conjugate equilateral hyperbolas,

whose equations, if the axes of symmetry of the ellipses are taken as the
coordinate axes, have the form xy =

2~

. 2069. a) The discri minant curve

y=O is the locus Qf pOints of inflection and of the envelope of the givell family; b) the discriminant curve y = 0 is the locus of cusps and of the envelopE of the fain i ly; c) the discriminant curve y == 0 is the locus of cusps and is not an en velope; d) the discriminant curve deconlposes into the straight hnes: x == 0 (locu~ 2 00 1 2 of nodes) and x=a(envelope). 2070.y= 2 _gx .2071. 7 .2072. y'9+4n 2 , 3 g 20: ..r- t In 10 2073. r 3 (e -1). 2074. 42. 2075. 5. 2076. X o -l- ze 2077. 11 -1- -92079. a) Straight line; b) parabola; c) ell ipse; d) hyperbola. 2) 2080. I)

'1n aO

2082. 4t (t 2 + 1). 2083. x = 3 cos t; Y = 4 sin t (ell ipse); for t =0, !' = 4j, tv =. -31; fo 1t 3 Y2 . r-3 y~r -. rt t=4,t1==--2- 1+ 2 r 2J, w=--2-1-2 Y2J;fort="2 ,t1=-31, w== = -4}. 2084. x == 2 cos t, Y = 2 sin t, Z = 3t (screw-line); !' = - 21 sin t -t + 2jcos t-I-3k; v= yf3 for any t; w=-2icost-2jsint; w-=2for any t
for

ad~O; 3)~ao+add~0.

2081.

:t(abC)=(~~bC )+(ad~c)+(ab~~)

t=O,

'D=2j+3k,

tv =-21;

[or

t=~,

'D=-2i+3k,

tv=-2J

Answe,~

443

2085. X= cos a cos rot; y = sin a cos rot; z = sin rot (circle); f1 = cos a sin rot -- oo} sin a sin rot rok cos rot; v = I ro I; tv = - ro 2i cos a cos rot - ro 2} sin a cos rot -

rot

-w2ksinrot; w=ro 2 2086.


l

w=g. 2088. 00 Ya + hI, where 00= ~t is the angular speed of rotatien of the
:screw. 2089. Va 2u>2+11;-2aoovoslnoot. 2090. 'f=
=

v=VV~+Vy2 +(v x -gt)2; wx=wy=O;wz=-g; o a


0 0

~2(l+k);
~3;

'Y=-};
k);

!J=
'11

yet
't

(i-k).

2091. 'f=

J-3- cos t - sin t) l +


yi05
R= 'II

(sin t

+ cos t)j +
2093

=-J2(Siut+Cost)l+(Slnt-cost)}); cos ('f-:'Z) =


2092

cos('Y~)=O.

x-acost = -a sin t

=.i+4j+2k.

y2f'

= -4i-f-5J-8k.
v

-2i+k

Y5

_y-asint __ z-bt (t t)o x-acost y-asint z-bt (b' I). - acost - b angen, b sin t -beast a lnorma, x-a cos t = y-a sin t =z-bt I Th e dtrec tIon cosines 0 f t 0 - (pnnclpa I norma). cos t sin a sin t t the tangent are cos a = - y 2 ; cos ~ = ya cos ; cos Y= ~r 2b l . a + b2 a2 + b2 f a +b The direction cosines of the principal normal are cos a l =cos t; cos ~l = sin t; cos '\'1 = O. 2094. 2x-z = 0 (normal plane); y-I = 0 (osculating plane); x-2 y-4 z-8 x+2z-5=0 (rectifying plane). 2095. -1-=-4-=12 (tangent); x+ +4y+12z-114=O (normal plane); 12x-6y+z-8=0 (oscutating plane).
0

t3 Y2096. -t2 - = - t - = - I - (tangent); t 3 +2t = I - tol

x- 4

t2

t' y-at"

z-2
t3

t2

x-T

tf.

a
MJ

= -2t
({-,

z-2
3

t~

-t

(prin-

x-- y-4 cipal normal); -1-- = -2}=~ 8) x-2 M 2 ( 4, -3"' 2 2097. -1x-2 y+2 z-2 ing plane); -I-=--=t= - I
(bJJ10rmV~~ cos a l =

t2 z--

(binormal);

-+,

~);

y+2 z-2 =-=1=-2-

(tan~ent);

(principal normal);

+1=-1-=-0-

x+y=O (osculatx-2 y+2 z-2

Yi";

cos ~I=

Y2 '

Y-7[ COSY2=Q. 2098. a) -2-= - 0 - ==

R x-2'

z-T R
_

Y2

..;x-I y-I z-2 (tangent); x y 2 -z= 0 (normal plane); b) -1-= -1-= -4-

(tangent); x+U+ 4z- 10 =O (normal plane); c)

1 2 r a -2 3 (tangent); 2 Va x+y-2 y3i=O (normal plane); 2099. x+y=O. 2100. xy-z V2-.=O. 2101. a) 4x-y-z-9=0; b) 9x- 6U+ 2z- 18=0; c) b2 x:x-a 2 y:u + (a 2 _b 2 ) = a2 b2 (a 2 -b!). 2102. 6.Y-8y-z + a = 0 (osculat_

x~,.~ u-2 V3 =

z-:

z:z

ing plane);

x-I

31 =26= -22

y-I

z-I

(principal normal);

x-I

-6 =

y-l

z-1 -S-==-r

444

Answers

(binormal). 2103. bx-z -= 0 (osculating plane); ; :

+bz = O,} (bmormal);" . i + bk - bl + k = y ; p = y--; Y= O Itb l+b


2 2

g' } (principal
'V

normal);

J.
3

2106. 2x

+ 3y + 19z-27 =
b) K
2111.

O. 2107. a) Y2; b)

--

T.

yEf

2108. a) K =

e- t

Y2

; T

= 31

e- t

= T = 2a C~Shlt'
a2 +bl
av l
.

2109. a) R = Q = 2112.

(y~a)1
t=O,

; b) R = Q =

(P~;:~~)'
w-,;=O,

When

K=2,
22
wn =2

w n =2;

when

t=l,

K=7 JI 14'
Chapter VII

1 ,/'19

w'1;= Y14'

,/19

JI

14

2 25 2113. 4"3. 2114. In 24

2115. 12' 2116.

11:

4".

2117. 50.4. 2118.

T.

na 2

2119. 2.4.

2120.

~.

2121. x=y; -I; x=2-y; y=-6; y=2. 2122. y=xl ; y=x+9; 2123. y=x; Y= IO-x;

x= I; x=3.

y=O; y=4. 2124.


1
J

y=3; y=2x;
2 I

x= I; x=3. 2125. y=O; Y= Y25-x 2 ; x=O; x=3. 2126. y=x 2 ; y=x+2;


x=-I;
I

x=2.
1

2127.
1

Sdy Sf(x,
o
X
0
0

y)dx=
1

Sdx Sf(x,
0 0

y) dy.

J-y

2128.
1 1

Sdy Sf (x,
o
y.

y) dx=
I

Sdx Sf (x,
0

y) dy. 2129.

Sdy S f (x,
o
I 0

y) dx =

2-X

Sdx Sf (x,
o
~

y) dy+

Sdx S f (x, y) dy.


1 0

2130.

S dx S f(x,
1 IX

tx+lt

y)dy=

.!L.
2 I
J

Sdy Sf(x,
2 1

y) dx+

Sdy Sf(x, y) dx+ Sdy S f(x, y)dx.


4 1 I
~

II

VI

V 2_ YI
1
_

y2

'V 2 -

X2

2131.

Sdy S f (x, y) dx + S dy S
o - y
111-

f (x, y) dx=
yl
1

S dx S
1
J - X

f (x, y) dy+

x"

Sdx S
o
~

f (x, y) dy. 2132.

S dx Sf (x, y) dy= Sdy


-I

V~

f(x,

y) dx.

IX 2

-v f

..

-j-

Answers
1

445
1

V4="X2

V1=Xi

V~ - r

2133. S dx S {(x, y)dy+ S dx S {(x,y) dy+ S dx S f(x.gldy+ - 2 _ V 4 - x2 - V 4 - x2 -] y, - t'2 - 1 J Yi"'=X2 -1 V4 _y2 1-~

t- SdX

S
- y4
- X2

{(x, y)dy= S dy
-

S
-

{(x, y)dx+ S dy
-

S
-

f(x.y)d+
11 2 y2

2
1

1/4 IJ2 1/4 - l/~

1
2

Jl 4 V4 -

+ S dy S f (x, y) dx + S dy -] vJ=Yi ] - y,
2

f (X,

y),

tIL.

_11 2

'V e -

X2

J! 1 +

x2

2134. S dx
- a a

S
_ Ve .- x 2

{(x,y)dy+ S dx
-

S
_

{(x,y)dy+

1 r 1 + ..(2

Ye-x 2

-I

-Vy2-1

t SdX
2
- 1

S
_

{(x, y)dy=
J

S dy
1/ I

V;-:-xi
V e - lP

- JI i - V 9

5
-

f (x,
lJ2

y) dx+
II', -

- 1.1 2

t- SdY S _. Vi V y 2
JI"5

{(x, y) dx +
t
lJ2

S dy S
1 _

V.Y2"'=1
S {(X, y) dy-JY

{(x, y) dx+ S dy
y2
1
1 _

V, a)

V. _ yl

+
b)

5 5
dy
J

JI e -

1 - X I I -

{(x, y)dx.

2135.
a

Sdx S {(x, y) dy= SdY S {(x. y) dx~


0 0 0 0

V;;;-:-i
-

V CJ2

x2

V a2 - 1/2

1 ] 1X - x 2

S dx S
- a _

{(x, y) dy= S dy
-

{(x, y) dA; c) SdX


1 1

S
VX
-. x.l

f(x, lJ)dll-=

V CJ2
1+

xJ

- ~ . a 2 - lJ2
J

VJ:4iii
2

5dy 5
- J/J
a
1 -

1/'J

{(x, y)dx; d)
a x

5dx 5{(x, y) dy= S


-J

dy S {(x, y)dx;
aa
a

VI4';i2
2

--I-I

y + 2a

e) S dy
o

{lx, y) dx= S dx
0

u Y

{If
S
3...
12

5
0

{lX, y) dy
1

+5 dx 5 f (x, y) dy +S tlxS
a
0 I

fa

f(x. y)dy.

IQ

x- 10
y)

2136.

5dy
o

{(x, y) dx. 2137. S dy

Sf (x, y) dx +S dy S f(x.
2

dx..

O!!..

a
- y2

JL
a

2"
2138. S dy
o

Vu

Va

2 - y2

S
VU~-Jay

{(x, y) dx+

Sdy 5
a
I
0

f (x. y) dx.

444 (binormal). 2103.


bX-J -

Answers
0 (osculating plane);
l' =

x + bz =

Y= O

O,} (binormal); .
1

+ bk ; p= ..-r bi + k ; \' = J. ..I r 2 2


rl+b

;:~'

} (principal normal); 2106. 2x + ; T

+ 3y + 19z-27 = O.
b) K = T

2107. a) Y2; b)

--

T 2108. a) K =
(y + a)2

yEf

rl+b

e- t

Y2

= 3

e- t

= 2a cosh1t 2109. a) R =
av l

Q = - a - ; b) R = Q =

(pC + 2xl )'

Bpcx:'

2111.
wn

a2 +bi .
when

2112.

When

t = 0,

=2;

t= I,

1(=+

-V:;,
4 .
9

w~=

;14'

K = 2,
wn =2

w-,;= 0,

-V~;.

Chapter VII
2 2113. 43". 2114.
2120.

1"24

25

2115. 12

2116.

2117. 50.4. 2118.

T.

na2

2119.2.4.

6.

yl. 2121. x=4- 1, x=2-y; y=-6; y=2. 2122. y=x l ; y=x+9;

x=l;

x=3.

2123. y=x; y=10-x;

y=O; y=4. 2124. Y=3;


1 2 2 I

y=2x;

x=l; x=3. 2125. y=O; Y= Y25-x 2; x=O; x=3. 2126. y=x2 ; y=x+2;

x=- 1;
I

x=2.
1

2127.
1
X

S dy S f (x. y) dx= S dx S f(x,


o
0
1

y) dy. y) dx =

2-y

2128.
1 1

S dy S f (x, y) dx= S dx S f(x, y) dy. 2129.


o
y"
20
X

S dy
0 2

S f (x,
0 IX + I
2X

=SdXSf(X,y)dy+SdX
o
4 0 1 I

S f(x,y)dy.
0 2 7

2130.

SdX S f(x,y)dy=
1

.JL
2 2

=SdySf(X, y)dx+SdySf(X, y)dX+SdY S f(x,y)dx.


2 1 4 1 I
~

2131.

Sdy S f(x, y) dx+l'"f dy YIY'


o - y
_ y
1
2 _

f(x, y) dx=

Sdx YIX'f(x,y)dY+
- ]
2

yl
2

Y2

-{

+
f (x, y) dx.

+ S dx
o

S
~

f (x,

y)

dy. 2132.

S dx S f (x, y; dy = S dy
-1

Ix l

.-j-

-V

Answers

445

2133.

5 v"5 dx
2 _

-I

Y~

f(x, g)dg+
x2

5 5
dx
J V'" _y2

-Y~

f(x,g) dg+

5 5
dx
J

~-r

f(x.gldg+

'V" - x 2

dX +5

y~

J-Y1='Yi

'V r

t'2

S
- 'V4=X2

f (x, g) dy= S dg
2 1 _

S
V" _. lJ2

f (x, g) dx+ S dg
2 1 _

S
-

JI" _ y2
114.
y2

f(x.!I)d~ +
f (x,
1/). ttL

+
-

5
1
-1

11" -

'1 2

dg
VI
2

S
_ y2

f (x, g) dx + S dg
J _

y" _ y2

2134.

5
I _

Y8="Xi

J! I + x 2

dx
_

S
'V It
-_

f (x, y) dg
x2

+S
2

dx
_

S
1 r 1 + x2

f (x, g) dg + f (x,
y) dx+

5
2

Ye-x 2

-V'y2-l

dx

S
V;-:-xi

f (x, g) dy =
- 1/ 2 1

S dy
- V,"

S
- V9
- lJ 2

- 1

JI 8

V .. -

11 2

V,

1-

Sdg S f(x, y) dx+ S dy _. Jli JI y2 - 1 - 1 - 'V.. JI i JI.. - lJ2 S dg


1

S
y2
0

YY2="i
S
Y

fIX, y)dx+ S dg
1
J -

f(x,g)dg-J,..

11.. _ yl

X I I -

+
b)

S f (x, y) dx. Vii'2=J


11a 2 - x 2

2135.
a

a)

S dx S f (x, gl dg= SdY S f (x, g) dx:


0
1

0
-

V a 2 - '/~
y~
1

VX

x2

S dx S
- a _ Va
I
1/ 2
2 J

f (x,
2

x + Vt:4ij2

y) dg = S dy S - V al - a
1

f (x, g) dx; c) S dx
-

S
-

{(x, lJ)dll
x~
1/

a::

11 x --

S dy
_1/ 2

S
I-V~
2

{(x. y) dx; d)

S dx
-I

5f (x, y) dy = S
X

dy S '(x, y) dx;
aa
a

--1-1

e)

5dy
o

y+ 2a

fa

S f (x, y) dx = S dx S f (x, y) dy + S dx S f (x, y) dg + S tlxS 1(x, y)dU.


{

48 II

~
S
.!!..
12

f
a

la x - la

2136. S dy
o

f (x, y) dx. 2137. S dy S f (x, y) dx + S dy S f (x, y) tU~


O.!!..
-

2.1L
a

.!!. 2
2138. S dy
o

VUl

y1.

Ya

2 - y2

S Vu~ -

f (x, g) dx + dy
2ay

S
.E..
I

S
0

'lx, y) dx.

446
aY""'I
I

Answers
a a a

2139.

S dy S1 (x, y) dx + 5 dy S
o
~
2

f (X,

y) dx.

a'V a
2

a-l'a 2 2a

_y2

a
2140.

a-

Jla;-::y;
S
~

JI"ia 2a
0
J

S dy
o
o

1(x, y) dx+ Sdy

S
a+V a 2 _ y 2

1(x, y) dx+ S dy S 1 (x, y) dx.


~

4a

ta
I 1-

'V 1 - x 2
0 0

-;:

'V
0

2141.

S dx S
-I

1(x, y) dy+ SdX


'V'-

S I(x, y)dy.
0

2142.

5dx S 1(x, y)dy+


0

Y"-;:-

5
.!...
2

dxSI(X,y)dY+
0
J

VI
n - arc sin y

5 5
dx
0

Y"i'=X2

RV"I
- 2-

f(x, y)dy. 2143.

5 5
dy
0

1/ R2 _ y2

I(x,y)dx.

2144.

5dy 5
o arc
~ln

f (x,

y) dx. 2145.

i
21 5. 2

2146.

i
3";
4

2147.

~ a.

2148.

i-.
5

2149. 6. 2153. 8

2150.

2.

2151.

In 2
I

a)

b) 15n - 16 150

) 2 2

Y2 P5 . -21Hint.

Vl-(X -2)2

2154.

5dx S
1 0

XYdy=i
Y=f (t')

2155.

~ a Y2a.

2156.
13t

~nRI.

55
(8)
(I -

231R

ydxdy=
cos t)

5 5
dx
0

y dy=

R (I-cos t) dt

y dy, where the last integral Is obtained from . R 80


4

the preceding one by the substitution x= R (t-SiD t). 2157.


3t
4 1

2158.

1 . "6

2159.

2160.

5 S rl
cos q>
o

dq>

(r cos '1', r

sin '1') dr+

o
n
1

'2

sin q>

.:!..

+S dlp T
n

5
0

r/(rcos<p, rsluijl)dr.

2161.

5
4.

cos q>

dlp

S rl (r
0

dr,

Answers
'31
4.

441
n . sin cp
ces 2 cp

2162.
an
~

5 5
dcp
n
0

sin cp

rf (r cos cp, r sin cp) dr.


1

2163.

o
sin cp

5f

4.

(tan lp) dcp

S r dr+

n
an
~

C8,2

sin cp cp

+S f (tan cp) dcp n "


~
4.

S r dr + Sf (tan cp) dcp S r dr.


4)

2164.

S dcp S
n
31:

aY~

rf (r cos cp, r sin cp) dr +

5d cp S
an
4.

uta'V~

rt (r cos cp, r sin q dr.

2165.

5dcp 5 2168. (~+ i )


2 3

a cos cp

2166.

2167.

ai,

2169.

2170.

Y2 -20) ~ ( ~_16 3 9 2

2171.

nab. Hint. The Jacobian is 1= ubr. The limits of integration are


f\

c
I-V

l+fJ

o.;;;; III E;;; 2rr,

0.;;;; r E;;; I. 2172.

S dv S f (u 0

uv,

uv) u duo

Solution.

We

a
l+a

have x = u (1- v) and y === uv; the Jacobian is 1= u. We define the limits u as functions of v: when x=O, u (l-v)=O, whence u=O (since 1- v

i:. 0); when x=c,

= -c 1 .

-v

Linlits

of variation of v:

since

y=ax, it follo\\s that llv=au (I-v), whence v=-1 a ; for y=px we find
2173.

+ ~ d<
+ ~(~ dv

t>
5f
II

I=~ (~ du ~t f (U~V,
2

+a

u2

V) dv +

(U

v,

2V) dV] = ~ ( _~ dv
~~l +::].

f (U

~ v,

U2 V) du +

2 -V

(u+V u2 V) dU.] Hint. After change of variables, the equa-2-'


Solution. The equation of the curve

2174. ab [(~- ~)arc tan

tions of the sides of the square will be u=v; u+v=2; u-v=2; u=-v.

448

Answers

r4 .. r' (:: cosacp- :: slna cp). whence the lower limit for r will be 0 and

.ahe upper limit. r=

Y ::
b2

cosl cp- :: sin l cp. Since r must be

real. It

follows that hI cos 2 q> -Ji2 sin l q> ~ 0; whence for the first quadrantal angle
ak we have tan q> ~ bh . Due to symmetry of the region of integration relative

a2

10 lhe axel, we can compute

1 4" of the entire integral, confining ourselves


arc tan bii
ak .. /~ a~

10 Ihe ftrst

Quadrant:
t

SS dx dy = 4
V II
(S)
2

5
0

hi

COS2

cP -

b2

ki sln2 cp

dcp

S
0

abr dr.
aya;-::.-;z a-

~171 ) 4~;
.178 9.' b) ) 2

Yg

Sdy
o _

S dx+ S dy S dx;
Vy
1

b)

~I_a;; SdX S dt/.


0

Y- 2

J&

(2 +4n) a.
2

217 7. 2181.

7a 120

17 2 8.

10 "3 a I .

2179 n

I H nt.

-l<x<;l.

2180.

16 ,r-

"3

15.

(' n 1) 3\ 4+2 . 2182.

4:t

3- ..f r-;3 .

~183.

{naa 2184. 6.
2186.
t

2185.

IOn.
x

Hint.

Change the variables

x-2y = u,

b+4,1=u.
,

l i b "3(b-a)(~-a). 2187. 3(~-a)lna.


t

tll8. v =

)dy. SdYSO-X) t!x= SdX 0 - X


o
y
0 0

2193.

n;a.
88

2194.}. 2195.
al

i
abc

2196.

"3. 2197.4Q. 2198. - 5 - 2199. 105 2200. 18 . 2201. T 4 nal ,r4 .. r2202. na' (a-~). 2203. 3 (2 f 2 -1). 2204. '3 real (r 2 -1).
na' T
2206.
RI

as

mt

48

V6

2205.

4 3 nabc .

2207.

na 3

(6

lr-;rf

3 -5).

2208.

32

"9 a3

2209. JUI(l-'-

3xab 3 ). 2210. -2-. 2211.


variables

va2 V2 lr-2 .2212. -2-(2 r 2 -J).


2213.

tllnt.

Change

the

xy=u. ~ =v.

~ Valbl+blcJ+clal

2214. 4 (m-n) RI. 2215. -2- a2 Hint. Integrafe in the uz-plane. 2216. 4a l .
b 1 ,r2217. Sallfe alna-. 2218. "'3 nal (3 , 3 -I).
Pus to polar coordinates. 2221.

Y2

2219.
I

8a2

2220.

3M2

Hint.

0=

:rra 2

[(

+ ~: )';- -1 ].

Hint. Pass to

Answers
polar coordinates.
2222.

449
Hint. Pass to polar coordinates.
a

~ a'
a

and
~

aaa.
2

2223. 8a l arc tan

Y52

Hint.

0'=5' dx5! y a -x a dy i-Sa 51 arc sin 2 .. r~~. -Y r a -x


l

Integrate by parts, and then change the variable x= a


the answer. 2224

~ 3 sin to

lransform

31 (b Yb 2 +cl -a a l +c1+c l In b+ ~~) . Hint. 4 a + al+cl 2 . 2nfJR a'b a2b~ 12-312 Pass to polar coordInates 2225. -3-. 2226 .12; 2T' 2227. x=3(4_n);

31 5 2~ sin a 2 Y 6(4-31) 2228. x=1fa; y=O. 2229. x==3<l; y=O. 2230. ~==5; n n y =0. 2231. Ix=4 2232. a) I O Z::: (D4_d 4 ); b) IX==64 (DC-dt ).
32
2234. 2235.

8 5 aC

Hint.

1-

5 5
dx
G

tJ

'Vax
(y+a)1 dg.

161n 2-9

i.

Hint.

Th~ distance of the point (x, y) from the

-JI'ax

straight lin.

x=y is equal to d=V-2~ and is found by means of the normal equation

of the straight line. 2236. l=~ ka 5 [7 Y 2 +3 In (Y2 + I)), where k is the proportional ity factor. Hint. Placi ng the coordinate origin at the vertex, the distance froln which is proportional to the density of the lamina, we direcf the coordillate axes along the sides of the square. The mOlllent of inertia i5 detennlned relative to the x-axis Passing to polar coordinates, \\'e have
n
t

n
2

I" =

J J
d<p
R

a sec cp

a cosec cp

kr (r sin <p)! r dr

+~
C

dq>

~
l-X-y

nac 35 2238. 10 =2' 2239. 12nac. Hint. For the variables of infeiratlon take Land
1 I-X

II (see Problem 2156). 2240. \ dx

5 S
dy

I (x,

II. ,) da

2241.

dx dll -R -V'RI-x1 b _ _0 y (J G a2 - x 2 -a

5 5 5f
dx
b

Jl'RI_X:il

0 0 '

(x, y, ,) d,.

2242.

dy

f (Je, 1/, .) d.

__

-a-JI'a 2 -xl

450
I

Answers
Yi'='Xi YI-x2 _y2

2243.

-1

Sdx S -x2
- 'VI

dy

S
s (

f (x, y,
2245.
.. r -

z) dz.

8 2244. 15 (31 2248.

+ 12

.. r .. rf 2 -27 f 3). 2249.


2

-3-

4xt

Y2

2246.

8
I

:llal

1 2247. 720
xtabc'

1 5 2" In 2- 16

na 5

18 f 3

-"6

97)

2250. 480
2 a.

59

nR. 2251. -4-


(

4 bc. 2252. 5111


2257.
2a

RI 2253. xth -4-


xt 2258. 10. x2 +/p
X2

2254 22 59. n
2

n Ra

8 2255. 9

8 r8 22 56 ."3

4) n- '3
[1=

4 31 RI . 15
'V 2QX -

32 2h ga.
2a

60 -:rna. 3 3 SIr 22. e u Ion.

,2
cos cp
la

=25 dx

o
3t
2

S
r'dr 2a

dy

S dz=2 5 dq> S
0 3t 0 0

2a

rdr 5 dh =
0

la cos q>
't'

--25 dm
o

=.! a

S(2aCOSljl)4 dm=!nal.
4
't'

2261.

2na

V2

. Hint. Pass

to spherical coordinates. 2262.


aI (3jt-). 4 2263. "9
2267. 2268.

~9 n.
22 65.

Hint. Pass to cylindrical coordinates.


a be (a+ b -I-c. ) T
ab (6c 2 -a 2 - b2 ) 22 66 24

4 226.
= 0;

rcabc.

x = 0; y
-

z=
-

2 "5 a.

Hint.
2

Introduce

spherical

coordinates.

4 ="3'

y =0, z =0. 2269.

na h 12 (3a 2 +4h 2 ).

Hint. For the axis of

the cylinder we t~ke the z-axis, for the plane of the base of the cylinder, the xy-plane The moment of inertia is computed about the x-axis. After passing to cylindrical coordinates, the square of the distance of an elelnent
r dip dr dz from the x-axis is equal to r a sin a <p

+za.

2270.

n~~al (2h Z +3a2).

Hint. The base of the cone is taken for the xy-plane, the axis of the cone, for the z-axis. The filoment of inertia is computed about the x-axis. Passing to cylindrical coordinates, we have for points of the surface of the cone:
r

=f

(11- z); and the square of the distance of the element r dq> dr dz fronl

the x-axis is equal to r2 sin 2 <p+z2. 2271. 2rtkQh(l-cosa), \vhere k is the proportionality factor and Q is the density. Solution. The vertex of the cone is tal<en for the coordinate origin and its axis is the z-axis. If we introduce spherical coordinates, the equation of the lateral surface of the cone will be

'" = ~2 -

a, and the

equation of the plane of the base will be r

=--/!:- .
Sin '"

From the symmetry it follows that the resulting stress is directed along the z-axis. The nlass of an element of volume dm=Qr 2 cos", dcp d", dr, where Q is the density. The component of attraction, along the z-axis, by this element
of unit mass lying at the pOint 0 is equal to k

~m

sin "I' = kQ sIn", cos '1' d'1' dq> dr.

Answers

451

The resulting attraction is equal to

5 5
dlp

2n

n '2 - a
d'ljl

h cosec

11'
kQ sin 'Ijl cos 'Ijl dr.

0 0 0

2272. Solution. We introduce cylindrical coordinates (Q, cp, z) with origin

at the centre of the sphere and with the z-axis passing through a material point whose mass we assume equal to m. We denote by ~ the distance of this point from the centre of the sphere. Let r= Y QI+(;-Z)2 be the distance from the element of volume dv to the mass m. The attractive force of the element of volume dv of the sphere and the material point m is directed along r and is numerically equal to -kym

~.
r

where y= ~ is the
-1tR8

density of the sphere and dv = Q dcp dQ dz is the elelnent of volume. The projection of this force on the z-axis is kmydv A E- z dF = - - - I- cos (rz) = - ktny _ . -3- Q dq> dQ dz. r r Whence
231

YRJ - zJ

F=-kmy5 d lp

S(~-z)dz
-R

Qr~Q=kmy4n:RI~~.

4 ynRI:.=.:: M, it follows that But since "3 2275. a)

F=~. 2273. -

kMm

Se
x

ex>

y2 -XJ

dy _e- X3 .
(p> 0)

-.!. P
2

(p> 0); b) _1_ for p p-a

>

a; c)

2276.

_..!... n

2277.!. Hint. DilJerentiate p8


~

r
o

RZ PI-I-'

z~

(p

> 0); d) 2,P


P

R2

-t"'

e- pl dt

=...!.. P

twice.

2278. In

~.

2279. arc tan--arctan-.

a m

2280. -2 In(l+a).

2282. arc cot

a If

2283. 1.

2284.

2"

2285.

2286. 4a 2

1.

HInt. Pass to

polar coordinates. 2287.

~"it.

2288.

~z 2289. Converges. Solution. Eliminate

from S the coordinate origin together with its e-neighbourhood, that is, consider II = ~ ~ In V Xl + yZ dx dy. where the eliminated region is a circle 01 radius e with c~ntre at the origin.
ut:
1

(8 )

Pa5sing to polar coordinates, we have


1

1.=

S eSr Inr dr= 5[r~ In r /; -~ eSrdr]


dlp
0

,n;

dlp= 2n:

(~-~

In

e-f)
=
(x-g)

Whence lim Ii =- - ~. 2290. Converges for a > 1. 2291. Con,oerges. Hint. Sur8-.0 2 XdY round the straight line g=x with a narrow strip and put S\~ Z
(5)

452

Answers

== lim
8-+0

Sdx J r dy V
o
0

x-e

+ linl r dx
6-+oJ
0

(X-y)1

> "2 .
~
I [

2293. O.

2294.

Y-5 +3 . In 2
a"
21Tb

x+~

dy V(X-y)2
2295.
2

2292.

Converges
l )

for

ab (a l

2297.

(1 +411 2)1 -1

.!]
l

3 (a + b)

+ ab + b

2296.

15 a

2Cl6 a

. 2298.

Y5m 1+ m

.2299. a 2
2

Y2.

2300.

54 (56

Y7-.
Hint.

-1). 2301.

ab

+b

arc tan

a . 2302.

231a. 2303.

16 .. r27 (10 y 10-1).

~ f (x. y) ds may be Interpreted geometrically as the area of a cylindrical sure face with generatrix parallel to the z-axis, with base, the contour of integra tion, and with altitudes equal to the values of the integrand. Therefore, S = ~ x ds. where C is the arc OA of the parabola y= ~- xl that connects the

points (0, 0) and (4,6). 2304. a


2306. Y a2+b 2

Y3.

2305. 2 21rb

(n Ya2+4nb2+~ln
~;

(b + .r aa b arc sin YQi=bi) . -b a


2
l

+ V"-:-2-+-4-n b -2).
-2

2307.

(ia.; a)2

2308. 2312. a)

2na Ya 2 +b 2

--

2309. Y(at+bl)I'

kMmb

2310.40

f;

30 ,

19

2311. -2na

b) 0; c)

d) -4; e) 4. 2313. In all cases 4. 2314. -2n. Hint.

Use the parametric equations of a circle. 2315.


2317.
liz

ab 2

2316. -2 sin 2.
%2

O. 2318.

a) 8;

b) 12; c)

2; d)

~;

e) In (x+ y); f)

~ q> (x) dx +

XI

+~""(Y)dY.

2319.

a) 62; b) I; C){+ln2; d) 1+Y2. 2320. YI+a 2_


a)

2322. c) eX-Y(x+y)+C; d) 2325.

- V 1 + b2

YI_-

x2+ 3xy-2y 2 C;

(i+ n 1~2) RI.


2328. -

Inlx+yl+C.

b) xl -x2y xy2_ yl C; 2323. -2na(a+b). 2324. -nR2cos2a

2326. a) -20; b) abc-I; c) 5

V2;

d) O. 2327. 1=
2332.

= SSy2 dx dy.
(5)

~.

2329.

n:

t .

2330. - { .

2331. O.

a)

0;

b) 2nn. Hint In Case (b). Green's formula is used in the region between the contour C and a circle of sufficiently small radiuc;; with centre at the coordinate origin 2333. Solution. I f we consider that the direction of the tangent coincides with that of positive circulation of the con1our, then cos (X. n) =

= cos (Y.

t)=1!-. hence. 1 cos (x. ds= 11t ds= 1 dy=O Sec s c


n)

2334. 28. where

S is the area bounded by the contour C. 2335. -4. Hint. Green's formula is
not applicable. 2336. nab. 2337.

~ nat,

2338. 6nat

2339.

a l Hint. Put

Answers
y = tx, where t is a para meter. 2340.

453"
2341. n (R

a 00.

+ r) (R +2r);
point of

6nR2 for

R=r Hint. The equation of an epicycloid is of the form x=(R+r)cost_ r cos R +r t, y= (R + r) sin t-r sin R +r t, where t is the angle of turn 01 , r
the
2342.

radius of

a sta tionary

11 (R - r) (R - 2r). llR2 for r = ~ Hint. The equation of the hypocycloid is obtained from the equation of the corresponding epicycloid (see Problem 2341) by replacing r by - r 2343. FR. 2344. mg (Z.-2 2 ).

circle

drawn

to

the

tangency.

2345.

(a l -b2 ). where k

is a proportionality factor. 2346. a) Potential, b) potential.

U =mgz.

U =1:... work. ~l ~ r a2 + b2 +C2 k k 8 c) potential, U=-T(X 2+y2+ Z2), work, 2" (R 2- r l ). 2347. 3 M .
work.

mg (Zl-ZI);
2

2348. 2353.

2na2
25

V a" + b 3
V5 + 1
5 1

2349. O.
n

2350.
4

3" nabc.

2351.

nat

2 .

2352.

(y-- _ ) a. 2354. - 2 - h
2356. O. 2357.
411.

V2

2355. a) 0, b) -lla2

10 5

rr (cos a + cos p + JJ
(5)

+ cos yl dS.
oP oR oQ iJz = ox' ox 2363. 2

2358.

2359. -al .

2360.

iJP = iJy 2361. O. 2362. 2


.1. 2

rr J J (x + y +z) dx dy dz.
(V)

~: = ~~ .

rr V d1C +dyy2dz+Z2' .\J


2366.

2361.

r~5 (d U iJ2U iJ U) J ox2 + oyl + OZ2 dxdydz.


, )

(V)

2365. 3a4

2.

as

2367.

5" na.

12

na 2 b2 2363. -2-

2371. Spheres, cylinders.

2372. Cones. 2373. Circles, X2+y2=C~, z=c,. 2376. grad U (A)=91-3j-3k;

\ grad U (A) 1= Y99 =3


d)

yo; Z2=xy;
= e;
2U

x= y=z.

2377. a) !.-; r b) 2r. c)- ,~ ;

I' (r) !... r

2378. grad (er)


2379
;

the vector e. ens (', r)


= -

iJU

a=- f (r) + f (r). r ' 2385. a) div r=3, rot r= 0; b) div (re) -= ~ , rot (re) = rx c ; c) div (f (r) c) = r ,
,2
for I

au -iJl =0
rot

or

=-'-'
J..r.

the level surfaces are planes perpendicular to iJU au a;-=/gradU/whena=b=c, 2380.

ar=
I

2 . 2382. - . 2383. dlV

I' (r) (c,


r

r),

(I (r) e) = I'

2388.

m = ook 2387. 2oon. where is a unit vector parallel to the axis of rotation. iJ2U o"U 02U dlV grad U = iJ>. 2 + iJ 2 +OZ2; rot grad U = O. 2391. 31tR 2 H.
y

,,0

(r) ex r. r

2386. div!'::.:: 0;

rot 'D = 2m, where

2392. a) kllR2H (3R I +2H 2); b)

~nR2H (R2+2H2).

2393. div F=O at aU

points except the origin. The flux is equal to -4nm. Hint. When calculatine

454

Answers
2395.

the flux, use the Ostrogradsky-Gauss theorem. 2394. 2n1hl


r

-~R

'2396. U =

Srf (r) dr.


r.,

2397. ; .

2398.

a)

No

polential;

b) U =xyz +c;

c) U=xy+xz+yz+C. 2400. Yes.

Chapter VIII
2401. 2n-l

2402. 2n 2403. 2n-1 2408.

11

2404.

1
n2'

2405. (n

n +2

+ 1)2 2406. 3n + 2 .
2410.
n+ 1
n(-I)

2n

1 2407. n(n+l).

135 1.4.7

(2n-l) (3n-2)

2409. (_I)n+l.

2416. Diverges. 2417. Converges. 2418. Diverges. 2419. Diverges. 2420. Diverges. 2421. Diverges. 2422. Diverges. 2423. Diverges. 2424. Diverges. 2425. Converges. 2426. Converges. 2427. Converges. 2428. Converges. 2429. Converges. 2430. Converges. 2431. Converges. 2432. Converges. 2433. Converges. 2434. Diverges. 2435. Diverges. 2436. Converges. 2437. Diverges. 2438. Converges. 2439. Converges. 2440. Converges. 2441. Diverges. 2442. Converges. 2443. Converges. 2444. Converges. 2445. Converges. 2446. Converges. 2447. Converges. 2448. Converges. 2449. Converges. 2450. Diverges. 2451. Converges. 2452. "-Diverges. 2453. Converges. 2454. Diverges. 2455. Diverges. 2456. Converges. 2457. Diverges. 2458. Converges. 2459. Diverges. 2460. Converges. 2461. Diverges. 2462. Converges. 2463. Diverges. 2464. Converges. 2465. Converges. 2466. Converges. 2467. Diverges. 2468. Diverges. Hint. all+ 1 > I verges conditionally. 2471. Converges conditionally. 2472. Converges absolutely 2473. Diverges. 2474. Converges conditionally. 2475. Converges absolutely. 2476. Converges conditionally. 2477. Converges absolutely. 2478. Converges absolutely. 2479. Diverges. 2480. Converges absolutely. 2481. Converges conditionally. 2482. Converges absolutely. 2484. a) Diverges; b) converges absolutely; c) diverges; d) converges conditionally. Hint. In exaillpies (a) and (d)
00

an

2470. Con-

consider the series ~ (a 2k-l +a2k)


k=1
00

and
00

in examples (b) and (c) investigate 2485. Diverges. 2486. Converges

-separately the series ~ a2k-l and ~ a2 k'


k=1 k=1

absolutely. 2487. Converges absolutely. 2488. Converges conditionally. 2489. Diverges. 2490. Converges absolutely. 2491. Converges absolutely. 2492. Con(IJ 1 (_I)n verges absolutely. 2493. Yes. 2494. No. 2495. an ; converges. 2496. (IJ 1 n=1 (2n-l); converges. 2497. Diverges. 2499. Converges. 2500. Converges.

L2n

n=1

"2501. I R4,1 < 120' I R., < 720' R4,<O, R,>O. 2502. R n < 2n+ 1 =2 n (2n 1) n! Hint. The remainder of the series may be evaluated by means of the sum of 1 a geometric progression exceeding this remainder: Rn = an [2 _1_

1 .

an

+(i ) (n + 1)1(It +2) +.]


2

< all

ri ~ 1 +( ~ )
n

(n

~ 1 + .]
)1

n+l

Answers
2503. Rn

455

< (n+ n+2 I) (n+ 1)1


1_

RIo

<

I 310-'. 2504. n+ I I I

<

Rn

i < Ii
_

Solution.

R
n

= __ 1

_(_I I) (I I) I n + 1- n + ~ + n + 2- n + 3 + ... = n + 1 ' R


I + (n + I)1(n + 2) + .. =(i

(n+ 1)2+ (n+2)2+ ... > (n+ I) (n + 2) + (n +2) (n+3) + ... n

<

n (n

I+ I) +

2505. For the given series it is easy to find the exact value of the remainder:

Rn=fs (n+ ~~) ( }


Solution.

Rn=(n+ I) (}

We multiply by

(f
n

rn

yn-z.

+(n+2) ({- rn+Z + ...

~Rn=(n+l)
Whence we obtain

(f yn+z+<n+2) ({-yn+4+ ..

16 R"

(~)2n
4

_ + (!.)2n 4 + (!)2n+2 4 + (!.)2n+.& 4 + ... -

-n
Fronl this
\ve

_ (!)2n _ (-} 4 -t

r
16

n _ (

I -

1-

n+ ~) 15

(.!.)2n 4
11

find the above value of R n Putting

the series S -= (

r.

=0, \ve find the sunl of


2508. S = I.

2506. 99; 999.

2507. 2; 3; 5.

Hint.

I I a'J~----1 2509.8::::1 when x>O, S=-I \vhen x<O; 8==0 when n n-tx == O. 2510. COllvcrge~ absolutel y for x> 1, diverges for x ~ 1. 2511. Converges absolutely for x > 1, converges conditionally for 0 < x ~ I, diverges for x ~ O. 2512. Converges absolutely for x> e, converges conditionally for l<x~c, diverges for x~1. 2513. -oo<x<oo. 2514. -oo<X<():). 2515. Convergls ahsolutely for x> 0, diverges for x~O. Solution. I) Ian I~

..;;;; ~x; and when x > 0 the series with general term ),; converges;

2)

)x ~ 1

for x ~ 0, and cos nx does 110t tend to zero as n ---+ 00, since from cos ,1X -~ 0 it would follow that cos 2nx -+- -1; thus, the necessary condition for convergence is violated when x ~ O. 2516. Converges absolutely when 2kn < x < < (2k + I) n (k == 0, I, 2, ... ); at the remaining points it diverges. 2517. Diverges everjwhere. 2518. Converges absolutely for x :1= O. 2519. x > I, x ~-l. 1 2 2520. x>3, x<l. 2521. x~l, x~-I. 2522. x~53' x<4"3' 2523.

x> I, x < -I.


of

2524. - I
~

< x <- ;, ; < x <


the series

I. Hint. For these values


converge. When

x, bol h the series

L xfl and
k=l

L 2x
k=l

1l k

I x I ;;;::= I

456

Answers
when
I 'x I ~ "2'

-and
2525.

the general term of the series does not tend to zero

-1<x<O,

O<x<l.
I
2533. -

2528. -l<x<l
2532. - I .2536. -4

2529.-Y2~X";;;;V2' 2530.-1<x~1.2531.-1<x<1
00

2526. -l<x<L I
00.

2527. -2~x<2.

2540. 2541. - I < x < I 2542. gence of the series for I x I ~ 1 is obvious (it is interesting, however, to note that the divergence of the series at the end-points of the interval of convergence x= I is detected not only with the aid of the necessary condition of convergence, but also by means of the d' Alembert test). When I x I < 1 we have

< x < 1. < x < 4. -3 ~ x < 3.

2537.

I -3

<x<
< x<

2534. x -= O. 2538. -2

I 3'

2535. - 00 < x < 00. < x < 2. 2539.-e < x <e. -I < x < I Solution. The diver-

lim n -+ :t>

l(n+l)IX~n+I)J
n! x n

1= lim
n

-+ '"

l(n+l)xnfn'~lim(n+I)lxln= lim n+~==O n -+ '" n -+ ~ ~

II

(this equality is readily obtained by means of l'Hospital's rule). ~ x ~ I Hint. USing the d' Alelnbert test, it is possible not only to find the interval of convergence, but also to investigate the convergence of the given series at the extremities of the interval of convergence. 2544. -I ~ x ~ 1. Hint. USIng the Cauchy test, it is rosslble not onl y to find the interval of convergence, but also to investigate the convergence of the given series at the extremities of the interval of convergence. 2~45. 2 < x ~ 8. 2546. -2~ x < 8. 2547. -2 < x < 4. 2548. 1 ~ x ~ 3 2549. -4 ~ x ~-2.
2543. -I

2550. x =
2558.

2551.

- 7< x< - 3 2559.

2552. 0 ~ x < 4. 2553.

-"4 < x <"4 .

13

2554. -e-3<x<e-3. 2555.

-3~x~-1
.
d'Iverges, . sInce

-2~x~O.

2556.2<x<4 2557.1

1--!.<x<I+J.
e I ( 1+n I--n --= lr eye

)n

Hint. For x=1

~
e

<x~3.

the

. series

n~C7J

l'1m

# 0

2560.

-2

< x<

2561. 1 < x~3 2562. 1 ~x < 5. 2563. 2~x~4. 2564. I z 1< 1 2fi6:>.' z 1< 1

2566.
2576. 2578.

I z-2t 1< 3 2567. I z 1<

V22568. z=O 2569.1 z 1< 00.2570.

I z 1< {

-In(l-x) (-I~x<l) 2577. In(l+x) (-I<x~l). 1 tx I i-x (I x f < I) 2579. arc tan x (f x I ~ 1). 2580. (x- 1)2 (I xl < I). l-x 2 2 x 2581. (1+x2)2(1xl<l) 2582. (l_x)I(lxl<I). 2583 (X_I)2(Ix l >I).

"2 111

2584.

1 ( arctanx-2"ln
the series x2587.

I I-X) +
l
x'

sum of
2586.

3 +5"-...

x x

Va (Ixl<l). 2585. n -6 - . Hint. Consider the


(see Problem
2579) for
X=

I V3.

3.

aX=l+I. xn~~na.

-oo<x<oo. 2588. sin (

X+~)=

= ~2

[l+X_~_~+~:~_ ... _H_l)n.:n ~+ ...].

Answers
2589.
n

451
x2

cos (x + a) = cos a- x sin a -

2f cos a + 3f sin a +'4f cos a -f- ...


2 2 2t 2S 4 25 x' x=2T-4f-l-m-

Xl

x;4

x Sln . r -l-nr a+ (n -t-21) nl +... ,-oo<x<00.2590.s1n

... -l-'(-l)"-'
x,2

2:. n - J x2n 2 )' + ... , - 0 0 <x< \ n .


+{_I)n-I

00.

2591

-2.22 +3.2 3-"


gating
2592.
~

x'

n.2 n + ....

xn

In(2+x)=ln2+--2
Hint. When investi-

-2<x~2.

the

relnainder,

use the theorem on integrating a power series


00

(X_W=-L(n+3)x".lx 1<1. 2593.

2x-3

x2 -4x+3

3x-5

=-~
~

n=o
00

( 1+3 2) x
n +1

n==o

n,

Jxl< 1. 2594. xe-1X=x+


(SJ

L(-I)n-12n-Jxn
(n-l)l
n=2
00

'
x%n t
1

< xa<

00. 2595. eX'::=: I -1- ~~, ' ~ n. n=1

2n

00

<

<

2596.

~(2n+I)' n=o

,~

00

(-oo<x<oo) 2597. I-t-'''' (_I)"2"x " . 2598. ~ \2n)1


n=1

I+~~
2

(-1)"(2x)2" (2n)'

-oo<x<oo.

2599.

2L(-ll
n=o

~..,

(n

+ 2) 32n.x21l+1
(2n+I)'
260).

11=1

(-oo<x<oo).

~ X 2n + 1 2600. L(-I)n gn + J

(-3<x<3).

1.3.5 x' -1- 2.4.627 1-

n=o

135 . . (2n -I) x1n 2.4.6 2ft 22n+1

+ ...
n

l-2 < x < 2)

2602.

~x~n+, 2~ 211 + I
n=o
00

(I

xI

<
x,"

I) 2603

. ~
n=1

~(-1)n+12n-l.n(
x
QCI

"2 < . < 2

1)

2604
2606.

x+L.<-l)n(n_l)n (lxl~I). 2605.


n=2

X2n+1

L(-J)"2n+1
n=o

('x,~).
(I.I~)

1 x' 13 x' 13.5 ... (211-1) X 2n + 1 x-f-"23+2.4S++ 2.4.6 . . 2n 2n+l+


S

1 x' I 3 X 11 13 5 . . (2n I) x + 1 2607. x- 2 3"+2.4S-+(-1) 2.4.6 ... 2n 2n+I+'" (lxl~I). f7J 24n-1 x2n go 1 2608. (_1)"1'\ (2n)1 (-oo<x<oo). 2609. 1 t(-l)"-lnnl x" n=1 n=2 00 1 2n + 3n-, (-oo<x<oo). 2610. 8--t- 3 nl X,n (-oo<x<oo).

2n

n=1

2x 2 25x' l)n-l 258 .. . (3tl-4) xn 2611.2-1-22.3.11-25.32.21+28.:33.31+ ... + (--23n-l.3~-I- t,


X

458
(-00 <X<oo).

Answers
2612.
2n - l )

(-2

<

<

2).

"26t3. 1+"4

3 L (I +(2n)1
C

x2n

<I x

I<
n

00).

2614.

x L 4n+1
Q)

4n

(.I xl
2616.

< Y2).

n=1

n=o

26tS.ln2+L(-I)n-l(I+2- n) :
n=1
X2n + 1

a:>

(-I

<

x..;;; I).

L (_I)n X
n=o

X (2n

+ I) (2n + I)!
Q)

( - 00

<x<

(0). 2617. x+ (_I)n (2n -r- 1) ti! (lxl<oo). n=l

x!n+1

2618. L(-W+l~1
n=l

(I xl".;;; I).

2619.
(lxl<I).

I s 1.3. + + 2.5 x +2 .9.2r x + ...


2

~2n-l) 4n+l + + 1.3.5... 2n (4n+l)n! x . . .

2620.

x+ 3

x3

2x +U-+ ...

2621. x- 3 +15-'

2x5

2622. e

4 (1-2"+6"-'.' X2 X )

.2623.1+2"+24+.

xl

5x4

2624. -

(~+ ~+ ~+ ... )

. 2625.

ing from the parametric equations pute the length of the elli pse and expand the expression obtained in a series of powers of 8. 2628. x'-2x2 -5x-2= -78+59 (x +~4)-14 (x+ 4)2 +(X+4)' (-00 <x< (0). 2629. f(x+h)=5x ' -4x2 -3x+2+ +(15x 2 -8x-3) h+ (15x-4) h2 +5h l (-00 <x<oo; -oo<h<oo). 1n 1 2630. (X n ) (0 <x";;;2). 2631.l:(-W(X-W (Q<x<2). n=l n=o

x+x + ~ x+ ... 2626. Hint. Proceed of the elli pse x = a cos eft Y = b sin CPt com
l

L(-WIX)

ct)

ct)

2632.

L (n+ 1) (x+ I)n


n=o

(-2
Q)

<x<

0).

2633.
2n

L (2- n-l_3- n n=o

(x+4)n

(-6<x<-2). 2634. L(-I)n(\-t!


2835. e-' [ I

(-2-3<x<-2+3).
2636. 2

+~ (x~12)n]
46.8 28
Ql

n=o

(I x 1<00).

+ x '}/ - ~
22n

(x 244 )1

-J- ~ {X-4)8 _ 135 {X-4)4 +


4.6 2'

+ (_I)n-l I 3.5 ... (2n-3) (x-4)n +


(

(0~x~8).

2637.

n=l

~(_l)n

468 .. . 2n x-~ yn-l


(2n-=-l)!

(Ixl <

00).

2638.
1

2+

+ \1 n.fd (-I)
(0 < x

4n 1 n -

yn-l (2n-l)!

X-T

1I x 1< 00). 2639. -2 ~

i,

2n

+1

(I-X)2n+l
1 +x

<

ClO).

Answers
Hint. Make the substitution

4590

~640.

+}( I :-x +~:~C ~x + ... + ~:~:~:: :~~:=~~( I :-x +'" ... ( - ; ,,;;;;; x < <Xl ) . 2641. I R I < ~ < ~. 2642. I R I < 1\ . 2643. ~ :::::: I I (~r 1'3(~r. , ::::="2+2 Hint. To prove that the error does not
I :-x
-3-+~-5-:::::0.523.

+=

t and expand In x in powers of t.

r
I

r
7

exceed 0.001, It is necessary to evaluate the reillainder by nlcans of a geo.. metric progression that exceeds this renlainder. 2644. Two ternlS, th3t is,
I -

~.

2645. Two terms, i. e., x-if. 2646. Eight terms, i. e . I

+ I: ~I
n=1

2647. 99; 999. 2648. 1.92 2649. 4.8 I R 1< 0.005. 2650. 2.087.2651. J x 1< 0.69;
x

1< 0.39; / x / <


0.~68.
00

0.22.2652./ x 1<0 39;

I x 1< 0
0 621

18 2653. }-23 ';'3!:::::: 0.4931.

2654.

2655.

0.608
(211)!

2656.

2657.

0.2505

2658.

0.026.

2659.1-t-~(-1)" x-y)
~
11=1

(2n

(-oo<x<oo; -co<y<oo). (-co<x<oo; -<Xl<y<co).

2660.

~
11=1

~(_I)II(X_y)3n_(x+y)2n
2(211)'
00 ( 2 X

2661. ~(_l)n-I
~
11=1
oJ)

-!I ----(2n-l)!

I-

2)'2n-1

(-oo<x<oo;

-oo<y<oo).

2662.1-1-2

tt=1

~ (y-x)n;lx-YI<1
(X)

. I-x+y 2 Hint. I =-1+1 ( . Use -J-x-y - y-x)


n

a geometric progression 2663. - "':""').'


~
tt=1

+ 'I 11

(-1
JJ

~ x < 1;
X 2n + 1

-1

~y <

I).

Hin:. I-x-y+xy=(l-x) (i-g). 2664.


-1

I: (-1)/1
n=o

211:1

2n+l

(-I~";;;;;I;

~y ~ 1). Hint. (lrc tan


f(x-t-h,

IX+ Y =arc tan x+arc tan y (for


-~\y

I x I ~ 1, I y J ~ 1).

2665.

y-t-k) =ax2+2bxy+cy2_r-2 (ax-j-by) h+2(bx+cy) k+ah 2


t1'

1-2bh+ck 2 2666. {(1+", 2-j-k)-f(1, 2):=9h-21k+31z 2 -t-3Izk-12k 2 +h!-2kl. 2667. 1+~,,[(x-2)+(Y+2)]


n=l

,,00
I

It'

.2668.1-L~(-1)n

[ x + ( y - ~) 1 zn
(2n)!

i.J

n=1

2669.

1+x+2

x2 _y!

-,-+

x8 -3xy2

3'

+ ...
+

2670.

-t- X -I-XY+"2
S ( n) =

xlJ +
(1

2671. c1 + c2 _ 2 (CI-C2~ ,..., sin (2/1 + l) x; S (0) = c\ 2 11: ~ 21l 1 n=o

-L c2 ; 21

+.~ C2
-

-460

Answers

'2672.

b-an_2lb-a)~~COS(2n+l)x+(a+b
4

n=o
CD

(2n

+ 1)2

)~
n=1

~(_l)n-I sinnx.
n

.s ( n)= -2- n.

b-a

2673.

'3 + 4

nl

cos nx 2 L (-1 )nfi"l ; S ( n) = nl. 2674. 1t sin h anx


n=1

1 ~ ( - 1)11 ] 2 sin axt X [ 2"a+ ~a2+nl(aCosnx-nsinnx) ; S(n)=coshan. 2675. - n - X


R=I

X ~ (_l)n n n: if a is nonintegral; sin ax if a is an integer; S ( n) =0. ~ a-n


R=1

ClIO

:1"

2 sin axt [ 1 CD a Cf'S nx] . .. . 2676. - 2- + ~ (_l)n 2- - I If a IS nonlntegral; cos ax If a Is an n

~ n=1

a-n

integer; S ( xt) =cos an.

2677. 2 sinh axt


no

~ n=1

CD

(_l)n-l n

~i+"n~; S'l n) =0.


n

2678.

2Si"ha:rr[2~+~(_Wa~('+sn:]; xt a..... a n
n=1
CD

S(:n:)=coshan.2679.
2~
~

~ n=l

~slnnx. n
1n
1

~.., ~t'1 (211 - 1) 2680. ~ 2rt- 1


n=1

x . ) ~.
,a

4'

b) ~.

3' c
xt
l

V3

xt

2681

a)

~ (- ) - X
where
2

n=1

stnnx. n'

b)

~_.! ~c0s(2n-l)x.
2
xt

t2a-l)1

'

"8.

2682. a)
xt
2

~bnsinnx,
n=1
011

n=1

blk 2)

8 xt =2k-l-n(2k-l)' and b2k =-/i; b)


2683.
'I)

211

~ " cos nx I) n 3+ 4 .L(-1)"11"2; 6'


n=1

~121 .

~;..
n

L...

[1-( _.I)n ea1t] n Jstn nx.

a +n2
rt>

b)
nxt

n=1

+ 11 ~
n=1
(X)

rt>[( 2 ~ ~~

Inan -) e - I)' ens nx a +n


l

2684

2IJI-COS2 . ) _
xt

SIn

nx,

b)

~+ 2

+-

2 \.1 sin 2 cos nx. 2685. 11...... n

nn

n=1

'J --

n=1

_~ ,~ crs 2t2'1-1) x 2606.


11

(2n-l)"

.J

"(~ k 1 1...bnSlnnx, where b2k =(-1) -12k,b2k+l=


n=1

n=1

Answers

461

2689.
2691.

~ ({ +i: SI::h cos nx). 2690. ~ [~+ i: cl~:hr cos nx].


1-~+2 ~ (-1)"-1 COlnx. 2 ~ n 1
l
n~1

n-I

2892.

..!. 1'

[i
2

~ n-I

~ (_l)n-I ens 21tX] 4n - l .


l

n=1

2694.
n

Solution.

I)

alII

= ~ Sf (x) cos 2nx dx = ~


0

Sf (x) cos 2nx dx +


0

+~S f (x) cos 2nx dx.


11:

If we make the substitution t - ; -x in the first


in the second, then, taking advantage of the assumed

Integ:al and t = x - ; identity

f( ;

+t ) = n

f(~ - t ),
3t

It wl\l readily be seen


n

that

0.11 =0

(n = 0, I,", ... );
2)
bill

=;

Sf (x) sin 2nx dx = ~ Sf (x) sin 2nx dx +- ; Sf (x) sin 2nx dx.
o
0

n
2

The same substitution as in Case (I), with account taken of the assumed
Identity 2695.
2

f (~
n2

+- t
~
11=0

) = f (~- t ) leads to the equalities


2696. 1_
(2/1
~-

bZII

= 0 (n = l, 2, ... ).

~ _ i. ~ cos (2n +- 1) 1lX


1)2

~ ~~
;t......

sin 2nnx . n

2697.

sinh 1

Ii., [T f-2 l.J


1l=1

Icos n;x -lttl SIIlIl;X]


(-1)"
IlJ'tX

n=l

l2+ n2Jt2

2698.

nL

10 ~
n=1

00

(_I)n

SlI1 - ' - 1. fl nx S1Il -1-

2699. a)
l

nL
n;.1

""

4' 2(
SIn

21~~1 n~:

1)

b)

2700

a) n

21

lJ
~
11=1
a)

n=l
Il'J (2'1 - I) nx oc 41 ~ cos - - / - - .. fl \' .2701.a) bn Sll17)' (2n - 1)2 .... '"

(_1)n+l _ _ ; b) - - 11 2 nl

where

blk +- 1

=n
cos -

8 [

2k+I-\ 2k 1- 1)'

n2

11=1

b2k =-/i;

4n

b)
b)

4 n::!

-S--

nx

00.

-16 ,,( -1)"-1 _ _ 2 . 2702. a) 2


n=1
ctJ

I.J

~ n2 1"J

(2'1 1- 1) 3lX sin - - - : - - (_I)n ')


{2.1

n=o

+ 1)2

_ i. ~
2

n ~

n=o

cos (2n + 1) fiX 2703, (2n+ 1)2

462

Answers

Chapter IX
2704. Yes. 2705. No. 2706. Yes. 2707. Yes. 2708. Yes. 2709. a) Yes; b) no. 2710. Yes. 2714. y-xy'=O. 2715. xy'-2y=0. 2716. y-2xy'=0. 2717. xdx+ydy=O. 2718. y'=y. 2719. 3y2_X!=2xyy'. 2720. xyy'(xy2+1)=1.

2721. y=-xy' In~. 2722. 2xy"+y'=O. 2723. y"-y'-2y=O. 2724. y"+4y=O. Y 2725. y"-2y'+y=O. 2726. y"=O. 2727. y"'=O. 2728. (l+y'2)y"'-3y'y"2-=O. 2729. y2_ X 2=25. 2730. y=xe2X 2731. y==-cosx. 2732. y= 1 X X 2X =(f(-5e- +ge -4e ). 2738.2.593 (exact value y=e). 2739.4.780 [exact
value y =3 (e-l)]. 2740. 0.946 (exact value y= 1). 2741. 1.826 (exact value

Y=

,ry 3)

2742. cottl/=tantx+C. 2743. .

~ x=y__ ; y=O. 1 + 1/2

2744.

X2 +y2.=

= In Cxt. 2745.

y =a+ I ~ax' 2746. tan Y =C (I-eX)I; x=O. 2747. y = C sinx.


2749.

2748. 2e 2

IJ

= Ye (l-t-e x ).

2 1 +y2=-1- 2

-x

2750.

y= I.

2751.

arctan(x+y)=x+C.
I

2752. 8x+2y+l=2tan(4x-I-C). 2753. x+2y+ +3InI2x+3y-7J=C. 2754. 5x+lOy+C=3InllOx-5y-t-61. 2755. Q=

C
- cos (J>
x

or y2=2Cx+C2. 2756. InQ=-2-.--1nlcosq>I+C or Inlxlcos" <p


x '

y22 =C. 2757. Straight line y = Cx or hyperbola y=... Hint. -2

The seg-

ment of the tangent is equal to Yyt+(f.-f.2758.yt-xt=C.2759.y=


x
x

=Ce a . 2760. y2~2px. 2761. y=ax". Hint. By hypothesis

~ xydx o 3 -x--=4 x.
) ydx
o

Differentiating twice with respect to x, we get a differential equation. 2762. y2 = } x.


..r 2-Y4-x 2 2763. y == f 4-x 2 21n . 2764. Pencil of lines y = kx. 2765. Fax 2 roily of similar ellipses 2X'-t- y 2=C 2766. Family of hyperbolas X 2 _ y 2::=:.C.

2767. Family of circles x 2 +(y_b)2=b 2 2768. y=x In


x

s.. 2769. y=.E._!-2 . x x


y=
2

2770._x=Ce y.

,ix V y+ln1yl=C.

2771.

(X-C)2_ y2=C2;

(x-2)!_y2=4;

x.

2772.

2773.

y=2 X2 -2C;

x=O.

2774.

(X +y2)3(X+y)IC.

2775. y=XY1-: ;c. 2776. (;c+y-l)'=C(x-y+3). 2777.3x+y+2X

Xln 1x+u-1J =C. 2778. In 14x +8y +51 + 8y-4x =C. 2779. xl = 1-2y.

Answers

463

2780. Paraboloid of revolution. Solution. By virtue of symmetry Ule sought-

for mirror is a surface of revolution. The coordinate origin is located in the source of light; the x-axis is the direction of the pencil of rays. If a tangent at any point M (x, y) of the curve, generated by the desired surface bein~ cut by the xy-plane, forms with the x-axis an angle cp, and the segment connecting the origin with the point M (x, y) forms an angle a, then ta-n a = tan 2cp = I 2t~n ~ . But tan a. =JL ; tan cp = y'. The desired diiTerential equation is - an q> x y-yy'!= 2xy' and its solution is y2:= 2Cx +C2. The plane section is a parabola. The desired surface is a paraboloid of revolution. 2781. (X-y)2_Cy=O. 2782. xl =C(2y+C). 2783. (2 y 2_ X ')I.=CX 2. Hint. Use the fact that the area
x

is equal to

~y
tl

dx.

2784. y =Cx-x In I x

I.

278li.

y=Cx +x l .

2786.

y=

,-::i-X4+~I' 2787.
respect
I
t~x

x VI+yl+cosy=C. Hint. The equation is linear with

I eX ab-e fJ and d-. 2788. X=C y 2 _ - . 2789. y=--f---. y y x x

dx

=-2 (x r l - x 2 +

.. r -

. arc Sill x)

yl+X
1

2790.

y=

-1- .

-x
y

2791. y = 2795.

x . 2792. II (x 2 -t- ex) = 1. cos x 2797.


XlJ=

2793. y2=xln-. 2794. x 2 :=: +C

2.

y'(3+CeC09X )=x

~Cy2+a2. 2q98. y2+ X +ay==O. 2799. x=y

a y In - . 2800. -

b += I. y

2801.

,\2_t- y 2-Cy+a 2=O. 2802.

x!

+xy+y2=C. 2803.
X

Xl "3+xy2+X2=C.
X 2_

2804.

"4-2"X 2y 2_1-2x+ 3 =C.


?807. -2

x4

yS

2805.

Y 2+yl_2 arc tan-:t=C. 2806.

y 2=Cy.
-lnx+

x2

+ ye l/=2
-

II" X Xl 2808. Inlxl-~=C. 2809. --1-- =C. 2810. x y 2


(.t C 2 + '

1 y

I + '2 y2 :--::: C. 2811. (x Sill Y + y cos y -sin y) c"(::::L C. 2812. X (x + C2- 2Cy) = 0; singular integral :.;2_ y2= O. 2813.
2

1- 2Cy) X
Inte~ral

Qeneral

(y

+ C)I =x'; there is

X ( x-

RO

singular integral. 2814. General integral (

f-

y+C ) X

+ C ) =0; there is no singular integral. 2815. General integral

UI+C1=-=2Cx; singular integral X1_yl=0. 2816.

y=~cosx ~-3Sil1X. 2817.

x=sinp+lnp, { Y = P sin 191- cos P + p +


Sin~ular

c.

2818

{x=eP+peP+c, 2819. y = p2eP .

JX=2P--p~ +C.
\

y=p'+21np.
x

solution:
__

y=O.
p

2820.

4Y=X +pl.

2821. In yp2+y2+arctan-=C, x=ln-- . Singular solution: y=e". y 2p

y2+ p 2

Inlp-x/=C+--. p-x

464
2822.
1 C I 2. Y=2 x +C
t

Answers
y::;g

2x.

2824.
1

x=Ce-P-2p+2, { y=C (1 +p) e- P - pZ+2.


-1

x=- (Cp 2 -p), x= In I p I-arc sin p + C, 2825 3 I Hint. The differential 2823. { y=p+ VI-pl. 1 _ y="6 (2Cp I p2).

equation from which x is defined as a function of p IS honlogeneous. 2826. x2 y=-CX+C2; Y=-"4. 2827. y=Cx+C; no singular solution 2828. y=--Cx+
2829. y = Cx + ~ ; yl = 4x. 2830. xy = C 2831. A ci rcle and the family of its tangents. 2832. The astroid x 2 /1+ y2JI=a 2 ;1. 2833. a) Honlogeneous, y=xu; b) linear in x; x=uv: c) linear in y; y=uv; d) Bernoulli's equation; y= uv; e) with variables separable; f) Clairaut's equation; reduce to y =xy' y g'l; g) Lagrange's equation; differentiate with respect to x; h) Bernoulli's equation; y = uv; i) leads to eq uatIoll with variable~ separa ble; u =x+ y; j) Lagrange's equation: differentiate with respect to x; k) Bernoulli's equation in x; x= uv; 1) exact differential equation; m) linear; y = uv;

+ y I + CI;

Xl + yl

= I.

n) Bernoulli's equation; y=uv. 2834. a) sin 1t.

= -In I x, + C;
1

b) x= y.eCY+1.
2838.

2835. xl+yC=Cyl. 2836. y= xl+C. 2837. xy(C-

1n 1 x)= 1.

y=

=Cx+C In C; singular solution, y=t-(x+l). 2839. y=Cx+ V -aC; singular a I x'-l I 1 solution, Y=4i. 2840. 3y+ln (y+l)'=C. 2841. 2etX-eY-arc tany-

-'2 1n (1+y')=C.
slny

2842. y=xI(l+Ce X

).

2843. x=yl(C-e-Y ).

2844. y=

=C,-slnx+Sinx:-l. 2845. y=ax+CVl-xl 2846. y=X~l (x+1nlxl+C).


2847. x=Ce

-2a (1 + sin y). 2848.

2 +3x+y+ In [(X_3)IO I y-11 3 ] = c.


2 -2

x2

2849. 2arc tan -2-=lnCx. 2850. x 2 .=1--+Ce y. 2851. x'~CeY-y-2

2852.

V~ + In I x I =C.

y-l x

2853. y=x arc sin (Cx). 2854. y'=Ce- 1X +

~ sin x+
+C):.:O.

+:cosx. 2855. xy=C(y-I). 2856. x=CeY - - } (siny+cosy). 2857. py __


333 c::C(p-l). 2858. x4 =CecY -y'-4 yl-SY-32
-x Yx +y2 __ =C. 2861.xeY l

2859. (xy+C) (x 2y

2860.

y2=C. 2862.

VI +p2 1 C x=t+.In(p+ ..~ _ _ 2p 2p ( + f 1+p2), Y = 2px + V 1 + pl.

2863.IJ=xeCx 2864. 2eX _1t=Cy l. 2865. In 1Y +21+2arctan;+:=C.2868.

Answers
~~
, 2

465
8 286.

-t- Ce - 2
4(

1 2 = 0. +-x
I

2867 . x 2 'Y= C'I ew

x x+-=C. y

2869.

y=
2872.

C-x" . a 2 In(x+Ya 2 +x 2 )+C 2 1)3,2.2870.y==CS1l1X-a.2871.Y= x X+ y a2 +x2

(y -Cx) (y2_x 2

+C):.= 0.

2873. Y=-= Cx

I + C2'

Y==-

2" Vi \ 2x 2

2874.

x'

+ x y2

2879. y=O. 2880. Y=~(SiI1X+COSX). 2881. Y={-(2x 2 +2x+I). 2882. y= =_-::e- x -i-2x-2. 2883. a) y=x; b) y=Cx, where C is arbitrary; the poinf (0,0) is a siugular pOint of the differential equation. 2884. a) y2==X; h) y2== 2px; (0,0) is a singular pOint. 2885. a) (x -C)2 + y! = C!; b) no sol ution; c) x 2 + y2 =--= x~
x

_y2x _ y3===C. 2875. p2+4y2:-=Cy 3. 2876. y=-=x-l. 2877. y-=-=x. 2878. y=2.

(0,0) is a singular point. 2886. y -=-e". 2887. y=-=(Y2a YX)2. 2888. y2=-= --::l-e- x . 2889. r=Cea~. Hint. Pass to polar coordinates. 2890. 3y2_2~=O 2891. r--kcp 2892. x 2 -1-(y-b)2=.-b 2. 2893. y2--l-16x=O. 2894. Hyperbola 1 (eX -)- e- x ). Hint. Use t h~ fact !/ _x 2 -~ "C or circle x 2 1- y2 --- C2 2895. Y = 2
x
%

Ilut the area


2

IS

equal to

Sydx
u

and

the arc leni:tll, to ~ I


~

l-ry'2

1r..

2896. X~:.!.-+ CII.


II'

2897. 1/2_4C (C-;-a-x). 2898. Hint. Use the fact th:lt th~

re"ultant of the force oi ~ra\'lt~ and the centrIfugal force is nornlJI to thl' surface. '1 akll1~ the y-axl~ as the :tXIS of rotation and denotlIlI1: by (J) the ,11lgular velocity -of rotatll)l1, \\ e ~t't for the plane aXIal cross-section of the deSired surface the differential equation g ~~! ={t)~x. 2899. p -=e- IJ .I)QIl1671l. Hint. The pr<:s..
dr

sure at each level of a vertical CUIUlll11 of air IBay be consHlcrcd HS due solely to the pre~"1I1C uf the upper-lyIng layers Usc the la\v of Boyh~-.J\LI!" utte, ~c cordll1~ to \\ 11Ich the dCllsIty IS proportional to the pressure. The sought-for
dtflefl\J1tlal equation
IS

dp -- - kp dll. 2900. s -:.: 2"

kl~.

Hint. EqUation ds-=

_'k'WI-~xdx.

2901. S=(Pl-{W)kl. 2902. T=a-j-(To-a)e- kl 2903. In

one hour. 2904. w'-' 100 (

y
I.

rpm. 2905. 4 2% of Ihe initial quantity Qo

will decay in 100 years. Hint. Equation

dd~ =kQ.
1t

Q -= Q. (

)00.
(k

2906.

t =:::
HInt.

=::: 35.2 sec. Hint. Equation


dQ =-kQ dll. Q = Qo

1t

(h'-2h) dh =
2908. v ~

(~r rJ dt.

2907. Id24'
-+- 00

I)!!' (2"

.V . / grn T as t
2 ; U=

IS

a propor-

tionality factor). Hint. Equation

m~~ =mg-ku

g;; tanh

(t V~ ).

2909. 18.1 kg. Hint. Equation ~~ = k (


16-1900

~ - 3~O)

2910. i =

R' :L2W2 [(R sin wt-

466
- Lro cos rot) + Lroe
-- t
L ].

Answers
R

di Hint. Equation Ri + L dt = E sin rot.

2911.

Y ==

=xlnlxl+C l x+C 2. 2912.

1+Cly2=(C2+~;r.
2918.

2913. y=ln/e

2x +C I

!-

-x+C 2. 2914. y=C 1 +C 2 In/xl. 2915. y=Clec~x 2916.y=YCtx-l-C~


2917.
2919.

y=(l+C~)lnlx+Cll-C1X+C2'

(x-C I )

=aln\sinY~C21
2923. Y =

y=

(In I x 1)2+ Cl ln Ixl +C2 2920.X=~ In II -:C "1 C2 ; Y= C. 2921. Y =


I

Clee," +

~2'

2922. Y =

[x V C~ _x + C~ arc Sin~] + C2
2

c=(C 1ex +I)x+C 2

x -+J e 2924. y==(C1x-C:)e Ct +C 2 ; y=2X2+C

(singular salu2926.
~

tion). 2925. y=C,x(x-C 1 )+C 2 ; Y= ~

+C

(singular solution).
2

Y=

=12+"2 + C1x In Ix I +C 2x+C.


=xl +3x.
x

2927. Y=

sin (C, x) + C x-t- Ca.

2928. Y---=

1 2929. Y-=="2 (x 2
2933.

+ I).

2930. y=x+ 1. 2931. y=Cx2. 2932. y--=C.X

+ C2e y=C. X 1 l-C~x;


'2935.
x 2 _1 2 (e 2 -1)

2 1 In \ y+C !I x==C 1 +ln y-C y-t-C \ 2934. X==Cl-~ 2 2 X=C ty2+ y Iny+C 2 2936. 2y 2_4x 2=:: 1. 2937. !J=-x-~-l. 2938. y:-=.

e2 -1

-4- 1n I xl or y=2 (e2+ 1)


2941. Y =-= 2e

I-x!

+ 4 - 1n I x I.
2

e2 +1
x ==

2939.
I .

!I -

2 x

2940. y == 2 x2
2944.

1 3 x
2942.

-"2

(y -~. 2)

2943.
8

Y - eX.

y2="e_I+I_e

e- x

2945.

Y-== -:~-x-

JI2

+-1.

Answers
xcosh x-t-C +C 2, where H

461

IS

a constant hOrizontal tension, and

The differential equation


2

::~=1i

VI -/- (:~r
gt
2

!!..=a. q

Hint_

2965. Equation of motion,


2966.

ds dt 2 =g(sina-l-tcosa). La\v of nl0tlon,


Xincosh(t

s~2(sin(L-l-tcosU)

s=/i

yg ~).Hint.EqUatiOnOfmotJon.m:;~=mg-k(~r.
g
t

2967. In

6.45 seconds. Hint. Equation of nlotlon, 300 dd:!~ =-10 v. 2968. a) No, b) yes.

c)

d) yes, e) no, f) no, g) no, 11) yes 2969. a) t/, + y = 0; b) y" -2y' + y -= 0; c) x 2 y"-2xy' +2y=O, d) y"'-3y"-J-4t/-2y-=-=O 2970. 1I:::.=3x-5x 2 +2x 3 2971. Y-=
Yl~S,

=~(CISlnx-J-C2COSX).
x

Hint. Use Ule substitution Y=YIU, 2972. y=C1x+


x

+C

x2 B In x. 2973. Y = A -t- Bx 2 +Xl. 2974. y == -3 -f- Ax + -. Hint. Particular so-

lutlons~

the honlof1:cneolls equation

YI~=X' .tI2-=--=~' x
x

By the 1l1cthod of the

variation of paranlcters we find: C I :-:: ~

+ A,

C2 =

x:s 6 -t- B

2975.

y= A
2X

-1-C2e3.\: 2977. !I:....:Cle-3~ j-C2,e 3 ';. 2978. y--=CI-1-C2e~ 2979. y=C.Losx+C 2 sinx. 2980. !J --=. eX (C I cos x -1- C 2 Sill x) 2981. !I =-= e- 2X (C l co~ 3t -1- C:! 5111 3x) 2982. Y =

-I B Sl1l x., C cos x -t-In/ sec x + tan x 1-1- "in x Inl cos \" I-x cos x. 2976. Y == C 1 e -1- (CI-!-C~x)e-x. 2983. y~-:..e2.\ (Cle t J -; -f- C2e-X~'2).
2984.

If

Il> 0,
Slll

Y-=

=-

Cle J Ii

C 2e-d -1/;

If

<

O.

y-~CI cos

V -llX

-t- C 2

V -kx.
l 11
x

x --

298t).y=e

.' 5" x -Jr-:- x --t 2 (Cle 2 -1-C 2e 2 ) 2986.y=-e 6

(~rJ' 11

Clcos~x-t-C2Sill-6-X

r- )

. 2987. !I =- 4e x -1- elX". 298R.!I ~ e-". 2989. Y:=-;: sin 2x. 2990. y = 1. 2991. y

== a cosha-

2992.

II -== 0 2993. !I =- C sin rrx 2994. a) xe (Ax:! + Bx -t- C); b) A C()~ ~~ t-1- n "in 2x; c) A co~ ~\" -t-1J Sill 2\' +- C\"2e2X; d) eX (A cos x-I- B "in \), c) e\:;< ).' (A \:! -1- Bt -t- C) 1- xe:!'; (!)r.,- E); f) xe X [(Ax 2 -i- B\ + C) cos 2x+ (Dx 2 + Ex t-F) X
2X

XSln2x)
,-

2995. II=(C I -!

,. x 2C 2 s1n
0

Y3)

c.x)eX-/-~(2x2+4X+3).
lj---

2996.
l

Y=e:(clcosX~3+

'~ 2 2997 +x 3 -1- .\.

__ (C I-f-:!X C ,) e -x -19 L 1

,2.'\:

2998. 11_ Cle x 1- C 2e7X

+2
I

2999. Y ~ C .ex

+ C .e-\ + xe~.
eX

3000. y

= C I cos x+
Y=
X _ 2

+C2SiIlX-l-ixSinx. 3001.

Y=C,l!x+C2e-2X-i(:~S1l12X+COS2X). 3002.
2X

=
_
X

C1e:!''\ -t-C 2e- 3X

Xl) + x l 10 - 25 e
sin 2x) -/-

3003. y = (C 1 -J- C 2x)

1 x + 2" cos x + 4" e

~e-x
lC, cos 2x

3004.

y=c,+c2e-x+~x+~(2COS2x-Sin2X).

3005.

y=eXx.

+C

TeX

sin 2x.

3006.

-= cos 2x +

(sin x -/- sill 2x).

16*

468

Answers A
2) x= C1 cos rot + CI sin oot-

-8007. 1) x== C. cos rot + C. sin rot + -.--. sin pt; ro -p

A x Xl x - 200 tcos rot. 3008. y= C1e'x + Cae4X-xe4X. 3009. y =C 1 +Cae"x + 4-4-"6 1 5 -8010. y:::eX(C 1 +C 2X+X 2). 3011. y=Cl+C2e2X+2xe2X-2"x. 3012. y:::::l =Cle-IX+czeU'_~ eX+ ~ (3cos2x+sln2x). 3013. y=C1+Cze-X+eX +
5 + 2XI-5x.
1 xe- x + 4 eX.

3014.y=C 1+C.ex -3xex -x-xl . 3015.y= ( C1+C1 X+ 1 x1)X


3016.
II = (C 1 cos 3x + C2 sin 3x) eX
x + 37 (sin 3x + 6 cos 3x) + e~}

3017.
Xl

y=(Cl+caX+x~elx+xtl.
x
3019.

3018. y=C1+CiI'X-f6(COsx+3sinX)-

- 6-9

x3 x 2 X X _X (4x+ 1)- 6- 4+"4 3020. II=C 1e +C 2e e2X - x sin x-cos x. 3021. y = e1e- 2X + C~e2X -20 (sin 2x 2 cos 2x). 3022. Y ==

Y=g e

1 2X

c:

C1 cos 2x+ Casin 2x -

f (3 sin 2x+2 COS2X)++.

+ C 1 sin x-2x cos x). 3024. y= C1e x + Cze-x + +C a sin3x+

+
x

3023.

y=eX t~l cos x +

(xa-x)eX. 3025.y =C1cos3x+

xSinx-~cosx+51 4 (3x-l)e lX 3026. y=C1e,x+cae-x+{x


3029.

)( (2-3x) +

1 3 3 16 (2x 2-x)e3X 3027.y=Cl+C2e2X-2xex-4x-4"xl 3028.y:::::a

-=(CI+Clx+~)eIX.
)( (2xl + 3x) eX.
+
8030.

Y=Cle-,x+cr-i(2xl+x)e-'x+~x
4
cosx+"4slnx-acos3x+
Xl

II=C1cosx+C 2 sinx+

~ sin 3x.

Hint. Transform the product of cosines to the sum of cosines.

8031. Y= C1e- x 'VI; +Cr 'VI +xeX sin x+e x cos x. 3032. y=C 1 cos x+C I slnx+ +COSXln/cot
3034. y=(C 1 +C 2 X) In I x I. 3035. 11= (C 1 +C 2x) e- x +xe- x In I x ,. 8036. y=C 1 cosx+C 2 sinx+xsinx+cosxlnlcosxl 3037. y=-=C 1 cosx+ + Clsin x-xcos x+sin x'VIn I sin x I. 3038. a) y=C1ex +Cst- X+ (r +e-X)x yxarc tanr; b) y=C 1ex I +C~-x I +exi . 3040. Equation of motion,

(-i-+~ )/.

3033.

y=C1COSx+Caslnx+slnx.tnltan;

eX +xex

I.

i-(~a:')=2-k(X+2);

(k=l);

T=2n

vi

sec.

3041.

x-

-=

2gsin30t-60 Visin Vit . g-900 cm. Hint. If x Is reckoned from the position of

x"=4-k (X.+X-II-I), where X o Is the distance of the point of rest of the load from the Initial point of suspension of the spring. 4 dlx I Is the length of the spring af rest; therefore, k (xo-l) = 4, hence, - dt l ~ dx g ---k(x-y), where ~:a:4, g=981 cm/secl. 8042. mdil=k(b-x)-k(b+x)

rest of the load, then

Answers
and

469

x=ccos (t
(e
lUt

V2~). 3043.6 ::~=gs; t=


b) r = ;~
(elllf _e- mt )

V;

In (6+

Yi5). 3044.a)r=

= ~
is

e- wf );

Hint. The differential equation of motion


3046.

:;~

=ro 2r.

3045.

y=C.+C~x+Cael2~.

y=C.+C~-x+Carr.

X( 3047. y=C.e-X+el c 2 cos-+x+C,sin


3051. y::= (C I + C 2 x) cos 2x
I

y--

y;-3 )
2 x

3048. y=-=Cl+C2X+CaexV;: -f-C 4 e- x lr 2 - 3049. y-=-e x tCI1-C2X+Cax!). 3050. y =-- eX (C I cos x -1- C 2 sin x) e- x (C. cos x -f- Col sin x) 3052. y=--CI-rC~e-x

-1- e~

(caCOSTX1-C4SlIl-2-x. yJ . Y3 )
+-

+ (Ca-l- C

x) sin 2x

3053. y = (C I -t- C 2 x) e-'Y: --1- (C a -1- C4~) eX. 3054. y === Cle QX -t- C2e-a:~ -1- Ca ("('S at Col sin a~ 1ra 1 3055. y =- (C I 1- C 2x) e t -t- (C a -I C 4 x) e- 1";- t'

+ Cs cO~J'K -1- Col sin a~.

3060. y=C. +C 2 x 1- ( C, -I- C~X+~) eX,


3061 y==C I -l-C 2 X_II 3062. y==Clex-!-e

3056. Y = CI C 2x 3057. U _::.- CI 1- C 2x -1- (C a C4 t) e- x 3058. y:-::: (C I -f- C2 x) cos x -t- (C a + G\x) sin x. 3059. y -- e-o,; (C I -t- C2 "< -!- ... C'l"(n-I).

+ +

12r:!_L:3X3---l_~ r "I 2'\4_L~.\5_; I 20


I

(C S -l-C t)c'Y: I -l
X

-~
2

3 (rtf C cOS-i-X-1 Ca<;':lT


}'F
2

-x 3 -5.

3063. y=C.+C 2 X+C aX 2 -t 3064. y =c C,e- x -I- C. -1- CaX

c~e-'>;+1~88(4CnS'lx-sin4X)

-I-{ x
J

+Xl

-I-

I~ \:~ -I- e~ ( {X-~) .

'C ' n x --,I .3065 . Y:- - C Ie -x + C 2 C( IS x -t 3 "1 e~

3066. Y =.3067.

x - ~ ~3 ) . "4 C I -t- C2 cos x 1- C3 sin x -1 sec x -1- cos x In I cos t 1- tan x sin x +x sin .'(.
(

( COS--X1--sin-x V"3 1 }/3) -1-x-2 y=-e-X+-e - ~ 2


2

Y3

3068. Y :-- (C 1-t- C2 In

1 x) . - . 3069. Y =-= CIXS


X

C
_2

3070. !I =-= C J cos (21n x) -{-- C 2 sin (2111 x). 3071. y:=.: C IX -1- C 2 X 2 -1- Cax s . 3072. Y ==- CI -1- C2
3073. y

(:~x

-1- 2) - 4/3

~ CIX~ +- C x

3074. Y == C 1 cos (In x) + C 2 sin (In x).

3075.

y=C.XI+C~2

-I-{ x.
2

3076. 3078.

y=(x+ I)" IC. -I-C 2 1n (x -I-

1)1 tex -I- I)'.

3077. y=x(lnx+ln x).

3079. y =e- x (C. cos x + C2 sin x),


3080. Y= (C 1 -C 2 -C.x)e2X ,

y=C1COSx+C 2 sinx, z=C 2 cnsx-C t sinx. x Z= ~ e- [(C:-2C.) cos x- (C -I- 2C ) sin xl.
I
2
2X

z=(C.x+C 2 )e-

470

Answers

t -~ ( C CosTt+Casln-3081. x=Cle +e 2 2

ya

. ya- t ) ,
C2
3 Y3-I-C y~r 2 SID -2-

C t+ -{- (C s Y = Ie e
Z -

Y3-C Y3 t 2 2 cos -22

t) ,
.

3082.

s ,. 3 t) t+ e--~(-Cs Va-c :3" t+ C2 Y3-C 2 cos 2 2 n 2 t t 2t t 2t x= Cle- + C e , Y =-=Cae- + C e , z = -(C + Cal e- + C2e~t.

-C

Ie

81

3083. y=c t +C 2eU

3084. y = C I C2x+ 2 sin x, z = -2C I -C 2 (2x + I) -3 sin x-2 cos x. 3085. y=(C 2 -2C l -2C 2 x) e- x -6x+ 14, z==(C 1 +C 2 x) e- x +5x-9; CI =9, C2 ==4, y = 14 (1-e- X )-2x (3 + 4e- X ), Z == -9 (1 _e- X ) + x (5 + 4e- X ). 3086. x = lOe 2t -8e at -et + 6t -- I; y = - 20e2t + 8e at + 3et -I- 12t + 10. 2C 1 C1 * (X 2 +y2)y Z _ _ 3087.y (C )2' z - C - - ' 3088. a) =C I, ---C 2 , 2- X 2- X x,( OY 2+y2.=arc tan JL +C , lr z b) In ==C 2. Hint. Integrating the horno1 x y x2 + y2

"-f (x!+x). z=C e


2 2
2

2X

d-f (x -x-I).
2

Yx

geneous equation = arcX tan

x-y

~=~,
x+y

\ve

find the first

tlltegral In

y X +p2-:-:-:
2

1L + CI' x

Then, uSing the properties of derivative proportions, \ve havl'


x~+y~

~ y~ = (x~ ) = y ( ) z x x-y x +y

+ y2

Whence 1n z == -2 111 (x

+ y 2) -1- InC 2 and,

hence,

.. r z
y

x2+ y2

=C 2 ; c) x+y+-z-==O, X2 +y2+ Z2==6. flint. Applying till'"


dx dtj dz dx+dl/-I-dz --=----=-==--== 0 y-z z-x x,-y
1

. t'Ive propor t'Ions, we h ave 0 f derlva proper t les whence dx

-r dy -t- d.J~--== 0 and, consequentl y, x + y + z == C


+ +

S, nlilarl y,

x Y-Z

(x dx )-

z dz x dx II dy z dz Y dy - - - - - - . xdx-'-ydtj-t-zdz-=--Q and x 2 fy(z-x) z(x-y) 0 ' I. 2 2 +Z2 == C2 Thus, the integra I curves are the circles x +!I + z -:: C1, x y -I- Z2 - C~ From the initial condition<;, x=l, y==l,z=-=-2, \VC will haveC 1 =O,C 2 ={). C x2 3089. y==C l x2 +-f-T8 (31n 2 x-2ln x),

+y0)

z= 1-2C,x + :~ +i (31n 2 x+ In x- \).


C 2e- ~ V'2 Cs cos x C" sin . x ex - 2x, 3090 . y = C Ie ~ V 2x Vi- C -xV;- C s C4 1 x z=- C Ie - 2e -4-cosx-TslIlx-2e+x.

3091.

X=

v m cos a ( _.!!0 k l-e m

t) 'Y=/i2(kv m (_!- t) - T mg t ' o sina+mg) l-e m

Solution. m d;t = -kv g ; m ~;: = -kvy-mg for the initial conditions: when

Answers

471
k

t=o,

xo==Yo==O,
k

l'XO=vocosa, Vt' =uosina.


0

Integrating, we obtain v x 3092. x ==a cos ~r-' i, r In


Il

== V o cos ae
t'
=_0_ _

- -- t
til ,

kV y

+mg == (kv o sin f1 + mg) em.


x2
-:;+-~- -=-

-- t

y=

JIm sin --:..-:: k


YIll
~
III

t,

u"

k'2~l nzv 2 o
2

l. Hint. The differential equations of motIon:

III

dt 2 =-= - k"x,

d!t

d2~

dt;- --= - k y.

3093. y= -2-2t-x 2 3094. y=


5 _ 309 . y3096

(Yo+{ ) e (X-I)_~ x+~ .


2

2" -f-"4 X -1- ~


X
3

1.

-1- 16

a
X

+ 32 X

-1- 320 X -1-

21

1 Y =- : 3"

-7.9 x

2 II -t- 7. 11 .27 X -

3097. y~x.+T:2-1-2.3+a.4+ ... ; thc series converges for -l~x<l. 3098. lj-~X-(1f)2.2-i-(2!)2.3-(3!)q 1- ; thc series conven~es for -00<
r2
XS

x2

x3

x4

x4

<x< -1- 00.


3099. Y --= ) ~

Hint. Use the 111ethod of undeternlloed coefficients.

3!
x~

-t- 6! x 6 -

)4

1 47
~}-,-

x' . .. ; the

sene~

converges for-

00

<X< +

CX).

3100. 11-== sin x . Hint. U"e the Illethod of undctcnnincd corfficH:nts.

3101. y;:-=. 1-22-t-22.4~-2l ..P.6~-1- ... ; Hint. 2 -1- ~r!


.

~'I

xG

the

senes converges for

I (/.::

0).

U~C

the IlIl,thod of UI1(ktrrnlinl'd coefflclents. 3102.


U t 6 -1- 8! 55 t 8 -...
U (1.
)

x -- a

(I-~ t 2 -t2!

t 4 - ()!

t S1I1 . T :rl X 31 03 u =-= A cos art -l HInt. l'Jse t 1h~ condiU

tlon~: Ll

(0, t)-::-::o,
00

t)-:--o,

(x, 0) --. (x, 0) --= A' sin nx T' iJu dt


rzrx

3104. U -=

L 1 (I-cos no-a
21
-f)-

-2

It

l1n) Sill - l - sin -1-.

. l11tut

Hint.

Usc

the

conditions:

rt=1

u (0, t) =-= 0,
3105.
U

Ii

(l, t) --= 0,
00

(x, 0) -= 0,
ll:Tat.

au (x, dt
IUtX

0)

= 1.

== 312

Biz

~
11=1

1t2 SlIt 2

11:rl

cos -l- SUI -l- Hint. Use the conoitlOl1s:

iJu

~/

0) _-=-0,
00

It

(0, t)=O,

It

(I, t)=O, u (x, 0)=

J t (1-3.-.\\
2h

211t -1-

for O::::x ~ :l" , I for- < x 1J 2

< I

,-., 3106 u ~ ~

An Cos --2-[-(2n -1- I) ant . (211 + 1) nx wllcrc tile fn t 1 sIn 2t' coe 11Clf11l S / II =:

n=o

472
x = T S-1
1

Answers
2
. (2n+I)1tx dx. Hint. Use the conditions 21
(1, t) x u (0, t)=O, ---ax=0, u (x, 0) 1
CJ)

Sill

3107. u =

au

'

au (x, 0) iJt o.

a 2 n:!j't2t

400

na ~ fi3
n=l

. nnx -1""002 (I-cos n1t) Sin 100 - e

Hint. Use the conditions: II (0, t) == 0, U (100, t) == 0, u (x, 0) = 0.01 x (laO-x).

Chapter X
3108. a) ~ I"; ~0.0023!o; b) ~ I mm; ~0.26/0; c) ~ I gm; ~0.OOI6%. 3109. a) ~ 0.05; ~ 0.021%~ b) ~ 0.0005; ~ 1.45% ; c) ~ 0.005; ~ 0.16/0. 3110. a) two deciInals; 48.108 or 49.10 3 , since the number lies between 47,877 and 48,845; b) two decimals; 15; c) one decinlal; 6.10 2. For practi~" purpo"es there is sense in writing the result in the fonn (5.90.1).102. 31 I t. a) 29.5; b) 1.6-102 ; c) 43.2. 3112. a) 84.2; b) 18.5 or 18 47 0.01; c) the result of

subtraction does not have any correct decimals, since the difference is equal to one hundredth with a possible absolute error of one hundredth. 31131&. 1.80.3 cm 2. Hint. Use the fornlula for increase in area of a square. 3114. a) 30.00.2; b) 43.70.1; c) 0.30.1. 3115. 19.90.1 In 2 3116. a) 1.12950.0002; b) 0.120O.006; c) the quotient nlay vary behveen 48 and 62. Hence, not a single decinlal place in the quotient may be con~id ered certain. 3117. 0.480. The last digit 111ay vary by unity. 3118. a) 0.1729; b) 277.10 3 ; c) 2. 3119. (2.050.01)103 cm 2 3120. a) 1.648; b) 4.0250.001; c) 9.0060.003. 3121. 4.01.10 3 cm 2. Absolute error, 6 5 crn!. Relative error, 0.16/0.3122. The side is equal to 13.8+0.2 cm; sin a=0.440.01, (1,=---=2615' 35'. 3123. 2 7 O.l. 3124. 0.27 ampere 3125. The length of the pendululll should be measured to within 0.3 enl; take the nurnbers :t and q to three decinlals (on the principle of equal effects). 3126. Measure the radii and the generatrix with relative error 1/300. Take the number 1t to three decimal places (on the principle of equal effects). 3127. l\\easure the quantity 1 to within 0.20/ 0 , and s to within 0.7/0 (on the principle of equal effects).
3128.

/).y

&2y

/).3 y

/).4 y

ti 5 y

3 2 10 15
12

-2
-8
0

-6
8
-1

14

-23

-9

-3

-3 -4

-I

5
6

9
5

4nswers
J129.

473

&y

/).2 y

flSy
48
4,Q

-4
3
fi

-12
20

32 80 12R
176

-16
4

100 228 40t

48

7
9

104
332

11

7:36

3130.
.\
.\ :l

IJ

I
- - I- - -

,--------

I
I

I
Hint. Compute the first live values of y and, after obtaining L\4 yo 24, repeat the nUlllber 24 throughout the C01Un1l1 of fourth differences. After this the remaining part of the table is tilled HI by the operation of additlo.1 (nl0ving
-=..:;

from right to left).

474

Answers

3131. a) 0.211; 0.389; 0.490; 0.660; b) 0.229;0.399;0.491;0.664.3132.0 1822;

0.1993; 0.2165; 0.2334; 0.2503. 3133. 1 + x + XZ + x s 3134. y = :6

x~-~~ Xs +

+~~ XZ-~x+8;
y(2)=1l; b)
3140. 3144. 3149. 3154. 3157.

y=::::22 for x=5.5; y=20 for x=::::5.2. Hint. When computing

x for y=20 take Yo== 11. 3135. The interpolating polynolnial is y =-= x 2 - lOx -t- 1; Y= 1 \vhen x==O. 3136. 158 kgf (approxilllately). 3137. a) y (0.5)...:;;;- 1,

Y(0.5)=-~,

y(2)=-3.

3138.

-1.325

3139.

1.01.

-1.86; -0.25; 2.11. 3141. 2.09. 3142.245 and 0 019. 3143. O.:~I and 4 2.506. 3145. 0.02. 3146. 0 24. 3147. 1 27 3148. -1.88; 0 35; 1 5:) 1.84. 3150. 1.31 and -0.67. 3151. 7.13. 3152. 0.165. 3153. 1.73 and O. 1.72. 3155. 1:38 3156. x=0.83; y-==O 56; x~ -0.83; y== -0.56 x== 1.67; Y== 1 22. 3158. 4 493. 3159. 1 1997 3160. Bv the trapezoidal formula, 11.625: by Sinlpson's formula, 11 417. 3161. -0 995; -1; 0.005; 0.5/o~ 6. -=0.005. 3162. 0.3068; 6. == 1.:~10-5. 3163. 0 69 3164. 0.79. 3165.0.84. 3166.0.28. 3167.0.10. 3168. 161. 3169. 1.85 3170.0.09. 3171. 0.67. 3172.0.75. 3173. 0.79. 3174. 4.93. 3175. 1 29. Hint. Make use of the parametric equation of the elli pse x ~ cos t, Y -== 0.6222 sin t r'::11d tran~1t

fOlm the formula of the arc length to the form ~ VI-Ezcos z tdt, where E

IS

the eccentnclty of the ellipse. 3176. Yl (x) ==:3 'Y2 (x)


x7 2x JJ
2

. .

x3

== 3" +. 63 ' Y3 (x) -= ""3 -t3x 2 x4


3

x3

x7

x3

+ 63 +2079 + 59535 3177. Yl (x)==2- x + 1, Y2(X)==6 + 2 -x + 1, !/:s(x) =-:-- 12x 3x x' 7x -"6-l-2- x + 1; Zt(x)=-:3x-2, z2(x)-==1f-2x2+3x-2, z3(x)-==T3

XIS

x2

XS

x3 x3 XS -2x2 +3x-2. 3178. Yl(X)=X, Y2(X)=X-tf' Y3(x)~x-6+1~0 31 79. Y (1) == 3. 36 . 3180. Y (2) == 0 .80 . 3181 . Y (1) === 3 . 72; Z ( 1) == 2 . 72
3182. Y:::: 1.80. 3183. 3.15. 3184. 0.14. 3185. Y (0.5) --:-:3 15; z (0 5) --.: - 3 15. 3186. Y (0.5) = 0 . 55; Z (0 5) = - 0 . 18. 3 t 87. 1. 16. 3188. 0 87. 3189. x (It) - 3. 58~ x' (rt) --=0.79. 3190. 429+ 1739 cos x-l0:37 SUI x-6:321 ros 2x -t- 126:J Sill 2x- ]242 cos 3x-33 S111 3x. 3191. 6 49-1 96 cos x -t- 2.14 SUI x-l.6H Cos 2x -t-t- O. 53 ~in 2x - 1. 13 cos 3x 0.04 sin 3x. 3192. 0.960 0.851 cos x 0.915 Sill x -I0.542 cos 2x 0.620 sin 2x 0.271 cos 3x -I- 0.100 sin 3x. 3193. a) () 608 ~In x 1+ 0.076 sin 2x 0.022 sin 3x; b) 0.338 0.414 cos x 0.111 cos 2x 1- 0.056 cos 3x.

+ +

+.

APPENDIX

I. Greek Alphabet
Alpha-An

Beta- Bri Ganll11a - ry Delta-8b


Epsilon-Ep,

Iota-It Kappa-}(x
LaInbda-AA
Mu-l\f~l

Nu-Nv

Zeta -Z~ Eta - II.,


Theta-eO

Xi-3~

Ornicron-Oo Pi-Iln

Rho-PQ Slgnla -~O' Tau- Tt Upsl1on- r" Phi -(Dcp Chi-Xx Psi - 'l'~' Oll1ega-Qw

II. Some Constants


Quantity
1

x
I

log x

QU3nt It)

log x

:t

:~

14159

0.49713
0.79818

2:t
j(

G.28318

I e el

0.36788 7.3890fi
I.G48i2

r 56571
O.R6859 0.21715 0.14-176

-rt

"2n 4 1
n2

1.57080 0.78540 0.31831 9.86n60 1.77245 1.46459 2.7182R

0.19612

Ve

I .89509

V-;
1\1 = tog e

1 39561 0.4342) 2.30258 5717'45" 0.01745 9.81

t .50285
0.99130 0.24857 0.16572 0.43429

t .6:)778
O.~3G222

-=lnl0 1 radian arc 1


0

1 M

V;
e

Vn

2.24188 0.99167

476

Appendix

III. Inverse Quantities, Powers, Roots, Logarithms

xl

X2

X3

/ 1.000 1.331 1.728 2.197 2.744 3.375 4.096 4.913 5.832 6.859 8.000 9.261 10.65 12.17 13.82 15.62 17.58 19.68 21.95 24.39 27.00 29.79 32.77 35.94 *39.30 42.88 46.66 50.65 54.87 59.32 64.00 68.92 74.09 79.51 85.18 91.12 97.34 103.8 110.6 117.6 125.0 132.7 140.6 148.9 157.5

I I Iv: IVloxl V-IIOgX I


Vx VlO x
lOOx (t.nantlssas)

In x

1.0 1.000 1. 1 0.909 1.2 0.833 1.3 0.769 1.4 0.714 1.5 0.667 1.6 0.625 1.7 0.588 1.8 0.556 1.9 0.526 2.0 0.500 2. I 0.476 2.2 0.454 2.3 0.435 2.4 0.417 2.5 0.400 2.6 0.385 2.7 0.370 2.8 0.357 2.9 0.345 3.0 0.333 3.1 0.323 3.2 0.312 3.3 0.303 3.4 0.294 3.5 0.286 3.6 0.278 3.7 0.270 3.8 0.263 3.9 0.256 4.0 0.250 4.1 0.244 4.2 0.238 4.3 0.233 4.4 0.227 4.5 0.222 4.6 0.217 4.7 0.213 4.8 0.208 4.9 0.204 5.0 0.200 5.1 0.196 5.2 0.192 5.3 0.189 5.4 0.185

1.000 1.210 1.440 1.690 1.960 2.250 2.560 2.890 3.240 3.610 4.000 4.410 4.840 5.290 5.760 6.250 6.760 7.290 7.840 8.410 9.000 9.610 10.24 10.89 11.56 12.25 12.96 13.69 14.44 15.21 16.00 16.81 17.64 18.49 19.36 20.25 21.16 22.09 23.04 24.01 25.00 26.01 27.'()4 28.09 29.16

1.000 1.049 1.095 1.140 1.183 1.225 1.265 1.304 1.342 1 378 1.414 1.449 1.483 1.517 1.549 1.581 1.612 1.643 1.673 1.703 1.732 1.761 1.789 1.817 1.844 1.871 1.897 1.924 1.949 1.975 2.000 2.025 2.0 4 2.098 2.121 2.145 2.168 2.191 ~!. 214 2.236 2.258 2.280 2 302 2.324
2.~9

3.162 3.317 3.464 3.606 3.742 3.873 4.000 4.123 4.243 4.339 4.472 4.583 4.690 4.796 4.899 5.000 5.099 5.196 5.292 5.385 5.477 5.568 5.657 5.745 5.831 5.916 6.000 6.083 6.164 6.245 6.325 6.403 6.481 6.557 6.633 6.708 6.782 6.8G6 6.928 7.000 7.071 7.141 7.211 7.280 7.348

1.000 1.032 1.063 1.091 1.119 1.145 1.170 1 193 1.216 1.239 1.260 1.281 1.301 1.320 1.339 1.357 1.375 1.392 1.409 1.426 1.442 1.458 1.474 1.489 1.504 1.518 1.533 1.547 1.560 1.574 1.587 1.601 1.613 1.626 1.639 1.651 1.663 1.675 1.687 1.698 1.710 1.721 1.732 1.744 1.754

2.154 2 224 2.289 2.351 2.410 2.466 2.520 2.571 2.621 2.668 2.714 2.759 2 802 2.844 2.884 2.Q24 2.962 3.000 3.037 3.072 3.107 3.141 3.175 3.208 3.240 3.271 3.302 3.332 3.362 3.391 3.420 3.448 3.476 3.503 3.530 3.557 3.583 3.609 3.634 3.659 3.684 3.708 3.733 3.756 3.780

4.642 4.791 4.932 5.066 5.192 5.313 5.429 5.540 5.646 5.749 5.848 5.944 6.037 6.127 6.214 6.300 6.383 6.463 6.542 6.619 6.694 6.768 6.840 6.910 6.98e 7.047 7.114 7.179 7.243 7.306 7.368 7.429 7.489 7.548 7.606 7.663 7.719 7.775 7.830 7.884 7.937 7.990 8.041 8.093 8.143

0000 0414 0792 1139 1461 1761 2041 2304 2553 2788 30rv 3222 3424 3617 3802 3979 4150 4314 4472 4624 4771 4914 5051 5185 5315 5441 5563 5682 5798 5911 6021 6128 6232 6335 6435 6532 6628 6721 6812 6902 6990 7076 7160 7243 7324

0.0000 0.0953 0.1823 0.2624 0.3365 0.4 00 0.5306 o 5878 0.6419 0.6931 0.7419 0.7885 0.8329 0.8755 0.9163 0.9555 0.9933 1.0296 1 0647 1 0986 1.1314 1.1632 1.1939 1.2238 1.2528 1..2809 1.3083 1.3350 1. 3610 1.3863 1.4110 1 4351 1.4586 1. 4816 1.5041 1.5261 1.5476 1.5686 1.5892 1.6094 1.6292 1.6487 1.6677 1.6854
0.4~55

Appendix

471
Continued

1+1
o 182 0.179 0.175 0.172 o 169 0.167 0.164 o 161 o 159 0.156 ~,~~ 154 0.151 0.149 0.147 o 145 0.143 0.141 0.139 o 137 0.135 0.133 0.132 o 1:\0 o 128 o 127 o 125 o 12~ 0.122 0.120 o 119 0.118 0.116 0.115 0.114 o 112

t''J

x"

v"x

I IV-lv-/v-/ I
VTOX
x'
lOx
IOgX lOOx (~a n-

t ls~as)

In x

5.5 56 5.7 5.8 5.9 6.0 6.1 6.2 6.3 6 4 6.5 66 6 7 68 69 7.0 7.1 7.2 7.3 7.4 7.5 7.6 7.7 7.8 7.9

S.O
8.1
8.2

8 4 8.5
8.G

8.3

8.7 8.8 89

9.1 0.110 9.2 0.109 9.:3 0.108 9.4 0.106 9.5 0.105 9.6 0.104 9.7 0.103 9.8 0.102 9.9 0.101 10.0 0.100

9.0 O. III

30 25 166 4 31 36 175.6 32 49 185 2 33.64 195.1 34 81 205.4 36.00 216.0 37 21 227.0 38.44 2a8 3 39.69 250 0 40 96 262.1 42.23 274 6 43.56 287 5 44 89 300 8 46 24 314 4 47 61 ~i28 5 49.00 343 0 50.41 357.9 51.84 373.2 53.29 389.0 54 76 405.2 56.25 421.9 57.76 439 0 59 29 456 5 60.84 474 6 62.41 49~3. 0 64.00 512 0 6G.61 531.4 67.24 551. 4 68.89 571.8 70.56 592.7 72.23 614.1 7:3.96 636.1 75.69 658.5 77.44 681.5 79 21 705.0 81.00 729.0 82.81 753.6 84.64 778.7 86.49 804.4 88.36 830 6 90.25 857.4 92.1~ 884.7 94.09 912 7 96.04 941.2 98.01 970.3 100.00 1000.0

2.345 2 366 2 387 2 408 2 429 2.449 2.470 2 490 2.510 2.530 2.550 2 569 2 588 2 608 2.627
2.646

2 665 2.683 2.702 2.720 2.739 2 757 2 775 2.793 2.811 2.828 2 846 2.864 2.881 2.898 2.915 2 933 2.950 2.966 2 983 3.000 3.017 3.033 3.050 3 066 3.082 3.098 3.' 14 3.130 3.146 3.162

7.416 7 483 7 550 7 616 7.681 7 746 7.810 7 874 7.937 8.000 8.062 8.124 8.185 8.246 8 307 8.367 8 426 8.485 8.544 8 602 8.660 8.718 8.775 8.832 8.888 8.944 9.000 9 055 9.110 9.165 9.220 9.274 9.a27 9.381 9.431 9.487 9.539 9.592 9.644 9.695 9.747 9.798 9.849 9.899 9.950 10.000

1.765 3.803 1 776 3.826 1.786 3.849 1.797 3.871 1.807 3 893 1.817 3.915 I 827 3.936 1.837 3.958 1.847 3.979 1.857 4.000 1.866 4 021 I 876 4.041 I 885 4 062 1.895 4.08~ 1.904 4.102 1.91a 4.121 1.922 4.141 1.931 4.160 1.940 4.179 1.949 4.198 1.957 4.217 1.966 4.236 1.975 4.254 1.983 4 273 I 992 4 291 2.000 4.309 2 008 4.327 2 017 4.344 2 025 4.362 2.033 4 380 2.041 4.397 2 049 4.414 2.057 4.431 2.065 4.448 2 072 4.465 2.080 4.481 2.088 4.498 2.095 4.514 2.103 4.531 2.110 4.547 2.118 4.563 2.125 4.579 2.133 4.595 2.140 4.610 2.147 4.626 2.154 ,4.642

8.193 7404 1.7047 8.243 7482 1 7228 8.291 7559 1.7405 8.340 7634 1.7579 8 387 7709 1.7750 8.434 7782 1.7918 8 481 7853 1.8083 8 527 7924 1.8245 8 57:3 7993 1.8405 8 618 8062 1.8563 8.662 8129 1.8718 8.707 8195 1.8871 8 750 8261 I 9021 8 794 8:325 1 9169 8.837 8388 I 9315 8 879 8451 I 9459 8 921 8513 1.9601 8.963 8573 1.9741 9.004 8633 1.9879 9.045 8692 2.0015 9.086 8751 2.0149 9.126 8808 2.0281 9.166 8865 2.0412 9.205 8921 2.0541 9.244 8976 2.0669 9.283 9031 2.0794 9.322 9085 2'.0919 9 360 9138 ~.1041 9.398 9191 2.1163 9.435 9243 2.1282 9.473 9294 2.1401 9 510 9345 2.1518 9.546 9395 2.1633 9. 58~~ 9445 2.1748 9 619 9494 2.1861 9.655 9542 2 1972 9.691 9590 2.2083 9.726 9638 2.2192 9.761 9685 2.2300 9.796 9731 2 2407 9.830 9777 2.2513 9 865 ge23 2.2618 9 899 9868 2 2721 9.933 9912 2.2824 9.967 9956 2.2925 10.000 0000 2.3026

478 Appendix ---------------=-.....:..-_---------_._-

IV. Trigonometric Functions

,,0
0
1
2

x
(fad lans)

41ln .\

tan

cot x
00

cos x

I
1.5708 1.55:33 1 5:~5q 1. 5184 l.fiOl0 1.48:35 1.4661 1.4-lR6 1 4:H2 1 41:37 I ~963 l. 37~8 . I ~61~
1.34;~9

3 4 5 6 7 8
9

0.0000 0.0175 0.0349 0.0524 0.0698


0.087~

10 11 12 13 14 15 16 17 18 19 20 21 22 23 24 25 26

27

42

28 29 30 31 32 33 34 35 36 37 38 39 40 41

43 44 45

0.1047 0.1222 0.1396 0.1571 0.1745 0.1920 0.2094 0.2269 O. 244'~ 0.2618 0.2793 0.2967 0.3142 0.3316 0.3491 0.:~420 0.3665 0.3584 0.3840 0.3746 0.4014 0.3907 0.4189 0.4067 0.4363 0.4226 0.4538 .. o 4384 0.4712 0.4540 0.4887 0.4695 0.5061 o 484S 0.52:36 0.5000 0.5411 0.5150 0.5585 0.5299 0.5760 0.5446 0.5934 0.5592 0.6109 0.5736 0.6283 0.5878 0.6458 0.6018 0.6632 0.6157 0.6807 0.6293 0.0981 0.6428 0.7156 0.6561 0.7330 0.6691 0.7505 0.6820 0.7679 0.6947 0.7854 0.7071

0.0000 0.0175 0.0349 o .052:~ 0.0698 0.0872 0.1045 0.1219 0.1392 0.156t 0.1736 0.1908 0.2079 0.2250 0.2419 0.2588 0.2756 0.2924 0.3090 0.32G6

0.0000 0.0175 0.0349 0.0524


0.0()9~

0.0875 0.1051 0.1228 0.1405 o 1584 0.1944 0.2126 0.2309 0.2493 0.2679 0.2867 O. ~~057 0.3249 0.3640 0.3839 0.4040 0.4245 0.4452 0.4663 o 4877 0.5095 0.5317 0.5543 0.5774 0.6009 0.6249
0.~44:3 O.176~

57 29 28.64 19.08 14 30 11.43 9.514 8.144 7.115 6 314 5.671 5.145 4.703 4. :~~~1 4.011 3.732 3.487 3.271 3.078 2.904 2.747 2.605 2.475 2.356 2.246 2.145 2.050 1.963 I.R81 1.804 1 732 1.6643
I.GOO:3

1.0000 0.9998 0.9994 0.9986 0.9<)76 0.9962 O.994S 0.9925 0.9903 0.9877 0.9848 0.9816 o 9781 0.9744 0.9703 0.9639 0.961:3 O. 956;~ 0.9511 O.945[) o 9397 O. 9;t~6 0.9272
0.9~O5

90 89 88 87 86 85 84 83
82

RI 80
I

79
78

77
76

0.9135 0.906:3
0.89R8

o 64q4

0.6745 0.7002 0.7265 0.7536 0.7813 0.8098 0.8391 0.8693 0.9004 o 9325 0.9657 1.0000

1.5399 1.4826 1.4281 1.3764 1.3270 1.2799 1.2349 1.1918 1.1504 1.1106 1.0724 1.0355 1.0000

0.8910 0.8829 0.8716 0.8660 0.8G72 0.8480 0.8387 o 8290 0.81Q2 0.8090
O. 7~)86

0.7880 0.7771 0.7660 0.7547 0.7431 0.7314 0.7193 0.7071

1.3263 1.3090 1.2915 1.2741 I 2566 I 2:392 I 2217 1.2041 1.1868 1.1691 1.1519 I 1343 1.1170 1.0996 1.0821 l.n647 1.0472 1.0297 I .OI2:l 0.9948 0.9774 0.9599 0.9423 0.9250 0.9076 0.8901 0.8727 0.8552 0.8378 0.8203 0.8029 0.7854
I

75 74 73 72 71 70 69 68 67 66 6;) 64
():~

62 61 60 59 58 57
56 55

54
53

52 S!
50

49 48 47 46

45

cos x

rot x

tan x

~in

(rad~al") I

Appendix

479

V. Exponential, Hyperbolic and Trigonometric Functions


x

e"'C

e-;J;

sinh x

cosh x

tanh x

sin x

cos x

o 0 0.1

0.2 o :l

0.4 o 5 o6 o 7 0.8 o 9. I 0
1.1

1.0000 1.1052 I 2214 1.:3499 I 4918 1 6487 1.8221 2.0138 2.22;)3 2 4596 2. 718:~ 3.0042 :l 3201 :~ 66 3 4 O:l52 4 4817
t.9':>:lO

1 0000 o 9048 o 8187 0.7408 0.6703 0.6063 0.5488 0.4966 o 4493 0.4066 0.3679

0.0000 0.1002 0.201:3 0.3045 0.4108 o 5211 o 6367 o 7586 o 8881 1.0263 1.1752
1.3356

1.0000 1.0050 I 0201 1.0453 1.0811 1.1276 1.1853 1.2552 1.3:374 1.4331 1.5431
1.6685

0.0000 0.0997 0.1974 0.2913 0.3799 0.4621 0.5370 0.6044 0.6640 0.7163 o 7616
0.8005

0.0000 0.0998 0.1987 0.29n3 0.3894 0.4794 0.5646 0.6442 0.7174 0.7833

1.0000 0.9950 0.9801 0.9553 0.9211 0.8776 0.8253 0.7648 0.6967 0.6216 0.5403 0.4536 0.3624 0.2675 0.1700

12 1 :~ 1 t
15

o :3329

0.3012 o 2725 o 2166 () 22al

1.5095 1.6981 1. 9043


~.12n3

1.8107 1.9709 2.1309 2.3524 2 5775 2 8283 :3.1075 3 4177


:~ 7622 4 .144:~

o 8337 0.8617 o 8834 0.9051 o 9217 0.9354 0.9468

o 8912
0.9320 o 9636 0.9854 0.9973 0.9996 0.9917 0.9738 n 9463

o 8415

1.6

17
I H

1~ 2 () ~ 1 22
2
:~

:>.47:39 6.0496 h 68fi9 7 ~~8~} I H 1662


q
()~.so

o 1827 () 1653 o 11<)6


() () ()

o 2019

2 3756 2.6456 2.9422


:~.2682

o 9:)62

-0.0292 -0.1288 -0 2272 -0.3233

0007071

I;~~):~ 122:>
IIn8

:~

6269

24
2 ;) 26 27 2.8
2 9

11

n 974~
O~;~~

() 10n:~ o 0907 0.0821 o 0743 o 06i2

4 0219 4 4571 4.9370 5.46G2 6.0502 6 6(}47 7.4063 8.1919 9.0596 10.0179 11.0764 12.2459 I~l. 537n 14.9654 16.5426

4 567<)

S.0372 5.5569
6.1.12:~

o. ~640 0.9704 0.9757 D.U8ul 0.98:37

0.9093 -0.4161 O.86~~ -0 5048 o 8085 -0.5885 o 74.:17 -0.6663 (1 6753 -G 7374 0.5985 -0.8011 o 5155 -0.8569 0.4274 -0.9041 O. :3:350 -0.9422 0.23H2 -0.9710 0.1411 0.0416 -0.0584 -0 1577 -0.2555 -0.9900 -0.9991 -0.9983 -0.9875 -0.9668

1~ 182.) I:J. 4().~7 11.8797 16 4146 18.1741

o 0608

0.0550

6.7690 7. 4i~l5 8 2f)27 9.1146 10.0677 11.1215 12.23G6 13.5748 14.9a87

o ~)890
0.991U 0.9926 0.9940 0.9950 0.9959 0.9967 0.9973 0.9978

0.9866

:l.O

3.1 3.2
3.3

3 4 3.5

20.085!l 22.1979 24.5325 27.1126 29.9641 33.1154

0.0498 0.0450 0.0408 0.03()9 0.0334 0.0302

5728\ 0.9982 16 0

--0.3508 -0.9365

480

Appendix

VI. Some Curves (for Reference)

y
y

o
1. Parabola. y=x 2

X
2. Cubic parabola, y == Xl.

3. Rectangular
hyperbola,

Y:_-=x "''"'
y

-1 0

4. Graph of a fractional

function,
1 y=-X2

1 5. The witch of Agncsi, 1


y;;:. 1

-1

+x

o
6. Parabola (upper
branch),

7. Cubic parabola,
11=

y=x.

a -x. V

Appendix

481

o
Sa. Neile's parabola.
Y :..::
t'

x
I
,
x~

Bb Sem icubical parabola. f x=t'l. y2=X' or '

\ y=t'.

or

is

X: -

t-.

rtf

-f

9. Si llC curve and cosi ne curve, y - -= SA:l x ;]nd 'I = cos x.

10. Tangent curve and cotangent curve,


y=fanx and y=cotx.

Appendix

11. Gra phs of the f unct ions y -= sec x and y = cosec A.

""

y=arc sin :r

12. Graphs of the inverse trigonometric functions


II =

arc sin x and y = arc cos x.

Appendix

483

-----..,...----'-......,;;.-

y=arc

tan:l

------0------1[

13. Gra phs of the Inv~rse trigonoInetric functions y =- arc tan x and y = arc cot x.

-1

14. Graphs of the exponential functions y=ex and y=e- x .

486

Appendix

x
x
25. Bernoull i's lenlniscate, (x 2 _I- y2r~ = a 2 (x 2_ y2) or r2 == a2 cos 2<p.

24. Strophoi d,

a+x y 2 =x2 -. a-x


y

o
26. Cycloid, x=a (t -sin t), { y=a (1- cos t).

27. IIypocyclold (astroid), x=a cos 3 t, { y = a sin a t


2 2

or

.\ a

+ 11 a = a 3-.
y

28. Cardioid,

r=a (1 +cos <pl.

29. Evolvent (invol ute) of the circle X = a (cos t + t sin t) t { Y =a (sin t -t cos l).

Appendix

481

x
tI--................_ - - " " ' _........._~

X
.30. Spiral of Archinledcs,
r =aq>.

31. Hyperbolic spiral,


a r=-. q>

32. Logdrithl11ic spiral,


(-=

33. Th ree-Iea fed rose,

ea'l'.

r =a sin 3tp.

34. Four-leafed rose, r = a sin 2<p.

INDEX

Absolute error 367 Absolute value of a real nUlnber 11 Absolutely convergent series 296, 297 Acceleration vector 236 Adams' formula 390 Adarns' method 389, 390, 392 Agnesi Witch of 18, 156,480 Algebraic functions 48 Angle bet\veen two surfaces, 219 Angle of contingence 102, 243 Angle of contingence of second kind 243 Antiderivative 140, 141 generalized 143 Approximate numbers 367 addition of 368 division of 368 rnul ti plicat ion of 368 powers of 368 roots of 368 subtraction of 368 .Approxil11at ion successi ve 377, 385 J\rc length of a curve 158-161 J\rc length of a spacE' curve 234 .Archimedes spiral of 20, 65, 66, 105, 487 .\rea in polar coordinates 155, 256 Area in rectangular coordinates 153, 256 /\rea of a pI ane region 256 Area of a surf ace 166168, 259 Argument 11 Astroid 20, 63, 105, 486
Asymptot~

Bending point 84 Bernoulli's equation 333 Bernoulli's lemniscate 155, Beta-function 146, 150 B inorm al 238 Boundary conditions 363 Branch of a hyperbol a 20, 480 Broken-line Illethod Euler's 326

486

c
Ca rdioi d 20, 105, 486 Catenary 104, 105, 484 Catenoid 168 Cauchy's integral test 295 Cauchy's test 293, 295 Cauchy's theorcln 75, 326 Cavalieri's "leInon" 165 Centrc of curvature 103 Change of variable 211-217 ina de fi nit e in t eg raI 146 in a double integral 252-254 in an indefinite Integral 113 Characteristic equation ~~56 Characteristic points 96 Chebyshev's conditions 127 Chord method 376 Circle 20, 104 of convergence 306 of curvature 103 osculating 103 Circulation of a vector 289 Cissoid 232 of Diocles 18, 485 Clairaut's equation 339 Closed interval 11 Coefficients Fourier 318, 393, 394 Comparison test 143, 293, 294 Composite function 12, 49

93

Ifft horizontal 94 l~ft inclined 94 right horizontal 93 right inclined 93 vertical 93

Index
Coneave down 91 Concave up 91 Concavity direction of 91 Conchoid 232 Condition Li pschi t7. 385 Conditions boundary 363 Chebyshev's 127 Dirichlet 318, 319 initial 323, 363 Conditional extrelTIUm 223225 Conditionally (not absolutely) convergent series 296 Contingence angle of 102, 243 Continuity of functions 36 Continuous function 36 properdes of 38 Convergence circle of 306 interval of 305 radius of 305 region of 304 unifonn 306 Convergent i111prOper integral 143,
270

489

probability 19,484 sine 481 tangent 481 Cusp 230 Cycloid 105, 106, 486
D
D' Alembert's test 295

Convergent serie~ 293 Coordinates of centre of gravity 170 ~en('rali7ed polar 255 Correct deci n1 aI pI aces i 11 a hroad sense 367 Correct decilnal places in a narro\v sense 367 Cosine curve 481 Cotangent curve 481 Coupling equation 223 Cntical point of the second kind 92 en tical points 84 Cuhic parabola 17, 105, 234, 480 Curl of a vector field 288 Curvature centre of 103 circle of 10:~ of a curve 102, 242 radius of 102 second 243 Curve cosine 481 cotangent 481 discriminant 232, 234 Gaussian 92 integral 322 logarithmic 484

Decreasing function 83 Definite integral 138 Del 288 Dependent variable 11 Derivative 43 left-hand 44 logarithmic 55 nth 67 right -hand 44 second 66 Derivative of a function in a given direction 193 Derivative of functions represented parametrically 57 Derivative of an implicit functicn 57 Derivative of an inverse fUllction 57 Derivative of the second order 66 Deriv at ives of higher orders 66-69 one-si ded 43 table of 47 Descartes foliuln of 20, 21, 232, 485 Deternlinant functional 264 Determining coefficients first Inethod of 122 second method of 122 Diagonal table 389 Difference of two convergent series 298 Differential of a 11 arc 10 1, 234 first-order 71 higher-order 198 pri nci pal properties of 72 second 198 second -order 72 total, integration of 202-204 Differential equation 322 homogeneous linear 349 inhotnogeneous linear 349 Differential equations first-order 324 forming 329 higher~rder 345 linear 349, 351

490

Index
coupling 223 difTerenti al 322 Euler's 357 exact differential 335 first-order differential 324 homogeneous 330, 351, 356 honl0geneous linear difTerential 332,
349

Differential equations of higher powers first-order 337 Oifferenti als met hod of 343 of third and higher orders 72 Differentiating a composite function
47

Differentiation 43 of implicit functions 205-208 tabular 46 Diocles cissoid of 18, 485 Direction of concavity 91 Direction field 325 Dirichlet conditions 318, 319 function 40 series 295, 296 theorem 318 Discontinuity 37 of the first kind 37 infinite 38 removable 37 of the second kind 38 Discontinuous function 270 Discriminant 222 Dicriminant curve 232, 234 Divergence of a vector field 288 Divergent improper integral 143, 270 Di vergent seri es 293, 294 Domain 11 Domain of definition 11 Double integral 246 in curvilinear coordinatps 253 in polar coordinates 252 in rectangular coordinates 246 Double point 230
E

inhomogeneous 349, 351, Lagrange' s 339 Laplace's 289, 291 linear 332 of a normal 60, 218 of a tangent 60 of a tangent plane 218 with variables separable Equivalent functions 33 Error absolute 367 limiting absolute 367 limiting relative 367 relative 367 Euler integral 146 Euler-Poisson inte~ral 272 Euler's broken-line nlethod Euler's equation 357 Even function 13 Evolute of a curve 103 Evolvent of a circle 486 Evolvent of a curve 104 Exact diITerential equation Exponential functions 49, EX\remal point 84 Extremum conditional 223-225 of a functi 011 83, 83, 222
F

356

327, 328

326

335 55, 483

Elimination met hod of 359 Ellipse 18, 20, 104, 485 Energy kinetic 174 Envelope equations of 232 of a family of plane curves 232 E picycloi d 283 Equal effects principle of 369 Equation Bernoulli '5 333 charactrristic 356 Clairaut's 339

Factor integrating 335 Field direction fiel d 325 nonstati onary seal ar or vector 288 potenti al vector 289 scalar 288 solenoidal vector 289 Field (cont) stationary scalar or vector 288 vector 288 Field theory 288-292 First-order differential 71 First-order differential equations 324 Flow lines 288 FI ux of a vector field 288 Folium of Descartes 20, 21, 232, 485

Index Force lines 288 FOrl11 Lagrange's 311 Forn1ula Adams' 390 Green's 276, 281, 282 Lagrange's 145 Lagrange's interpolation 374 Leibniz 67 t.\aclaurin's 77, 220 Nc\vton-Leibniz 140, 141, ~75 Newton's interpolation 372 ()strogradsky-Gauss 286-~88 parabul1c 382
Sln1p~on's

491

logarit hm ic 49 . transcendental, integration of 135 trigonometric 48 trigonometric, integrating 128, 129 Fundalnental system of solutions 349
G Gamma-function 146, 150 Gaussian curve 92 General integral 322 General solution 359 General solution (of an equation) 323 General term 294 Generalized antiderivative 143 Generalized polar coordinates 255 Geometric progression 293, 294 Gradient of a field 288 Gradient of a function 194, 195 Graph of a function 12 Greatest value 85, 225,227 Green's forrnul a 276, 281, 282 Guldin's theorenls 171
H

382-384

Stokes' 285, 286, 289 T a\' I or's 77, 220 tra"'pezoidal 382 Forn1111clc; reduction 130, 135 Fourier'" c(_'efficlents 318, Fourier series 318, 319 Four-leafed rose 487 Fraction proper rational 121 I'unctlon 11 COllI posite 12. 49 contInuous 36 cont inUOll':>, properties of decreasing 83 DIrichlet 40 dl'icont Inuous 270 ('veil 13 (If C1 function 12 inlpl1clt 12 increasi n~ 83
Lagr~lnge

39~3,

394

:~8

223, 224 11

Illultiple-valued 11 periodic 14
sin~lc-vallled

vector 235 F11nctional detern1inant 264 Functional series 304 Functions


al~ebraic

Han11ltonian operator 288 Harn10nic series 294, 296, 297 HIgher-order dIfferential 198 l-ligher-order dIfferential equation'.' Higher-order partial derivative Hodograph of a vector 235 H0l110geneous equations 330, 351, l-Ion10geneous Iinear difTerent Ial equation 332. 349 llyperbola 17, 18,20,485 rectangular 480 l-Iyperbolic functions 49, 484 integration of 133 Hyperbolic spiral 20, 105, 487 Hyperbolic substitutions 114, 116, Ilypocycloid 283, 486
I 111 pI i cit fun c t i on 12

345 197 356

133

48

equivalent 33 ('\ ponenti al 49, 55, 483 hy perbol ic 49, 484 hyperbolic, integration of 133 inverse 12 Functions (cont) inverse circular 48 inv('rse hyperbolic 49 inverse trigonometric 482, 483 Ii nearl y dependent 349 linearly independent 349

Inlproper integral
conver~ent

270

divergent 270 Improper Jllultiple integrals 269, 270 Incoillplete Fourier series 318, 319 Increasing function 83 Increment of an arguI11ent 42 Incren1ent of a function 42 Independent variClble 11 Indetenn inate forll1s evaluating 78, 79

492

Index
Interpolation formula Lagrange's 374 Newton's 372 Interval of calculations 382 closed II of convergence 305 of monotonicity 83 I nterv al (cont) open 11 table interval 372 I nverse circular functions 48 I nverse functions 12 Inverse hyperbolic functions 49 Inverse interpolation 373 I nverse trigonometric functions 482,
483

Infinite discontinuities 38 Infinitely large quantities 33 Infinitely small quantities 33 Infinites 33 Infinitesimals 33 of higher order 33 of order n 33 of the same order 33 Inflection points of 91 Inhomogeneous equation 349. 351. 356 Inhomogeneous linear differential equation 349 Initial conditions 323. 363 Integral 322 convergent improper 143 definite 138 divergent improper 143 double 246 Euler 146 Euler-Poisson 272 general 322 improper multiple 269, 270 line 273-278 part icul ar 322 probability 144 singular 337 surface 284-286 triple 262 Integral curve 322 Integral sum 138 Integrating factor 335 Integration basic rules of 107 under the differential sign 109 direct 107 by parts 116, 117, 149 path of 273, 274, 280 region of 246-248 by substitution 113 Integration of differential equation by means of power series 361, 362 Integration of functions numerical 382, 383 Inte~ration of ordinary differential equation nurnerical 384-393 Integration of total difTerentials 202204

Involute of a circle 20, 106, 486 Involute of a curve 104 Isoclines 325 Isolated point 230 , Iterative method 377, 378, 380
J

Jacobian 253, 264

K Kinetic energy 174


L

Integration of transcendental functions 135 Jnterpolation of functions 372-374 inverse 373 linear 13. 372 quadratic 372

Lagrange's equation 339 Lagrange's form 311 Lagrange's fornlula 145 Lagrange's function 223, 224 Lagrange's interpolation fornlllla 374 Lagrange's theOrel11 75 Laplace equation 289, 291 Laplace transformation 271 Laplacian operator 289 Lamina coordinates of the centre of gravity of a. 261 mass and static nlornents of a 260 moments of inertia of a 261 Least value 85 Left-hand derivative 44 Left horizontal asymptote 94 Left inclined asymptote 94 Leibniz rule 67, 269 Leibniz test 296, 297 Lemniscate 20, 105, 232 Bernoulli's 155, 486 Level surf aces 288 L'Hospital-Bernoulli rule 78.. 82

Index,
Lima~on

493
of successive approxi.mation 38f,
385, 389

Pascal's 158 Limit of a function 22 Limit on the left 22 Limit on the right 22 Limit of a sequence 22 Limiting absolute error 367 Lilniting relative error 367 Lilnits one-sided 22 Line straight 17, 20 Line Integral application of 276, 28:3 of the first type 273, 274, 277, 278 Line integral of the second type 274,
275, 278-281

of tangents 377 of undetermined coefficients 121, 351 of variation of parameters 332, 349,
352

Minimum of a function 84, 222 Mininlum point 84 Mixed partial derivative 197 Moment of inertia 169 static 168 Monotonicity intervals of 8a Multiple-valued function 11 Multi pi icities root 121 N nth derivative 67 Nabla 288 Napier's number 28 Natural trlhedron ~3B Necessary condition for convergence 29:3 Necessary condition for an extrCJ11Unl
222

Linear differential equations 349, 351 Linear equation 332 Linear interpolation 372 of a hinrtion 13 Linearly dependent functions 349 Linearly independent functions 349 Lines flow 288 force 288 vector 288 Li pschitz condition 385 Logarithlllic curve 484 Logarithnlic derivative 55 Logarithlllic functions 49 Logarithnlic spiral 20, 21, 105, 106,
487
M

l\\aclaurin's formu] a 77, 220 Maclaurin's series 311, 31:3 }'v\axiUll1I11 of a fUllction 84, 222 MaxinlU111 point ~\ean value of a function 151 1\\ean-valuC' theorellls 75, 150 j\\ean rate of change 42 ~\\et hod l\dall1s' 389, 390, 392 chord lnethod 376 of differentials 343 of elinlinat ion 359 Method (cont) Euler's broken-line 326 iterative 377, 378, 380 Milne's 386, 387, 390 Newton's 377, 379 Ostrogradsky 123, 125 Picard's 384, 385 reduction 123 Runge-Kutta 385-387, 390

Newton trident of 18 Ne\vton-Leibniz fOrillula 140, 141, 275 Newton's interpolation formula 372 Newton's J11ethod 377, 379 Ne\\ton's serpentine 18 Niele's par abo! a 18, 234. 48] Node 230 Nonstationary scalar or vector field 288 Nonnal 217 to a curve 60 equations of 218 principal 238 Normal plClne 238 NtlInber Napier's 28 real 11 Number series 293 NUlllerical integration of functions 382,
383

NUlnerical integration of ordinary differential equations 384-393

o
One-sided derivatives 43 One-sided lilnits 22 Open interval 11

494

Index
critical 84 stationary 222, 225 Polar subnormal 61 Polar subtangent 61 Potential (of a field) 289 Potential vector field 289 Po\vcr series 305 Principal normal 238 Principle of equal effects 369 Runge 383, 386 of superposition of solutions 353 Probability curve 19, 484 Probability integral 144 Product of t\VO convergent series 298 Progression ~eometric 293, 294 Proper rational fraction 121 Proportionate parts rule of 376
Q

Operator Hamiltonian 288 Laplacian 289 Order of smallness 35 Orthagonal surfaces 219 Orthagonal trajectories 328 Osculating circle 103 Osculating plane 238 Ostrogradsky-Gauss formula 286-288 Ostrogradsky-Gauss theorem 291 Ostrogradsky method 123, 125
p

Parabola 17, 20, 104, 105, 480, 485 cubic 17, 105, 234 Niele's 18, 234, 481 safety 234 semicubical 18, 20, 234, 481 Parabolic formula 382 Parameters variation of 332, 349, 352 Parametric representation of a function 207 Partial derivative hirheg-order 197 "mixed" 197 second 197 Partial sum 293 Particular integral 322 Particular solution 339 Pascal's limac;on 158 Path of integration 273, 274, 280 Period of a function 14 Periodic function 14 Picard's met hod 384, 385 Plane normal 238 osculating 238 rect ifyi ng 238 tangent 217 Point bending 84 critical (of the second kind) 92 of discontinuity 37 double 230 extrern aI 84 of inflection 91 isol ated 230 nlaximum 84 minimum 84 singular 230 stationary 196 of tangency 217 Points characteristic 96

Quadratic interpolation 372 Quadratic trinonlial 118, 119, Quantity infinitely large 33 infinitely sl11a11 33

12:~

R
Radi liS of convergence 305 Radius of curvature 102, 243 Radius of second curvature 243 Radius of tor~ion 243 Rate of change of a function 43 mean 42 Ratio (of a geornetric progreC\sion) 294 Real nunlbers J I Rectangular hyperbola 480 Rectifying plane 238 Reduction fornlulas 130, 135, 150 Reduction nlcthod 123 Regi on of convergence ~04 Region of integration 246-248 Relative error 367 Remainder 311 Renlainder of a series 293, 304 Remainder term 311 Removab!e discontinuity 37 Right -hand derivative 44 Right horizontal asymptote 93 Right inclined aSYlnptote 93 Rolle's theorem 75 Root multi plici ties 121

Index
Rose four-leafed 487 three-leafed 20, 487 Rotation (of a vector field) 288 Rule Leibniz 67, 269 I' I-Iospit aI-Bernoulli 78-82 of proport j onat c parts 376 Runge-Kutta nlethod 385-387, 390 I~unge principle 383, 386
S Safety parabola 234 Scalar field 288 Scheme
twelve-ordin~lte

495

393-39!i

Second curvature 243 Second derivative 66 Second dJJTerent ial 198 Second-orde~ difl'crential 72 Second partIal derivative 197 Segment of the nonnal 61 Segnlent of the polar BOrIllal 61 Segment of the polar tangent 61 Segment of a straight line 20 Segrnent of the tangent 61 Senlicircle 20 Senlicubical parabola 18, 20, 2.34, 4~1 Series absolutely convergent 296. 297 with cornplex ternlS 297 condrtionally (not absolutely) cOllvergent 296 convcrgent 293 Series (cont) Dirichlet 295, 296 dl vergent 293, 294 Fourier 318. 319 functional 304 harnl0nic 2~}4, 296, 297 IIlC0l11plete Fourier :318, 319 Maclaurin's 311. 313 number series 293 operations on 297 power 305 Taylor's 311, 313 Serpentine Newton's 18 Simpson's forrllula 382-384 Si oe curve 481 Single-valurd function 11 Singular integral 337 Singular point 230 Slope (of a tan~ent) 43 Stnallest value 225, 227

Solenoidal vector field 289 Solution (of an equation) 322 general 323, 359 part icu Iar 339 Spiral of Archimedes 20, 65, 66, 105, 487 hyperbolic 20, 105, 487 logarithmic 20, 21, 105, 106, 487 Static moment 168 Stationary point 196, 222, 225 Stationary scalar or vector field 288 Stokes' formula 285, 286, 289 S t ra i g h t 11 ne 17, 20 Strophoid 157,232,234,486 Subnormal 61 polar 61 Substitutions hyperbolic 114, 116, 133 trigonolTIctric 114, 115, 133 Subtangent 61 polar 61 Successive approxitnation 377, 385 rl1 et hod of 384, 385, 389 Sufficient conditions (for an extrclnum)

222
Sunl

Inte~ral 138 partial 293 of a seri es 293. 304 of two convergent serres 298 Su perposlti on of sol uti ons principle of 353 Surfncc jnte~ral of the first type 284 Surface iI1t('~ral of the second type 284 Surface integrals 284-286 Surfaces level 288 orthogonal 219
T

Table d I ago naI tab I e 389 of standard integrals 107 Table interval 372 Tabular difTerentiation 46 Tacnode 230 Tangency point of 217 Tangent 238 Tangent curve 481 Tangent plane 217 equation of 218 Tangents met h od of 377 Taylor's formula 77, 220

496

Index
computing volumes by means of 268 evaluating a 265 in rectangular coordinates 262 Trochoid 157 Twelve-ordinate schenle 393 395
U

Taylor's series 311, 313 Term general 294 remainder 311 Test d' Alembert's 295 Cauchy's 293,295 Cauchy's integral 295 com parison 143, 293, 294 Leibniz 296, 297 Weierstrass' 306 Theorem Cauchy's 75, 326 Dirichlet's 318 Theorem (cont) Lagrange's 75 Ostrogradsky-Guuss 291 Rolle's 75 Theorems Guldin's 171 mean-value 75, 150 Theory field 288-292 Three-leafed rose 20, 487 Torsion 243 Tractrix 161 Trajec.tories orthogonal 328 Transcendental funct ions integrati on of 135 Transformation Laplace 271 Trapezoidal fornlula 382 Trident of Newton 18 Trigonometric ~unctions 48 integrating 128, 129 Trigonometric substitutions 114 115 133 ' t Trihedron natural 238 Trinorrlial quadratic 118, 119, 123 Triple integral 262 applications of 265, 268 change of variables in 263

Undetermined coefficients method of 121, 351 Uniform convergence 306

v
Value greatest 85, 225, 227 least 85 mean (of a function) 151, 252 smallest 225, 227 Variable dependent 11 independent 11 Variables sep.arable an equation \vith 327 328 Variation of paranleters '332. 349, 352 Vector acceleration 236 of binormal 238 of principal nOrIllal 238 of tangent line 238 velocity 236 Vector field 288 Vector function 235 Vector lines 288 Velocity vector 236 Vertex of a curve 104 Vertical asynlptote 93 Vertices of a curve 104 Volume of a cylindroid 258 Volume of ~olids 161-166
W

Weierstrass' test 306 Witch of Agnesi 18, 156, 480 Work of a force 174, 276, 271

You might also like